You are on page 1of 280

CONSTITUTIONAL LAW 2 Case Digest

[Pick the date]

INTRODUCTION TO CONSTITUTIONAL LAW 2


THE NATURE OF THE CONSTITUTION AND ITS RELATION WITH THE COURTS
FRANCISCO VS. HOUSE OF REPRESENTATIVES
[415 SCRA 44; G.R. No. 160261; 10 Nov 2003]
Facts:
Impeachment proceedings were filed against Supreme Court Chief Justice Hilario
Davide.
The justiciable controversy poised in front of the Court was the
constitutionality of the subsequent filing of a second complaint to controvert the rules
of impeachment provided for by law
Issue:
Whether or Not the filing of the second impeachment complaint against Chief Justice
Hilario G. Davide, Jr. with the House of Representatives falls within the one year bar
provided in the Constitution and whether the resolution thereof is a political question
has resulted in a political crisis.
Held:
In any event, it is with the absolute certainty that our Constitution is sufficient to
address all the issues which this controversy spawns that this Court unequivocally
pronounces, at the first instance, that the feared resort to extra-constitutional methods
of resolving it is neither necessary nor legally permissible. Both its resolution and
protection of the public interest lie in adherence to, not departure from, the
Constitution.
In passing over the complex issues arising from the controversy, this Court is ever
mindful of the essential truth that the inviolate doctrine of separation of powers among
the legislative, executive or judicial branches of government by no means prescribes
for absolute autonomy in the discharge by each of that part of the governmental power
assigned to it by the sovereign people.
At the same time, the corollary doctrine of checks and balances which has been
carefully calibrated by the Constitution to temper the official acts of each of these three
branches must be given effect without destroying their indispensable co-equality.
There exists no constitutional basis for the contention that the exercise of judicial
review over impeachment proceedings would upset the system of checks and balances.
Verily, the Constitution is to be interpreted as a whole and "one section is not to be
allowed to defeat another." Both are integral components of the calibrated system of
independence and interdependence that insures that no branch of government act
beyond the powers assigned to it by the Constitution.
Page 2

When suing as a citizen, the interest of the petitioner assailing the constitutionality of a
statute must be direct and personal. He must be able to show, not only that the law or
any government act is invalid, but also that he sustained or is in imminent danger of
sustaining some direct injury as a result of its enforcement, and not merely that he
suffers thereby in some indefinite way. It must appear that the person complaining has
been or is about to be denied some right or privilege to which he is lawfully entitled or
that he is about to be subjected to some burdens or penalties by reason of the statute
or act complained of. In fine, when the proceeding involves the assertion of a public
right, the mere fact that he is a citizen satisfies the requirement of personal interest.
In the case of a taxpayer, he is allowed to sue where there is a claim that public funds
are illegally disbursed, or that public money is being deflected to any improper
purpose, or that there is a wastage of public funds through the enforcement of an
invalid or unconstitutional law. Before he can invoke the power of judicial review,
however, he must specifically prove that he has sufficient interest in preventing the
illegal expenditure of money raised by taxation and that he would sustain a direct injury
as a result of the enforcement of the questioned statute or contract. It is not sufficient
that he has merely a general interest common to all members of the public.
At all events, courts are vested with discretion as to whether or not a taxpayer's suit
should be entertained. This Court opts to grant standing to most of the petitioners,
given their allegation that any impending transmittal to the Senate of the Articles of
Impeachment and the ensuing trial of the Chief Justice will necessarily involve the
expenditure of public funds.
As for a legislator, he is allowed to sue to question the validity of any official action
which he claims infringes his prerogatives as a legislator. Indeed, a member of the
House of Representatives has standing to maintain inviolate the prerogatives, powers
and privileges vested by the Constitution in his office. 83
The framers of the Constitution also understood initiation in its ordinary meaning. Thus
when a proposal reached the floor proposing that "A vote of at least one-third of all the
Members of the House shall be necessary to initiate impeachment proceedings," this
was met by a proposal to delete the line on the ground that the vote of the House does
not initiate impeachment proceeding but rather the filing of a complaint does.
To the argument that only the House of Representatives as a body can initiate
impeachment proceedings because Section 3 (1) says "The House of Representatives
shall have the exclusive power to initiate all cases of impeachment," This is a
misreading of said provision and is contrary to the principle of reddendo singula
singulis by equating "impeachment cases" with "impeachment proceeding."
Having concluded that the initiation takes place by the act of filing and referral or
endorsement of the impeachment complaint to the House Committee on Justice or, by
the filing by at least one-third of the members of the House of Representatives with the
Secretary General of the House, the meaning of Section 3 (5) of Article XI becomes
clear. Once an impeachment complaint has been initiated, another impeachment
complaint may not be filed against the same official within a one year period.
The Court in the present petitions subjected to judicial scrutiny and resolved on the
merits only the main issue of whether the impeachment proceedings initiated against
Page 3

the Chief Justice transgressed the constitutionally imposed one-year time bar rule.
Beyond this, it did not go about assuming jurisdiction where it had none, nor
indiscriminately turn justiciable issues out of decidedly political questions. Because it is
not at all the business of this Court to assert judicial dominance over the other two
great branches of the government.
No one is above the law or the Constitution. This is a basic precept in any legal system
which recognizes equality of all men before the law as essential to the law's moral
authority and that of its agents to secure respect for and obedience to its commands.
Perhaps, there is no other government branch or instrumentality that is most zealous in
protecting that principle of legal equality other than the Supreme Court which has
discerned its real meaning and ramifications through its application to numerous cases
especially of the high-profile kind in the annals of jurisprudence. The Chief Justice is not
above the law and neither is any other member of this Court. But just because he is the
Chief Justice does not imply that he gets to have less in law than anybody else. The law
is solicitous of every individual's rights irrespective of his station in life.
Thus, the Rules of Procedure in Impeachment Proceedings which were approved by the
House of Representatives on November 28, 2001 are unconstitutional. Consequently,
the second impeachment complaint against Chief Justice Hilario G. Davide, Jr is barred
under paragraph 5, section 3 of Article XI of the Constitution.
MANILA PRINCE HOTEL VS. GSIS
[267 SCRA 408; G.R. No. 122156; 3 Feb 1997]
Facts:
The controversy arose when respondent Government Service Insurance System (GSIS),
pursuant to the privatization program of the Philippine Government under Proclamation
No. 50 dated 8 December 1986, decided to sell through public bidding 30% to 51% of
the issued and outstanding shares of respondent Manila Hotel Corporation. In a close
bidding held on 18 September 1995 only two (2) bidders participated: petitioner Manila
Prince Hotel Corporation, a Filipino corporation, which offered to buy 51% of the MHC or
15,300,000 shares at P41.58 per share, and Renong Berhad, a Malaysian firm, with ITTSheraton as its hotel operator, which bid for the same number of shares at P44.00 per
share, or P2.42 more than the bid of petitioner.
Pending the declaration of Renong Berhad as the winning bidder/strategic partner and
the execution of the necessary contracts, matched the bid price of P44.00 per share
tendered by Renong Berhad.
On 17 October 1995, perhaps apprehensive that respondent GSIS has disregarded the
tender of the matching bid and that the sale of 51% of the MHC may be hastened by
respondent GSIS and consummated with Renong Berhad, petitioner came to this Court
on prohibition and mandamus.
In the main, petitioner invokes Sec. 10, second par., Art. XII, of the 1987 Constitution
and submits that the Manila Hotel has been identified with the Filipino nation and has
practically become a historical monument which reflects the vibrancy of Philippine
Page 4

heritage and culture. It is a proud legacy of an earlier generation of Filipinos who


believed in the nobility and sacredness of independence and its power and capacity to
release the full potential of the Filipino people. To all intents and purposes, it has
become a part of the national patrimony. 6 Petitioner also argues that since 51% of the
shares of the MHC carries with it the ownership of the business of the hotel which is
owned by respondent GSIS, a government-owned and controlled corporation, the hotel
business of respondent GSIS being a part of the tourism industry is unquestionably a
part of the national economy.
Issue:
Whether or Not the sale of Manila Hotel to Renong Berhad is violative of the
Constitutional provision of Filipino First policy and is therefore null and void.
Held:
The Manila Hotel or, for that matter, 51% of the MHC, is not just any commodity to be
sold to the highest bidder solely for the sake of privatization. The Manila Hotel has
played and continues to play a significant role as an authentic repository of twentieth
century Philippine history and culture. This is the plain and simple meaning of the
Filipino First Policy provision of the Philippine Constitution. And this Court, heeding the
clarion call of the Constitution and accepting the duty of being the elderly watchman of
the nation, will continue to respect and protect the sanctity of the Constitution. It was
thus ordered that GSIS accepts the matching bid of petitioner MANILA PRINCE HOTEL
CORPORATION to purchase the subject 51% of the shares of the Manila Hotel
Corporation at P44.00 per share and thereafter to execute the necessary clearances
and to do such other acts and deeds as may be necessary for purpose.
PEOPLE VS. POMAR
[46 Phil 126; G.R. No. L-22008; 3 Nov 1924]
Facts:
Macaria Fajardo was an employee of La Flor de la Isabela, a Tobacco factory. She was
granted a vacation leave, by reason of her pregnancy, which commenced on the 16 th of
July 1923. According to Fajardo, during that time, she was not given the salary due her
in violation of the provisions of Act No. 3071. Fajardo filed a criminal complaint based
on Section 13 and 15 of said Act against the manager of the tobacco Factory, Julio
Pomar, herein defendant. The latter, on the other hand, claims that the facts in the
complaint did not constitute an offense and further alleges that the aforementioned
provisions of Act No. 3071 was unconstitutional. Section 13, Act No. 3071 provides that,
Every person, firm or corporation owning or managing a factory, shop or place of labor
of any description shall be obliged to grant to any woman employed by it as laborer
who may be pregnant, thirty days vacation with pay before and another thirty days
after confinement: Provided, That the employer shall not discharge such laborer without
just cause, under the penalty of being required to pay to her wages equivalent to the
total of two months counting from the day of her discharge. Section 15 of the same
Act provides for the penalty of any violation of section 13. The latter was enacted by
Page 5

the legislature in the exercise of its supposed Police Power with the purpose of
safeguarding the health of pregnant women laborers in "factory, shop or place of labor
of any description," and of insuring to them, to a certain extent, reasonable support for
one month before and one month after their delivery. The trial court rendered a
decision in favor of plaintiff, sentencing the defendant to pay the fine of fifty pesos and
in case of insolvency, to suffer subsidiary imprisonment. Hence, the case was raised to
the Court of Appeals which affirmed the former decision.
Issue:
Whether or not Section 13 of Act No. 3071 is unconstitutional.
Whether or not the promulgation of the questioned provision was a valid exercise of
Police Power.
Held:
The Supreme Court declared Section 13 of Act No. 3071 to be unconstitutional for being
violative or restrictive of the right of the people to freely enter into contracts for their
affairs. It has been decided several times, that the right to contract about one's affairs
is a part of the liberty of the individual, protected by the "due process of law" clause of
the constitution. The contracting parties may establish any agreements, terms, and
conditions they may deem advisable, provided they are not contrary to law, morals or
public policy
The police power of the state is a very broad and expanding power. The police power
may encompass every law for the restraint and punishment of crimes, for the
preservation of the public peace, health, and morals. But that power cannot grow faster
than the fundamental law of the state, nor transcend or violate the express inhibition of
the constitution. The Police Power is subject to and is controlled by the paramount
authority of the constitution of the state, and will not be permitted to violate rights
secured or guaranteed by the latter.
LAMBINO VS. COMELEC
[G.R. No. 174153; 25 Oct 2006]
Facts:
Petitioners (Lambino group) commenced gathering signatures for an initiative petition
to change the 1987 constitution, they filed a petition with the COMELEC to hold a
plebiscite that will ratify their initiative petition under RA 6735. Lambino group alleged
that the petition had the support of 6M individuals fulfilling what was provided by art
17 of the constitution. Their petition changes the 1987 constitution by modifying
Page 6

sections 1-7 of Art 6 and sections 1-4 of Art 7 and by adding Art 18. the proposed
changes will shift the present bicameral- presidential form of government to
unicameral- parliamentary. COMELEC denied the petition due to lack of enabling law
governing initiative petitions and invoked the Santiago Vs. Comelec ruling that RA 6735
is inadequate to implement the initiative petitions.
Issue:
Whether or Not the Lambino Groups initiative petition complies with Section 2, Article
XVII of the Constitution on amendments to the Constitution through a peoples
initiative.
Whether or Not this Court should revisit its ruling in Santiago declaring RA 6735
incomplete, inadequate or wanting in essential terms and conditions to implement
the initiative clause on proposals to amend the Constitution.
Whether or Not the COMELEC committed grave abuse of discretion in denying due
course to the Lambino Groups petition.
Held:
According to the SC the Lambino group failed to comply with the basic requirements for
conducting a peoples initiative. The Court held that the COMELEC did not grave abuse
of discretion on dismissing the Lambino petition.
1. The Initiative Petition Does Not Comply with Section 2, Article XVII of
Constitution on Direct Proposal by the People

the

The petitioners failed to show the court that the initiative signer must be informed
at the time of the signing of the nature and effect, failure to do so is deceptive and
misleading which renders the initiative void.
2. The Initiative Violates Section 2, Article XVII of the Constitution Disallowing Revision
through Initiatives
The framers of the constitution intended a clear distinction between amendment
and revision, it is intended that the third mode of stated in sec 2 art 17 of the
constitution may propose only amendments to the constitution. Merging of the
legislative and the executive is a radical change, therefore a constitutes a revision.
3. A Revisit of Santiago v. COMELEC is Not Necessary
Even assuming that RA 6735 is valid, it will not change the result because the
present petition violated Sec 2 Art 17 to be a valid initiative, must first comply with
the constitution before complying with RA 6735
Page 7

Petition is dismissed.
SANTIAGO VS. COMELEC
[270 SCRA 106; G.R. No.127325; 19 Mar 1997]
Facts:
Private respondent Atty. Jesus Delfin, president of Peoples Initiative for Reforms,
Modernization and Action (PIRMA), filed with COMELEC a petition to amend the
constitution to lift the term limits of elective officials, through Peoples Initiative. He
based this petition on Article XVII, Sec. 2 of the 1987 Constitution, which provides for
the right of the people to exercise the power to directly propose amendments to the
Constitution. Subsequently the COMELEC issued an order directing the publication of
the petition and of the notice of hearing and thereafter set the case for hearing. At the
hearing, Senator Roco, the IBP, Demokrasya-Ipagtanggol ang Konstitusyon, Public
Interest Law Center, and Laban ng Demokratikong Pilipino appeared as intervenorsoppositors. Senator Roco filed a motion to dismiss the Delfin petition on the ground
that one which is cognizable by the COMELEC. The petitioners herein Senator Santiago,
Alexander Padilla, and Isabel Ongpin filed this civil action for prohibition under Rule 65
of the Rules of Court against COMELEC and the Delfin petition rising the several
arguments, such as the following: (1) The constitutional provision on peoples initiative
to amend the constitution can only be implemented by law to be passed by Congress.
No such law has been passed; (2) The peoples initiative is limited to amendments to
the Constitution, not to revision thereof. Lifting of the term limits constitutes a revision,
therefore it is outside the power of peoples initiative. The Supreme Court granted the
Motions for Intervention.
Issue:
Whether or not Sec. 2, Art. XVII of the 1987 Constitution is a self-executing provision.
Whether or not COMELEC Resolution No. 2300 regarding the conduct of initiative on
amendments to the Constitution is valid, considering the absence in the law of specific
provisions on the conduct of such initiative.
Whether the lifting of term limits of elective officials would constitute a revision or an
amendment of the Constitution.
Held:
Sec. 2, Art XVII of the Constitution is not self executory, thus, without implementing
legislation the same cannot operate. Although the Constitution has recognized or
Page 8

granted the right, the people cannot exercise it if Congress does not provide for its
implementation.
The portion of COMELEC Resolution No. 2300 which prescribes rules and regulations on
the conduct of initiative on amendments to the Constitution, is void. It has been an
established rule that what has been delegated, cannot be delegated (potestas delegata
non delegari potest). The delegation of the power to the COMELEC being invalid, the
latter cannot validly promulgate rules and regulations to implement the exercise of the
right to peoples initiative.
The lifting of the term limits was held to be that of a revision, as it would affect other
provisions of the Constitution such as the synchronization of elections, the
constitutional guarantee of equal access to opportunities for public service, and
prohibiting political dynasties. A revision cannot be done by initiative. However,
considering the Courts decision in the above Issue, the issue of whether or not the
petition is a revision or amendment has become academic.
GONZALES VS. COMELEC
[21 SCRA 774; G.R. No. L-28196; 9 Nov 1967]
Facts:
The case is an original action for prohibition, with preliminary injunction.
The main facts are not disputed. On March 16, 1967, the Senate and the House of
Representatives passed the following resolutions:
1. R. B. H. (Resolution of Both Houses) No. 1, proposing that Section 5, Article VI, of the
Constitution of the Philippines, be amended so as to increase the membership of the
House of Representatives from a maximum of 120, as provided in the present
Constitution, to a maximum of 180, to be apportioned among the several provinces as
nearly as may be according to the number of their respective inhabitants, although
each province shall have, at least, one (1) member;
2. R. B. H. No. 2, calling a convention to propose amendments to said Constitution, the
convention to be composed of two (2) elective delegates from each representative
district, to be "elected in the general elections to be held on the second Tuesday of
November, 1971;" and
3. R. B. H. No. 3, proposing that Section 16, Article VI, of the same Constitution, be
amended so as to authorize Senators and members of the House of Representatives to
become delegates to the aforementioned constitutional convention, without forfeiting
their respective seats in Congress.
Subsequently, Congress passed a bill, which, upon approval by the President, on June
17, 1967, became Republic Act No. 4913, providing that the amendments to the
Page 9

Constitution proposed in the aforementioned Resolutions No. 1 and 3 be submitted, for


approval by the people, at the general elections which shall be held on November 14,
1967.
Issue:
Whether or Not a Resolution of Congress, acting as a constituent assembly, violates the
Constitution.
Held:
Inasmuch as there are less than eight (8) votes in favor of declaring Republic Act 4913
and R. B. H. Nos. 1 and 3 unconstitutional and invalid, the petitions in these two (2)
cases must be, as they are hereby, dismiss and the writs therein prayed for denied,
without special pronouncement as to costs. It is so ordered.
As a consequence, the title of a de facto officer cannot be assailed collaterally. It may
not be contested except directly, by quo warranto proceedings. Neither may the validity
of his acts be questioned upon the ground that he is merely a de facto officer. And the
reasons are obvious: (1) it would be an indirect inquiry into the title to the office; and
(2) the acts of a de facto officer, if within the competence of his office, are valid, insofar
as the public is concerned.
"The judicial department is the only constitutional organ which can be called upon to
determine the proper allocation of powers between the several departments and
among the integral or constituent units thereof."
Article XV of the Constitution provides:
. . . The Congress in joint session assembled, by a vote of three-fourths of
all the Members of the Senate and of the House of Representatives voting
separately, may propose amendments to this Constitution or call a
contention for that purpose. Such amendments shall be valid as part of
this Constitution when approved by a majority of the votes cast at an
election at which the amendments are submitted to the people for their
ratification.
From our viewpoint, the provisions of Article XV of the Constitution are satisfied so long
as the electorate knows that R. B. H. No. 3 permits Congressmen to retain their seats as
legislators, even if they should run for and assume the functions of delegates to the
Convention.
SANIDAD VS. COMELEC
Page 10

[78 SCRA 333; G.R. No. 90878; 29 Jan 1990]


Facts:
This is a petition for certiorari assailing the constitutionality of Section 19 of Comelec
Resolution No. 2167 on the ground that it violates the constitutional guarantees of the
freedom of expression and of the press. On October 23, 1989, Republic Act No. 6766,
entitled "AN ACT PROVIDING FOR AN ORGANIC ACT FOR THE CORDILLERA
AUTONOMOUS REGION" was enacted into law. Pursuant to said law, the City of Baguio
and the Cordilleras which consist of the provinces of Benguet, Mountain Province,
Ifugao, Abra and Kalinga-Apayao, all comprising the Cordillera Autonomous Region,
shall take part in a plebiscite for the ratification of said Organic Act originally scheduled
last December 27, 1989 which was, however, reset to January 30, 1990 by virtue of
Comelec Resolution No. 2226 dated December 27, 1989. The Commission on Elections,
by virtue of the power vested by the 1987 Constitution, the Omnibus Election Code (BP
881), said R.A. 6766 and other pertinent election laws, promulgated Resolution No.
2167, to govern the conduct of the plebiscite on the said Organic Act for the Cordillera
Autonomous Region. In a petition dated November 20, 1989, herein petitioner Pablito V.
Sanidad, who claims to be a newspaper columnist of the "OVERVIEW" for the BAGUIO
MIDLAND COURIER, a weekly newspaper circulated in the City of Baguio and the
Cordilleras, assailed the constitutionality of Section 19 of Comelec Resolution No. 2167,
which provides:
Section 19. Prohibition on columnists, commentators or announcers.
During the plebiscite campaign period, on the day before and on the
plebiscite day, no mass media columnist, commentator, announcer or
personality shall use his column or radio or television time to campaign for
or against the plebiscite Issue.
It is alleged by petitioner that said provision is void and unconstitutional because
it violates the constitutional guarantees of the freedom of expression and of the
press enshrined in the Constitution. Unlike a regular news reporter or news
correspondent who merely reports the news, petitioner maintains that as a
columnist, his column obviously and necessarily contains and reflects his
opinions, views and beliefs on any issue or subject about which he writes.
Petitioner likewise maintains that if media practitioners were to express their
views, beliefs and opinions on the issue submitted to a plebiscite, it would in fact
help in the government drive and desire to disseminate information, and hear, as
well as ventilate, all sides of the issue.
Issue:
Whether or not Section 19 of Comelec Resolution No. 2167 is unconstitutional.
Held:
The Supreme Court ruled that Section 19 of Comelec Resolution No. 2167 is
unconstitutional. It is clear from Art. IX-C of the 1987 Constitution that what was
granted to the Comelec was the power to supervise and regulate the use and
enjoyment of franchises, permits or other grants issued for the operation of
transportation or other public utilities, media of communication or information to the
Page 11

end that equal opportunity, time and space, and the right to reply, including
reasonable, equal rates therefor, for public information campaigns and forums among
candidates are ensured. The evil sought to be prevented by this provision is the
possibility that a franchise holder may favor or give any undue advantage to a
candidate in terms of advertising space or radio or television time. This is also the
reason why a "columnist, commentator, announcer or personality, who is a candidate
for any elective office is required to take a leave of absence from his work during the
campaign period (2nd par. Section 11(b) R.A. 6646). It cannot be gainsaid that a
columnist or commentator who is also a candidate would be more exposed to the
voters to the prejudice of other candidates unless required to take a leave of absence.
However, neither Article IX-C of the Constitution nor Section 11 (b), 2nd par. of R.A.
6646 can be construed to mean that the Comelec has also been granted the right to
supervise and regulate the exercise by media practitioners themselves of their right to
expression during plebiscite periods. Media practitioners exercising their freedom of
expression during plebiscite periods are neither the franchise holders nor the
candidates. In fact, there are no candidates involved in a plebiscite. Therefore, Section
19 of Comelec Resolution No. 2167 has no statutory basis.
Plebiscite Issue are matters of public concern and importance. The people's right to be
informed and to be able to freely and intelligently make a decision would be better
served by access to an unabridged discussion of the Issue, including the forum. The
people affected by the Issue presented in a plebiscite should not be unduly burdened
by restrictions on the forum where the right to expression may be exercised. Comelec
spaces and Comelec radio time may provide a forum for expression but they do not
guarantee full dissemination of information to the public concerned because they are
limited to either specific portions in newspapers or to specific radio or television times.
The instant petition is GRANTED. Section 19 of Comelec Resolution No. 2167 is declared
null and void and unconstitutional.
BONDOC VS. PINEDA
[201 SCRA 792; G.R. No. 97710; 26 Sep 1991]
Facts:
In the elections held on May 11, 1987, Marciano Pineda of the LDP and Emigdio Bondoc
of the NP were candidates for the position of Representative for the Fourth District of
Pampanga. Pineda was proclaimed winner. Bondoc filed a protest in the House of
Representatives Electoral Tribunal (HRET), which is composed of 9 members, 3 of whom
are Justices of the SC and the remaining 6 are members of the House of
Representatives (5 members belong to the LDP and 1 member is from the NP).
Thereafter, a decision had been reached in which Bondoc won over Pineda.
Congressman Camasura of the LDP voted with the SC Justices and Congressman
Cerilles of the NP to proclaim Bondoc the winner of the contest.
On the eve of the promulgation of the Bondoc decision, Congressman Camasura
Page 12

received a letter informing him that he was already expelled from the LDP for allegedly
helping to organize the Partido Pilipino of Eduardo Cojuangco and for allegedly inviting
LDP members in Davao Del Sur to join said political party. On the day of the
promulgation of the decision, the Chairman of HRET received a letter informing the
Tribunal that on the basis of the letter from the LDP, the House of Representatives
decided to withdraw the nomination and rescind the election of Congressman
Camasura to the HRET.
Issue:
Whether or not the House of Representatives, at the request of the dominant political
party therein, may change that partys representation in the HRET to thwart the
promulgation of a decision freely reached by the tribunal in an election contest pending
therein
Held:
The purpose of the constitutional convention creating the Electoral Commission was to
provide an independent and impartial tribunal for the determination of contests to
legislative office, devoid of partisan consideration.
As judges, the members of the tribunal must be non-partisan. They must discharge
their functions with complete detachment, impartiality and independence even
independence from the political party to which they belong. Hence, disloyalty to party
and breach of party discipline are not valid grounds for the expulsion of a member of
the tribunal. In expelling Congressman Camasura from the HRET for having cast a
conscience vote in favor of Bondoc, based strictly on the result of the examination
and appreciation of the ballots and the recount of the votes by the tribunal, the House
of Representatives committed a grave abuse of discretion, an injustice and a violation
of the Constitution. Its resolution of expulsion against Congressman Camasura is,
therefore, null and void.
Another reason for the nullity of the expulsion resolution of the House of
Representatives is that it violates Congressman Camasuras right to security of tenure.
Members of the HRET, as sole judge of congressional election contests, are entitled to
security of tenure just as members of the Judiciary enjoy security of tenure under the
Constitution. Therefore, membership in the HRET may not be terminated except for a
just cause, such as, the expiration of the members congressional term of office, his
death, permanent disability, resignation from the political party he represents in the
tribunal, formal affiliation with another political party or removal for other valid cause.
A member may not be expelled by the House of Representatives for party disloyalty,
short of proof that he has formally affiliated with another
MIRASOL VS CA
[351 SCRA 44; G.R. No. 128448; 1 Feb 2001]
Page 13

Facts:
The Mirasols are sugarland owners and planters. Philippine National Bank (PNB)
financed the Mirasols' sugar production venture FROM 1973-1975 under a crop loan
financing scheme. The Mirasols signed Credit Agreements, a Chattel Mortgage on
Standing Crops, and a Real Estate Mortgage in favor of PNB. The Chattel Mortgage
empowered PNB to negotiate and sell the latter's sugar and to apply the proceeds to
the payment of their obligations to it.
President Marcos issued PD 579 in November, 1974 authorizing Philippine Exchange
Co., Inc. (PHILEX) to purchase sugar allocated for export and authorized PNB to finance
PHILEX's purchases. The decree directed that whatever profit PHILEX might realize was
to be remitted to the government. Believing that the proceeds were more than enough
to pay their obligations, petitioners asked PNB for an accounting of the proceeds which
it ignored. Petitioners continued to avail of other loans from PNB and to make unfunded
withdrawals from their accounts with said bank. PNB asked petitioners to settle their
due and demandable accounts. As a result, petitioners, conveyed to PNB real properties
by way of dacion en pago still leaving an unpaid amount. PNB proceeded to
extrajudicially foreclose the mortgaged properties. PNB still had a deficiency claim.
Petitioners continued to ask PNB to account for the proceeds, insisting that said
proceeds, if properly liquidated, could offset their outstanding obligations. PNB
remained adamant in its stance that under P.D. No. 579, there was nothing to account
since under said law, all earnings from the export sales of sugar pertained to the
National Government.
On August 9, 1979, the Mirasols filed a suit for accounting, specific performance, and
damages against PNB.
Issue:
Whether or not the Trial Court has jurisdiction to declare a statute unconstitutional
without notice to the Solicitor General where the parties have agreed to submit such
issue for the resolution of the Trial Court.
Whether PD 579 and subsequent issuances thereof are unconstitutional.
Whether or not said PD is subject to judicial review.
Held:
It is settled that Regional Trial Courts have the authority and jurisdiction to consider the
constitutionality of a statute, presidential decree, or executive order. The Constitution
Page 14

vests the power of judicial review or the power to declare a law, treaty, international or
executive agreement, presidential decree, order, instruction, ordinance, or regulation
not only in this Court, but in all Regional Trial Courts.
The purpose of the mandatory notice in Rule 64, Section 3 is to enable the Solicitor
General to decide whether or not his intervention in the action assailing the validity of a
law or treaty is necessary. To deny the Solicitor General such notice would be
tantamount to depriving him of his day in court. We must stress that, contrary to
petitioners' stand, the mandatory notice requirement is not limited to actions involving
declaratory relief and similar remedies. The rule itself provides that such notice is
required in "any action" and not just actions involving declaratory relief. Where there is
no ambiguity in the words used in the rule, there is no room for construction. 15 In all
actions assailing the validity of a statute, treaty, presidential decree, order, or
proclamation, notice to the Solicitor General is mandatory.
Petitioners contend that P.D. No. 579 and its implementing issuances are void for
violating the due process clause and the prohibition against the taking of private
property without just compensation. Petitioners now ask this Court to exercise its power
of judicial review.
Jurisprudence has laid down the following requisites for the exercise of this power: First,
there must be before the Court an actual case calling for the exercise of judicial review.
Second, the question before the Court must be ripe for adjudication. Third, the person
challenging the validity of the act must have standing to challenge. Fourth, the
question of constitutionality must have been raised at the earliest opportunity, and
lastly, the issue of constitutionality must be the very lis mota of the case.

DUMLAO VS. COMELEC


[95 SCRA 392; G.R. No.L-52245; 22 Jan 1980]
Facts:
Petitioner Dumlao questions the constitutionality of Sec. 4 of Batas Pambansa Blg 52 as
discriminatory and contrary to equal protection and due process guarantees of the
Constitution. Sec. 4 provides that any retired elective provincial or municipal official
who has received payments of retirement benefits and shall have been 65 years of age
at the commencement of the term of office to which he seeks to be elected, shall not
be qualified to run for the same elective local office from which he has retired.
According to Dumlao, the provision amounts to class legislation. Petitioners Igot and
Salapantan Jr. also assail the validity of Sec. 4 of Batas Pambansa Blg 52, which states
that any person who has committed any act of disloyalty to the State, including those
amounting to subversion, insurrection, rebellion, or other similar crimes, shall not be
qualified for any of the offices covered by the act, or to participate in any partisan
activity therein: provided that a judgment of conviction of those crimes shall be
conclusive evidence of such fact and the filing of charges for the commission of such
Page 15

crimes before a civil court or military tribunal after preliminary investigation shall be
prima facie evidence of such fact.
Issue:
Whether or Not the aforementioned statutory provisions violate the Constitution and
thus, should be declared null and void
Whether or not the requisites of judicial review are complied with
Held:
No constitutional question will be heard and decided by the Court unless there is
compliance with the requisites of a judicial inquiry, which are: 1) There must be an
actual case or controversy; 2) The question of constitutionality must be raised by the
proper party; 3) The constitutional question must be raised at the earliest possible
opportunity; and 4) The decision of the constitutional question must be necessary to
the determination of the case itself.
As to (1), Dumlao has not been adversely affected by the application of the provision.
His question is posed merely in the abstract, and without the benefit of a detailed
factual record. As to (2), neither Igot nor Salapantan has been charged with acts of
loyalty to the State, nor disqualified from being candidates for local elective positions.
They have no personal nor substantial interest at stake. Igot and Salapantan have
institute the case as a taxpayers suit, but the institution of a taxpayers suit per se is
no assurance of judicial review. As to (4), there is no cause of action in this particular
case. Therefore, the necessity for resolving the issue of constitutionality is absent.
In regards to the unconstitutionality of the provisions, Sec. 4 of BP Blg 52 remains
constitutional and valid. The constitutional guarantee of equal protection of the laws is
subject to rational classification. One class can be treated differently from another
class. In this case, employees 65 years of age are classified differently from younger
employees. The purpose of the provision is to satisfy the need for new blood in the
workplace. In regards to the second paragraph of Sec. 4, it should be declared null and
void for being violative of the constitutional presumption of innocence guaranteed to an
accused.

LACSON VS. PEREZ


[357 SCRA 756; G.R. No. 147780 ;10 May 2001]
Facts:
Page 16

President Macapagal-Arroyo declared a State of Rebellion (Proclamation No. 38) on May


1, 2001 as well as General Order No. 1 ordering the AFP and the PNP to suppress the
rebellion in the NCR. Warrantless arrests of several alleged leaders and promoters of
the rebellion were thereafter effected. Petitioner filed for prohibition, injunction,
mandamus and habeas corpus with an application for the issuance of temporary
restraining order and/or writ of preliminary injunction. Petitioners assail the declaration
of Proc. No. 38 and the warrantless arrests allegedly effected by virtue thereof.
Petitioners furthermore pray that the appropriate court, wherein the information against
them were filed, would desist arraignment and trial until this instant petition is
resolved. They also contend that they are allegedly faced with impending warrantless
arrests and unlawful restraint being that hold departure orders were issued against
them.
Issue:
Whether or Not Proclamation No. 38 is valid, along with the warrantless arrests and
hold departure orders allegedly effected by the same.
Held:
President Macapagal-Arroyo ordered the lifting of Proc. No. 38 on May 6, 2006,
accordingly the instant petition has been rendered moot and academic. Respondents
have declared that the Justice Department and the police authorities intend to obtain
regular warrants of arrests from the courts for all acts committed prior to and until May
1, 2001. Under Section 5, Rule 113 of the Rules of Court, authorities may only resort to
warrantless arrests of persons suspected of rebellion in suppressing the rebellion if the
circumstances so warrant, thus the warrantless arrests are not based on Proc. No. 38.
Petitioners prayer for mandamus and prohibition is improper at this time because an
individual warrantlessly arrested has adequate remedies in law: Rule 112 of the Rules of
Court, providing for preliminary investigation, Article 125 of the Revised Penal Code,
providing for the period in which a warrantlessly arrested person must be delivered to
the proper judicial authorities, otherwise the officer responsible for such may be
penalized for the delay of the same. If the detention should have no legal ground, the
arresting officer can be charged with arbitrary detention, not prejudicial to claim of
damages under Article 32 of the Civil Code. Petitioners were neither assailing the
validity of the subject hold departure orders, nor were they expressing any intention to
leave the country in the near future. To declare the hold departure orders null and void
ab initio must be made in the proper proceedings initiated for that purpose. Petitioners
prayer for relief regarding their alleged impending warrantless arrests is premature
being that no complaints have been filed against them for any crime, furthermore, the
writ of habeas corpus is uncalled for since its purpose is to relieve unlawful restraint
which Petitioners are not subjected to.
Petition is dismissed. Respondents, consistent and congruent with their undertaking
earlier adverted to, together with their agents, representatives, and all persons acting
Page 17

in their behalf, are hereby enjoined from arresting Petitioners without the required
judicial warrants for all acts committed in relation to or in connection with the May 1,
2001 siege of Malacaang.

SANLAKAS VS. EXECUTIVE SECRETARY


[421 SCRA 656; G.R. No. 159085; 3 Feb 2004]
Facts:
During the wee hours of July 27, 2003, some three-hundred junior officers and enlisted
men of the AFP, acting upon instigation, command and direction of known and unknown
leaders have seized the Oakwood Building in Makati. Publicly, they complained of the
corruption in the AFP and declared their withdrawal of support for the government,
demanding the resignation of the President, Secretary of Defense and the PNP Chief.
These acts constitute a violation of Article 134 of the Revised Penal Code, and by virtue
of Proclamation No. 427 and General Order No. 4, the Philippines was declared under
the State of Rebellion. Negotiations took place and the officers went back to their
barracks in the evening of the same day. On August 1, 2003, both the Proclamation and
General Orders were lifted, and Proclamation No. 435, declaring the Cessation of the
State of Rebellion was issued.
In the interim, however, the following petitions were filed: (1) SANLAKAS AND PARTIDO
NG MANGGAGAWA VS. EXECUTIVE SECRETARY, petitioners contending that Sec. 18
Article VII of the Constitution does not require the declaration of a state of rebellion to
call out the AFP, and that there is no factual basis for such proclamation. (2)SJS
Officers/Members v. Hon. Executive Secretary, et al, petitioners contending that the
proclamation is a circumvention of the report requirement under the same Section 18,
Article VII, commanding the President to submit a report to Congress within 48 hours
from the proclamation of martial law. Finally, they contend that the presidential
issuances cannot be construed as an exercise of emergency powers as Congress has
not delegated any such power to the President. (3) Rep. Suplico et al. v. President
Macapagal-Arroyo and Executive Secretary Romulo, petitioners contending that there
was usurpation of the power of Congress granted by Section 23 (2), Article VI of the
Constitution. (4) Pimentel v. Romulo, et al, petitioner fears that the declaration of a
state of rebellion "opens the door to the unconstitutional implementation of warrantless
arrests" for the crime of rebellion.
Issue:
Whether or Not Proclamation No. 427 and General Order No. 4 are constitutional?
Whether or Not the petitioners have a legal standing or locus standi to bring suit?
Held:
The Court rendered that the both the Proclamation No. 427 and General Order No. 4 are
constitutional. Section 18, Article VII does not expressly prohibit declaring state or
rebellion. The President in addition to its Commander-in-Chief Powers is conferred by
the Constitution executive powers. It is not disputed that the President has full
Page 18

discretionary power to call out the armed forces and to determine the necessity for the
exercise of such power. While the Court may examine whether the power was exercised
within constitutional limits or in a manner constituting grave abuse of discretion, none
of the petitioners here have, by way of proof, supported their assertion that the
President acted without factual basis. The issue of the circumvention of the report is of
no merit as there was no indication that military tribunals have replaced civil courts or
that military authorities have taken over the functions of Civil Courts. The issue of
usurpation of the legislative power of the Congress is of no moment since the President,
in declaring a state of rebellion and in calling out the armed forces, was merely
exercising a wedding of her Chief Executive and Commander-in-Chief powers. These are
purely executive powers, vested on the President by Sections 1 and 18, Article VII, as
opposed to the delegated legislative powers contemplated by Section 23 (2), Article VI.
The fear on warrantless arrest is unreasonable, since any person may be subject to this
whether there is rebellion or not as this is a crime punishable under the Revised Penal
Code, and as long as a valid warrantless arrest is present.
Legal standing or locus standi has been defined as a personal and substantial interest
in the case such that the party has sustained or will sustain direct injury as a result of
the governmental act that is being challenged. The gist of the question of standing is
whether a party alleges "such personal stake in the outcome of the controversy as to
assure that concrete adverseness which sharpens the presentation of Issue upon which
the court depends for illumination of difficult constitutional questions. Based on the
foregoing, petitioners Sanlakas and PM, and SJS Officers/Members have no legal
standing to sue. Only petitioners Rep. Suplico et al. and Sen. Pimentel, as Members of
Congress, have standing to challenge the subject issuances. It sustained its decision in
Philippine Constitution Association v. Enriquez, that the extent the powers of Congress
are impaired, so is the power of each member thereof, since his office confers a right to
participate in the exercise of the powers of that institution.

JOYA VS. PCGG


[225 SCRA 568; G.R. No. 96541; 24 Aug 1993]
Facts:
On 9 August 1990, Mateo A.T. Caparas, then Chairman of PCGG, wrote then President
Corazon C. Aquino, requesting her for authority to sign the proposed Consignment
Agreement between the Republic of the Philippines through PCGG and Christie, Manson
and Woods International, Inc concerning the scheduled sale on 11 January 1991 of
eighty-two) Old Masters Paintings and antique silverware seized from Malacaang and
the Metropolitan Museum of Manila alleged to be part of the ill-gotten wealth of the late
President Marcos, his relatives and cronies. On 14 August 1990, then President Aquino,
through former Executive Secretary Catalino Macaraig, Jr., authorized Chairman
Caparas to sign the Consignment Agreement allowing Christie's of New York to auction
off the subject art pieces for and in behalf of the Republic of the Philippines. On 15
August 1990, PCGG, through Chairman Caparas, representing the Government of the
Republic of the Philippines, signed the Consignment Agreement with Christie's of New
York. According to the agreement, PCGG shall consign to CHRISTIE'S for sale at public
auction the eighty-two Old Masters Paintings then found at the Metropolitan Museum of
Page 19

Manila as well as the silverware contained in seventy-one cartons in the custody of the
Central Bank of the Philippines, and such other property as may subsequently be
identified by PCGG and accepted by CHRISTIE'S to be subject to the provisions of the
agreement.
On 26 October 1990, the Commission on Audit through then Chairman Eufemio C.
Domingo submitted to President Aquino the audit findings and observations of COA on
the Consignment Agreement of 15 August 1990 to the effect that: the authority of
former PCGG Chairman Caparas to enter into the Consignment Agreement was of
doubtful legality; the contract was highly disadvantageous to the government; PCGG
had a poor track record in asset disposal by auction in the U.S.; and, the assets subject
of auction were historical relics and had cultural significance, hence, their disposal was
prohibited by law.
After the oral arguments of the parties on 9 January 1991, we issued immediately our
resolution denying the application for preliminary injunction to restrain the scheduled
sale of the artworks on the ground that petitioners had not presented a clear legal right
to a restraining order and that proper parties had not been impleaded.
On 11 January 1991, the sale at public auction proceeded as scheduled and the
proceeds of $13,302,604.86 were turned over to the Bureau of Treasury.
Issue:
Whether or not petitioners have legal standing.
Whether or not the Old Masters Paintings and antique silverware are embraced in the
phrase "cultural treasure of the nation".
Whether or not the paintings and silverware are properties of public dominion on which
can be disposed of through the joint concurrence of the President and Congress.
Whether or not PCGG has complied with the due process clause and other statutory
requirements for the exportation and sale of the subject items.
Whether or not the petition has become moot and academic, and if so, whether the
above Issue warrant resolution from this Court.
Held:
This is premised on Sec. 2, Rule 3, of the Rules of Court which provides that every
action must be prosecuted and defended in the name of the real party-in-interest, and
that all persons having interest in the subject of the action and in obtaining the relief
demanded shall be joined as plaintiffs. The Court will exercise its power of judicial
review only if the case is brought before it by a party who has the legal standing to
raise the constitutional or legal question. "Legal standing" means a personal and
substantial interest in the case such that the party has sustained or will sustain direct
injury as a result of the governmental act that is being challenged. The term "interest"
is material interest, an interest in issue and to be affected by the decree, as
distinguished from mere interest in the question involved, or a mere incidental interest.
Page 20

Moreover, the interest of the party plaintiff must be personal and not one based on a
desire to vindicate the constitutional right of some third and related party.
There are certain instances however when this Court has allowed exceptions to the rule
on legal standing, as when a citizen brings a case for mandamus to procure the
enforcement of a public duty for the fulfillment of a public right recognized by the
Constitution, and when a taxpayer questions the validity of a governmental act
authorizing the disbursement of public funds.
Petitioners' arguments are devoid of merit. They lack basis in fact and in law. The
ownership of these paintings legally belongs to the foundation or corporation or the
members thereof, although the public has been given the opportunity to view and
appreciate these paintings when they were placed on exhibit.

The confiscation of these properties by the Aquino administration however should not
be understood to mean that the ownership of these paintings has automatically passed
on the government without complying with constitutional and statutory requirements of
due process and just compensation. If these properties were already acquired by the
government, any constitutional or statutory defect in their acquisition and their
subsequent disposition must be raised only by the proper parties the true owners
thereof whose authority to recover emanates from their proprietary rights which are
protected by statutes and the Constitution. Having failed to show that they are the
legal owners of the artworks or that the valued pieces have become publicly owned,
petitioners do not possess any clear legal right whatsoever to question their alleged
unauthorized disposition.
Neither can this petition be allowed as a taxpayer's suit. Obviously, petitioners are not
challenging any expenditure involving public funds but the disposition of what they
allege to be public properties. It is worthy to note that petitioners admit that the
paintings and antique silverware were acquired from private sources and not with
public money.
Anent the second requisite of actual controversy, petitioners argue that this case
should be resolved by this Court as an exception to the rule on moot and academic
cases; that although the sale of the paintings and silver has long been consummated
and the possibility of retrieving the treasure trove is nil, yet the novelty and importance
of the Issue raised by the petition deserve this Court's attention. They submit that the
resolution by the Court of the Issue in this case will establish future guiding principles
and doctrines on the preservation of the nation's priceless artistic and cultural
possessions for the benefit of the public as a whole.
For a court to exercise its power of adjudication, there must be an actual case of
controversy one which involves a conflict of legal rights, an assertion of opposite
legal claims susceptible of judicial resolution; the case must not be moot or academic
or based on extra-legal or other similar considerations not cognizable by a court of
justice. A case becomes moot and academic when its purpose has become stale, such
as the case before us. Since the purpose of this petition for prohibition is to enjoin
respondent public officials from holding the auction sale of the artworks on a particular
date 11 January 1991 which is long past, the Issue raised in the petition have
become moot and academic.
Page 21

The cultural properties of the nation which shall be under the protection of the state are
classified as the "important cultural properties" and the "national cultural treasures."
On the other hand, a "national cultural treasures" is a unique object found locally,
possessing outstanding historical, cultural, artistic and/or scientific value which is highly
significant and important to this country and nation. This Court takes note of the
certification issued by the Director of the Museum that the Italian paintings and
silverware subject of this petition do not constitute protected cultural properties and
are not among those listed in the Cultural Properties Register of the National Museum.
WHEREFORE, for lack of merit, the petition for prohibition and mandamus is DISMISSED.

OPOSA VS. FACTORAN, JR.


[224 SCRA 792; G.R. No. 101083; 30 Jul 1993]
Facts:
Principal petitioners, are all minors duly represented and joined by their respective
parents. Impleaded as an additional plaintiff is the Philippine Ecological Network, Inc.
(PENI), a domestic, non-stock and non-profit corporation organized for the purpose of,
inter alia, engaging in concerted action geared for the protection of our environment
and natural resources. The original defendant was the Honorable Fulgencio S. Factoran,
Jr., then Secretary of the Department of Environment and Natural Resources (DENR).
His substitution in this petition by the new Secretary, the Honorable Angel C. Alcala,
was subsequently ordered upon proper motion by the petitioners. The complaint was
instituted as a taxpayers' class suit and alleges that the plaintiffs "are all citizens of the
Republic of the Philippines, taxpayers, and entitled to the full benefit, use and
enjoyment of the natural resource treasure that is the country's virgin tropical forests."
The same was filed for themselves and others who are equally concerned about the
preservation of said resource but are "so numerous that it is impracticable to bring
them all before the Court."
On 22 June 1990, the original defendant, Secretary Factoran, Jr., filed a Motion to
Dismiss the complaint based on two grounds, namely: the plaintiffs have no cause of
action against him and, the issue raised by the plaintiffs is a political question which
properly pertains to the legislative or executive branches of Government. In their 12
July 1990 Opposition to the Motion, the petitioners maintain that, the complaint shows
a clear and unmistakable cause of action, the motion is dilatory and the action presents
a justiciable question as it involves the defendant's abuse of discretion.
On 18 July 1991, respondent Judge issued an order granting the aforementioned motion
to dismiss. In the said order, not only was the defendant's claim that the complaint
states no cause of action against him and that it raises a political question sustained,
the respondent Judge further ruled that the granting of the relief prayed for would
result in the impairment of contracts which is prohibited by the fundamental law of the
land.
Plaintiffs thus filed the instant special civil action for certiorari under Rule 65 of the
Revised Rules of Court and ask this Court to rescind and set aside the dismissal order
on the ground that the respondent Judge gravely abused his discretion in dismissing the
Page 22

action. Again, the parents of the plaintiffs-minors not only represent their children, but
have also joined the latter in this case.
Petitioners contend that the complaint clearly and unmistakably states a cause of
action as it contains sufficient allegations concerning their right to a sound environment
based on Articles 19, 20 and 21 of the Civil Code (Human Relations), Section 4 of
Executive Order (E.O.) No. 192 creating the DENR, Section 3 of Presidential Decree
(P.D.) No. 1151 (Philippine Environmental Policy), Section 16, Article II of the 1987
Constitution recognizing the right of the people to a balanced and healthful ecology,
the concept of generational genocide in Criminal Law and the concept of man's
inalienable right to self-preservation and self-perpetuation embodied in natural law.
Petitioners likewise rely on the respondent's correlative obligation per Section 4 of E.O.
No. 192, to safeguard the people's right to a healthful environment.
It is further claimed that the issue of the respondent Secretary's alleged grave abuse of
discretion in granting Timber License Agreements (TLAs) to cover more areas for
logging than what is available involves a judicial question.
Anent the invocation by the respondent Judge of the Constitution's non-impairment
clause, petitioners maintain that the same does not apply in this case because TLAs are
not contracts. They likewise submit that even if TLAs may be considered protected by
the said clause, it is well settled that they may still be revoked by the State when the
public interest so requires.
Issue:
Whether or not the petitioners have locus standi.
Whether or not the petiton is in a form of a class suit.
Whether or not the TLAs can be out rightly cancelled.
Whether or not the petition should be dismissed.
Held:
As to the matter of the cancellation of the TLAs, respondents submit that the same
cannot be done by the State without due process of law. Once issued, a TLA remains
effective for a certain period of time usually for twenty-five (25) years. During its
effectivity, the same can neither be revised nor cancelled unless the holder has been
found, after due notice and hearing, to have violated the terms of the agreement or
other forestry laws and regulations. Petitioners' proposition to have all the TLAs
indiscriminately cancelled without the requisite hearing would be violative of the
requirements of due process.
The subject matter of the complaint is of common and general interest not just to
several, but to all citizens of the Philippines. Consequently, since the parties are so
numerous, it, becomes impracticable, if not totally impossible, to bring all of them
before the court. The plaintiffs therein are numerous and representative enough to
ensure the full protection of all concerned interests. Hence, all the requisites for the
filing of a valid class suit under Section 12, Rule 3 of the Revised Rules of Court are
Page 23

present both in the said civil case and in the instant petition, the latter being but an
incident to the former.
Petitioners minors assert that they represent their generation as well as generations
yet unborn. Their personality to sue in behalf of the succeeding generations can only be
based on the concept of intergenerational responsibility insofar as the right to a
balanced and healthful ecology is concerned. Nature means the created world in its
entirety. Every generation has a responsibility to the next to preserve that rhythm and
harmony for the full enjoyment of a balanced and healthful ecology. The minors'
assertion of their right to a sound environment constitutes, at the same time, the
performance of their obligation to ensure the protection of that right for the generations
to come.
The complaint focuses on one specific fundamental legal right the right to a balanced
and healthful ecology which, for the first time in our nation's constitutional history, is
solemnly incorporated in the fundamental law. Section 16, Article II of the 1987
Constitution.
While the right to a balanced and healthful ecology is to be found under the Declaration
of Principles and State Policies and not under the Bill of Rights, it does not follow that it
is less important than any of the civil and political rights enumerated in the latter. Such
a right belongs to a different category of rights altogether for it concerns nothing less
than self-preservation and self-perpetuation aptly and fittingly stressed by the
petitioners the advancement of which may even be said to predate all governments
and constitutions. As a matter of fact, these basic rights need not even be written in
the Constitution for they are assumed to exist from the inception of humankind. If they
are now explicitly mentioned in the fundamental charter, it is because of the wellfounded fear of its framers that unless the rights to a balanced and healthful ecology
and to health are mandated as state policies by the Constitution itself, thereby
highlighting their continuing importance and imposing upon the state a solemn
obligation to preserve the first and protect and advance the second, the day would not
be too far when all else would be lost not only for the present generation, but also for
those to come generations which stand to inherit nothing but parched earth incapable
of sustaining life.
Conformably with the enunciated right to a balanced and healthful ecology and the
right to health, as well as the other related provisions of the Constitution concerning
the conservation, development and utilization of the country's natural resources, then
President Corazon C. Aquino promulgated on 10 June 1987 E.O. No. 192, Section 4 of
which expressly mandates that the Department of Environment and Natural Resources
"shall be the primary government agency responsible for the conservation,
management, development and proper use of the country's environment and natural
resources, specifically forest and grazing lands, mineral, resources, including those in
reservation and watershed areas, and lands of the public domain, as well as the
licensing and regulation of all natural resources as may be provided for by law in order
to ensure equitable sharing of the benefits derived therefrom for the welfare of the
present and future generations of Filipinos." Section 3 thereof makes the following
statement of policy:
The above provision stresses "the necessity of maintaining a sound ecological balance
and protecting and enhancing the quality of the environment." Section 2 of the same
Title, on the other hand, specifically speaks of the mandate of the DENR; however, it
Page 24

makes particular reference to the fact of the agency's being subject to law and higher
authority.
It may, however, be recalled that even before the ratification of the 1987 Constitution,
specific statutes already paid special attention to the "environmental right" of the
present and future generations. On 6 June 1977, P.D. No. 1151 and P.D. No. 1152 were
issued. Thus, the right of the petitioners to a balanced and healthful ecology is as clear
as the DENR's duty under its mandate and by virtue of its powers and functions under
E.O. No. 192 and the Administrative Code of 1987 to protect and advance the said
right.
A denial or violation of that right by the other who has the correlative duty or obligation
to respect or protect the same gives rise to a cause of action. Petitioners maintain that
the granting of the TLAs, which they claim was done with grave abuse of discretion,
violated their right to a balanced and healthful ecology; hence, the full protection
thereof requires that no further TLAs should be renewed or granted.
It is settled in this jurisdiction that in a motion to dismiss based on the ground that the
complaint fails to state a cause of action; the question submitted to the court for
resolution involves the sufficiency of the facts alleged in the complaint itself. No other
matter should be considered; furthermore, the truth of falsity of the said allegations is
beside the point for the truth thereof is deemed hypothetically admitted. Policy
formulation or determination by the executive or legislative branches of Government is
not squarely put in issue. What is principally involved is the enforcement of a right visa-vis policies already formulated and expressed in legislation. It must, nonetheless, be
emphasized that the political question doctrine is no longer, the insurmountable
obstacle to the exercise of judicial power or the impenetrable shield that protects
executive and legislative actions from judicial inquiry or review.
In the second place, even if it is to be assumed that the same are contracts, the instant
case does not involve a law or even an executive issuance declaring the cancellation or
modification of existing timber licenses. Hence, the non-impairment clause cannot as
yet be invoked. Nevertheless, granting further that a law has actually been passed
mandating cancellations or modifications, the same cannot still be stigmatized as a
violation of the non-impairment clause. This is because by its very nature and purpose,
such as law could have only been passed in the exercise of the police power of the
state for the purpose of advancing the right of the people to a balanced and healthful
ecology, promoting their health and enhancing the general welfare.
Finally, it is difficult to imagine, as the trial court did, how the non-impairment clause
could apply with respect to the prayer to enjoin the respondent Secretary from
receiving, accepting, processing, renewing or approving new timber licenses for, save
in cases of renewal, no contract would have as of yet existed in the other instances.
Moreover, with respect to renewal, the holder is not entitled to it as a matter of right.
Petition is hereby GRANTED, and the challenged Order of respondent Judge of 18 July
1991 dismissing Civil Case No. 90-777 is hereby set aside. The petitioners may
therefore amend their complaint to implead as defendants the holders or grantees of
the questioned timber license agreements.

Page 25

AGAN JR. VS. PIATCO


[402 SCRA 612; G.R. No. 155001; 5 May 2003]
Facts:
Some time in 1993, six business leaders, explored the possibility of investing in the new
NAIA airport terminal, so they formed Asians Emerging Dragon Corp. They submitted
proposals to the government for the development of NAIA Intl. Passenger Terminal III
(NAIA IPT III). The NEDA approved the NAIA IPT III project. Bidders were invited, and
among the proposal Peoples Air Cargo (Paircargo) was chosen. AEDC protested alleging
that preference was given to Paircargo, but still the project was awarded to Paircargo.
Because of that, it incorporated into, Phil. Intl. Airport Terminals Co. (PIATCO). The DOTC
and PIATCO entered into a concession agreement in 1997 to franchise and operate the
said terminal for 21years. In Nov. 1998 it was amended in the matters of pertaining to
the definition of the obligations given to the concessionaire, development of facilities
and proceeds, fees and charges, and the termination of contract. Since MIAA is charged
with the maintenance and operations of NAIA terminals I and II, it has a contract with
several service providers. The workers filed the petition for prohibition claiming that
they would lose their job, and the service providers joined them, filed a motion for
intervention. Likewise several employees of the MIAA filed a petition assailing the
legality of arrangements. A group of congressmen filed similar petitions. Pres. Arroyo
declared in her speech that she will not honor PIATCO contracts which the Exec.
Branch's legal office concluded null and void.
Issue:
Whether or Not the 1997 concession agreement is void, together with its amendments
for being contrary to the constitution.
Held:
The 1997 concession agreement is void for being contrary to public policy. The
amendments have the effect of changing it into and entirely different agreement from
the contract bidded upon. The amendments present new terms and conditions which
provide financial benefit to PIATCO which may have the altered the technical and
financial parameters of other bidders had they know that such terms were available.
The 1997 concession agreement, the amendments and supplements thereto are set
aside for being null and void.

Page 26

The petitioners have local standi. They are prejudiced by the concession agreement as
their livelihood is to be taken away from them.

UMALI VS. GUINGONA


[305 SCRA 533; G.R. No. 131124; 21 Mar 1999]
Facts:
Osmundo Umali the petitioner was appointed Regional Director of the Bureau of
Internal Revenue by Pres Fidel V. Ramos. He assigned him in Manila, November 29,
1993 to March 15, 1994 and Makati, March 16, 1994 to August 4, 1994. On August 1,
1994, President Ramos received a confidential memorandum against the petitioner for
alleged violations of internal revenue laws, rules and regulations during his incumbency
as Regional Director, more particularly the following malfeasance, misfeasance and
nonfeasance. upon receipt of the said confidential memorandum, former President
authorized the issuance of an Order for the preventive suspension of the petitioner and
immediately referred the Complaint against the latter to the Presidential Commission
on Anti-Graft and Corruption (PCAGC), for investigation. Petitioner was duly informed of
the charges against him. And was directed him to send in his answer, copies of his
Statement of Assets, and Liabilities for the past three years (3), and Personal Data
Sheet. Initial hearing was set on August 25, 1994, at 2:00 p.m., at the PCAGC Office. On
August 23, the petitioner filed his required answer. After evaluating the evidence on
record, the PCAGC issued its Resolution of September 23, 1994, finding a prima facie
evidence to support six (6) of the twelve (12) charges against petitioner. On October 6,
1994, acting upon the recommendation of the PCAGC, then President Ramos issued
Administrative Order No. 152 dismissing petitioner from the service, with forfeiture of
retirement and all benefits under the law.
Issue:
Whether or Not AO No. 152 violated petitioner's Right to Security of Tenure.
Whether or Not Petitioner was denied due process of law
Whether or Not the PCAGC is a validly Constituted government agency and whether the
petitioner can raise the issue of constitutionality belatedly in its motion for
reconsideration of the trial courts decision.
Whether or Not the ombudsman's resolution dismissing the charges against the
petitioner is still basis for the petitioner's dismissal with forfeiture of benefits as ruled in
AO No. 152

Page 27

Held:
Petitioner maintains that as a career executive service officer, he can only be removed
for cause and under the Administrative Code of 1987, 6 loss of confidence is not one of
the legal causes or grounds for removal. Consequently, his dismissal from office on the
ground of loss confidence violated his right to security of tenure, petitioner theorized.
After a careful study, we are of the irresistible conclusion that the Court of Appeals
ruled correctly on the first three Issue. To be sure, petitioner was not denied the right to
due process before the PCAGC. Records show that the petitioner filed his answer and
other pleadings with respect to his alleged violation of internal revenue laws and
regulations, and he attended the hearings before the investigatory body. It is thus
decisively clear that his protestation of non-observance of due process is devoid of any
factual or legal basis. Neither can it be said that there was a violation of what petitioner
asserts as his security of tenure. According to petitioner, as a Regional Director of
Bureau of Internal Revenue, he is CESO eligible entitled to security of tenure. However,
petitioner's claim of CESO eligibility is anemic of evidentiary support. It was incumbent
upon him to prove that he is a CESO eligible but unfortunately, he failed to adduce
sufficient evidence on the matter. His failure to do so is fatal. As regards the issue of
constitutionality of the PCAGC, it was only posed by petitioner in his motion for
reconsideration before the Regional Trial Court of Makati. It was certainly too late to
raise for the first time at such late stage of the proceedings. As to last issue, It is
worthy to note that in the case under consideration, the administrative action against
the petitioner was taken prior to the institution of the criminal case. The charges
included in Administrative Order No. 152 were based on the results of investigation
conducted by the PCAGC and not on the criminal charges before the Ombudsman. In
sum, the petition is dismissable on the ground that the Issue posited by the petitioner
do not constitute a valid legal basis for overturning the finding and conclusion arrived
at by the Court of Appeals. However, taking into account the antecedent facts and
circumstances aforementioned, the Court, in the exercise of its equity powers, has
decided to consider the dismissal of the charges against petitioner before the
Ombudsman, the succinct and unmistakable manifestation by the Commissioner of the
Bureau of Internal Revenue that his office is no longer interested in pursuing the case,
and the position taken by the Solicitor General, that there is no more basis for
Administrative Order No. 152, as effective and substantive supervening events that
cannot be overlooked.

IN RE CUNANAN
[94 Phil 534; Resolution; 18 Mar 1954]
Facts:
Congress passed Republic Act Number 972, commonly known as the Bar Flunkers Act
of 1953. In accordance with the said law, the Supreme Court then passed and
admitted to the bar those candidates who had obtained an average of 72 per cent by
raising it to 75 percent.
Page 28

After its approval, many of the unsuccessful postwar candidates filed petitions for
admission to the bar invoking its provisions, while other motions for the revision of their
examination papers were still pending also invoked the aforesaid law as an additional
ground for admission. There are also others who have sought simply the
reconsideration of their grades without, however, invoking the law in question. To avoid
injustice to individual petitioners, the court first reviewed the motions for
reconsideration, irrespective of whether or not they had invoked Republic Act No. 972.
Issue:
Whether or Not RA No. 972 is constitutional and valid.
Held:
RA No. 972 has for its object, according to its author, to admit to the Bar, those
candidates who suffered from insufficiency of reading materials and inadequate
preparation.
In the judicial system from which ours has been evolved, the admission, suspension,
disbarment and reinstatement of attorneys at law in the practice of the profession and
their supervision have been indisputably a judicial function and responsibility. We have
said that in the judicial system from which ours has been derived, the admission,
suspension, disbarment or reinstatement of attorneys at law in the practice of the
profession is concededly judicial.
On this matter, there is certainly a clear distinction between the functions of the judicial
and legislative departments of the government.
It is obvious, therefore, that the ultimate power to grant license for the practice of law
belongs exclusively to this Court, and the law passed by Congress on the matter is of
permissive character, or as other authorities may say, merely to fix the minimum
conditions for the license.
Republic Act Number 972 is held to be unconstitutional.

REPUBLIC ACT 6735, INITIATIVE AND REFERENDUM ACT


R.A. No. 6735 was, as its history reveals, intended to cover initiative to propose
amendments to the Constitution. The Act is a consolidation of House Bill No. 21505 and
Senate Bill No. 17. The former was prepared by the committee on Suffrage and
Page 29

Electoral Reforms of Representatives on the basis of two House Bills referred to it, viz.,
(a) House Bill No. 497, which dealt with the initiative and referendum mentioned in
Sections 1 and 32 of Article VI of the Constitution; and (b) House Bill No. 988, which
dealt with the subject matter of House Bill No. 497, as well as with initiative and
referendum under Section 3 of Article XVII of the Constitution. Senate Bill No. 17 solely,
dealt with initiative and referendum concerning ordinances or resolutions of local
government units. The Bicameral Conference Committee consolidated Senate Bill No.
17 and House Bill No. 21505 into a draft bill, which was subsequently approved on 8
June 1989 by the Senate and by the House of Representatives. This approved bill is now
R.A. No. 6735.

Page 30

THE FUNDAMENTAL POWERS OF


THE STATE
THE POLICE POWER

AGUSTIN VS. EDU


[88 SCRA 195; G.R. No. L-49112; 2 Feb 1979]
Facts:
President Marcos issued the Letter of Instruction No. 229 which states that all owners,
users or drivers shall have at all times one pair of early warning devise (EWD) in their
cars acquire from any source depending on the owners choice. The Letter of Instruction
was assailed by petitioner Leovillo Agustin to have violated the constitution guarantee
of due process against Hon Edu, Land Transportation Commissioner, Hon. Juan Ponce
Enrile, Minister of national Defense, Hon. Juinio, Minister of Public Works, Transportation
and Communication and Hon. Aquino, Minister of Public Highways. Because of such
contentions, the Implementing Rules and Regulation was ordered to be suspended for a
period of 6 months. Petitioner alleges that EWD are not necessary because vehicles
already have hazard lights (blinking lights) that can be use as a warning device. Also
petitioner contest that the letter of instruction violates the delegation of police power
because it is deemed harsh, oppressive and unreasonable for the motorists and those
dealers of EWD will become instant millionaires because of such law.
Issue:
Whether or not Petitioners contentions possess merit.
Held:
Petitioners contentions are without merit because the exercise of police power may
interfere with personal liberty or property to ensure and promote the safety, health and
prosperity of the State. Also, such letter of instruction is intended to promote public
safety and it is indeed a rare occurrence that such contention was alleged in a
instruction with such noble purpose. Petitioner also failed to present the factual
foundation that is necessary to invalidate the said letter of instruction. In cases where
there is absence in the factual foundation, it should be presumed that constitutionality
shall prevail. Pres. Marcos on the other hand possesses vital statistics that will justify
the need for the implementation of this instruction. As signatory to the 1968 Vienna
Page 31

Conventions on Road Signs and Signals, our country must abide with the standards
given as stated in our Constitution that the Philippines adopts the generally accepted
principles of International Law as part of the law of the land. In the case at bar, the
Vienna Convention also requires the use of EWD. Vehicle owners are not obliged to buy
an EDW. They can personally create a EWD provided that it is in accordance to the
specifications provided by law. Petitioners allegation against the manufacturers of EDW
being millionaires is deemed to be an unfounded speculation. Wherefore, the petition is
dismissed. The restraining order regarding the implementation of the Reflector Law is
lifted making the said law immediately executory.

ICHONG VS. HERNANDEZ


[101 Phil 1117; G.R. No. L-7995; 31 May 1957]
Facts:
Republic Act 1180 or commonly known as An Act to Regulate the Retail Business was
passed. The said law provides for a prohibition against foreigners as well as
corporations owned by foreigners from engaging from retail trade in our country. This
was protested by the petitioner in this case. According to him, the said law violates the
international and treaty of the Philippines therefore it is unconstitutional. Specifically,
the Treaty of Amity between the Philippines and China was violated according to him.
Issue:
Whether or Not Republic Act 1180 is a valid exercise of police power.
Held:
According to the Court, RA 1180 is a valid exercise of police power. It was also then
provided that police power can not be bargained away through the medium of a treaty
or a contract. The Court also provided that RA 1180 was enacted to remedy a real and
actual danger to national economy posed by alien dominance and control. If ever the
law infringes upon the said treaty, the latter is always subject to qualification or
amendment by a subsequent law and the same may never curtain or restrict the scope
of the police power of the state.

LUTZ VS. ARANETA


[98 Phil 148; G.R. No. L-7859; 22 Dec 1955]
Page 32

Facts:
Walter Lutz, as the Judicial Administrator of the Intestate Estate of Antonio Jayme
Ledesma, seeks to recover from J. Antonio Araneta, the Collector of Internal Revenue,
the sum of money paid by the estate as taxes, pursuant to the Sugar Adjustment Act.
Under Section 3 of said Act, taxes are levied on the owners or persons in control of the
lands devoted to the cultivation of sugar cane. Furthermore, Section 6 states all the
collections made under said Act shall be for aid and support of the sugar industry
exclusively. Lutz contends that such purpose is not a matter of public concern hence
making the tax levied for that cause unconstitutional and void. The Court of First
Instance dismissed his petition, thus this appeal before the Supreme Court.
Issue:
Whether or Not the tax levied under the Sugar Adjustment Act ( Commonwealth Act
567) is unconstitutional.

Held:
The tax levied under the Sugar Adjustment Act is constitutional. The tax under said Act
is levied with a regulatory purpose, to provide means for the rehabilitation and
stabilization of the threatened sugar industry. Since sugar production is one of the
great industries of our nation, its promotion, protection, and advancement, therefore
redounds greatly to the general welfare. Hence, said objectives of the Act is a public
concern and is therefore constitutional. It follows that the Legislature may determine
within reasonable bounds what is necessary for its protection and expedient for its
promotion. If objectives and methods are alike constitutionally valid, no reason is seen
why the state may not levy taxes to raise funds for their prosecution and attainment.
Taxation may be made with the implement of the states police power. In addition, it is
only rational that the taxes be obtained from those that will directly benefit from it.
Therefore, the tax levied under the Sugar Adjustment Act is held to be constitutional.

TIO VS. VIDEOGRAM REGULATORY BOARD


[151 SCRA 208; G.R. No. L-75697; 18 Jun 1987]
Facts:

Page 33

The case is a petition filed by petitioner on behalf of videogram operators adversely


affected by Presidential Decree No. 1987, An Act Creating the Videogram Regulatory
Board" with broad powers to regulate and supervise the videogram industry.
A month after the promulgation of the said
National Internal Revenue Code provided that:

Presidential Decree, the amended the

"SEC. 134. Video Tapes. There shall be collected on each processed


video-tape cassette, ready for playback, regardless of length, an annual
tax of five pesos; Provided, That locally manufactured or imported blank
video tapes shall be subject to sales tax."
"Section 10. Tax on Sale, Lease or Disposition of Videograms.
Notwithstanding any provision of law to the contrary, the province shall
collect a tax of thirty percent (30%) of the purchase price or rental rate, as
the case may be, for every sale, lease or disposition of a videogram
containing a reproduction of any motion picture or audiovisual program.
Fifty percent (50%) of the proceeds of the tax collected shall accrue to
the province, and the other fifty percent (50%) shall accrue to the
municipality where the tax is collected; PROVIDED, That in Metropolitan
Manila, the tax shall be shared equally by the City/Municipality and the
Metropolitan Manila Commission.
The rationale behind the tax provision is to curb the proliferation and unregulated
circulation of videograms including, among others, videotapes, discs, cassettes or any
technical improvement or variation thereof, have greatly prejudiced the operations of
movie houses and theaters. Such unregulated circulation have caused a sharp decline
in theatrical attendance by at least forty percent (40%) and a tremendous drop in the
collection of sales, contractor's specific, amusement and other taxes, thereby resulting
in substantial losses estimated at P450 Million annually in government revenues.
Videogram(s) establishments collectively earn around P600 Million per annum from
rentals, sales and disposition of videograms, and these earnings have not been
subjected to tax, thereby depriving the Government of approximately P180 Million in
taxes each year.
The unregulated activities of videogram establishments have also affected the viability
of the movie industry.
Issue:
Whether or not tax imposed by the DECREE is a valid exercise of police power.
Page 34

Whether or nor the DECREE is constitutional .


Held:
Taxation has been made the implement of the state's police power. The levy of the 30%
tax is for a public purpose. It was imposed primarily to answer the need for regulating
the video industry, particularly because of the rampant film piracy, the flagrant
violation of intellectual property rights, and the proliferation of pornographic video
tapes. And while it was also an objective of the DECREE to protect the movie industry,
the tax remains a valid imposition.
We find no clear violation of the Constitution which would justify us in pronouncing
Presidential Decree No. 1987 as unconstitutional and void. While the underlying
objective of the DECREE is to protect the moribund movie industry, there is no question
that public welfare is at bottom of its enactment, considering "the unfair competition
posed by rampant film piracy; the erosion of the moral fiber of the viewing public
brought about by the availability of unclassified and unreviewed video tapes containing
pornographic films and films with brutally violent sequences; and losses in government
revenues due to the drop in theatrical attendance, not to mention the fact that the
activities of video establishments are virtually untaxed since mere payment of Mayor's
permit and municipal license fees are required to engage in business."
WHEREFORE, the instant Petition is hereby dismissed. No costs.

ASSO. OF SMALL LANDOWNERS VS. SEC. OF DAR


[175 SCRA 343; G.R. NO. L-78742; 14 JUL 1989]
Facts:
Several petitions are the root of the case:
a. A petition alleging the constitutionality of PD No. 27, EO 228 and 229 and RA
6657. Subjects of the petition are a 9-hectare and 5 hectare Riceland worked
by four tenants. Tenants were declared full owners by EO 228 as qualified
farmers under PD 27. The petitioners now contend that President Aquino
usurped the legislatures power.
b. A petition by landowners and sugarplanters in Victorias Mill Negros
Occidental against Proclamation 131 and EO 229. Proclamation 131 is the
creation of Agrarian Reform Fund with initial fund of P50Billion.

Page 35

c. A petition by owners of land which was placed by the DAR under the coverage
of Operation Land Transfer.
d. A petition invoking the right of retention under PD 27 to owners of rice and
corn lands not exceeding seven hectares.
Issue:
Whether or Not the aforementioned EOs, PD, and RA were constitutional.
Held:
The promulgation of PD 27 by President Marcos was valid in exercise of Police power
and eminent domain.
The power of President Aquino to promulgate Proc. 131 and EO 228 and 229 was
authorized under Sec. 6 of the Transitory Provisions of the 1987 Constitution. Therefore
it is a valid exercise of Police Power and Eminent Domain.
RA 6657 is likewise valid. The carrying out of the regulation under CARP becomes
necessary to deprive owners of whatever lands they may own in excess of the
maximum area allowed, there is definitely a taking under the power of eminent domain
for which payment of just compensation is imperative. The taking contemplated is not a
mere limitation of the use of the land. What is required is the surrender of the title and
the physical possession of said excess and all beneficial rights accruing to the owner in
favour of the farmer.
A statute may be sustained under the police power only if there is concurrence of the
lawful subject and the method.
Subject and purpose of the Agrarian Reform Law is valid, however what is to be
determined is the method employed to achieve it.

LOZANO VS. MARTINEZ


[146 SCRA 323; G.R. No. L-63419; 18 Dec 1986]
Facts:
A motion to quash the charge against the petitioners for violation of the BP 22 was
made, contending that no offense was committed, as the statute is unconstitutional.
Such motion was denied by the RTC. The petitioners thus elevate the case to the
Page 36

Supreme Court for relief. The Solicitor General, commented that it was premature for
the accused to elevate to the Supreme Court the orders denying their motions to
quash. However, the Supreme Court finds it justifiable to intervene for the review of
lower court's denial of a motion to quash.
Issue:
Whether or not BP 22 is constitutional as it is a proper exercise of police power of the
State.
Held:
The enactment of BP 22 a valid exercise of the police power and is not repugnant to the
constitutional inhibition against imprisonment for debt.
The offense punished by BP 22 is the act of making and issuing a worthless check or a
check that is dishonored upon its presentation for payment. It is not the non-payment
of an obligation which the law punishes. The law is not intended or designed to coerce
a debtor to pay his debt.
The law punishes the act not as an offense against property, but an offense against
public order. The thrust of the law is to prohibit, under pain of penal sanctions, the
making of worthless checks and putting them in circulation. An act may not be
considered by society as inherently wrong, hence, not malum in se but because of the
harm that it inflicts on the community, it can be outlawed and criminally punished as
malum prohibitum. The state can do this in the exercise of its police power.

KWONG SING VS. CITY OF MANILA


[41 Phil 103; G.R. No. 15972; 11 Oct 1920]
Facts:
Kwong Sing, in his own behalf and of other Chinese laundrymen who has general and
the same interest, filed a complaint for a preliminary injunction. The Plaintiffs also
questioned the validity of enforcing Ordinance No. 532 by the city of Manila. Ordinance
No. 532 requires that the receipt be in duplicate in English and Spanish duly signed
showing the kind and number of articles delivered by laundries and dyeing and
cleaning establishments. The permanent injunction was denied by the trial court. The
appellants claim is that Ordinance No. 532 savors of class legislation; putting in mind
that they are Chinese nationals. It unjustly discriminates between persons in similar
circumstances; and that it constitutes an arbitrary infringement of property rights.
Page 37

They also contest that the enforcement of the legislation is an act beyond the scope of
their police power. In view of the foregoing, this is an appeal with the Supreme Court.
Issue:
Whether or Not the enforcement of Ordinance no, 532 is an act beyond the scope of
police power
Whether or Not the enforcement of the same is a class legislation that infringes
property rights.

Held:
Reasonable restraints of a lawful business for such purposes are permissible under the
police power. The police power of the City of Manila to enact Ordinance No. 532 is
based on
Section 2444, paragraphs (l) and (ee) of the Administrative Code, as
amended by Act No. 2744, authorizes the municipal board of the city of Manila, with the
approval of the mayor of the city:
(l) To regulate and fix the amount of the license fees for the following:
xxxx xxxxxlaundries xxxx.
(ee) To enact all ordinances it may deem necessary and proper for the
sanitation and safety, the furtherance of the prosperity, and the promotion
of the morality, peace, good order, comfort, convenience, and general
welfare of the city and its inhabitants.
The court held that the obvious purpose of Ordinance No. 532 was to avoid disputes
between laundrymen and their patrons and to protect customers of laundries who are
not able to decipher Chinese characters from being defrauded. (Considering that in the
year 1920s, people of Manila are more familiar with Spanish and maybe English.)
In whether the ordinance is class legislation, the court held that the ordinance invades
no fundamental right, and impairs no personal privilege. Under the guise of police
regulation, an attempt is not made to violate personal property rights. The ordinance is
neither discriminatory nor unreasonable in its operation. It applies to all public
laundries without distinction, whether they belong to Americans, Filipinos, Chinese, or
any other nationality. All, without exception, and each every one of them without
distinction, must comply with the ordinance.
The obvious objection for the
implementation of the ordinance is based in sec2444 (ee) of the Administrative Code.
Although, an additional burden will be imposed on the business and occupation
affected by the ordinance such as that of the appellant by learning even a few words in
Page 38

Spanish or English, but mostly Arabic numbers in order to properly issue a receipt, it
seems that the same burdens are cast upon the them. Yet, even if private rights of
person or property are subjected to restraint, and even if loss will result to individuals
from the enforcement of the ordinance, this is not sufficient ground for failing to uphold
the power of the legislative body. The very foundation of the police power is the control
of private interests for the public welfare.
Finding that the ordinance is valid, judgment is affirmed, and the petition for a
preliminary injunction is denied, with costs against the appellants.

TABLARIN VS. GUTIERREZ


[152 SCRA 730; G.R. No. 78164; 31 July 1987]
Facts:
The petitioners sought to enjoin the Secretary of Education, Culture and Sports, the
Board of Medical Education and the Center for Educational Measurement from enforcing
Section 5 (a) and (f) of Republic Act No. 2382, as amended, and MECS Order No. 52,
series of 1985, dated 23 August 1985 and from requiring the taking and passing of the
NMAT as a condition for securing certificates of eligibility for admission, from
proceeding with accepting applications for taking the NMAT and from administering the
NMAT as scheduled on 26 April 1987 and in the future. The trial court denied said
petition on 20 April 1987. The NMAT was conducted and administered as previously
scheduled.
Republic Act 2382, as amended by Republic Acts Nos. 4224 and 5946, known as the
"Medical Act of 1959" defines its basic objectives in the following manner:
"SECTION 1. Objectives. This Act provides for and shall govern (a) the
standardization and regulation of medical education; (b) the examination
for registration of physicians; and (c) the supervision, control and
regulation of the practice of medicine in the Philippines."
The statute, among other things, created a Board of Medical Education. Its functions as
specified in Section 5 of the statute include the following:
"(a) To determine and prescribe requirements for admission into a
recognized college of medicine;
x x x

Page 39

(f)
To accept applications for certification for admission to a medical
school and keep a register of those issued said certificate; and to collect
from said applicants the amount of twenty-five pesos each which shall
accrue to the operating fund of the Board of Medical Education;
Section 7 prescribes certain minimum requirements for applicants to medical schools:
"Admission requirements. The medical college may admit any student
who has not been convicted by any court of competent jurisdiction of any
offense involving moral turpitude and who presents (a) a record of
completion of a bachelor's degree in science or arts; (b) a certificate of
eligibility for entrance to a medical school from the Board of Medical
Education; (c) a certificate of good moral character issued by two former
professors in the college of liberal arts; and (d) birth certificate. Nothing in
this act shall be construed to inhibit any college of medicine from
establishing, in addition to the preceding, other entrance requirements
that may be deemed admissible.
MECS Order No. 52, s. 1985, issued by the then Minister of Education, Culture and
Sports and dated 23 August 1985, established a uniform admission test called the
National Medical Admission Test (NMAT) as an additional requirement for issuance of a
certificate of eligibility for admission into medical schools of the Philippines, beginning
with the school year 1986-1987. This Order goes on to state that: "2. The NMAT, an
aptitude test, is considered as an instrument toward upgrading the selection of
applicants for admission into the medical schools and its calculated to improve the
quality of medical education in the country. The cutoff score for the successful
applicants, based on the scores on the NMAT, shall be determined every year by the
Board of Medical Education after consultation with the Association of Philippine Medical
Colleges. The NMAT rating of each applicant, together with the other admission
requirements as presently called for under existing rules, shall serve as a basis for the
issuance of the prescribed certificate of eligibility for admission into the medical
colleges.
Issue:
Whether or not Section 5 (a) and (f) of Republic Act No. 2382, as amended, and MECS
Order No. 52, s. 1985 are constitutional.
Held:
Yes. We conclude that prescribing the NMAT and requiring certain minimum scores
therein as a condition for admission to medical schools in the Philippines, do not
constitute an unconstitutional imposition.

Page 40

The police power, it is commonplace learning, is the pervasive and non-waivable power
and authority of the sovereign to secure and promote all the important interests and
needs in a word, the public order of the general community. An important
component of that public order is the health and physical safety and well being of the
population, the securing of which no one can deny is a legitimate objective of
governmental effort and regulation. Perhaps the only issue that needs some
consideration is whether there is some reasonable relation between the prescribing of
passing the NMAT as a condition for admission to medical school on the one hand, and
the securing of the health and safety of the general community, on the other hand. This
question is perhaps most usefully approached by recalling that the regulation of the
practice of medicine in all its branches has long been recognized as a reasonable
method of protecting the health and safety of the public.
MECS Order No. 52, s. 1985 articulates the rationale of regulation of this type: the
improvement of the professional and technical quality of the graduates of medical
schools, by upgrading the quality of those admitted to the student body of the medical
schools. That upgrading is sought by selectivity in the process of admission, selectivity
consisting, among other things, of limiting admission to those who exhibit in the
required degree the aptitude for medical studies and eventually for medical practice.
The need to maintain, and the difficulties of maintaining, high standards in our
professional schools in general, and medical schools in particular, in the current stage
of our social and economic development, are widely known. We believe that the
government is entitled to prescribe an admission test like the NMAT as a means for
achieving its stated objective of "upgrading the selection of applicants into [our]
medical schools" and of "improv[ing] the quality of medical education in the country.
We are entitled to hold that the NMAT is reasonably related to the securing of the
ultimate end of legislation and regulation in this area. That end, it is useful to recall, is
the protection of the public from the potentially deadly effects of incompetence and
ignorance in those who would undertake to treat our bodies and minds for disease or
trauma.
WHEREFORE, the Petition for Certiorari is DISMISSED and the Order of the respondent
trial court denying the petition for a writ of preliminary injunction is AFFIRMED. Costs
against petitioners.

CITY GOVERNMENT OF QUEZON CITY VS. ERICTA


[122 SCRA 759; G.R. No. L-34915; 24 Jun 1983]
Facts:
Section 9 of Ordinance No. 6118, S-64, entitled "Ordinance Regulating The
Establishment, Maintenance And Operation Of Private Memorial Type Cemetery Or
Burial Ground Within The Jurisdiction Of Quezon City And Providing Penalties For The
Violation Thereof" provides:
Page 41

Sec. 9. At least six (6) percent of the total area of the memorial park
cemetery shall be set aside for charity burial of deceased persons who are
paupers and have been residents of Quezon City for at least 5 years prior
to their death, to be determined by competent City Authorities. The area
so designated shall immediately be developed and should be open for
operation not later than six months from the date of approval of the
application.
For several years, the aforequoted section of the Ordinance was not enforced but seven
years after the enactment of the ordinance, the Quezon City Council passed a
resolution to request the City Engineer, Quezon City, to stop any further selling and/or
transaction of memorial park lots in Quezon City where the owners thereof have failed
to donate the required 6% space intended for paupers burial.
The Quezon City Engineer then notified respondent Himlayang Pilipino, Inc. in writing
that Section 9 of the ordinance would be enforced.
Respondent Himlayang Pilipino reacted by filing a petition for declaratory relief,
prohibition and mandamus with preliminary injunction seeking to annul Section 9 of the
Ordinance in question. Respondent alleged that the same is contrary to the
Constitution, the Quezon City Charter, the Local Autonomy Act, and the Revised
Administrative Code.
Issue:
Whether or Not Section 9 of the ordinance in question is a valid exercise of police
power.
Held:
Section 9 of the City ordinance in question is not a valid exercise of police power.
Section 9 cannot be justified under the power granted to Quezon City to tax, fix the
license fee, and regulate such other business, trades, and occupation as may be
established or practiced in the City.
Bill of rights states that 'no person shall be deprived of life, liberty or property without
due process of law' (Art. Ill, Section 1 subparagraph 1, Constitution). On the other
hand, there are three inherent powers of government by which the state interferes with
the property rights, namely-. (1) police power, (2) eminent domain, (3) taxation.
The police power of Quezon City is defined in sub-section 00, Sec. 12, Rep. Act 537 that
reads as follows:
Page 42

To make such further ordinance and regulations not repugnant to law as


may be necessary to carry into effect and discharge the powers and duties
conferred by this act and such as it shall deem necessary and proper to
provide for the health and safety, , and for the protection of property
therein; and enforce obedience thereto with such lawful fines or penalties
as the City Council may prescribe under the provisions of subsection (jj) of
this section.
The power to regulate does not include the power to prohibit. The power to regulate
does not include the power to confiscate. The ordinance in question not only
confiscates but also prohibits the operation of a memorial park cemetery, because
under Section 13 of said ordinance, 'Violation of the provision thereof is punishable with
a fine and/or imprisonment and that upon conviction thereof the permit to operate and
maintain a private cemetery shall be revoked or cancelled. The confiscatory clause and
the penal provision in effect deter one from operating a memorial park cemetery.
Moreover, police power is defined by Freund as 'the power of promoting the public
welfare by restraining and regulating the use of liberty and property'. It is usually
exerted in order to merely regulate the use and enjoyment of property of the owner. If
he is deprived of his property outright, it is not taken for public use but rather to
destroy in order to promote the general welfare.
It seems to the court that Section 9 of Ordinance No. 6118, Series of 1964 of Quezon
City is not a mere police regulation but an outright confiscation. It deprives a person of
his private property without due process of law, nay, even without compensation.

MMDA Vs. Bel-Air Village


[328 SCRA 836; G.R. No. 135962; 27 Mar 2000]
Facts:
Metropolitan Manila Development Authority (MMDA), petitioner herein, is a Government
Agency tasked with the delivery of basic services in Metro Manila. Bel-Air Village
Association (BAVA), respondent herein, received a letter of request from the petitioner
to open Neptune Street of Bel-Air Village for the use of the public. The said opening of
Neptune Street will be for the safe and convenient movement of persons and to
regulate the flow of traffic in Makati City. This was pursuant to MMDA law or Republic
Act No. 7924. On the same day, the respondent was appraised that the perimeter wall
separating the subdivision and Kalayaan Avenue would be demolished.
The respondent, to stop the opening of the said street and demolition of the wall, filed a
preliminary injunction and a temporary restraining order. Respondent claimed that the
Page 43

MMDA had no authority to do so and the lower court decided in favor of the
Respondent. Petitioner appealed the decision of the lower courts and claimed that it
has the authority to open Neptune Street to public traffic because it is an agent of the
State that can practice police power in the delivery of basic services in Metro Manila.

Issue:
Whether or not the MMDA has the mandate to open Neptune Street to public traffic
pursuant to its regulatory and police powers.
Held:
The Court held that the MMDA does not have the capacity to exercise police power.
Police power is primarily lodged in the National Legislature. However, police power may
be delegated to government units. Petitioner herein is a development authority and not
a political government unit. Therefore, the MMDA cannot exercise police power because
it cannot be delegated to them. It is not a legislative unit of the government. Republic
Act No. 7924 does not empower the MMDA to enact ordinances, approve resolutions
and appropriate funds for the general welfare of the inhabitants of Manila. There is no
syllable in the said act that grants MMDA police power.
It is an agency created for the purpose of laying down policies and coordinating with
various national government agencies, peoples organizations, non-governmental
organizations and the private sector for the efficient and expeditious delivery of basic
services in the vast metropolitan area.

TATEL VS. MUNICIPALITY OF VIRAC


[207 SCRA 157; G.R. No. 40243; 11 Mar 1992]
Facts:
Petitioner Celestino Tatel owns a warehouse in barrio Sta. Elena, Municipality of Virac.
Complaints were received by the municipality concerning the disturbance caused by
the operation of the abaca bailing machine inside petitioners warehouse. A committee
was then appointed by the municipal council, and it noted from its investigation on the
matter that an accidental fire within the warehouse of the petitioner created a danger
to the lives and properties of the people in the neighborhood. Resolution No. 29 was
then passed by the Municipal council declaring said warehouse as a public nuisance
within a purview of Article 694 of the New Civil Code. According to respondent
Page 44

municipal officials, petitioners warehouse was constructed in violation of Ordinance No.


13, series of 1952, prohibiting the construction of warehouses near a block of houses
either in the poblacion or barrios without maintaining the necessary distance of 200
meters from said block of houses to avoid loss of lives and properties by accidental fire.
On the other hand, petitioner contends that Ordinance No. 13 is unconstitutional.
Issue:
Whether or not petitioners warehouse is a nuisance within the meaning Article 694 of
the Civil Code
Whether or not Ordinance No. 13, series of 1952 of the Municipality of Virac is
unconstitutional and void.

Held:
The storage of abaca and copra in petitioners warehouse is a nuisance under the
provisions of Article 694 of the Civil Code. At the same time, Ordinance No. 13 was
passed by the Municipal Council of Virac in the exercise of its police power. It is valid
because it meets the criteria for a valid municipal ordinance: 1) must not contravene
the Constitution or any statute, 2) must not be unfair or oppressive, 3) must not be
partial or discriminatory, 4) must not prohibit but may regulate trade, 5) must be
general and consistent with public policy, and 6) must not be unreasonable. The
purpose of the said ordinance is to avoid the loss of property and life in case of fire
which is one of the primordial obligation of government. The lower court did not err in
its decision.

Page 45

THE POWER OF EMINENT DOMAIN

REPUBLIC VS. TAGLE


[299 SCRA 549; G.R. No. 129079; 2 Dec 1998]
Facts:
Private respondent Helena Z. Benitez is the registered owner of two (2) parcels of land
located in Barangay Salawag, Dasmarias, Cavite containing an area of 483,331 square
meters more or less.
The Philippine Government, through the Philippine Human Resources Development
Center (PHRDC), negotiated with the Japanese International Cooperation Agency (JICA)
Survey Team on the technicalities of the establishment of the ASEAN Human Resources
Development Project in the Philippines. Among the five (5) main programs of the
proposed project was Program III (Construction Manpower Development) which involved
the establishment of a Construction Manpower Development Center (CMDC). PHRDC
and private respondent Helena Z. Benitez, signed a Memorandum of Agreement which
provides, among others, that Benitez undertakes to lease within the period of twenty
(20) years and/or sell a portion of that property (which is no less than ten-hectares) in
favor of PHRDC which likewise agrees to lease within a period of twenty (20) years
and/or buy said property site.
The Philippine Womens University (PWU) and Benitez granted a permit to PHRDC to
occupy and use the land in question and to undertake land development, electrical and
road network installations and other related works necessary to attain its objectives.
Pursuant thereto, the CMDC took possession of the property and erected buildings and
other related facilities necessary for its operations. A deposit made by the plaintiff with
the Philippine National Bank (PNB) in the amount of P708,490.00 which is equivalent to
the assessed value of the property subject matter hereof based on defendants 1990
tax declaration, was made.
In view of the agreement on the sale of the land in question, PHRDC prepared a Deed of
Absolute Sale with Benitez, as vendor, and PHRDC and CMDC, as vendees, duly
represented by then Undersecretary Gloria M. Arroyo, for the signature of Benitez.
Benitez in her own capacity did not sign the deed of absolute sale.
Failing to acquire the property involved through negotiated sale, petitioner, through the
Department of Trade and Industry, to which CMDC is attached, instituted a complaint
for Eminent Domain, pursuant to the provisions of Executive Order No. 1035, dated
June 25, 1985.
A Motion for Issuance of Writ of Possession was granted by the court but quashed it
subsequently.
Issue:
Whether or Not the respondent judge may quash a writ of possession on the ground
that the expropriating government agency is already occupying the property sought to
Page 46

be expropriated.
Held:
No. Under Section 7 of EO 1035, when the government or its authorized agent makes
the required deposit, the trial court has a ministerial duty to issue a writ of possession.
The expropriation of real property does not include mere physical entry or occupation
of land. Although eminent domain usually involves a taking of title, there may also be
compensable taking of only some, not all, of the property interests in the bundle of
rights that constitute ownership.
In the instant case, it is manifest that the petitioner, in pursuit of an objective beneficial
to public interest, seeks to realize the same through its power of eminent domain. In
exercising this power, petitioner intended to acquire not only physical possession but
also the legal right to possess and ultimately to own the subject property. Hence, its
mere physical entry and occupation of the property fall short of the taking of title,
which includes all the rights that may be exercised by an owner over the subject
property.

CITY OF MANILA VS. CHINESE COMMUNITY


[40 Phil 349; No. 14355; 31 Oct 1919]
Facts: The City of Manila, plaintiff herein, prayed for the expropriation of a portion
private cemetery for the conversion into an extension of Rizal Avenue. Plaintiff claims
that it is necessary that such public improvement be made in the said portion of the
private cemetery and that the said lands are within their jurisdiction.
Defendants herein answered that the said expropriation was not necessary because
other routes were available. They further claimed that the expropriation of the
cemetery would create irreparable loss and injury to them and to all those persons
owing and interested in the graves and monuments that would have to be destroyed.
The lower court ruled that the said public improvement was not necessary on the
particular-strip of land in question. Plaintiff herein assailed that they have the right to
exercise the power of eminent domain and that the courts have no right to inquire and
determine the necessity of the expropriation. Thus, the same filed an appeal.
Issue:
Page 47

Whether or not the courts may inquire into, and hear proof of the necessity of the
expropriation.
Held:
The courts have the power of restricting the exercise of eminent domain to the actual
reasonable necessities of the case and for the purposes designated by the law. The
moment the municipal corporation or entity attempts to exercise the authority
conferred, it must comply with the conditions accompanying the authority. The
necessity for conferring the authority upon a municipal corporation to exercise the right
of eminent domain is admittedly within the power of the legislature. But whether or not
the municipal corporation or entity is exercising the right in a particular case under the
conditions imposed by the general authority, is a question that the courts have the
right to inquire to.

REPUBLIC VS. PLDT


[26 SCRA 320; G.R. No. L-18841; 27 Jan 1969]
Facts:
The plaintiff Republic of the Philippines is a political entity exercising government
powers through one of its branches, the Bureau of Telecommunication. Herein
defendant, PLDT is a public service corporation holding a franchise to install operates
and maintains a telephone system. After its creation, the BOT set up its own
government telephone system by utilizing its own appropriations and other equipment
and by renting trunk lines of the PLDT to enable the govt offices to call privately. BOT
entered into an agreement with the RCA communications for joint overseas telephone
service whereby BOT would convey overseas calls received by RCA to local residents.
PLDT complained to the BOT that it was a violation of the condition of their agreement
since the BOT had used trunk lines only for the use of government offices but even to
serve private persons or the general public in competition with the business of PLDT.
Subsequently, the plaintiff commenced suit against PLDT asking the court judgment be
rendered ordering the PLDT to execute a contract with the plaintiff, through the BOT for
the use of the facilities of PLDT's telephone system throughout the country under such
conditions as the court may consider reasonable. The CFI rendered judgment stating
that it could not compel PLDT to enter into such agreement. Hence this petition.
Issue:
Whether or Not PLDT may be compelled to enter into such agreement.

Page 48

Held:
Yes, the state, may, in the interest of national welfare transfer utilities to public
ownership upon payment of just compensation, there is no reason why the state ma not
require a public utility to render services in the general interest provided just
compensation is paid.

PEOPLE VS. FAJARDO


[104 Phil 443; G.R. No. L-12172; 29 Aug 1958]
Facts:
The municipal council of baao, camarines sur stating among others that construction of
a building, which will destroy the view of the plaza, shall not be allowed and therefore
be destroyed at the expense of the owner, enacted an ordinance. Herein appellant filed
a written request with the incumbent municipal mayor for a permit to construct a
building adjacent to their gasoline station on a parcel of land registered in Fajardo's
name, located along the national highway and separated from the public plaza by a
creek. The request was denied, for the reason among others that the proposed building
would destroy the view or beauty of the public plaza. Defendants reiterated their
request for a building permit, but again the mayor turned down the request.
Whereupon, appellants proceeded with the construction of the building without a
permit, because they needed a place of residence very badly, their former house
having been destroyed by a typhoon and hitherto they had been living on leased
property. Thereafter, defendants were charged in violation of the ordinance and
subsequently convicted. Hence this appeal.
Issue:
Whether or Not the ordinance is a valid exercise of police power.
Held:
No. It is not a valid exercise of police power. The ordinance is unreasonable and
oppressive, in that it operates to permanently deprive appellants of the right to use
their own property; hence, it oversteps the bounds of police power, and amounts to a
taking of appellants property without just compensation. We do not overlook that the
modern tendency is to regard the beautification of neighborhoods as conducive to the
comfort and happiness of residents.

Page 49

As the case now stands, every structure that may be erected on appellants' land,
regardless of its own beauty, stands condemned under the ordinance in question,
because it would interfere with the view of the public plaza from the highway. The
appellants would, in effect, be constrained to let their land remain idle and unused for
the obvious purpose for which it is best suited, being urban in character. To legally
achieve that result, the municipality must give appellants just compensation and an
opportunity to be heard.

CITY OF BAGUIO V. NAWASA


[106 Phil; G.R. No. L-12032; 31 Aug 1959]
Facts:
Plaintiff a municipal corporation filed a complaint against defendant a public
corporation, created under Act.1383. It contends that the said act does not include
within its purview the Baguio Water Works system, assuming that it does, is
unconstitutional because it deprives the plaintiff ownership, control and operation of
said water works without just compensation and due process of law. The defendant filed
a motion to dismiss ion the ground that it is not a proper exercise of police power and
eminent domain. The court denied the motion and ordered the defendants to file an
answer. The court holds that the water works system of Baguio belongs to private
property and cannot be expropriated without just compensation. Sec. 8 of R.A.1383
provides for the exchange of the NAWASA assets for the value of the water works
system of Baguio is unconstitutional for this is not just compensation. Defendants
motion for reconsideration was denied hence this appeal.
Issue:
Whether or Not there is a valid exercise of police power of eminent domain.
Held:
R.A. 1383 does not constitute a valid exercise of police power. The act does not
confiscate, destroy or appropriate property belonging to a municipal corporation. It
merely directs that all water works belonging to cities, municipalities and municipal
districts in the Philippines to be transferred to the NAWASA. The purpose is placing
them under the control and supervision of an agency with a view to promoting their
efficient management, but in so doing does not confiscate them because it directs that
they be paid with equal value of the assets of NAWASA.
The Baguio water works system is not like a public road, the park, street other public
property held in trust by a municipal corporation for the benefit of the public. But it is a
Page 50

property of a municipal corporation, water works cannot be taken away except for
public use and upon payment of just compensation. Judgment affirmed.

NATIONAL POWER CORP. VS. GUTIERREZ


[193 SCRA 1; G.R. No. 60077; 18 Jan 1991]
Facts:
Petitioner filed an action to acquire a right of way over the land of Respondents for the
construction of transmission lines. Petitioner was adjudged to pay the full market value
of land traversed by the transmission lines. Petitioner argued that it was only asking for
a right of way.
Issue:
Whether or Not the acquisition of the right of way constitutes "taking" and such the
case will be entitled just compensation.
Held:
The acquisition of the right of way constitutes taking. It perpetually deprives
Respondents of their proprietary rights. No plant higher than three meters is allowed
below the transmission lines. Because of high tension current conveyed through the
transmission lines, danger to life and limbs cannot be discounted. The owner of the
property is entitled to just compensation.

REPUBLIC VS. CASTELVI


[58 SCRA 336; G.R. No. L-20620; 15 Aug 1974]
Facts:
In 1947, the republic, through the Armed Forces of the Philippines (AFP), entered into a
lease agreement with Castelvi on a year-to-year basis. When Castelvi gave notice to
terminate the lease in 1956, the AFP refused. She then instituted an ejectment
proceeding against the AFP. In 1959, however, the republic commenced the
expropriation proceedings for the land in question.
Issue:
Page 51

Whether or Not the compensation should be determined as of 1947 or 1959.


Held:
The Supreme Court ruled that the taking should not be reckoned as of 1947, and that
just compensation should not be determined on the basis of the value of the property
as of that year.
The requisites for taking are: 1) the expropriator must enter a private property, 2) the
entry must be for more than a momentary period, 3) it must be under warrant or color
of authorities, 4) the property must be devoted for public use or otherwise informally
appropriated or injuriously affected, and 5) the utilization of the property for public use
must be such a way as to oust the owner and deprive him of beneficial enjoyment of
the property. Under Sec. 4 Rule 67 of the Rules of Court, just compensation is to be
determined as of the date of the filing of the complaint. The Supreme Court has ruled
that when the taking of the property sought to be expropriated coincides with the
commencement of the expropriation proceedings, or takes place subsequent to the
filing of the complaint for eminent domain, the just compensation should be
determined as of the date of the filing of the complaint. In the instant case, it is
undisputed that the Republic was placed in possession of the Castelvi property, by
authority of court, on August 10, 1959. The taking of the Castelvi property for the
purposes of determining the just compensation to be paid must, therefore, be reckoned
as of June 26, 1959 when the complaint for eminent domain was filed. There is no basis
to the contention of the Republic that a lease on a year-to-year basis can give rise to
permanent right to occupy since by express provision a lease made for a determinate
time, as was the lease of Castelvi land in the instant case, ceases upon the day fixed,
without need of a demand (Art. 1669, New Civil Code). The Supreme Court, however,
did not apply Art. 1250 of the New Civil Code for the adjustment of the peso rate in
times of extraordinary inflation or deflation because in eminent domain cases the
obligation to pay arises from law independent of contract.

EPZA VS. DULAY


[148 SCRA 305; G.R. No. L-59603; 29 Apr 1987]
Facts:
The four parcels of land which are the subject of this case is where the Mactan Export
Processing Zone Authority in Cebu (EPZA) is to be constructed. Private respondent San
Antonio Development Corporation (San Antonio, for brevity), in which these lands are
registered under, claimed that the lands were expropriated to the government without
them reaching the agreement as to the compensation. Respondent Judge Dulay then
issued an order for the appointment of the commissioners to determine the just
compensation. It was later found out that the payment of the government to San
Page 52

Antonio would be P15 per square meter, which was objected to by the latter contending
that under PD 1533, the basis of just compensation shall be fair and according to the
fair market value declared by the owner of the property sought to be expropriated, or
by the assessor, whichever is lower. Such objection and the subsequent Motion for
Reconsideration were denied and hearing was set for the reception of the
commissioners report. EPZA then filed this petition for certiorari and mandamus
enjoining the respondent from further hearing the case.
Issue:
Whether or Not the exclusive and mandatory mode of determining just compensation in
PD 1533 is unconstitutional.
Held:
The Supreme Court ruled that the mode of determination of just compensation in PD
1533 is unconstitutional.
The method of ascertaining just compensation constitutes impermissible encroachment
to judicial prerogatives. It tends to render the courts inutile in a matter in which under
the Constitution is reserved to it for financial determination. The valuation in the decree
may only serve as guiding principle or one of the factors in determining just
compensation, but it may not substitute the courts own judgment as to what amount
should be awarded and how to arrive at such amount. The determination of just
compensation is a judicial function. The executive department or the legislature may
make the initial determination but when a party claims a violation of the guarantee in
the Bill of Rights that the private party may not be taken for public use without just
compensation, no statute, decree, or executive order can mandate that its own
determination shall prevail over the courts findings. Much less can the courts be
precluded from looking into the justness of the decreed compensation.

AMIGABLE VS. CUENCA


[43 SCRA 360; G.R. No. L-26400; 29 Feb. 1972]
Facts:
Victoria Amigable is the registered owner of a particular lot. At the back of her Transfer
Certificate of Title (1924), there was no annotation in favor of the government of any
right or interest in the property. Without prior expropriation or negotiated sale, the
government used a portion of the lot for the construction of the Mango and Gorordo
Avenues. On 1958, Amigables counsel wrote the President of the Philippines,
requesting payment of the portion of the said lot. It was disallowed by the Auditor
Page 53

General in his 9th Endorsement. Petitioner then filed in the court a quo a complaint
against the Republic of the Philippines and Nicolas Cuenca, in his capacity as
Commissioner of Public Highways for the recovery of ownership and possession of the
lot. According to the defendants, the action was premature because it was not filed first
at the Office of the Auditor General. According to them, the right of action for the
recovery of any amount had already prescribed, that the Government had not given its
consent to be sued, and that plaintiff had no cause of action against the defendants.

Issue:
Whether or Not, under the facts of the case, appellant may properly sue the
government.
Held:
In the case of Ministerio v. Court of First Instance of Cebu, it was held that when the
government takes away property from a private landowner for public use without going
through the legal process of expropriation or negotiated sale, the aggrieved party may
properly maintain a suit against the government without violating the doctrine of
governmental immunity from suit without its consent. In the case at bar, since no
annotation in favor of the government appears at the back of the certificate of title and
plaintiff has not executed any deed of conveyance of any portion of the lot to the
government, then she remains the owner of the lot. She could then bring an action to
recover possession of the land anytime, because possession is one of the attributes of
ownership. However, since such action is not feasible at this time since the lot has been
used for other purposes, the only relief left is for the government to make due
compensationprice or value of the lot at the time of the taking.

PHILIPPINE PRESS INSTITUTE VS. COMELEC


[244 SCRA 272; G.R. No. 119694; 22 May 1995]

Facts:
Respondent Comelec promulgated Resolution No. 2772 directing newspapers to provide
free Comelec space of not less than one-half page for the common use of political
parties and candidates. The Comelec space shall be allocated by the Commission, free
of charge, among all candidates to enable them to make known their qualifications,
their stand on public Issue and their platforms of government. The Comelec space shall
also be used by the Commission for dissemination of vital election information.

Page 54

Petitioner Philippine Press Institute, Inc. (PPI), a non-profit organization of newspaper


and magazine publishers, asks the Supreme Court to declare Comelec Resolution No.
2772 unconstitutional and void on the ground that it violates the prohibition imposed
by the Constitution upon the government against the taking of private property for
public use without just compensation. On behalf of the respondent Comelec, the
Solicitor General claimed that the Resolution is a permissible exercise of the power of
supervision (police power) of the Comelec over the information operations of print
media enterprises during the election period to safeguard and ensure a fair, impartial
and credible election.
Issue:
Whether or not Comelec Resolution No. 2772 is unconstitutional.

Held:
The Supreme Court declared the Resolution as unconstitutional. It held that to compel
print media companies to donate Comelec space amounts to taking of private
personal property without payment of the just compensation required in expropriation
cases. Moreover, the element of necessity for the taking has not been established by
respondent Comelec, considering that the newspapers were not unwilling to sell
advertising space. The taking of private property for public use is authorized by the
constitution, but not without payment of just compensation. Also Resolution No. 2772
does not constitute a valid exercise of the police power of the state. In the case at
bench, there is no showing of existence of a national emergency to take private
property of newspaper or magazine publishers.

REYES VS. NATIONAL HOUSING AUTHORITY


[395 SCRA 494; GR NO. 147511; 20 JAN 2003]

Facts:
Respondent National Housing Authority (NHA) filed complaints for the expropriation of
sugarcane lands belonging to the petitioners. The stated public purpose of the
expropriation was the expansion of the Dasmarias Resettlement Project to
accommodate the squatters who were relocated from the Metropolitan Manila area. The
trial court rendered judgment ordering the expropriation of these lots and the payment
of just compensation. The Supreme Court affirmed the judgment of the lower court.
A few years later, petitioners contended that respondent NHA violated the stated public
purpose for the expansion of the Dasmarias Resettlement Project when it failed to
Page 55

relocate the squatters from the Metro Manila area, as borne out by the ocular
inspection conducted by the trial court which showed that most of the expropriated
properties remain unoccupied. Petitioners likewise question the public nature of the use
by respondent NHA when it entered into a contract for the construction of low cost
housing units, which is allegedly different from the stated public purpose in the
expropriation proceedings. Hence, it is claimed that respondent NHA has forfeited its
rights and interests by virtue of the expropriation judgment and the expropriated
properties should now be returned to herein petitioners.
Issue:
Whether or not the judgment of expropriation was forfeited in the light of the failure of
respondent NHA to use the expropriated property for the intended purpose but for a
totally different purpose.
Held:
The Supreme Court held in favor of the respondent NHA. Accordingly, petitioners
cannot insist on a restrictive view of the eminent domain provision of the Constitution
by contending that the contract for low cost housing is a deviation from the stated
public use. It is now settled doctrine that the concept of public use is no longer limited
to traditional purposes. The term "public use" has now been held to be synonymous
with "public interest," "public benefit," "public welfare," and "public convenience." Thus,
whatever may be beneficially employed for the general welfare satisfies the
requirement of public use."
In addition, the expropriation of private land for slum clearance and urban development
is for a public purpose even if the developed area is later sold to private homeowners,
commercials firms, entertainment and service companies, and other private concerns.
Moreover, the Constitution itself allows the State to undertake, for the common good
and in cooperation with the private sector, a continuing program of urban land reform
and housing which will make at affordable cost decent housing and basic services to
underprivileged and homeless citizens in urban centers and resettlement areas. The
expropriation of private property for the purpose of socialized housing for the
marginalized sector is in furtherance of social justice.

MUNICIPALITY OF PARAAQUE VS. VM REALTY CORPORATION


[292 SCRA 676; G. R. NO. 127820; 20 JUL 1998]
Facts:
Petitioner sought to exercise its power of eminent domain based on a resolution by the
municipal council. Petitioner cites a previous case wherein a resolution gave authority
to exercise eminent domain. Petitioner also relies on the Implementing Rules, which
provides that a resolution authorizes a Local Government Unit to exercise eminent
domain.

Page 56

Issue:
Whether or Not an LGU can exercise its power of eminent domain pursuant to a
resolution by its law-making body.
Held:
Under Section 19, of the present Local Government Code (RA 7160), it is stated as the
first requisite that LGUs can exercise its power of eminent domain if there is an
ordinance enacted by its legislative body enabling the municipal chief executive. A
resolution is not an ordinance, the former is only an opinion of a law-making body, the
latter is a law. The case cited by Petitioner involves BP 337, which was the previous
Local Government Code, which is obviously no longer in effect. RA 7160 prevails over
the Implementing Rules, the former being the law itself and the latter only an
administrative rule which cannot amend the former.

ASLP VS. SEC. OF AGRARIAN REFORM


[175 SCRA 343; G.R. NO. 78742; 14 JUL 1989]
Facts:
Several petitions are the root of the case:
e. A petition alleging the constitutionality of PD No. 27, EO 228 and 229 and RA
6657. Subjects of the petition are a 9-hectare and 5 hectare Riceland worked
by four tenants. Tenants were declared full owners by EO 228 as qualified
farmers under PD 27. The petitioners now contend that President Aquino
usurped the legislatures power.
f.

A petition by landowners and sugarplanters in Victorias Mill Negros


Occidental against Proclamation 131 and EO 229. Proclamation 131 is the
creation of Agrarian Reform Fund with initial fund of P50Billion.

g. A petition by owners of land which was placed by the DAR under the coverage
of Operation Land Transfer.
h. A petition invoking the right of retention under PD 27 to owners of rice and
corn lands not exceeding seven hectares.
Issue:
Page 57

Whether or Not the aforementioned EOs, PD, and RA were constitutional.


Held:
The promulgation of PD 27 by President Marcos was valid in exercise of Police power
and eminent domain.
The power of President Aquino to promulgate Proc. 131 and EO 228 and 229 was
authorized under Sec. 6 of the Transitory Provisions of the 1987 Constitution. Therefore
it is a valid exercise of Police Power and Eminent Domain.
RA 6657 is likewise valid. The carrying out of the regulation under CARP becomes
necessary to deprive owners of whatever lands they may own in excess of the
maximum area allowed, there is definitely a taking under the power of eminent domain
for which payment of just compensation is imperative. The taking contemplated is not a
mere limitation of the use of the land. What is required is the surrender of the title and
the physical possession of said excess and all beneficial rights accruing to the owner in
favour of the farmer.
A statute may be sustained under the police power only if there is concurrence of the
lawful subject and the method.
Subject and purpose of the Agrarian Reform Law is valid, however what is to be
determined is the method employed to achieve it.

ESLABAN VS. ONORIO


[360 SCRA 230; G.R. NO. 146062; 28 JUN 2001]
Facts:
Clarita Vda. De Onorio is the owner of the land in Barangay M. Roxas, Sto. Nino, South
Cotabato. Such land is the subject for the construction of an irrigation canal of the
National Irrigation Administration (NIA). Mr. Santiago Eslaban Jr. is the project manager
of NIA. The parties agreed to the construction of the canal provided that the
government will pay for the area that has been taken. A right-of-way agreement was
entered into by the parties in which respondent was paid the amount of P4, 180.00 as
right of way damages. Subsequently, respondent executed an Affidavit of Waiver of
Rights and Fees which waives her rights for the damage to the crops due to
construction of the right of way. After which, respondent demands that petitioner pay
P111, 299.55 for taking her property but the petitioner refused. Petitioner states that
Page 58

the government had not consented to be sued and that the respondent is not entitled
for compensation by virtue of the homestead patent under CA no. 141. The RTC held
that the NIA should pay respondent the amount of P107, 517.60 as just compensation
for the 24,660 sq meters that have been used for the construction of the canal. The
Court of Appeals also affirmed the decision of the RTC.
Issue:
Whether or Not the CA erred in affirming the decision of the RTC.
Held:
The CA is correct in affirming the decision of the RTC but modifications shall be made
regarding the value of the just compensation. The following are the points to be
considered in arriving in this decision.
First, Rule 7 par 5 of the Rule of Civil Procedure provides that the certification against
forum shopping should only be executed by the plaintiff or the principal. The petition
for review was filed by Mr. Eslaban jr. while the verification or certification were signed
by Mr. Cesar Gonzales, an administrator of the agency. Neither of the two has the
authority to sign such certificate for they are not the plaintiff or principal. Such case is a
sufficient ground for dismissing this petition.
Second, PD NO. 1529 provides that the owner is required to recognize in favor of the
government the easement of a public highway, way, private way established by law,
or any government canal where the certificate of title does not state that the
boundaries thereof have been pre-determined. In the case at bar, the irrigation canal
was constructed on Oct 1981 after the property had been registered in May of 1976. In
this case, prior expropriation proceedings must be filed and just compensation shall be
paid to the owner before the land could be taken for public use.
Third, In this case, just compensation is defined as not only the correct amount to be
paid but the reasonable time for the Government to pay the owner. The CA erred in this
point by stating that the market value (just compensation) of the land is determined in
the filing of the complaint in 1991.The determination of such value should be from the
time of its taking by the NIA in 1981.
Lastly, the petitioner cannot argue that the Affidavit of waiver of rights and fees
executed by the respondent pertains to the payment of the value of the land therefore
exempting NIA to pay the value of the land taken. Such waiver pertains only to the
crops and improvements that were damage due to the construction of the right-of-way
not the value of the land.

Page 59

Wherefore, decision of CA affirmed with modification regarding the just compensation


in the amount of P16, 047.61 per hectare.

KNECHT VS. COURT OF APPEALS


[290 SCRA 223; G.R. NO. 108015, 20 MAY 1998]
Facts:
The instant case is an unending sequel to several suits commenced almost twenty
years ago involving a parcel of land located at the corner of the south end of EDSA
and F.B. Harrison in Pasay City. The land was owned by petitioners Cristina de Knecht
and her son, Rene Knecht. On the land, the Knechts constructed eight houses, leased
out the seven and occupied one of them as their residence. In 1979, the government
filed for the expropriation of Knechts property. The government wanted to use the land
for the completion of the Manila Flood Control and Drainage Project and the extension
of the EDSA towards Roxas Boulevard. In 1982, the City Treasurer of Pasay discovered
that the Knechts failed to pay real estate taxes on the property from 1980 to 1982. As
a consequence of this deficiency, the City Treasurer sold the property at public auction
for the same amount of their deficiency taxes. The highest bidders were respondent
Spouses Anastacio and Felisa Babiera (the Babieras) and respondent Spouses Alejandro
and Flor Sangalang (the Sangalangs). Subsequently, Sangalang and Babiera sold the
land to respondent Salem Investment Corporation. On February 17, 1983, the Batasang
Pambansa passed B.P. Blg. 340 authorizing the national government to expropriate
certain properties in Pasay City for the EDSA Extension. The property of the Knechts
was part of those expropriated under B.P. Blg. 340. The government gave out just
compensation for the lands expropriated under B.P. Blg. 340. Salem was included and
received partial payment. Seven of the eight houses of the Knechts were demolished
and the government took possession of the portion of land on which the houses stood.
Since the Knechts refused to vacate their one remaining house, Salem filed a case
against them for unlawful detainer. As defense, the Knechts claimed ownership of the
land and building. The Municipal Trial Court however ordered the Knechts' ejectment
thus their residence was demolished.
The Knechts continuously claimed ownership of the property and allege that they must
be given just compensation.
Issue:
Whether or not Knechts are the lawful owners of the land at subject.
Held:

Page 60

The Supreme Court held that the Knechts were not the owners anymore of the said
land. The Knechts' right to the land had been foreclosed after they failed to redeem it
one year after the sale at public auction. Since the petitions questioning the order of
dismissal were likewise dismissed by the Court of Appeals and this Court, the order of
dismissal became final and res judicata on the issue of ownership of the land.
Petitioners contended that they did not receive notice of their tax delinquency. Neither
did they receive notice of the auction sale. However, this question has been previously
raised in the cases which have been already set aside. The court is not a trier of facts.
Res judicata has already set it. The Knechts therefore are not the lawful owners of the
land and are not any longer accountable for just compensation given by the
government.
Note: Res judicata is a ground for dismissal of an action. It is a rule that precludes
parties from relitigating Issue actually litigated and determined by a prior and final
judgment. It pervades every well-regulated system of jurisprudence, and is based upon
two grounds embodied in various maxims of the common law one, public policy and
necessity, that there should be a limit to litigation; and another, the individual should
not be vexed twice for the same cause. When a right of fact has been judicially tried
and determined by a court of competent jurisdiction, or an opportunity for such trial
has been given, the judgment of the court, so long as it remains unreversed, should be
conclusive upon the parties and those in privity with them in law or estate. To follow a
contrary doctrine would subject the public peace and quiet to the will and neglect of
individuals and prefer the gratification of the litigious disposition of the parties to the
preservation of the public tranquility.
Res judicata applies when: (1) the former judgment or order is final; (2) the judgment or
order is one on the merits; (3) it was rendered by a court having jurisdiction over the
subject matter and the parties; (4) there is between the first and second actions,
identity of parties, of subject matter and of cause of action.

REPUBLIC VS. KER


[383 SCRA 584; G.R. NO. 136171, 2 JULY 2002]
Facts:
Petitioner filed before the Regional Trial Court of Davao City a petition for expropriation
of portions of two parcels of land owned by respondent. Petitioner needed the parcels
of land for the widening of the road component of J.P. Laurel-Buhangin Interchange in
Davao City. The Regional trial court rendered decision of a fair just compensation for
defendant Ker Corporation. However, it was challenged by Petitioner Republic of the
Philippines, represented by the Department of Public Works and Highways alleging that
just compensation for site must be reduced. Petitioner alleged that when the petition
for expropriation was filed, the tax declaration of the property indicated its assessed
value at a lower price.
Page 61

Issue:
Whether or not respondent Ker Company was given a decision for fair just
compensation.
Held:
The Supreme Court held that the valuation for the lot Sites are excessive and
unreasonable. Just compensation cannot be measured by the assessed value of the
property as stated in the tax declaration and schedule of market values. For the
purpose of appraisal, the fair market value of the property is taken into account and
such value refers to the highest price in terms of money which a property will bring if
exposed for sale in the public market.
In computing just compensation for expropriation proceedings, it is the value of the
land at the time of the taking or at the time of the filing of the complaint not at the time
of the rendition of judgment which should be taken into consideration. 4 Section 4, Rule
67 of the 1997 Rules of Civil Procedure provides that just compensation is to be
determined as of the date of the taking or the filing of the complaint whichever came
first. On this matter, the appellate court is correct in disregarding petitioner's claim.

MANOSCA VS. COURT OF APPEALS


[252 SCRA 412; G.R. NO. 106440, 29 JAN. 1996]
Facts:
The National Historical Institute declared the parcel of land owned by Petitioners as a
national historical landmark, because it was the site of the birth of Felix Manalo, the
founder of Iglesia ni Cristo. The Republic of the Philippines filed an action to appropriate
the land. Petitioners argued that the expropriation was not for a public purpose.
Issue:
Whether or Not the taking or exercise of eminent domain may be granted.
Held:
Public use should not be restricted to the traditional uses. The taking is for a public use
because of the contribution of Felix Manalo to the culture and history of the Philippines.
Page 62

Page 63

THE POWER OF TAXATION

PASCUAL VS. SEC. OF PUBLIC WORKS


[110 PHIL 331; G.R. NO.L-10405; 29 DEC 1960]
Facts:
Petitioner, the governor of the Province of Rizal, filed an action for declaratory relief
with injunction on the ground that RA 920, Act appropriating funds for public works,
providing P85,000 for the construction, reconstruction, repair, extension and
improvement of Pasig feeder road terminals, were nothing but projected and planned
subdivision roads within Antonio Subdivision. Antonio Subdivision is owned by the
respondent, Jose Zulueta, a member of the Senate of the Philippines. Respondent
offered to donate the said feeder roads to the municipality of Pasig and the offer was
accepted by the council, subject to a condition that the donor would submit plan of the
roads and an agreement to change the names of two of the street. However, the
donation was not executed, which prompted Zuleta to write a letter to the district
engineer calling attention the approval of RA 920. The district engineer, on the other
hand, did not endorse the letter that inasmuch the feeder roads in question were
private property at the time of passage and approval of RA 920, the appropriation for
the construction was illegal and therefore, void ab initio. Petitioner, prayed for RA 920
be declared null and void and the alleged deed of donation be declared
unconstitutional. Lower court dismissed the case and dissolved the writ of preliminary
injunction.
Issue:
Whether or Not the deed of donation and the appropriation of funds stipulated in RA
920 are constitutional.
Held:
The ruling case law rules that the legislature is without power to appropriate public
revenue for anything but public purpose. The taxing power must be exercised for public
purposes only and the money raised by taxation can be expended only for public
purposes and not for the advantage of private individuals.
In the case at bar, the legality of the appropriation of the feeder roads depend upon
whether the said roads were public or private property when the bill was passed by
congress or when it became effective. The land which was owned by Zulueta, the
Page 64

appropriation sought a private purpose and hence, null and void. The donation did not
cure the nullity of the appropriation; therefore a judicial nullification of a said donation
need not precede the declaration of unconstitutionality of the said appropriation.
The decision appealed from is reversed.

PUNSALAN VS. MUNICIPAL BOARD OF MANILA


[95 PHIL 46; NO.L-4817; 26 MAY 1954]
Facts:
Petitioners, who are professionals in the city, assail Ordinance No. 3398 together with
the law authorizing it (Section 18 of the Revised Charter of the City of Manila). The
ordinance imposes a municipal occupation tax on persons exercising various
professions in the city and penalizes non-payment of the same. The law authorizing
said ordinance empowers the Municipal Board of the city to impose a municipal
occupation tax on persons engaged in various professions. Petitioners, having already
paid their occupation tax under section 201 of the National Internal Revenue Code, paid
the tax under protest as imposed by Ordinance No. 3398. The lower court declared the
ordinance invalid and affirmed the validity of the law authorizing it.
Issue:
Whether or Not the ordinance and law authorizing it constitute class legislation, and
authorize what amounts to double taxation.
Held:
The Legislature may, in its discretion, select what occupations shall be taxed, and in its
discretion may tax all, or select classes of occupation for taxation, and leave others
untaxed.
It is not for the courts to judge which cities or municipalities should be
empowered to impose occupation taxes aside from that imposed by the National
Government. That matter is within the domain of political departments. The argument
against double taxation may not be invoked if one tax is imposed by the state and the
other is imposed by the city. It is widely recognized that there is nothing inherently
terrible in the requirement that taxes be exacted with respect to the same occupation
by both the state and the political subdivisions thereof. Judgment of the lower court is
reversed with regards to the ordinance and affirmed as to the law authorizing it.

Page 65

OSMEA VS. ORBOS


[220 SCRA 703; G.R. NO. 99886; 31 MAR 1993]
Facts:
On October 10, 1984, Pres. Marcos issued P.D. 1956 creating a Special Account in the
General Fund, designated as the Oil Price Stabilization Fund (OPSF). The OPSF was
designed to reimburse oil companies for cost increases in crude oil and imported
petroleum products resulting from exchange rate adjustments and from increases in
the world market prices of crude oil.
Subsequently, the OPSF was reclassified into a "trust liability account," in virtue of E.O.
1024, and ordered released from the National Treasury to the Ministry of Energy.
Pres. Aquino, amended P.D. 1956. She promulgated Executive Order No. 137 on
February 27, 1987, expanding the grounds for reimbursement to oil companies for
possible cost underrecovery incurred as a result of the reduction of domestic prices of
petroleum products, the amount of the underrecovery being left for determination by
the Ministry of Finance.
The
petition
avers
that
the
creation
of
the
trust
fund
violates
29(3), Article VI of the Constitution, reading as follows:
(3) All money collected on any tax levied for a special purpose shall be
treated as a special fund and paid out for such purposes only. If the
purpose for which a special fund was created has been fulfilled or
abandoned, the balance, if any, shall be transferred to the general funds
of the Government.
The petitioner argues that "the monies collected pursuant to . . P.D. 1956, as amended,
must be treated as a 'SPECIAL FUND,' not as a 'trust account' or a 'trust fund,' and that
"if a special tax is collected for a specific purpose, the revenue generated therefrom
shall 'be treated as a special fund' to be used only for the purpose indicated, and not
channeled to another government objective." Petitioner further points out that since "a
'special fund' consists of monies collected through the taxing power of a State, such
amounts belong to the State, although the use thereof is limited to the special
purpose/objective for which it was created."
He also contends that the "delegation of legislative authority" to the ERB violates 28
(2). Article VI of the Constitution, viz.:
(2) The Congress may, by law, authorize the President to fix, within
specified limits, and subject to such limitations and restrictions as it may
impose, tariff rates, import and export quotas, tonnage and wharfage
dues, and other duties or imposts within the framework of the national
development program of the Government;
and, inasmuch as the delegation relates to the exercise of the power of taxation, "the
limits, limitations and restrictions must be quantitative, that is, the law must not only
Page 66

specify how to tax, who (shall) be taxed (and) what the tax is for, but also impose a
specific limit on how much to tax." 12
Issue:
Whether or Not the invalidity of the "TRUST ACCOUNT" in the books of account of the
Ministry of Energy (now, the Office of Energy Affairs), created pursuant to 8,
paragraph 1, of P.D. No. 1956, as amended, "said creation of a trust fund being contrary
to Section 29 (3), Article VI of the Constitution.
Whether or Not the unconstitutionality of 8, paragraph 1 (c) of P.D. No. 1956, as
amended by Executive Order No. 137, for "being an undue and invalid delegation of
legislative power to the Energy Regulatory Board.

Held:
The OPSF is a "Trust Account" which was established "for the purpose of minimizing the
frequent price changes brought about by exchange rate adjustment and/or changes in
world market prices of crude oil and imported petroleum products." Under P.D. No.
1956, as amended by Executive Order No. 137 dated 27 February 1987, this Trust
Account may be funded from any of the following sources:
a) Any increase in the tax collection from ad valorem tax or customs duty
imposed on petroleum products subject to tax under this Decree arising from
exchange rate adjustment, as may be determined by the Minister of Finance in
consultation with the Board of Energy;
b) Any increase in the tax collection as a result of the lifting of tax exemptions of
government corporations, as may be determined by the Minister of Finance in
consultation with the Board of Energy;
c) Any additional amount to be imposed on petroleum products to augment the
resources of the Fund through an appropriate Order that may be issued by the
Board of Energy requiring payment of persons or companies engaged in the
business of importing, manufacturing and/or marketing petroleum products;
d) Any resulting peso cost differentials in case the actual peso costs paid by oil
companies in the importation of crude oil and petroleum products is less than the
peso costs computed using the reference foreign exchange rate as fixed by the
Board of Energy.
Hence, it seems clear that while the funds collected may be referred to as taxes, they
are exacted in the exercise of the police power of the State. Moreover, that the OPSF is
a special fund is plain from the special treatment given it by E.O. 137. It is segregated
from the general fund; and while it is placed in what the law refers to as a "trust liability
account," the fund nonetheless remains subject to the scrutiny and review of the COA.
Page 67

The Court is satisfied that these measures comply with the constitutional description of
a "special fund." Indeed, the practice is not without precedent.
With regard to the alleged undue delegation of legislative power, the Court finds that
the provision conferring the authority upon the ERB to impose additional amounts on
petroleum products provides a sufficient standard by which the authority must be
exercised. In addition to the general policy of the law to protect the local consumer by
stabilizing and subsidizing domestic pump rates, 8(c) of P.D. 1956 expressly
authorizes the ERB to impose additional amounts to augment the resources of the
Fund.
What petitioner would wish is the fixing of some definite, quantitative restriction, or "a
specific limit on how much to tax." The Court is cited to this requirement by the
petitioner on the premise that what is involved here is the power of taxation; but as
already discussed, this is not the case. What is here involved is not so much the power
of taxation as police power. Although the provision authorizing the ERB to impose
additional amounts could be construed to refer to the power of taxation, it cannot be
overlooked that the overriding consideration is to enable the delegate to act with
expediency in carrying out the objectives of the law which are embraced by the police
power of the State.
The interplay and constant fluctuation of the various factors involved in the
determination of the price of oil and petroleum products, and the frequently shifting
need to either augment or exhaust the Fund, do not conveniently permit the setting of
fixed or rigid parameters in the law as proposed by the petitioner. To do so would
render the ERB unable to respond effectively so as to mitigate or avoid the undesirable
consequences of such fluidity. As such, the standard as it is expressed suffices to guide
the delegate in the exercise of the delegated power, taking account of the
circumstances under which it is to be exercised.

LLADOC VS. COMMISSIONER OF INTERNAL REVENUE


[14 SCRA 292; NO.L-19201; 16 JUN 1965]
Facts:
Sometime in 1957, M.B. Estate Inc., of Bacolod City, donated 10,000.00 pesos in cash
to Fr. Crispin Ruiz, the parish priest of Victorias, Negros Occidental, and predecessor of
Fr. Lladoc, for the construction of a new Catholic church in the locality. The donated
amount was spent for such purpose.
On March 3, 1958, the donor M.B. Estate filed the donor's gift tax return. Under date of
April 29, 1960. Commissioner of Internal Revenue issued an assessment for the donee's
gift tax against the Catholic Parish of Victorias of which petitioner was the parish priest.
Page 68

Issue:
Whether or not the imposition of gift tax despite the fact the Fr. Lladoc was not the
Parish priest at the time of donation, Catholic Parish priest of Victorias did not have
juridical personality as the constitutional exemption for religious purpose is valid.
Held:
Yes, imposition of the gift tax was valid, under Section 22(3) Article VI of the
Constitution contemplates exemption only from payment of taxes assessed on such
properties as Property taxes contra distinguished from Excise taxes The imposition of
the gift tax on the property used for religious purpose is not a violation of the
Constitution. A gift tax is not a property by way of gift inter vivos.
The head of the Diocese and not the parish priest is the real party in interest in the
imposition of the donee's tax on the property donated to the church for religious
purpose.

Page 69

CASSANOVAS VS. HORD


[8 Phil 125; No. 3473; 22 Mar 1907]
Facts:
The Spanish Govt. by virtue of a royal decree granted the plaintiff certain mines. The
plaintiff is now the owner of those mines. The Collector of Internal Revenue imposed tax
on the properties, contending that they were valid perfected mine concessions and it
falls within the provisions of sec.134 of Act No. 1189 known as Internal Revenue Act.
The plaintiff paid under protest. He brought an action against the defendant Collector of
Internal Revenue to recover the sum of Php. 9, 600 paid by him as taxes. Judgment was
rendered in favor of the defendant, so the plaintiff appealed.
Issue:
Whether or Not Sec. 164 is void or valid.
Held:
The deed constituted a contract between the Spanish Government and the plaintiff. The
obligation of which contract was impaired by the enactment of sec. 134 of the Internal
Revenue Law infringing sec. 5 of the Act of Congress which provides that no law
impairing the obligation of contracts shall be enacted. Sec. 134 of the Internal
Revenue Law of 1904 is void because it impairs the obligation of contracts contained in
the concessions of mine made by the Spanish Government. Judgment reversed.

Page 70

THE BILL
OF RIGHTS

Page 71

DUE PROCESS OF LAW

Art 3, Sec. 1.
process of law

No person shall be deprived of life, liberty, or property without due

ERMITA-MALATE HOTEL AND MOTEL OPERATORS ASSO. VS. MAYOR OF MANILA


[20 SCRA 849; G.R. NO.L-24693; 31 JULY 1967]
Facts:
Petitioners Ermita-Malate Hotel and Motel Operators Association with one of its
members, Hotel del Mar Inc., and Go Chiu, the president and general manager of the
second petitioner, filed a petition for prohibition against Ordinance No. 4760 against
the respondent Mayor of the City of Manila who was sued in his capacity as such
charged with the general power and duty to enforce ordinances of the City of Manila
and to give the necessary orders for the
execution and enforcement of such
ordinances. It was alleged that the petitioner non-stock corporation is dedicated to the
promotion and protection of the interest of its eighteen members operating hotels and
motels, characterized as legitimate businesses duly licensed by both national and city
authorities and regularly paying taxes. It was alleged that on June 13, 1963, the
Municipal Board of the City of Manila enacted Ordinance No. 4760, approved on June
14, 1963 by the then acting City Mayor, Vice-Mayor Herminio Astorga. After which the
alleged grievances against the ordinance were set forth in detail. There was the
assertion of its being beyond the powers of the Municipal Board of the City of Manila to
enact insofar as it regulate motels, on the ground that in the revised charter of the City
of Manila or in any other law, no reference is made to motels. it also being provided
that the premises and facilities of such hotels, motels and lodging houses would be
open for inspection either by the City Mayor, or the Chief of Police, or their duly
authorized representatives. The lower court on July 6, 1963 issued a writ of preliminary
injunction ordering respondent Mayor to refrain from enforcing said Ordinance No. 4760
from and after July 8, 1963.
Issue:
Whether or Not Ordinance No. 4760 of the City of Manila is unconstitutional, therefore,
null and void.
Held:
A decent regard for constitutional doctrines of a fundamental character ought to have
admonished the lower court against such a sweeping condemnation of the challenged
Page 72

ordinance. Its decision cannot be allowed to stand, consistently with what has been the
accepted standards of constitutional adjudication, in both procedural and substantive
aspects.
Primarily what calls for a reversal of such a decision is the absence of any evidence to
offset the presumption of validity that attaches to a challenged statute or ordinance. As
was expressed categorically by Justice Malcolm: "The presumption is all in favor of
validity x x x . The action of the elected representatives of the people cannot be lightly
set aside. The councilors must, in the very nature of things, be familiar with the
necessities of their particular municipality and with all the facts and circumstances
which surround the subject and necessitate action. The local legislative body, by
enacting the ordinance, has in effect given notice that the regulations are essential to
the well being of the people x x x . The Judiciary should not lightly set aside legislative
action when there is not a clear invasion of personal or property rights under the guise
of police regulation.
It admits of no doubt therefore that there being a presumption of validity, the necessity
for evidence to rebut it is unavoidable, unless the statute or ordinance is void on its
face which is not the case here. The principle has been nowhere better expressed than
in the leading case of O'Gorman & Young v. Hartford Fire Insurance Co. where the
American Supreme Court through Justice Brandeis tersely and succinctly summed up
the matter thus: The statute here questioned deals with a subject clearly within the
scope of the police power. We are asked to declare it void on the ground that the
specific method of regulation prescribed is unreasonable and hence deprives the
plaintiff of due process of law. As underlying questions of fact may condition the
constitutionality of legislation of this character, the resumption of constitutionality must
prevail in the absence of some factual foundation of record for overthrowing the
statute." No such factual foundation being laid in the present case, the lower court
deciding the matter on the pleadings and the stipulation of facts, the presumption of
validity must prevail and the judgment against the ordinance set aside.

VILLEGAS VS. HIU CHIONG


[86 SCRA 270; NO.L-29646; 10 NOV 1978]
Facts:
The controverted Ordinance no. 6537 was passed by the Municipal Board of Manila on
February 22, 1968 and signed by Mayor Villegas. It is an ordinance making it unlawful
for any person not a citizen of the Philippines to be employed in any place of
employment or to be
engaged in any kind of trade business or occupation within
the city of Manila without securing an employment permit from the Mayor of Manila and
for other purposes.

Page 73

Hiu Chiong Tsai Pao Ho, who was employed in Manila filed a petition praying for the writ
of preliminary injunction and restraining order to stop the enforcement of said
ordinance.
Issue:
Whether or Not Ordinance no.6537 violates the due process and equal protection
clauses of the
Constitution.
Held:
It is a revenue measure. The city ordinance which imposes a fee of 50.00 pesos to
enable aliens generally to be employed in the city of Manila is not only for the purpose
of regulation.
While it is true that the first part which requires the alien to secure an employment
permit from the Mayor involves the exercise of discretion and judgment in processing
and approval or disapproval of application is regulatory in character, the second part
which requires the payment
of a sum of 50.00 pesos is not a regulatory but a
revenue measure.
Ordinance no. 6537 is void and unconstitutional. This is tantamount to denial of the
basic human right of the people in the Philippines to engaged in a means of livelihood.
While it is true that the Philippines as a state is not obliged to admit aliens within it's
territory, once an alien is admitted he cannot be deprived of life without due process of
law. This guarantee includes the means of livelihood. Also it does not lay down any
standard to guide the City
Mayor in the issuance or denial of an alien employment
permit fee.

NAMIL VS. COMELEC


[414 SCRA 553; G.R. NO. 150540; 28 OCT 2003]
Facts:
On May 20, 2001, the Municipal Board of Canvassers of Palimbang, Sultan Kudarat
proclaimed the petitioners as winning candidates for their Sangguniang Bayan. The
following day, herein private respondents were proclaimed winners as well. Private
respondents claimed that they should be recognized as the winners, and not the
petitioners. Upon receipt of such letter, the Commissioner-in-charge for Region XII
asked the Law Department, the Regional Election Registrar and the Provincial Elections
Supervisor to submit their reports on the matter. All of them found the second
proclamation valid. Hence, the COMELEC issued a Resolution ordering the immediate
Page 74

installation of the private respondents as the newly elected members of the


Sangguniang Bayan, even though petitioners herein have already taken their oath and
have assumed office. Petitioners contend that such Resolution is null and void because
they were not accorded due notice and hearing, hence constituting a violation of the
due process principle.
Issue:
Whether or Not due the COMELEC has the power to suspend a proclamation or the
effects thereof without notice and hearing.
Held:
No. The COMELEC is without power to partially or totally annul a proclamation or
suspend the effects of a proclamation without notice and hearing. The proclamation on
May 20, 2001 enjoys the presumption of regularity and validity since no contest or
protest was even filed assailing the same. The petitioners cannot be removed from
office without due process of law. Due process in quasi-judicial proceedings before the
COMELEC requires due notice and hearing. Furthermore, the proclamation of a winning
candidate cannot be annulled if he has not been notified of any motion to set aside his
proclamation. Hence, as ruled in Farias vs. COMELEC, Reyes vs. COMELEC and
Gallardo vs. COMELEC, the COMELEC is without power to partially or totally annul a
proclamation or suspend the effects of a proclamation without notice and hearing.
ICHONG VS. HERNANDEZ
[101 PHIL 1155; L-7995; 31 MAY 1957]
Facts:
Republic Act 1180 or commonly known as An Act to Regulate the Retail Business was
passed. The said law provides for a prohibition against foreigners as well as
corporations owned by foreigners from engaging from retail trade in our country. This
was protested by the petitioner in this case. According to him, the said law violates the
international and treaty of the Philippines therefore it is unconstitutional. Specifically,
the Treaty of Amity between the Philippines and China was violated according to him.
Issue:
Whether or Not Republic Act 1180 is a valid exercise of police power.
Held:

Page 75

According to the Court, RA 1180 is a valid exercise of police power. It was also then
provided that police power can not be bargained away through the medium of a treaty
or a contract. The Court also provided that RA 1180 was enacted to remedy a real and
actual danger to national economy posed by alien dominance and control. If ever the
law infringes upon the said treaty, the latter is always subject to qualification or
amendment by a subsequent law and the same may never curtain or restrict the scope
of the police power of the state.

PHIL. PHOSPHATE FERTILIZER CORP. VS. TORRES


[231 SCRA 335; G.R. NO.98050; 17 MAR 1994]
Facts:
Philphos Movement for Progress, Inc. (PMPI for brevity), filed with the Department of
Labor and Employment a petition for certification election among the supervisory
employees of petitioner, alleging that as a supervisory union duly registered with the
Department of Labor and Employment it was seeking to represent the supervisory
employees of Philippine Phosphate Fertilizer Corporation. Mediator-Arbiter Rodolfo S.
Milado issued an order directing the holding of a certification election among the
supervisory employees of petitioner, excluding therefrom the superintendents and the
professional and technical employees. However, the PMPI filed an amended petition
with the Mediator-Arbiter wherein it sought to represent not only the supervisory
employees of petitioner but also its professional/technical and confidential employees.
The parties therein agreed to submit their respective position papers and to consider
the amended petition submitted for decision on the basis thereof and related
documents. Mediator-Arbiter Milado issued an order granting the petition and directing
the holding of a certification election among the "supervisory, professional (engineers,
analysts, mechanics, accountants, nurses, midwives, etc.), technical, and confidential
employees. PHILPHOS appealed the order to the Secretary of Labor and Employment
who rendered a decision through Undersecretary Bienvenido Laguesma dismissing the
appeal. PHILPHOS moved for reconsideration but the same was denied; hence, the
instant petition alleging denial of due process on the part of the DOLE to which the
mediator-arbiter was under.
Issue:
Whether or Not there was denial of due process.
Held:
There was no denial of due process. The essence of due process is simply an
opportunity to be heard or, as applied to administrative proceedings, an opportunity to
explain one's side or an opportunity to seek a reconsideration of the action or ruling
complained of petitioner PHILPHOS agreed to file its position paper with the MediatorArbiter and to consider the case submitted for decision on the basis of the position
papers filed by the parties, there was sufficient compliance with the requirement of due
process, as petitioner was afforded reasonable opportunity to present its side.
Moreover, petitioner could have, if it so desired, insisted on a hearing to confront and
examine the witnesses of the other party. But it did not; instead it opted to submit its
Page 76

position paper with the Mediator-Arbiter. Besides, petitioner had all the opportunity to
ventilate its arguments in its appeal to the Secretary of Labor.

RUBI VS. PROVINCIAL BOARD OF MINDORO


[39 PHIL 660; G.R. NO. 14078; 7 MAR 1919]
Facts:
This is an application for habeas corpus in favor of Rubi and other Manguianes of the
Province of Mindoro.
The provincial board of Mindoro adopted resolution No. 25 which states that provincial
governor of any province in which non-Christian inhabitants (uncivilized tribes) are
found is authorized, when such a course is deemed necessary in the interest of law and
order, to direct such inhabitants to take up their habitation on sites on unoccupied
public lands to be selected by him and approved by the provincial board. It is resolved
that under section 2077 of the Administrative Code, 800 hectares of public land in the
sitio of Tigbao on Naujan Lake be selected as a site for the permanent settlement of
Mangyanes in Mindoro. Further, Mangyans may only solicit homesteads on this
reservation providing that said homestead applications are previously recommended by
the provincial governor.
Thereafter, the provincial governor of Mindoro issued executive order No. 2, which says
that the provincial governor has selected a site in the sitio of Tigbao on Naujan Lake for
the permanent settlement of Mangyanes in Mindoro. In that case, pursuant to Section
2145 of the Revised Administrative Code, all the Mangyans in the townships of Naujan
and Pola and the Mangyans east of the Baco River including those in the districts of
Dulangan and Rubi's place in Calapan, were ordered to take up their habitation on the
site of Tigbao, Naujan Lake. Also, that any Mangyan who shall refuse to comply with
this order shall upon conviction be imprisoned not exceed in sixty days, in accordance
with section 2759 of the revised Administrative Code.
Said resolution of the provincial board of Mindoro were claimed as necessary measures
for the protection of the Mangyanes of Mindoro as well as the protection of public
forests in which they roam, and to introduce civilized customs among them.
It appeared that Rubi and those living in his rancheria have not fixed their dwelling
within the reservation of Tigbao and are liable to be punished.
It is alleged that the Manguianes are being illegally deprived of their liberty by the
provincial officials of that province. Rubi and his companions are said to be held on the
reservation established at Tigbao, Mindoro, against their will, and one Dabalos is said to
Page 77

be held under the custody of the provincial sheriff in the prison at Calapan for having
run away form the reservation.
Issue:
Whether or Not Section 2145 of the Administrative Code deprive a person of his liberty
without due process of law.
Whether or Not Section 2145 of the Administrative Code of 1917 is constitutional.
Held:
The Court held that section 2145 of the Administrative Code does not deprive a person
of his liberty without due process of law and does not deny to him the equal protection
of the laws, and that confinement in reservations in accordance with said section does
not constitute slavery and involuntary servitude. The Court is further of the opinion that
section 2145 of the Administrative Code is a legitimate exertion of the police power,
somewhat analogous to the Indian policy of the United States. Section 2145 of the
Administrative Code of 1917 is constitutional.
The preamble of the resolution of the provincial board of Mindoro which set apart the
Tigbao reservation, it will be read, assigned as reasons fort the action, the following: (1)
The failure of former attempts for the advancement of the non-Christian people of the
province; and (2) the only successfully method for educating the Manguianes was to
oblige them to live in a permanent settlement. The Solicitor-General adds the following;
(3) The protection of the Manguianes; (4) the protection of the public forests in which
they roam; (5) the necessity of introducing civilized customs among the Manguianes.
Considered purely as an exercise of the police power, the courts cannot fairly say that
the Legislature has exceeded its rightful authority. It is, indeed, an unusual exercise of
that power. But a great malady requires an equally drastic remedy. One cannot hold
that the liberty of the citizen is unduly interfered without when the degree of civilization
of the Manguianes is considered. They are restrained for their own good and the
general good of the Philippines. Nor can one say that due process of law has not been
followed.
None of the rights of the citizen can be taken away except by due process of law. To
constitute "due process of law," as has been often held, a judicial proceeding is not
always necessary. In some instances, even a hearing and notice are not requisite a rule
which is especially true where much must be left to the discretion of the administrative
officers in applying a law to particular cases.

Page 78

The idea of the provision in question is to unify the people of the Philippines so that
they may approach the highest conception of nationality. The public policy of the
Government of the Philippine Islands is shaped with a view to benefit the Filipino people
as a whole. The Manguianes, in order to fulfill this governmental policy, must be
confined for a time, as we have said, for their own good and the good of the country.
Therefore, petitioners are not unlawfully imprisoned or restrained of their liberty.
Habeas corpus can, therefore, not issue.

KWONG SING VS. CITY OF MANILA


[41 PHIL 103; G.R. NO. 15972; 11 OCT 1920]
Facts:
Kwong Sing, in his own behalf and of other Chinese laundrymen who has general and
the same interest, filed a complaint for a preliminary injunction. The Plaintiffs also
questioned the validity of enforcing Ordinance No. 532 by the city of Manila. Ordinance
No. 532 requires that the receipt be in duplicate in English and Spanish duly signed
showing the kind and number of articles delivered by laundries and dyeing and
cleaning establishments. The permanent injunction was denied by the trial court. The
appellants claim is that Ordinance No. 532 savors of class legislation; putting in mind
that they are Chinese nationals. It unjustly discriminates between persons in similar
circumstances; and that it constitutes an arbitrary infringement of property rights.
They also contest that the enforcement of the legislation is an act beyond the scope of
their police power. In view of the foregoing, this is an appeal with the Supreme Court.
Issue:
Whether or Not the enforcement of Ordinance no, 532 is an act beyond the scope of
police power
Whether or not the enforcement of the same is a class legislation that infringes
property rights.
Held:
Reasonable restraints of a lawful business for such purposes are permissible under the
police power. The police power of the City of Manila to enact Ordinance No. 532 is
based on
Section 2444, paragraphs (l) and (ee) of the Administrative Code, as
amended by Act No. 2744, authorizes the municipal board of the city of Manila, with the
approval of the mayor of the city:
Page 79

(l) To regulate and fix the amount of the license fees for the following: xxxx
xxxxxlaundries xxxx.
(ee) To enact all ordinances it may deem necessary and proper for the sanitation
and safety, the furtherance of the prosperity, and the promotion of the morality,
peace, good order, comfort, convenience, and general welfare of the city and its
inhabitants.
The court held that the obvious purpose of Ordinance No. 532 was to avoid disputes
between laundrymen and their patrons and to protect customers of laundries who are
not able to decipher Chinese characters from being defrauded. (Considering that in the
year 1920s, people of Manila are more familiar with Spanish and maybe English.)
In whether the ordinance is class legislation, the court held that the ordinance invades
no fundamental right, and impairs no personal privilege. Under the guise of police
regulation, an attempt is not made to violate personal property rights. The ordinance is
neither discriminatory nor unreasonable in its operation. It applies to all public
laundries without distinction, whether they belong to Americans, Filipinos, Chinese, or
any other nationality. All, without exception, and each every one of them without
distinction, must comply with the ordinance.
The obvious objection for the
implementation of the ordinance is based in sec2444 (ee) of the Administrative Code.
Although, an additional burden will be imposed on the business and occupation
affected by the ordinance such as that of the appellant by learning even a few words in
Spanish or English, but mostly Arabic numbers in order to properly issue a receipt, it
seems that the same burdens are cast upon the them. Yet, even if private rights of
person or property are subjected to restraint, and even if loss will result to individuals
from the enforcement of the ordinance, this is not sufficient ground for failing to uphold
the power of the legislative body. The very foundation of the police power is the control
of private interests for the public welfare.
Finding that the ordinance is valid, judgment is affirmed, and the petition for a
preliminary injunction is denied, with costs against the appellants.

YU CONG ENG VS. TRINIDAD


[47 PHIL 385; G.R. NO. 20479; 6 FEB 1925]
Facts:
The petitioner, Yu Cong Eng, was charged by information in the court of first instance of
Manila, with a violation of Act 2972, which provides that (Section 1) it shall be unlawful
for any person, company, or partnership or corporation engaged in commerce, industry
or any other activity for the purpose of profit in the Philippine Islands, in accordance
with existing law, to keep its account books in any language other than English,
Page 80

Spanish or any local dialect. He was arrested, his books were seized, and the trial was
about to proceed, when he and the other petitioner, Co Liam, on their own behalf, and
on behalf of all the other Chinese merchants in the Philippines, filed the petition against
the fiscal, or prosecuting attorney of Manila, and the collector of internal revenue
engaged in the prosecution, and against the judge presiding.
Issue:
Whether or Not Act 2972 is unconstitutional.

Held:
Yes. The Philippine government may make every reasonable requirement of its
taxpayers to keep proper records of their business transactions in English or Spanish or
Filipino dialect by which an adequate measure of what is due from them in meeting the
cost of government can be had. But we are clearly of opinion that it is not within the
police power of the Philippine Legislature, because it would be oppressive and arbitrary,
to prohibit all Chinese merchants from maintaining a set of books in the Chinese
language, and in the Chinese characters, and thus prevent them from keeping advised
of the status of their business and directing its conduct.

ANIAG VS. COMELEC


[237 SCRA 194; G.R. NO. 104961; 7 OCT 1994]
Facts:
In preparation for the synchronized national and local elections, the COMELEC issued
Resolution No. 2323, Gun Ban, promulgating rules and regulations on bearing,
carrying and transporting of firearm or other deadly weapons on security personnel or
bodyguards, on bearing arms by members of security agencies or police organizations,
and organization or maintenance of reaction forces during the election period.
COMELEC also issued Resolution No. 2327 providing for the summary disqualification of
candidates engaged in gunrunning, using and transporting of firearms, organizing
special strike forces, and establishing spot checkpoints. Pursuant to the Gun Ban, Mr.
Serrapio Taccad, Sergeant at Arms of the House of Representatives, wrote petitioner for
the return of the two firearms issued to him by the House of Representatives. Petitioner
then instructed his driver, Arellano, to pick up the firearms from petitioners house and
return them to Congress. The PNP set up a checkpoint. When the car driven by Arellano
approached the checkpoint, the PNP searched the car and found the firearms. Arellano
was apprehended and detained. He then explained the order of petitioner. Petitioner
also explained that Arellano was only complying with the firearms ban, and that he was
not a security officer or a bodyguard. Later, COMELEC issued Resolution No.92-0829
Page 81

directing the filing of information against petitioner and Arellano for violation of the
Omnibus Election Code, and for petitioner to show cause why he should not be
disqualified from running for an elective position. Petitioner then questions the
constitutionality of Resolution No. 2327. He argues that gunrunning, using or
transporting firearms or similar weapons and other acts mentioned in the resolution
are not within the provisions of the Omnibus Election Code. Thus, according to
petitioner, Resolution No. 2327 is unconstitutional. The issue on the disqualification of
petitioner from running in the elections was rendered moot when he lost his bid for a
seat in Congress in the elections.
Issue:
Whether or Not petitioner can be validly prosecuted for instructing his driver to return
the firearms issued to him on the basis of the evidence gathered from the warrant less
search of his car

Held:
A valid search must be authorized by a search warrant issued by an appropriate
authority. However, a warrantless search is not violative of the Constitution for as long
as the vehicle is neither searched nor its occupants subjected to a body search, and the
inspection of the vehicle is merely limited to a visual search. In the case at bar, the
guns were not tucked in Arellanos waist nor placed within his reach, as they were
neatly packed in gun cases and placed inside a bag at the back of the car. Given these
circumstances, the PNP could not have thoroughly searched the car lawfully as well as
the package without violating the constitutional injunction. Absent any justifying
circumstance specifically pointing to the culpability of petitioner and Arellano, the
search could not have been valid. Consequently, the firearms obtained from the
warrantless search cannot be admitted for any purpose in any proceeding. It was also
shown in the facts that the PNP had not informed the public of the purpose of setting up
the checkpoint. Petitioner was also not among those charged by the PNP with violation
of the Omnibus Election Code. He was not informed by the City Prosecutor that he was
a respondent in the preliminary investigation. Such constituted a violation of his right to
due process. Hence, it cannot be contended that petitioner was fully given the
opportunity to meet the accusation against him as he was not informed that he was
himself a respondent in the case. Thus, the warrantless search conducted by the PNP is
declared illegal and the firearms seized during the search cannot be used as evidence
in any proceeding against the petitioner. Resolution No. 92-0829 is unconstitutional,
and therefore, set aside.

JAVIER VS. COMELEC


Page 82

[144 SCRA 194; G.R. NOS. L-68379-81; 22 SEPT 1986]


Facts:
The petitioner and the private respondent were candidates in Antique for the Batasang
Pambansa in the May 1984 elections. The former appeared to enjoy more popular
support but the latter had the advantage of being the nominee of the KBL with all its
perquisites of power. On May 13, 1984, the eve of the elections, the bitter contest
between the two came to a head when several followers of the petitioner were
ambushed and killed, allegedly by the latter's men. Seven suspects, including
respondent Pacificador, are now facing trial for these murders. Owing to what he
claimed were attempts to railroad the private respondent's proclamation, the petitioner
went to the Commission on Elections to question the canvass of the election returns.
His complaints were dismissed and the private respondent was proclaimed winner by
the Second Division of the said body. The petitioner thereupon came to this Court,
arguing that the proclamation was void because made only by a division and not by the
Commission on Elections en banc as required by the Constitution. Meanwhile, on the
strength of his proclamation, the private respondent took his oath as a member of the
Batasang Pambansa.

Issue:
Whether or Not the Second Division of the Commission on Elections authorized to
promulgate its decision of July 23, 1984, proclaiming the private respondent the winner
in the election.
Held:
This Court has repeatedly and consistently demanded "the cold neutrality of an
impartial judge" as the indispensable imperative of due process. To bolster that
requirement, we have held that the judge must not only be impartial but must also
appear to be impartial as an added assurance to the parties that his decision will be
just. The litigants are entitled to no less than that. They should be sure that when their
rights are violated they can go to a judge who shall give them justice. They must trust
the judge, otherwise they will not go to him at all. They must believe in his sense of
fairness, otherwise they will not seek his judgment. Without such confidence, there
would be no point in invoking his action for the justice they expect.
Due process is intended to insure that confidence by requiring compliance with what
Justice Frankfurter calls the rudiments of fair play. Fair play cans for equal justice. There
cannot be equal justice where a suitor approaches a court already committed to the
other party and with a judgment already made and waiting only to be formalized after
Page 83

the litigants shall have undergone the charade of a formal hearing. Judicial (and also
extra-judicial) proceedings are not orchestrated plays in which the parties are supposed
to make the motions and reach the denouement according to a prepared script. There
is no writer to foreordain the ending. The judge will reach his conclusions only after all
the evidence is in and all the arguments are filed, on the basis of the established facts
and the pertinent law.

YNOT VS. IAC


[148 SCRA 659; G.R. NO. 74457; 20 MAR 1987]
Facts:
Executive Order No. 626-A prohibited the transportation of carabaos and carabeef from
one province to another. The carabaos of petitioner were confiscated for violation of
Executive Order No 626-A while he was transporting them from Masbate to Iloilo.
Petitioner challenged the constitutionality of Executive Order No. 626-A. The
government argued that Executive Order No. 626-A was issued in the exercise of police
power to conserve the carabaos that were still fit for farm work or breeding.
Issue:
Whether or Not EO No. 626-A is a violation of Substantive Due Process.
Held:
The challenged measure is an invalid exercise of police power, because it is not
reasonably necessary for the purpose of the law and is unduly oppressive. It is difficult
to see how prohibiting the transfer of carabaos from one province to another can
prevent their indiscriminate killing. Retaining the carabaos in one province will not
prevent their slaughter there. Prohibiting the transfer of carabeef, after the slaughter of
the carabaos, will not prevent the slaughter either.

PHILCOMSAT VS. ALCUAZ


[180 SCRA 218; G.R. NO.84818; 18 DEC 1989]
Facts:
Herein petitioner is engaged in providing for services involving telecommunications.
Charging rates for certain specified lines that were reduced by order of herein
Page 84

respondent Jose Alcuaz Commissioner of the National Telecommunications Commission.


The rates were ordered to be reduced by fifteen percent (15%) due to Executive Order
No. 546 which granted the NTC the power to fix rates. Said order was issued without
prior notice and hearing.
Issue:
Whether or Not E.O. 546 is unconstitutional.
Held:
Yes. Respondents admitted that the application of a policy like the fixing of rates as
exercised by administrative bodies is quasi-judicial rather than quasi-legislative. But
respondents contention that notice and hearing are not required since the assailed
order is merely incidental to the entire proceedings and temporary in nature is
erroneous. Section 16(c) of the Public Service Act, providing for the proceedings of the
Commission, upon notice and hearing, dictates that a Commission has power to fix
rates, upon proper notice and hearing, and, if not subject to the exceptions, limitations
or saving provisions.
It is thus clear that with regard to rate-fixing, respondent has no authority to make such
order without first giving petitioner a hearing, whether the order be temporary or
permanent, and it is immaterial whether the same is made upon a complaint, a
summary investigation, or upon the commission's own motion as in the present case.
WHEREFORE, the writ prayed for is GRANTED and the order of respondents is hereby
SET ASIDE.

EASTERN BROADCASTING CORP (DYRE) V. DANS JR.


[137 SCRA 628; L-59329; 19 JUL 1985]
Facts:
A petition was filed to reopen the Radio Station DYRE. DYRE was summarily closed on
grounds of national security. The radio station was allegedly used to incite people to
sedition. Petitioner, DYRE contends that they were denied due process. There was no
hearing to establish factual evidence for the closure. Furthermore, the closure of the
radio station violates freedom of expression. Before the court could even promulgate a
decision upon the Issue raised, Petitioner, through its president Mr. Rene Espina, filed a
motion to withdraw the petition. The rights of the station were sold to a new owner,
Manuel Pastrana; who is no longer interested in pursuing the case. Despite the case
becoming moot and academic, (because there are no longer interested parties, thus
Page 85

the dismissal of the case) the Supreme Court still finds that there is need to pass a
RESOLUTION for the guidance of inferior courts and administrative tribunals in
matters as this case.
Issue:
Whether or not due process was exercised in the case of DYRE.
Whether or not the closure of DYRE is a violation of the Constitutional Right of Freedom
of Expression.
Held:
The court finds that the closure of the Radio Station in 1980 as null and void. The
absence of a hearing is a violation of Constitutional Rights. The primary requirements
in administrative proceedings are laid down in the case of Ang Tibay v. Court of
Industrial Relation (69 Phil.635). The Ang Tibay Doctrine should be followed before any
broadcast station may be closed. The Ang Tibay Doctrine provides the following
requirements:
(1)

The right to hearing, includes the right to present ones case and
submit evidence presented.

(2)

The tribunal must consider the evidence presented

(3)

The decision must have something to support itself.

(4)

Evidence must be substantial (reasonable evidence that is adequate to


support conclusion)

(5)

Decision must be based on the evidence presented at hearing

(6)

The tribunal body must act on its own independent consideration of


law and facts and not simply accept subordinates views

(7)

Court must render decision in such a manner that the proceeding can
know the various issued involved and reasons for decisions rendered.

The court stresses that while there is no controlling and precise definition of Due
Process, it gives an unavoidable standard that government actions must conform in
order that deprivation of life, liberty and property is valid.
The closure of the radio station is like wise a violation of the constitutional right of
freedom of speech and expression. The court stresses that all forms of media, whether
print or broadcast are entitled to this constitutional right. Although the government still
has the right to be protected against broadcasts which incite the listeners to violently
overthrow it. The test for the limitation of freedom of expression is the clear and
present danger rule. If in the circumstances that the media is used in such nature as
Page 86

to create this danger that will bring in such evils, then the law has the right to prevent
it. However, Radio and television may not be used to organize a rebellion or signal a
start of widespread uprising. The freedom to comment on public affairs is essential to
the vitality of a representative democracy. The people continues to have the right to be
informed on public affairs and broadcast media continues to have the pervasive
influence to the people being the most accessible form of media. Therefore, broadcast
stations deserve the the special protection given to all forms of media by the due
process and freedom of expression clauses of the Constitution.

ANG TIBAY VS. COURT OF INDUSTRIAL RELATIONS (CIR)


[69 PHIL 635; G.R. NO. 46496; 27 FEB 1940]
Facts:
There was agreement between Ang Tibay and the National Labor Union, Inc (NLU). The
NLU alleged that the supposed lack of leather material claimed by Toribio Teodoro was
but a scheme adopted to systematically discharge all the members of the NLU, from
work. And this averment is desired to be proved by the petitioner with the records of
the Bureau of Customs and Books of Accounts of native dealers in leather. That National
Worker's Brotherhood Union of Ang Tibay is a company or employer union dominated
by Toribio Teodoro, which was alleged by the NLU as an illegal one. The CIR, decided
the case and elevated it to the Supreme Court, but a motion for new trial was raised by
the NLU. But the Ang Tibay filed a motion for opposing the said motion.
Issue:
Whether or Not, the motion for new trial is meritorious to be granted.
Held:
To begin with the issue before us is to realize the functions of the CIR. The CIR is a
special court whose functions are specifically stated in the law of its creation which is
the Commonwealth Act No. 103). It is more an administrative board than a part of the
integrated judicial system of the nation. It is not intended to be a mere receptive organ
of the government. Unlike a court of justice which is essentially passive, acting only
when its jurisdiction is invoked and deciding only cases that are presented to it by the
parties litigant, the function of the CIR, as will appear from perusal of its organic law is
more active, affirmative and dynamic. It not only exercises judicial or quasi-judicial
functions in the determination of disputes between employers and employees but its
functions are far more comprehensive and extensive. It has jurisdiction over the entire
Philippines, to consider, investigate, decide, and settle any question, matter
controversy or disputes arising between, and/ or affecting employers and employees or
Page 87

laborers, and landlords and tenants or farm-laborers, and regulates the relations
between them, subject to, and in accordance with, the provisions of CA 103.
As laid down in the case of Goseco v. CIR, the SC had the occasion to point out that the
CIR is not narrowly constrained by technical rules of procedure, and equity and
substantial merits of the case, without regard to technicalities or legal forms and shall
not be bound by any technical rules of legal evidence but may inform its mind in such
manner as it may deem just and equitable.

The fact, however, that the CIR may be said to be free from rigidity of certain
procedural requirements does not mean that it can in justiciable cases coming before it,
entirely ignore or disregard the fundamental and essential requirements of due process
in trials and investigations of an administrative character. There cardinal primary rights
which must be respected even in proceedings of this character:
(1) the right to a hearing, which includes the right to present one's cause
and submit evidence in support thereof;
(2) The tribunal must consider the evidence presented;
(3) The decision must have something to support itself;
(4) The evidence must be substantial;
(5) The decision must be based on the evidence presented at the hearing;
or at least contained in the record and disclosed to the parties affected;
(6) The tribunal or body or any of its judges must act on its own
independent consideration of the law and facts of the controversy, and
not simply accept the views of a subordinate;
(7) The Board or body should, in all controversial questions, render its
decision in such manner that the parties to the proceeding can know
the various Issue involved, and the reason for the decision rendered.
The failure to grasp the fundamental issue involved is not entirely attributable to the
parties adversely affected by the result. Accordingly, the motion for a new trial should
be, and the same is hereby granted, and the entire record of this case shall be
remanded to the CIR, with instruction that it reopen the case receive all such evidence
as may be relevant, and otherwise proceed in accordance with the requirements set
forth. So ordered.

ATENEO DE MANILA UNIVERSITY VS. HON. JUDGE IGNACIO CAPULONG


[222 SCRA 644; G.R. 99327; 27 MAY 1993]

Page 88

Facts:
Leonardo H. Villa, a first year law student of Petitioner University, died of serious
physical injuries at Chinese General Hospital after the initiation rites of Aquila Legis.
Bienvenido Marquez was also hospitalized at the Capitol Medical Center for acute renal
failure occasioned by the serious physical injuries inflicted upon him on the same
occasion. Petitioner Dean Cynthia del Castillo created a Joint Administration-FacultyStudent Investigating Committee which was tasked to investigate and submit a report
within 72 hours on the circumstances surrounding the death of Lennie Villa. Said notice
also required respondent students to submit their written statements within twenty-four
(24) hours from receipt. Although respondent students received a copy of the written
notice, they failed to file a reply. In the meantime, they were placed on preventive
suspension. The Joint Administration-Faculty-Student Investigating Committee, after
receiving the written statements and hearing the testimonies of several witness, found
a prima facie case against respondent students for violation of Rule 3 of the Law School
Catalogue entitled "Discipline." Respondent students were then required to file their
written answers to the formal charge. Petitioner Dean created a Disciplinary Board to
hear the charges against respondent students. The Board found respondent students
guilty of violating Rule No. 3 of the Ateneo Law School Rules on Discipline which
prohibits participation in hazing activities. However, in view of the lack of unanimity
among the members of the Board on the penalty of dismissal, the Board left the
imposition of the penalty to the University Administration. Accordingly, Fr. Bernas
imposed the penalty of dismissal on all respondent students. Respondent students filed
with RTC Makati a TRO since they are currently enrolled. This was granted. A TRO was
also issued enjoining petitioners from dismissing the respondents. A day after the
expiration of the temporary restraining order, Dean del Castillo created a Special Board
to investigate the charges of hazing against respondent students Abas and Mendoza.
This was requested to be stricken out by the respondents and argued that the creation
of the Special Board was totally unrelated to the original petition which alleged lack of
due process. This was granted and reinstatement of the students was ordered.
Issue:
Was there denial of due process against the respondent students.
Held:
There was no denial of due process, more particularly procedural due process. Dean of
the Ateneo Law School, notified and required respondent students to submit their
written statement on the incident. Instead of filing a reply, respondent students
requested through their counsel, copies of the charges. The nature and cause of the
accusation were adequately spelled out in petitioners' notices. Present is the twin
elements of notice and hearing.
Respondent students argue that petitioners are not in a position to file the instant
petition under Rule 65 considering that they failed to file a motion for reconsideration
Page 89

first before the trial court, thereby by passing the latter and the Court of Appeals. It is
accepted legal doctrine that an exception to the doctrine of exhaustion of remedies is
when the case involves a question of law, as in this case, where the issue is whether or
not respondent students have been afforded procedural due process prior to their
dismissal from Petitioner University.
Minimum standards to be satisfied in the imposition of disciplinary sanctions in
academic institutions, such as petitioner university herein, thus:
(1) the students must be informed in writing of the nature and cause of
any accusation against them;
(2) that they shall have the right to answer the charges against them with
the assistance of counsel, if desired:
(3) they shall be informed of the evidence against them
(4) they shall have the right to adduce evidence in their own behalf; and
(5) the evidence must be duly considered by the investigating committee
or official designated by the school authorities to hear and decide the
case.

US GOVERNMENT VS. JUDGE PURUNGAN


[389 SCRA 623; G.R. NO. 148571, 24 SEPT 2002]
Facts:
The United States of America, pursuant to the existing RP-US extradition treaty,
requested the extradition of Mark B. Jimenez. Upon receipt of the request, the secretary
of foreign affairs (SFA) transmitted them to the secretary of justice (SOJ) for appropriate
action. In such event, the RTC held that Jimenez shell be deprived of the right to notice
and hearing during the evaluation stage of the extradition process. Thereafter the US
government, through DOJ, filed Petition for Extradition and Jimenezs immediate arrest,
to avoid flight. Before the RTC could render its decision, Jimenez filed an "Urgent
Manifestation/Ex-Parte Motion," praying that his application for an arrest warrant be set
for hearing, which was granted. During which, the lower court issued its questioned July
3, 2001 Order, directing the issuance of a warrant for his arrest and fixing bail for his
temporary liberty at one million pesos in cash. After Jimenez had surrendered his
passport and posted the required cash bond, he was granted provisional liberty via the
challenged Order dated July 4, 2001. Thus, Petition prays for the lifting of the bail Order,
the cancellation of the bond, and the taking of Jimenez into legal custody.
Issue:
Page 90

Whether or not Jimenez is entitled to notice and hearing before a warrant for his arrest
can be issued.
Whether or not he is entitled to bail and to provisional liberty while the extradition
proceedings are pending.
Held:
By nature, extradition proceedings are not equivalent to a criminal case in which guilt
or innocence is determined. Consequently, an extradition case is not one in which the
constitutional rights of the accused are necessarily available. Having once escaped the
jurisdiction of the requesting state, the reasonable prima facie presumption is that the
person would escape again if given the opportunity. Hence, if the judge is convinced
that a prima facie case exists, he immediately Issue a warrant for the arrest of the
potential extraditee and summons him or her to answer and to appear at scheduled
hearings on the petition. Potential extraditees are entitled to the rights to due process
and to fundamental fairness. Due process does not always call for a prior opportunity to
be heard. A subsequent opportunity is sufficient due to the flight risk involved. Indeed,
available during the hearings on the petition and the answer is the full chance to be
heard and to enjoy fundamental fairness that is compatible with the summary nature of
extradition.
After being taken into custody, potential extraditees may apply for bail. Since the
applicants have a history of absconding, they have the burden of showing that (a) there
is no flight risk and no danger to the community; and (b) there exist special,
humanitarian or compelling circumstances. In extradition cases, bail is not a matter of
right; it is subject to judicial discretion in the context of the peculiar facts of each case.

Page 91

EQUAL PROTECTION

Art 3, Sec. 1.

nor shall any person be denied the equal protection of the laws.

PEOPLE VS. CAYAT


[68 PHIL 12; G.R. NO. 45987; 5 MAY 1939]
Facts:
Law prohibits any member of a non-Christian tribe to buy, receive, have in his
possession, or drink, any intoxicating liquors of any kind. The law, Act No. 1639,
exempts only the so-called native wines or liquors which the members of such tribes
have been accustomed to take.
Issue:
Whether or Not the law denies equal protection to one prosecuted and sentenced for
violation of said law.
Held:
No. It satisfies the requirements of a valid classification, one of which is that the
classification under the law must rest on real or substantial distinctions.
The distinction is reasonable. The classification between the members of the nonChristian and the members of the Christian tribes is not based upon accident of birth or
parentage but upon the degree of civilization and culture. The term non-Christian
tribes refers to a geographical area and more directly to natives of the Philippines of a
low grade civilization usually living in tribal relationship apart from settled communities.
The distinction is reasonable for the Act was intended to meet the peculiar conditions
existing in the non- Christian tribes
The prohibition is germane to the purposes of the law. It is designed to insure peace
and order in and among the non- Christian tribes has often resulted in lawlessness and
crime thereby hampering the efforts of the government to raise their standards of life
and civilization. This law is not limited in its application to conditions existing at the
time of the enactment. It is intended to apply for all times as long as those conditions
Page 92

exists. The Act applies equally to all members of the class. That it may be unfair in its
operation against a certain number of non- Christians by reason of their degree of
culture is not an argument against the equality of its operation nor affect the
reasonableness of the classification thus established.

PASEI VS. DRILON


[163 SCRA 386; L-81958; 30 JUN 1988]
Facts:
Petitioner, Phil association of Service Exporters, Inc., is engaged principally in the
recruitment of Filipino workers, male and female of overseas employment. It challenges
the constitutional validity of Dept. Order No. 1 (1998) of DOLE entitled Guidelines
Governing the Temporary Suspension of Deployment of Filipino Domestic and
Household Workers. It claims that such order is a discrimination against males and
females. The Order does not apply to all Filipino workers but only to domestic helpers
and females with similar skills, and that it is in violation of the right to travel, it also
being an invalid exercise of the lawmaking power. Further, PASEI invokes Sec 3 of Art 13
of the Constitution, providing for worker participation in policy and decision-making
processes affecting their rights and benefits as may be provided by law. Thereafter the
Solicitor General on behalf of DOLE submitting to the validity of the challenged
guidelines involving the police power of the State and informed the court that the
respondent have lifted the deployment ban in some states where there exists bilateral
agreement with the Philippines and existing mechanism providing for sufficient
safeguards to ensure the welfare and protection of the Filipino workers.
Issue:
Whether or not there has been a valid classification in the challenged Department
Order No. 1.
Held:

Page 93

SC in dismissing the petition ruled that there has been valid classification, the Filipino
female domestics working abroad were in a class by themselves, because of the special
risk to which their class was exposed. There is no question that Order No.1 applies only
to female contract workers but it does not thereby make an undue discrimination
between sexes. It is well settled hat equality before the law under the constitution does
not import a perfect identity of rights among all men and women. It admits of
classification, provided that:
1. Such classification rests on substantial distinctions
2. That they are germane to the purpose of the law
3. They are not confined to existing conditions
4. They apply equally to al members of the same class
In the case at bar, the classifications made, rest on substantial distinctions.
Dept. Order No. 1 does not impair the right to travel. The consequence of the
deployment ban has on the right to travel does not impair the right, as the right to
travel is subjects among other things, to the requirements of public safety as may be
provided by law. Deployment ban of female domestic helper is a valid exercise of police
power. Police power as been defined as the state authority to enact legislation that may
interfere with personal liberty or property in order to promote general welfare. Neither
is there merit in the contention that Department Order No. 1 constitutes an invalid
exercise of legislative power as the labor code vest the DOLE with rule making powers.

DUMLAO VS. COMELEC


[95 SCRA 392; L-52245; 22 JAN 1980]
Facts:
Petitioner questions the constitutionality of section 4 of Batas Pambansa Blg. 52 as
discriminatory and contrary to the equal protection and due process guarantees of the
Constitution.
Section 4 provided that any retired municipal or provincial city official that already
received retirement benefits and is 65 years of age shall not be qualified to run for the
same local elective office from which he has retired.
Issue:
Page 94

Whether or Not Sec. 4 of BP.52 is unconstitutional being contrary to the equal


protection and due process rights.
Held:
No. The guarantee of equal protection is subject to rational classification based on
reasonable and real differentiations. In the present case, employees 65 years of age
have been classified differently from younger employees. The former are subject to
compulsory retirement while the latter are not.
Retirement is not a reasonable disqualification for elective local officials because there
can be retirees who are even younger and a 65 year old retiree could be as good as a
65 year old official who is not a retiree. But there is reason to disqualify a 65 year old
elective official who is trying to run for office because there is the need for new blood
to assume relevance. When an official has retired he has already declared himself
tired and unavailable for the same government work.
WHEREFORE, the first paragraph of section 4 of Batas pambansa Bilang 52 is hereby
declared valid.

TELECOMMUNICATIONS AND BROADCAST ATTORNEYS OF THE PHILS. VS. COMELEC


[289 SCRA 337; G.R. NO. 132922; 21 APR 1998]
Facts:
Petitioner Telecommunications and Broadcast Attorneys of the Philippines, Inc.
(TELEBAP) is an organization of lawyers of radio and television broadcasting companies.
It was declared to be without legal standing to sue in this case as, among other
reasons, it was not able to show that it was to suffer from actual or threatened injury as
a result of the subject law. Petitioner GMA Network, on the other hand, had the
requisite standing to bring the constitutional challenge. Petitioner operates radio and
television broadcast stations in the Philippines affected by the enforcement of Section
92, B.P. No. 881.
Petitioners challenge the validity of Section 92, B.P. No. 881 which provides:
Comelec Time- The Commission shall procure radio and television time
to be known as the Comelec Time which shall be allocated equally and
impartially among the candidates within the area of coverage of all radio
and television stations. For this purpose, the franchise of all radio
broadcasting and television stations are hereby amended so as to provide
radio or television time, free of charge, during the period of campaign.
Page 95

Petitioner contends that while Section 90 of the same law requires COMELEC to procure
print space in newspapers and magazines with payment, Section 92 provides that air
time shall be procured by COMELEC free of charge. Thus it contends that Section 92
singles out radio and television stations to provide free air time.
Petitioner claims that it suffered losses running to several million pesos in providing
COMELEC Time in connection with the 1992 presidential election and 1995 senatorial
election and that it stands to suffer even more should it be required to do so again this
year. Petitioners claim that the primary source of revenue of the radio and television
stations is the sale of air time to advertisers and to require these stations to provide
free air time is to authorize unjust taking of private property. According to petitioners,
in 1992 it lost P22,498,560.00 in providing free air time for one hour each day and, in
this years elections, it stands to lost P58,980,850.00 in view of COMELECs
requirement that it provide at least 30 minutes of prime time daily for such.
Issue:
Whether of not Section 92 of B.P. No. 881 denies radio and television broadcast
companies the equal protection of the laws.
Whether or not Section 92 of B.P. No. 881 constitutes taking of property without due
process of law and without just compensation.

Held:
Petitioners argument is without merit. All broadcasting, whether radio or by television
stations, is licensed by the government. Airwave frequencies have to be allocated as
there are more individuals who want to broadcast that there are frequencies to assign.
Radio and television broadcasting companies, which are given franchises, do not own
the airwaves and frequencies through which they transmit broadcast signals and
images. They are merely given the temporary privilege to use them. Thus, such
exercise of the privilege may reasonably be burdened with the performance by the
grantee of some form of public service. In granting the privilege to operate broadcast
stations and supervising radio and television stations, the state spends considerable
public funds in licensing and supervising them.
The argument that the subject law singles out radio and television stations to provide
free air time as against newspapers and magazines which require payment of just
compensation for the print space they may provide is likewise without merit.
Regulation of the broadcast industry requires spending of public funds which it does not
do in the case of print media. To require the broadcast industry to provide free air time
for COMELEC is a fair exchange for what the industry gets.
Page 96

As radio and television broadcast stations do not own the airwaves, no private property
is taken by the requirement that they provide air time to the COMELEC.

LACSON VS. EXECUTIVE SECRETARY


[301 SCRA 298; G.R. NO. 128096; 20 JAN 1999]
Facts:
Eleven persons believed to be members of the Kuratong Baleleng gang, an organized
crime syndicate involved in bank robberies, were slain by elements of the Anti-Bank
Robbery and Intelligence Task Group (ABRITG). Among those included in the ABRITG
were petitioners and petitioner-intervenors.
Acting on a media expose of SPO2 Eduardo delos Reyes, a member of the Criminal
Investigation Command, that what actually transpired was a summary execution and
not a shoot-out between the Kuratong Baleleng gang members and the ABRITG,
Ombudsman Aniano Desierto formed a panel of investigators to investigate the said
incident. Said panel found the incident as a legitimate police operation. However, a
review board modified the panels finding and recommended the indictment for
multiple murder against twenty-six respondents including herein petitioner, charged as
principal, and herein petitioner-intervenors, charged as accessories.
After a
reinvestigation, the Ombudsman filed amended informations before the
Sandiganbayan, where petitioner was charged only as an accessory.
The accused filed separate motions questioning the jurisdiction of the Sandiganbayan,
asserting that under the amended informations, the cases fall within the jurisdiction of
the Regional Trial Court pursuant to Section 2 of R.A. 7975. They contend that the said
law limited the jurisdiction of the Sandiganbayan to cases where one or ore of the
principal accused are government officals with Salary Grade 27 or higher, or PNP
officials with rank of Chief Superintendent or higher. Thus, they did not qualify under
said requisites. However, pending resolution of their motions, R.A. 8249 was approved
amending the jurisdiction of the Sandiganbayan by deleting the word principal from
the phrase principal accused in Section 2 of R.A. 7975.
Petitioner questions the constitutionality of Section 4 of R.A. 8249, including Section 7
which provides that the said law shall apply to all cases pending in any court over
which trial has not begun as of the approval hereof.
Issue:

Page 97

Whether or not Sections 4 and 7 of R.A. 8249 violate the petitioners right to due
process and the equal protection clause of the Constitution as the provisions seemed to
have been introduced for the Sandiganbayan to continue to acquire jurisdiction over
the Kuratong Baleleng case.
Whether or not said statute may be considered as an ex-post facto statute.
Whether or not the multiple murder of the alleged members of the Kuratong Baleleng
was committed in relation to the office of the accused PNP officers which is essential to
the determination whether the case falls within the Sandiganbayans or Regional Trial
Courts jurisdiction.
Held:
Petitioner and intervenors posture that Sections 4 and 7 of R.A. 8249 violate their right
to equal protection of the law is too shallow to deserve merit. No concrete evidence
and convincing argument were presented to warrant such a declaration. Every
classification made by the law is presumed reasonable and the party who challenges
the law must present proof of arbitrariness. The classification is reasonable and not
arbitrary when the following concur: (1) it must rest on substantial distinction; (2) it
must be germane to the purpose of the law; (3) must not be limited to existing
conditions only, and (4) must apply equally to all members of the same class; all of
which are present in this case.
Paragraph a of Section 4 provides that it shall apply to all cases involving certain
public officials and under the transitory provision in Section 7, to all cases pending in
any court. Contrary to petitioner and intervenors argument, the law is not particularly
directed only to the Kuratong Baleleng cases. The transitory provision does not only
cover cases which are in the Sandiganbayan but also in any court.
There is nothing ex post facto in R.A. 8249. Ex post facto law, generally, provides
retroactive effect of penal laws. R.A. 8249 is not a penal law. It is a substantive law on
jurisdiction which is not penal in character. Penal laws are those acts of the Legislature
which prohibit certain acts and establish penalties for their violations or those that
define crimes and provide for their punishment. R.A. 7975, as regards the
Sandiganbayans jurisdiction, its mode of appeal and other procedural matters, has
been declared by the Court as not a penal law, but clearly a procedural statute, one
which prescribes rules of procedure by which courts applying laws of all kinds can
properly administer justice. Not being a penal law, the retroactive application of R.A.
8249 cannot be challenged as unconstitutional.
In People vs. Montejo, it was held that an offense is said to have been committed in
relation to the office if it is intimately connected with the office of the offender and
perpetrated while he was in the performance of his official functions. Such intimate
relation must be alleged in the information which is essential in determining the
Page 98

jurisdiction of the Sandiganbayan. However, upon examination of the amended


information, there was no specific allegation of facts that the shooting of the victim by
the said principal accused was intimately related to the discharge of their official duties
as police officers. Likewise, the amended information does not indicate that the said
accused arrested and investigated the victim and then killed the latter while in their
custody. The stringent requirement that the charge set forth with such particularity as
will reasonably indicate the exact offense which the accused is alleged to have
committed in relation to his office was not established.
Consequently, for failure to show in the amended informations that the charge of
murder was intimately connected with the discharge of official functions of the accused
PNP officers, the offense charged in the subject criminal cases is plain murder and,
therefore, within the exclusive original jurisdiction of the Regional Trial Court and not
the Sandiganbayan.

INT'L. SCHOOL ALLIANCE VS. QUISUMBING


[333 SCRA 13; G.R. NO. 128845; 1 JUN 2000]
Facts:
Receiving salaries less than their counterparts hired abroad, the local-hires of private
respondent School, mostly Filipinos, cry discrimination. We agree. That the local-hires
are paid more than their colleagues in other schools is, of course, beside the point. The
point is that employees should be given equal pay for work of equal value.
Private respondent International School, Inc. (the School, for short), pursuant to
Presidential Decree 732, is a domestic educational institution established primarily for
dependents of foreign diplomatic personnel and other temporary residents. To enable
the School to continue carrying out its educational program and improve its standard of
instruction, Section 2(c) of the same decree authorizes the School to
employ its own teaching and management personnel selected by it either locally or
abroad, from Philippine or other nationalities, such personnel being exempt from
otherwise applicable laws and regulations attending their employment, except laws
that have been or will be enacted for the protection of employees.
Accordingly, the School hires both foreign and local teachers as members of its faculty,
classifying the same into two: (1) foreign-hires and (2) local-hires.
The School grants foreign-hires certain benefits not accorded local-hires. These include
housing, transportation, shipping costs, taxes, and home leave travel allowance.
Foreign-hires are also paid a salary rate twenty-five percent (25%) more than localhires. The School justifies the difference on two "significant economic disadvantages"
foreign-hires have to endure, namely: (a) the "dislocation factor" and (b) limited tenure.
Page 99

Issue:
Whether or Not the grants provided by the school to foreign hires and not to local hires
discriminative of their constitutional right to the equal protection clause.
Held:
The foregoing provisions impregnably institutionalize in this jurisdiction the long
honored legal truism of "equal pay for equal work." Persons who work with substantially
equal qualifications, skill, effort and responsibility, under similar conditions, should be
paid similar salaries. This rule applies to the School, its "international character"
notwithstanding.
The School contends that petitioner has not adduced evidence that local-hires perform
work equal to that of foreign-hires. The Court finds this argument a little cavalier. If an
employer accords employees the same position and rank, the presumption is that these
employees perform equal work. This presumption is borne by logic and human
experience. If the employer pays one employee less than the rest, it is not for that
employee to explain why he receives less or why the others receive more. That would
be adding insult to injury. The employer has discriminated against that employee; it is
for the employer to explain why the employee is treated unfairly.
While we recognize the need of the School to attract foreign-hires, salaries should not
be used as an enticement to the prejudice of local-hires. The local-hires perform the
same services as foreign-hires and they ought to be paid the same salaries as the
latter. For the same reason, the "dislocation factor" and the foreign-hires' limited tenure
also cannot serve as valid bases for the distinction in salary rates.
The Constitution enjoins the State to "protect the rights of workers and promote their
welfare," "to afford labor full protection." The State, therefore, has the right and duty to
regulate the relations between labor and capital. These relations are not merely
contractual but are so impressed with public interest that labor contracts, collective
bargaining agreements included, must yield to the common good. Should such
contracts contain stipulations that are contrary to public policy, courts will not hesitate
to strike down these stipulations.
In this case, we find the point-of-hire classification employed by respondent School to
justify the distinction in the salary rates of foreign-hires and local hires to be an invalid
classification. There is no reasonable distinction between the services rendered by
foreign-hires and local-hires.
Wherefore, the petition is given due course. The petition is hereby granted in part. The
orders of the secretary of labor and employment dated June 10, 1996 and march 19,
1997, are hereby reversed and set aside insofar as they uphold the practice of
respondent school of according foreign-hires higher salaries than local-hires.
Page 100

ORMOC SUGAR COMPANY VS. TREASURER OF ORMOC CITY


[22 SCRA 603; L-23794; 17 FEB 1968]
Facts:
On January 29, 1964, the Municipal Board of Ormoc City passed Ordinance No. 4, Series
of 1964, imposing "on any and all productions of centrifugal sugar milled at the Ormoc
Sugar Company, Inc., in Ormoc City a municipal tax equivalent to one per centum (1%)
per export sale to the United States of America and other foreign countries." Payments
for said tax were made, under protest, by Ormoc Sugar Company, Inc. on March 20,
1964 for P7, 087.50 and on April 20, 1964 for P5, 000, or a total of P12, 087.50.
On June 1, 1964, Ormoc Sugar Company, Inc. filed before the Court of First Instance of
Leyte, with service of a copy upon the Solicitor General, a complaint against the City of
Ormoc as well as its Treasurer, Municipal Board and Mayor, alleging that the aforestated ordinance is unconstitutional for being violative of the equal protection clause
(Sec. 1[1], Art. III, Constitution) and the rule of uniformity of taxation (Sec. 22[1]), Art.
VI, Constitution).
Answering, the defendants asserted that the tax ordinance was within defendant city's
power to enact under the Local Autonomy Act and that the same did not violate the
afore-cited constitutional limitations. After pre-trial and submission of the case on
memoranda, the Court of First Instance, on August 6, 1964, rendered a decision that
upheld the constitutionality of the ordinance and declared the taxing power of
defendant chartered city broadened by the Local Autonomy Act to include all other
forms of taxes, licenses or fees not excluded in its charter.
Issue:
Whether or Not the ordinance is unconstitutional for being violative of the equal
protection clause under Sec. 1[1], Art. III, Constitution.
Whether or not it was violative of the rule of uniformity of taxation under the Bill of
Rights, Sec. 22[1], Art. VI, Constitution.
Held:
The Constitution in the bill of rights provides: ". . . nor shall any person be denied the
equal protection of the laws." (Sec. 1 [1], Art. III) In Felwa vs. Salas, We ruled that the
equal protection clause applies only to persons or things identically situated and does
Page 101

not bar a reasonable classification of the subject of legislation, and a classification is


reasonable where (1) it is based on substantial distinctions which make real differences;
(2) these are germane to the purpose of the law; (3) the classification applies not only
to present conditions but also to future conditions which are substantially identical to
those of the present; (4) the classification applies only to those who belong to the same
class.
A perusal of the requisites instantly shows that the questioned ordinance does not meet
them, for it taxes only centrifugal sugar produced and exported by the Ormoc Sugar
Company, Inc. and none other. At the time of the taxing ordinance's enactment, Ormoc
Sugar Company, Inc., it is true, was the only sugar central in the city of Ormoc. Still, the
classification, to be reasonable, should be in terms applicable to future conditions as
well. The taxing ordinance should not be singular and exclusive as to exclude any
subsequently established sugar central, of the same class as plaintiff, for the coverage
of the tax. As it is now, even if later a similar company is set up, it cannot be subject to
the tax because the ordinance expressly points only to Ormoc City Sugar Company, Inc.
as the entity to be levied upon.
Appellant, however, is not entitled to interest; on the refund because the taxes were
not arbitrarily collected (Collector of Internal Revenue v. Binalbagan). 6 At the time of
collection, the ordinance provided a sufficient basis to preclude arbitrariness, the same
being then presumed constitutional until declared otherwise.
Wherefore, the decision appealed from is hereby reversed, the challenged ordinance is
declared unconstitutional and the defendants-appellees are hereby ordered to refund
the P12,087.50 plaintiff-appellant paid under protest. No costs. So ordered.
PHILIPPINE JUDGES ASSO. VS. PRADO
[227 SCRA 703; G.R. NO. 105371; 11 NOV 1993]
Facts:
The Philippine Postal Corporation issued circular No. 92-28 to implement Section 35 of
RA 7354 withdrawing the franking privilege from the SC, CA, RTCs, MeTCs, MTCs and
Land Registration Commission and with certain other government offices. It is alleged
that RA 7354 is discriminatory becasue while withdrawing the franking privilege from
judiciary, it retains the same for the President & Vice-President of the Philippines,
Senator & members of the House of Representatives, COMELEC, National Census &
Statistics Office and the general public. The respondents counter that there is no
discrimination because the law is based on a valid classification in accordance with the
equal protection clause.
Issue:
Whether or Not Section 35 of RA 7354 is constitutional.
Page 102

Held:
The equal protection of the laws is embraced in the concept of due process, as every
unfair discrimination offends the requirements of justice and fair play.
It has
nonetheless been embodied in a separate clause in Article III Section 1 of the
Constitution to provide for amore specific guarantee against any form of undue
favoritism or hostility from the government. Arbitrariness in general may be challenged
on the basis of the due process clause. But if the particular act assailed partakes of an
unwarranted partiality or prejudice, the sharper weapon to cut it down is the equal
protection clause. Equal protection simply requires that all persons or things similarly
situated should be treated alike, both as to rights conferred and responsibilities
imposed. What the clause requires is equality among equals as determined according
to a valid classification. Section 35 of RA 7354 is declared unconstitutional. Circular No.
92-28 is set aside insofar

Page 103

SEARCHES AND SEIZURES

Art 3, Sec. 2.
The right of the people to be secure in their persons, houses,
papers, and effects against unreasonable searches and seizures of whatever nature
and for any purpose shall be inviolable, and no search warrant or warrant of arrest shall
issue except upon probable cause to be determined personally by the judge after
examination under oath or affirmation of the complainant and the witnesses he may
produce, and particularly describing the place to be searched and the persons or things
to be seized.
Art 3, Sec. 3.
(1) The privacy of communication and correspondence shall be
inviolable except upon lawful order of the court, or when public safety or order requires
otherwise as prescribed by law.
(2)
Any evidence obtained in violation of this or the preceding section shall be
inadmissible for any purpose in any proceeding.

PEOPLE VS. MARTI


[193 SCRA 57; G.R. NO. 81561; 18 JAN 1991]
Facts:
Accused-appellant went to a forwarding agency to send four packages to a friend in
Zurich. Initially, the accused was asked by the proprietress if the packages can be
examined. However, he refused. Before delivering said packages to the Bureau of
Customs and the Bureau of Posts, the husband of the proprietress opened said boxes
for final inspection. From that inspection, included in the standard operating procedure
and out of curiosity, he took several grams of its contents.
He brought a letter and the said sample to the National Bureau of Investigation. When
the NBI was informed that the rest of the shipment was still in his office, three agents
went back with him. In their presence, the husband totally opened the packages.
Afterwards, the NBI took custody of said packages. The contents , after examination by
forensic chemists, were found to be marijuana flowering tops.
The appellant, while claiming his mail at the Central Post Office, was invited by the
agents for questioning. Later on, the trial court found him guilty of violation of the
Dangerous Drugs Act.
Issue:

Page 104

Whether or Not the items admitted in the searched illegally searched and seized.
Whether or Not custodial investigation properly applied.
Whether or Not the trial court not give credence to the explanation of the appellant on
how said packages came to his possession.

Held:
No. The case at bar assumes a peculiar character since the evidence sought to be
excluded was primarily discovered and obtained by a private person, acting in a private
capacity and without the intervention and participation of State authorities. Under the
circumstances, can accused/appellant validly claim that his constitutional right against
unreasonable searches and seizure has been violated. Stated otherwise, may an act of
a private individual, allegedly in violation of appellant's constitutional rights, be invoked
against the State. In the absence of governmental interference, the liberties
guaranteed by the Constitution cannot be invoked against the State. It was Mr. Job
Reyes, the proprietor of the forwarding agency, who made search/inspection of the
packages. Said inspection was reasonable and a standard operating procedure on the
part of Mr. Reyes as a precautionary measure before delivery of packages to the Bureau
of Customs or the Bureau of Posts. Second, the mere presence of the NBI agents did not
convert the reasonable search effected by Reyes into a warrantless search and seizure
proscribed by the Constitution. Merely to observe and look at that which is in plain sight
is not a search. Having observed that which is open, where no trespass has been
committed in aid thereof, is not search.
No. The law enforcers testified that accused/appellant was informed of his
constitutional rights. It is presumed that they have regularly performed their duties
(See. 5(m), Rule 131) and their testimonies should be given full faith and credence,
there being no evidence to the contrary.
No. Appellant signed the contract as the owner and shipper thereof giving more weight
to the presumption that things which a person possesses, or exercises acts of
ownership over, are owned by him (Sec. 5 [j], Rule 131). At this point, appellant is
therefore estopped to claim otherwise.

WATEROUS DRUG VS. NLRC


[280 SCRA 735 ; G.R.NO. 113271; 16 OCT 1997]
Facts:
Catolico was hired as a pharmacist by petitioner Waterous Drug Corporation on 15
August 1988. On 31 July 1989, Catolico received a memorandum from WATEROUS Vice
President-General Manager Emma R. Co warning her not to dispense medicine to
employees chargeable to the latter's accounts because the same was a prohibited
practice. On the same date, Co issued another memorandum to Catolico warning her
Page 105

not to negotiate with suppliers of medicine without consulting the Purchasing


Department, as this would impair the company's control of purchases and, besides she
was not authorized to deal directly with the suppliers.
As regards the first memorandum, Catolico did not deny her responsibility but
explained that her act was "due to negligence," since fellow employee Irene Soliven
"obtained the medicines in bad faith and through misrepresentation when she claimed
that she was given a charge slip by the Admitting Dept." Catolico then asked the
company to look into the fraudulent activities of Soliven.
In a memorandum dated 21 November 1989, WATEROUS Supervisor Luzviminda E.
Bautro warned Catolico against the "rush delivery of medicines without the proper
documents." On 29 January 1990, WATEROUS Control Clerk Eugenio Valdez informed Co
that he noticed an irregularity involving Catolico and Yung Shin Pharmaceuticals, Inc.
Forthwith, in her memorandum dated 37 January 1990, Co asked Catolico to explain,
within twenty-four hours, her side of the reported irregularity. Catolico asked for
additional time to give her explanation, and she was granted a 48-hour extension from
1 to 3 February 1990. However, on 2 February 1990, she was informed that effective 6
February 1990 to 7 March 1990, she would be placed on preventive suspension to
protect the interests of the company.
In a letter dated 2 February 1990, Catolico requested access to the file containing Sales
Invoice No. 266 for her to be able to make a satisfactory explanation. In said letter she
protested Saldaa's invasion of her privacy when Saldaa opened an envelope
addressed to Catolico.
In a letter to Co dated 10 February 1990, Catolico, through her counsel, explained that
the check she received from YSP was a Christmas gift and not a "refund of overprice."
She also averred that the preventive suspension was ill-motivated, as it sprang from an
earlier incident between her and Co's secretary, Irene Soliven.
On 5 March 1990, WATEROUS Supervisor Luzviminda Bautro, issued a memorandum
notifying Catolico of her termination. On 5 May 1990, Catolico filed before the Office of
the Labor Arbiter a complaint for unfair labor practice, illegal dismissal, and illegal
suspension. In his decision of 10 May 1993, Labor Arbiter Alex Arcadio Lopez found no
proof of unfair labor practice against petitioners. Nevertheless, he decided in favor of
Catolico because petitioners failed to "prove what alleged as complainant's
dishonesty," and to show that any investigation was conducted. Hence, the dismissal
was without just cause and due process. He thus declared the dismissal and suspension
illegal but disallowed reinstatement.
Petitioners seasonably appealed from the decision and urged the NLRC to set it aside
because the Labor Arbiter erred in finding that Catolico was denied due process and
that there was no just cause to terminate her services.
In its decision of 30 September 1993, the NLRC affirmed the findings of the Labor
Arbiter on the ground that petitioners were not able to prove a just cause for Catolico's
dismissal from her employment. It found that petitioner's evidence consisted only of
the check of P640.00 drawn by YSP in favor of complainant, which her co-employee saw
when the latter opened the envelope. But, it declared that the check was inadmissible
in evidence pursuant to Sections 2 and 3(1 and 2) of Article III of the Constitution. It
concluded:
Page 106

With the smoking gun evidence of respondents being rendered


inadmissible, by virtue of the constitutional right invoked by complainants,
respondents' case falls apart as it is bereft of evidence which cannot be
used as a legal basis for complainant's dismissal.
The NLRC
portion of
same was
amount of

then dismissed the appeal for lack of merit, but modified the dispositive
the appealed decision by deleting the award for illegal suspension as the
already included in the computation of the aggregate of the awards in the
P35,401.86.

Issue:
Whether or Not the dismissal of the private respondent is in violation of the
Constitution, under the Bill of Rights.
Held:
As to the first and second grounds, petitioners insist that Catolico had been receiving
"commissions" from YSP, or probably from other suppliers, and that the check issued to
her on 9 November 1989 was not the first or the last. They also maintained that
Catolico occupied a confidential position and that Catolico's receipt of YSP's check,
aggravated by her "propensity to violate company rules," constituted breach of
confidence. And contrary to the findings of NLRC, Catolico was given ample opportunity
to explain her side of the controversy.
In her Comment, Catolico asserts that petitioners' evidence is too "flimsy" to justify her
dismissal. The check in issue was given to her, and she had no duty to turn it over to
her employer. Company rules do not prohibit an employee from accepting gifts from
clients, and there is no indication in the contentious check that it was meant as a
refund for overpriced medicines. Besides, the check was discovered in violation of the
constitutional provision on the right to privacy and communication; hence, as correctly
held by the NLRC, it was inadmissible in evidence.
Catolico was denied due process. Procedural due process requires that an employee be
apprised of the charge against him, given reasonable time to answer the charge,
allowed ample opportunity to be heard and defend himself, and assisted by a
representative if the employee so desires. Ample opportunity connotes every kind of
assistance that management must accord the employee to enable him to prepare
adequately for his defense, including legal representation. In the case at bar, although
Catolico was given an opportunity to explain her side, she was dismissed from the
service in the memorandum of 5 March 1990 issued by her Supervisor after receipt of
her letter and that of her counsel. No hearing was ever conducted after the issues were
joined through said letters.
Catolico was also unjustly dismissed. It is settled that the burden is on the employer to
prove just and valid cause for dismissing an employee, and its failure to discharge that
burden would result in a finding that the dismissal is unjustified. It clearly appears then
that Catolico's dismissal was based on hearsay information. Catolico's dismissal then
was obviously grounded on mere suspicion, which in no case can justify an employee's
Page 107

dismissal. Suspicion is not among the valid causes provided by the Labor Code for the
termination of employment; and even the dismissal of an employee for loss of trust and
confidence must rest on substantial grounds and not on the employer's arbitrariness,
whims, caprices, or suspicion. Besides, Catolico was not shown to be a managerial
employee, to which class of employees the term "trust and confidence" is restricted.
As regards the constitutional violation upon which the NLRC anchored its decision, that
the Bill of Rights does not protect citizens from unreasonable searches and seizures
perpetrated by private individuals. It is not true, as counsel for Catolico claims, that the
citizens have no recourse against such assaults. On the contrary, and as said counsel
admits, such an invasion gives rise to both criminal and civil liabilities.
Finally, since it has been determined by the Labor Arbiter that Catolico's reinstatement
would not be to the best interest of the parties, he correctly awarded separation pay to
Catolico. Separation pay in lieu of reinstatement is computed at one month's salary for
every year of service. In this case, however, Labor Arbiter Lopez computed the
separation pay at one-half month's salary for every year of service. Catolico did not
oppose or raise an objection. As such, we will uphold the award of separation pay as
fixed by the Labor Arbiter.
WHEREFORE, the instant petition is hereby DISMISSED and the challenged decision and
resolution of the National Labor Relations Commission dated 30 September 1993 and 2
December 1993, respectively, in NLRC-NCR CA No. 005160-93 are AFFIRMED, except as
to its reason for upholding the Labor Arbiter's decision, viz., that the evidence against
private respondent was inadmissible for having been obtained in violation of her
constitutional rights of privacy of communication and against unreasonable searches
and seizures which is hereby set aside.

STONEHILL VS. DIOKNO


[20 SCRA 383; L-19550; 19 JUN 1967]
Facts:
Upon application of the officers of the government named on the margin 1 hereinafter
referred to as Respondents-Prosecutors several judges 2 hereinafter referred to as
Respondents-Judges issued, on different dates, 3 a total of 42 search warrants against
petitioners herein4 and/or the corporations of which they were officers, 5 directed to the
any peace officer, to search the persons above-named and/or the premises of their
offices, warehouses and/or residences, and to seize and take possession of the
following personal property to wit:
Books of accounts, financial records, vouchers, correspondence, receipts,
ledgers, journals, portfolios, credit journals, typewriters, and other documents
and/or papers showing all business transactions including disbursements
receipts, balance sheets and profit and loss statements and Bobbins (cigarette
wrappers).
as "the subject of the offense; stolen or embezzled and proceeds or fruits of the
offense," or "used or intended to be used as the means of committing the offense,"
which is described in the applications adverted to above as "violation of Central Bank
Laws, Tariff and Customs Laws, Internal Revenue (Code) and the Revised Penal Code."
Page 108

Petitioners contentions are:


(1) they do not describe with particularity the documents, books and things to be
seized;
(2) cash money, not mentioned in the warrants, were actually seized;
(3) the warrants were issued to fish evidence against the aforementioned petitioners in
deportation cases filed against them;
(4) the searches and seizures were made in an illegal manner; and
(5) the documents, papers and cash money seized were not delivered to the courts
that issued the warrants, to be disposed of in accordance with law
Respondents-prosecutors contentions
(1) that the contested search warrants are valid and have been issued in accordance
with law;
(2) that the defects of said warrants, if any, were cured by petitioners' consent; and
(3) that, in any event, the effects seized are admissible in evidence against herein
petitioners, regardless of the alleged illegality of the aforementioned searches and
seizures.
The documents, papers, and things seized under the alleged authority of the warrants
in question may be split into two (2) major groups, namely: (a) those found and seized
in the offices of the aforementioned corporations, and (b) those found and seized in the
residences of petitioners herein.
Issue:
Whether or not those found and seized in the offices of the aforementioned
corporations are obtained legally.
Whether or not those found and seized in the residences of petitioners herein are
obtained legally.
Held:
The petitioners have no cause of action to assail the legality of the contested warrants
and of the seizures made in pursuance thereof, for the simple reason that said
corporations have their respective personalities, separate and distinct from the
personality of herein petitioners, regardless of the amount of shares of stock or of the
interest of each of them in said corporations, and whatever the offices they hold therein
may be. Indeed, it is well settled that the legality of a seizure can be contested only by
the party whose rights have been impaired thereby, and that the objection to an
unlawful search and seizure is purely personal and cannot be availed of by third parties.
With respect to the documents, papers and things seized in the residences of
petitioners herein, the aforementioned resolution of June 29, 1962, lifted the writ of
preliminary injunction previously issued by this Court, thereby, in effect, restraining
herein Respondents-Prosecutors from using them in evidence against petitioners
herein.
Page 109

Two points must be stressed in connection with this constitutional mandate, namely: (1)
that no warrant shall issue but upon probable cause, to be determined by the judge in
the manner set forth in said provision; and (2) that the warrant shall particularly
describe the things to be seized.
None of these requirements has been complied with in the contested warrants. Indeed,
the same were issued upon applications stating that the natural and juridical person
therein named had committed a "violation of Central Ban Laws, Tariff and Customs
Laws, Internal Revenue (Code) and Revised Penal Code." In other words, no specific
offense had been alleged in said applications. The averments thereof with respect to
the offense committed were abstract. As a consequence, it was impossible for the
judges who issued the warrants to have found the existence of probable cause, for the
same presupposes the introduction of competent proof that the party against whom it
is sought has performed particular acts, or committed specific omissions, violating a
given provision of our criminal laws. As a matter of fact, the applications involved in
this case do not allege any specific acts performed by herein petitioners. It would be
the legal heresy, of the highest order, to convict anybody of a "violation of Central Bank
Laws, Tariff and Customs Laws, Internal Revenue (Code) and Revised Penal Code," as
alleged in the aforementioned applications without reference to any determinate
provision of said laws or
__________________________
1

Hon. Jose W. Diokno, in his capacity as Secretary of Justice, Jose Lukban, in his capacity
as Acting Director, National Bureau of Investigation, Special Prosecutors Pedro D.
Cenzon, Efren I. Plana and Manuel Villareal, Jr. and Assistant Fiscal Maneses G. Reyes,
City of Manila.
2
Hon. Amado Roan, Judge of the Municipal (now City) Court of Manila, Hon. Roman
Cansino, Judge of the Municipal (now City) Court of Manila, Hon. Hermogenes Caluag,
Judge of the Court of First Instance of Rizal, Quezon City Branch, Hon. Eulogio Mencias,
Judge of the Court of First Instance of Rizal, Pasig Branch, and Hon. Damian Jimenez,
Judge of the Municipal (now City) Court of Quezon City.
3
Covering the period from March 3 to March 9, 1962.
4
Harry S. Stonehill, Robert P. Brooks, John J. Brooks and Karl Beck.
5
U.S. Tobacco Corporation, Atlas Cement Corporation, Atlas Development Corporation,
Far East Publishing Corporation (Evening News), Investment Inc., Industrial Business
Management Corporation, General Agricultural Corporation, American Asiatic Oil
Corporation, Investment Management Corporation, Holiday Hills, Inc., Republic Glass
Corporation, Industrial and Business Management Corporation, United Housing
Corporation, The Philippine Tobacco-Flue-Curing and Redrying Corporation, Republic
Real Estate Corporation and Merconsel Corporation.

BURGOS, SR. V. CHIEF OF STAFF, AFP


[133 SCRA 800; G.R. NO. 64261; 26 DEC 1984]
Facts:
Petitioners assail the validity of 2 search warrants issued on December 7, 1982 by
respondent Judge Cruz-Pano of the then Court of First Instance of Rizal, under which the
premises known as No. 19, Road 3, Project 6, Quezon City, and 784 Units C & D, RMS
Page 110

Building, Quezon Avenue, Quezon City, business addresses of the "Metropolitan Mail"
and "We Forum" newspapers, respectively, were searched, and office and printing
machines, equipment, paraphernalia, motor vehicles and other articles used in the
printing, publication and distribution of the said newspapers, as well as numerous
papers, documents, books and other written literature alleged to be in the possession
and control of petitioner Jose Burgos, Jr. publisher-editor of the "We Forum" newspaper,
were seized. As a consequence of the search and seizure, these premises were
padlocked and sealed, with the further result that the printing and publication of said
newspapers were discontinued. Respondents contend that petitioners should have filed
a motion to quash said warrants in the court that issued them before impugning the
validity of the same before this Court. Respondents also assail the petition on ground
of laches (Failure or negligence for an unreasonable and unexplained length of time to
do that which, by exercising due diligence, could or should have been done earlier. It is
negligence or omission to assert a right within a reasonable time, warranting a
presumption that the party entitled to assert it either has abandoned it or declined to
assert it). Respondents further state that since petitioner had already used as evidence
some of the documents seized in a prior criminal case, he is stopped from challenging
the validity of the search warrants.
Petitioners submit the following reasons to nullify the questioned warrants:
1. Respondent Judge failed to conduct an examination under oath or affirmation of
the applicant and his witnesses, as mandated by the above-quoted constitutional
provision as well as Sec. 4, Rule 126 of the Rules of Court.
2. The search warrants pinpointed only one address which would be the former
abovementioned address.
3. Articles belonging to his co-petitioners were also seized although the warrants
were only directed against Jose Burgos, Jr.
4. Real properties were seized.
5. The application along with a joint affidavit, upon which the warrants were issued,
from the Metrocom Intelligence and Security Group could not have provided
sufficient basis for the finding of a probable cause upon which a warrant may be
validly issued in accordance with Section 3, Article IV of the 1973 Constitution.
Respondents justify the continued sealing of the printing machines on the ground that
they have been sequestered under Section 8 of Presidential Decree No. 885, as
amended, which authorizes sequestration of the property of any person engaged in
subversive activities against the government in accordance with implementing rules
and regulations as may be issued by the Secretary of National Defense.
Issue:
Whether or Not the 2 search warrants were validly issued and executed.
Held:
In regard to the quashal of warrants that petitioners should have initially filed to the
lower court, this Court takes cognizance of this petition in view of the seriousness and
Page 111

urgency of the constitutional Issue raised, not to mention the public interest generated
by the search of the "We Forum" offices which was televised in Channel 7 and widely
publicized in all metropolitan dailies. The existence of this special circumstance justifies
this Court to exercise its inherent power to suspend its rules. With the contention
pertaining to laches, the petitioners gave an explanation evidencing that they have
exhausted other extra-judicial efforts to remedy the situation, negating the
presumption that they have abandoned their right to the possession of the seized
property.
On the enumerated reasons:
1. This objection may properly be considered moot and academic, as petitioners
themselves conceded during the hearing on August 9, 1983, that an examination
had indeed been conducted by respondent judge of Col. Abadilla and his
witnesses.
2. The defect pointed out is obviously a typographical error. Precisely, two search
warrants were applied for and issued because the purpose and intent were to
search two distinct premises. It would be quite absurd and illogical for
respondent judge to have issued two warrants intended for one and the same
place.
3. Section 2, Rule 126, of the Rules of Court, does not require that the property to
be seized should be owned by the person against whom the search warrant is
directed. It may or may not be owned by him.
4. Petitioners do not claim to be the owners of the land and/or building on which the
machineries were placed. This being the case, the machineries in question,
while in fact bolted to the ground, remain movable property susceptible to
seizure under a search warrant.
5. The broad statements in the application and joint affidavit are mere conclusions
of law and does not satisfy the requirements of probable cause. Deficient of such
particulars as would justify a finding of the existence of probable cause, said
allegation cannot serve as basis for the issuance of a search warrant and it was a
grave error for respondent judge to have done so. In Alvarez v. Court of First
Instance, this Court ruled that "the oath required must refer to the truth of the
facts within the personal knowledge of the petitioner or his witnesses, because
the purpose thereof is to convince the committing magistrate, not the individual
making the affidavit and seeking the issuance of the warrant, of the existence of
probable cause."
Another factor which makes the search warrants under
consideration constitutionally objectionable is that they are in the nature of
general warrants. The description of the articles sought to be seized under the
search warrants in question are too general.
With regard to the respondents invoking PD 885, there is an absence of any
implementing rules and regulations promulgated by the Minister of National Defense.
Furthermore, President Marcos himself denies the request of military authorities to
sequester the property seized from petitioners. The closure of the premises subjected
to search and seizure is contrary to the freedom of the press as guaranteed in our
fundamental law. The search warrants are declared null and void.

TAMBASEN VS. PEOPLE


Page 112

[246 SCRA 184; G.R. NO. 89103; 14 JUL 1995]


Facts:
In August 1988, P/Sgt. Natuel applied for issuance of search warrant alleging that he
received information that Petitioner had in his possession at his house M-16 Armalite
rifles, hand grenades, .45 Cal. pistols, dynamite sticks and subversive documents,
which were used or intended to be used for illegal purposes. The application was
granted.
In September, a police team, searched the house of petitioner and seized 2 envelopes
containing P14000, handset with antennae, transceiver with antennae, regulator
supply, academy notebook and assorted papers and handset battery pack. In
October, petitioner moved that the search and seizure be declared illegal and that the
seized articles be returned to him. In December, MTCC, in its order, directed Lt. Col.
Torres to return the money seized to petitioner ruling that any seizure should be limited
to the specified items covered thereby. SolGen petitioned with the RTC for the
annulment of the order of MTCC citing that pending the determination of legality of
seizure of the articles, they should remain in custogia legis. RTC granted the petition.
Issue:
Whether or Not the seizure of the articles which were not mentioned in the search
warrant was legal.
Held:
Section 2 Article III of the 1987 Constitution requires that a search warrant should
particularly describe the things to be seized. The police acts beyond the parameters of
their authority if they seize articles not described in the search warrants. The evident
purpose and intent of the requirement is to limit the things to be seized, to leave the
officers of the law with no discretion; that unreasonable search and seizure may not be
made and that abuses may not be committed.
Petition granted. People of the Philippines is ordered to return the money seized.

PLACER VS. JUDGE VILLANUEVA


[126 SCRA 463; G.R. NOS. L-60349-62; 29 DEC 1983]
Facts:
Page 113

Petitioners filed informations in the city court and they certified that Preliminary
Investigation and Examination had been conducted and that prima facie cases have
been found. Upon receipt of said informations, respondent judge set the hearing of the
criminal cases to determine propriety of issuance of warrants of arrest. After the
hearing, respondent issued an order requiring petitioners to submit to the court
affidavits of prosecution witnesses and other documentary evidence in support of the
informations to aid him in the exercise of his power of judicial review of the findings of
probable cause by petitioners. Petitioners petitioned for certiorari and mandamus to
compel respondent to issue warrants of arrest. They contended that the fiscals
certification in the informations of the existence of probable cause constitutes sufficient
justification for the judge to issue warrants of arrest.
Issue:
Whether or Not respondent city judge may, for the purpose of issuing warrants of
arrest, compel the fiscal to submit to the court the supporting affidavits and other
documentary evidence presented during the preliminary investigation.
Held:
Judge may rely upon the fiscals certification for the existence of probable cause and on
the basis thereof, issue a warrant of arrest. But, such certification does not bind the
judge to come out with the warrant. The issuance of a warrant is not a mere ministerial
function; it calls for the exercise of judicial discretion on the part of issuing magistrate.
Under Section 6 Rule 112 of the Rules of Court, the judge must satisfy himself of the
existence of probable cause before issuing a warrant of arrest. If on the face of the
information, the judge finds no probable cause, he may disregard the fiscals
certification and require submission of the affidavits of witnesses to aid him in arriving
at the conclusion as to existence of probable cause.
Petition dismissed.

SOLIVEN VS. MAKASIAR


[167 SCRA 393; G.R. NO. 82585; 14 NOV 1988]
Facts:
In these consolidated cases, three principal issues were raised: (1) whether or not
petitioners were denied due process when informations for libel were filed against them
although the finding of the existence of a prima facie case was still under review by the
Secretary of Justice and, subsequently, by the President; and (2) whether or not the
constitutional rights of Beltran were violated when respondent RTC judge issued a
Page 114

warrant for his arrest without personally examining the complainant and the witnesses,
if any, to determine probable cause. Subsequent events have rendered the first issue
moot and academic. On March 30, 1988, the Secretary of Justice denied petitioners'
motion for reconsideration and upheld the resolution of the Undersecretary of Justice
sustaining the City Fiscal's finding of a prima facie case against petitioners. A second
motion for reconsideration filed by petitioner Beltran was denied by the Secretary of
Justice on April 7, 1988. On appeal, the President, through the Executive Secretary,
affirmed the resolution of the Secretary of Justice on May 2, 1988. The motion for
reconsideration was denied by the Executive Secretary on May 16, 1988. With these
developments, petitioners' contention that they have been denied the administrative
remedies available under the law has lost factual support.
Issue:
Whether or Not petitioners were denied due process when informations for libel were
filed against them although the finding of the existence of a prima facie case was still
under review by the Secretary of Justice and, subsequently, by the President.
Whether or Not the constitutional rights of Beltran were violated when respondent RTC
judge issued a warrant for his arrest without personally examining the complainant and
the witnesses, if any, to determine probable cause
Held:
With respect to petitioner Beltran, the allegation of denial of due process of law in the
preliminary investigation is negated by the fact that instead of submitting his counteraffidavits, he filed a "Motion to Declare Proceedings Closed," in effect waiving his right
to refute the complaint by filing counter-affidavits. Due process of law does not require
that the respondent in a criminal case actually file his counter-affidavits before the
preliminary investigation is deemed completed. All that is required is that the
respondent be given the opportunity to submit counter-affidavits if he is so minded.

The second issue, raised by petitioner Beltran, calls for an interpretation of the
constitutional provision on the issuance of warrants of arrest. The pertinent provision
reads:
Art. III, Sec. 2. The right of the people to be secure in their persons,
houses, papers and effects against unreasonable searches and seizures of
whatever nature and for any purpose shall be inviolable, and no search
warrant or warrant of arrest shall issue except upon probable cause to be
determined personally by the judge after examination nder oath or
affirmation of the complainant and the witnesses he may produce, and
particularly describing the place to be searched and the persons or things
to be seized.
The addition of the word "personally" after the word "determined" and the deletion of
the grant of authority by the 1973 Constitution to issue warrants to "other responsible
officers as may be authorized by law," has apparently convinced petitioner Beltran that
the Constitution now requires the judge to personally examine the complainant and his
Page 115

witnesses in his determination of probable cause for the issuance of warrants of arrest.
This is not an accurate interpretation.
What the Constitution underscores is the exclusive and personal responsibility of the
issuing judge to satisfy himself of the existence of probable cause. In satisfying himself
of the existence of probable cause for the issuance of a warrant of arrest, the judge is
not required to personally examine the complainant and his witnesses. Following
established doctrine and procedure, he shall: (1) personally evaluate the report and the
supporting documents submitted by the fiscal regarding the existence of probable
cause and, on the basis thereof, issue a warrant of arrest; or (2) if on the basis thereof
he finds no probable cause, he may disregard the fiscal's report and require the
submission of supporting affidavits of witnesses to aid him in arriving at a conclusion as
to the existence of probable cause.
Sound policy dictates this procedure, otherwise judges would be unduly laden with the
preliminary examination and investigation of criminal complaints instead of
concentrating on hearing and deciding cases filed before their courts. It has not been
shown that respondent judge has deviated from the prescribed procedure. Thus, with
regard to the issuance of the warrants of arrest, a finding of grave abuse of discretion
amounting to lack or excess of jurisdiction cannot be sustained. The petitions fail to
establish that public respondents, through their separate acts, gravely abused their
discretion as to amount to lack of jurisdiction. Hence, the writs of certiorari and
prohibition prayed for cannot issue.
WHEREFORE, finding no grave abuse of discretion amounting to excess or lack of
jurisdiction on the part of the public respondents, the Court Resolved to DISMISS the
petitions in G. R. Nos. 82585, 82827 and 83979. The Order to maintain the status quo
contained in the Resolution of the Court en banc dated April 7, 1988 and reiterated in
the Resolution dated April 26, 1988 is LIFTED.
SALAZAR VS. ACHACOSO
[183 SCRA 145; G.R. NO. 81510; 14 MAR 1990]
Facts:
Rosalie Tesoro of Pasay City in a sworn statement filed with the POEA, charged
petitioner with illegal recruitment. Public respondent Atty. Ferdinand Marquez sent
petitioner a telegram directing him to appear to the POEA regarding the complaint
against him. On the same day, after knowing that petitioner had no license to operate a
recruitment agency, public respondent Administrator Tomas Achacoso issued a Closure
and Seizure Order No. 1205 to petitioner. It stated that there will a seizure of the
documents and paraphernalia being used or intended to be used as the means of
committing illegal recruitment, it having verified that petitioner has (1) No valid
license or authority from the Department of Labor and Employment to recruit and
deploy workers for overseas employment; (2) Committed/are committing acts
prohibited under Article 34 of the New Labor Code in relation to Article 38 of the same
code. A team was then tasked to implement the said Order. The group, accompanied by
mediamen and Mandaluyong policemen, went to petitioners residence. They served
the order to a certain Mrs. For a Salazar, who let them in. The team confiscated
Page 116

assorted costumes. Petitioner filed with POEA a letter requesting for the return of the
seized properties, because she was not given prior notice and hearing. The said Order
violated due process. She also alleged that it violated sec 2 of the Bill of Rights, and the
properties were confiscated against her will and were done with unreasonable force and
intimidation.
Issue:
Whether or Not the Philippine Overseas Employment Administration (or the Secretary of
Labor) can validly issue warrants of search and seizure (or arrest) under Article 38 of
the Labor Code
Held:
Under the new Constitution, . . . no search warrant or warrant of arrest shall issue
except upon probable cause to be determined personally by the judge after
examination under oath or affirmation of the complainant and the witnesses he may
produce, and particularly describing the place to be searched and the persons or things
to be seized. Mayors and prosecuting officers cannot issue warrants of seizure or
arrest. The Closure and Seizure Order was based on Article 38 of the Labor Code. The
Supreme Court held, We reiterate that the Secretary of Labor, not being a judge, may
no longer issue search or arrest warrants. Hence, the authorities must go through the
judicial process. To that extent, we declare Article 38, paragraph (c), of the Labor Code,
unconstitutional and of no force and effect The power of the President to order the
arrest of aliens for deportation is, obviously, exceptional. It (the power to order arrests)
cannot be made to extend to other cases, like the one at bar. Under the Constitution, it
is the sole domain of the courts. Furthermore, the search and seizure order was in the
nature of a general warrant. The court held that the warrant is null and void, because it
must identify specifically the things to be seized.

WHEREFORE, the petition is GRANTED. Article 38, paragraph (c) of the Labor Code is
declared UNCONSTITUTIONAL and null and void. The respondents are ORDERED to
return all materials seized as a result of the implementation of Search and Seizure
Order No. 1205.

MORANO VS. VIVO


[20 SCRA 562; G.R. L-22196; 30 JUN 1967]
Facts:

Page 117

Chan Sau Wah, a Chinese citizen born in Fukien, China arrived in the Philippines on
November 1961 to visit her cousin, Samuel Lee Malaps. She left China and her children
by a first marriage: Fu Tse Haw and Fu Yan Kai both minors, in the care of neighbors in
Fukien, China. Chan Sau wah arrived in the Philippines with Fu Yan Fun, her minor son
also by the first marriage. Chan Sau Wah and her minor son Fu Yan Fun were permitted
only into the Philippines under a temporary visitor's visa for two months and after they
posted a cash bond of 4,000 pesos. On January 1962, Chan Sau Wah married Esteban
Morano, a native-born Filipino citizen. Born to this union on September 1962 was
Esteban Morano, Jr. To prolong their stay in the Philippines, Chan Sau Wah and Fu Yan
Fun obtained several extensions. The last extension expired on September 10, 1962. In
a letter dated August 31, 1962, the Commissioner of Immigration ordered Chan Sau
Wah and her son, Fu Yan Fun, to leave the country on or before September 10, 1962
with a warning that upon failure so to do, he will issue a warrant for their arrest and will
cause the confiscation of their bond.
Issue:
Whether or Not the issuance of the warrant of arrest is unconstitutional.
Held:
Chan Sau Wah entered the Philippines on a tourist-temporary visitor's visa. She is a
non-immigrant. Under Section 13 just quoted, she may therefore be admitted if she
were a qualified and desirable alien and subject to the provisions of the last paragraph
of Section 9. Therefore, first, she must depart voluntarily to some foreign country;
second, she must procure from the appropriate consul the proper visa; and third, she
must thereafter undergo examination by the officials of the Bureau of Immigration at
the port of entry for determination of her admissibility in accordance with the
requirements of the immigration Act. This Court in a number of cases has ruled, and
consistently too, that an alien admitted as a temporary visitor cannot change his or her
status without first departing from the country and complying with the requirements of
Section 9 of the Immigration Act. The gravamen of petitioners' argument is that Chan
Sau Wah has, since her entry, married in Manila a native-born Filipino, Esteban Morano.
It will not particularly help analysis for petitioners to appeal to family solidarity in an
effort to thwart her deportation. Chan Sau Wah, seemingly is not one who has a high
regard for such solidarity. Proof: She left two of her children by the first marriage, both
minors, in the care of neighbors in Fukien, China.Then, the wording of the statute
heretofore adverted to is a forbidding obstacle which will prevent this Court from
writing into the law an additional provision that marriage of a temporary alien visitor to
a Filipino would ipso facto make her a permanent resident in his country. This is a field
closed to judicial action. No breadth of discretion is allowed. We cannot insulate her
from the State's power of deportation. it would be an easy matter for an alien woman to
enter the Philippines as a temporary visitor, go through a mock marriage, but actually
live with another man as husband and wife, and thereby skirt the provisions of our
immigration law. Also, a woman of undesirable character may enter this country, ply a
pernicious trade, marry a Filipino, and again throw overboard Sections 9 and 13 of the
Act. Such a flanking movement, we are confident, is impermissible.Recently we
Page 118

confirmed the rule that an alien wife of a Filipino may not stay permanently without first
departing from the Philippines. Reason: Discourage entry under false pretenses.

HARVEY V. DEFENSOR-SANTIAGO
[162 SCRA 840; G.R. NO. 82544; 28 JUN 1988]
Facts:
This is a petition for Habeas Corpus. Petitioners are the following: American nationals
Andrew Harvey, 52 and Jonh Sherman 72. Dutch Citizen Adriaan Van Den Elshout, 58.
All reside at Pagsanjan Laguna respondent Commissioner Miriam Defensor Santiago
issued Mission Orders to the Commission of Immigration and Deportation (CID) to
apprehended petitioners at their residences. The Operation Report read that Andrew
Harvey was found together with two young boys. Richard Sherman was found with two
naked boys inside his room. While Van Den Elshout in the after Mission Report read
that two children of ages 14 and 16 has been under his care and subjects confirmed
being live-in for sometime now.
Seized during the petitioners apprehension were rolls of photo negatives and photos of
suspected child prostitutes shown in scandalous poses as well as boys and girls
engaged in sex. Posters and other literature advertising the child prostitutes were also
found.
Petitioners were among the 22 suspected alien pedophiles. They were apprehended 17
February1988 after close surveillance for 3 month of the CID in Pagsanjan, Laguna. 17
of the arrested aliens opted for self-deportation. One released for lack of evidence,
another charged not for pedophile but working with NO VISA, the 3 petitioners chose to
face deportation proceedings.
On 4 March1988, deportation proceedings were
instituted against aliens for being undesirable aliens under Sec.69 of Revised
Administrative Code.
Warrants of Arrest were issued 7March1988 against petitioners for violation of Sec37,
45 and 46 of Immigration Act and sec69 of Revised Administrative Code. Trial by the
Board of Special Inquiry III commenced the same date. Petition for bail was filed
11March 1988 but was not granted by the Commissioner of Immigration. 4 April1988
Petitioners filed a petition for Writ of Habeas Corpus. The court heard the case on oral
argument on 20 April 1988.

Issue:
Page 119

Whether or Not the Commissioner has the power to arrest and detain petitioners
pending determination of existence of probable cause.
Whether or Not there was unreasonable searches and seizures by CID agents.
Whether or Not the writ of Habeas Corpus may be granted to petitioners.
Held:
While pedophilia is not a crime under the Revised Penal Code, it violates the declared
policy of the state to promote and protect the physical, moral, spiritual and social well
being of the youth. The arrest of petitioners was based on the probable cause
determined after close surveillance of 3 months. The existence of probable cause
justified the arrest and seizure of articles linked to the offense. The articles were seized
as an incident to a lawful arrest; therefore the articles are admissible evidences (Rule
126, Section12 of Rules on Criminal Procedure).
The rule that search and seizures must be supported by a valid warrant of arrest is not
an absolute rule. There are at least three exceptions to this rule. 1.) Search is
incidental to the arrest. 2.) Search in a moving vehicle. 3.) Seizure of evidence in plain
view. In view of the foregoing, the search done was incidental to the arrest.
The filing of the petitioners for bail is considered as a waiver of any irregularity
attending their arrest and estops them from questioning its validity. Furthermore, the
deportation charges and the hearing presently conducted by the Board of Special
Inquiry made their detention legal. It is a fundamental rule that habeas corpus will not
be granted when confinement is or has become legal, although such confinement was
illegal at the beginning.
The deportation charges instituted by the Commissioner of Immigration are in
accordance with Sec37 (a) of the Philippine Immigration Act of 1940 in relation to sec69
of the Revised Administrative code. Section 37 (a) provides that aliens shall be
arrested and deported upon warrant of the Commissioner of Immigration and
Deportation after a determination by the Board of Commissioners of the existence of a
ground for deportation against them. Deportation proceedings are administrative in
character and never construed as a punishment but a preventive measure. Therefore,
it need not be conducted strictly in accordance with ordinary Court proceedings. What
is essential is that there should be a specific charge against the alien intended to be
arrested and deported. A fair hearing must also be conducted with assistance of a
counsel if desired.
Lastly, the power to deport aliens is an act of the State and done under the authority of
the sovereign power. It a police measure against the undesirable aliens whose
Page 120

continued presence in the country is found to be injurious to the public good and
tranquility of the people.

SALES VS. SANDIGANBAYAN


[369 SCRA 293 G.R. NO. 143802; 16 NOV 2001]
Facts:
The petitioner, the incumbent mayor of Pagudpud Ilocos Norte, shot the former mayor
and his political rival Atty. Benemerito. After the shooting, he surrendered himself and
hence the police inspector and wife of the victim filed a criminal complaint for murder
against him. The judge after conducting the preliminary examination (p.e. for brevity)
found probable cause and issued a warrant of arrest. Also after conducting the
preliminary investigation (p.i. for brevity), he issued a resolution forwarding the case to
the prosecutor for appropriate action. Petitioner received a subpoena directing him to
file his counter affidavit, affidavit of witnesses and other supporting documents. He did
it the following day. While proceedings are ongoing, he filed a petition for habeas
corpus with the C.A alleging that: the warrant was null and void because the judge who
issued it was a relative by affinity of the private respondent and the p.e. and the p.i.
were illegal and irregular as the judge doesnt have jurisdiction on the case. The C.A.
granted the petition holding that the judge was a relative by affinity by 3 rd degree to
the private respondent and the p.i. he conducted has 2 stages, the p.e. and the p.i.
proper. The proceeding now consists only of one stage. He conducted the requisite
investigation prior to the issuance of warrant of arrest. Moreover he did not complete it.
He only examined the witness of the complainant. But the prosecution instead of
conducting p.i. of his own forwarded the records to the Ombudsman (OMB for brevity)
for the latter to conduct the same. The OMB directed the petitioner to submit his
counter affidavit, but he did not comply with it finding the same superfluous. The graft
investigator recommended the filing of information for murder which the OMB
approved. Petitioner received a copy of the resolution but prevented seeking
reconsideration thereof he filed a motion to defer issuance of warrant of arrest pending
the determination of probable cause. The Sandiganbayan denied the motion. This is
now a petition for review on the decision of the Sandiganbayan,
Issue:
Whether or Not
investigation.

the

OMB

followed

the

procedure

in

conducting

preliminary

Whether or Not petitioner was afforded an opportunity to be heard and to submit


controverting evidence.

Page 121

Held:
The proper procedure in the conduct of preliminary investigation was not followed
because of the following reasons. Firstly, the preliminary investigation was conducted
by 3 different investigators, none of whom completed the preliminary investigation
There was not one continuous proceeding but rather, cases of passing the buck, the
last one being the OMB throwing the buck to the Sandiganbayan. Secondly, the charge
of murder is a non bailable offense. The gravity of the offense alone should have
merited a deeper and more thorough preliminary investigation. The OMB did nothing of
the sort but wallowed the resolution of the graft investigator. He did a worse job than
the judge, by actually adopting the resolution of the graft investigator without doing
anything and threw everything to the Sandiganbayan for evaluation. Thirdly, a person
under preliminary investigation by the OMB is entitled to a motion for reconsideration,
as maintained by the Rules of Procedure by the OMB. The filing of the motion for
reconsideration is an integral part of the preliminary investigation proper. The denial
thereof is tantamount to the denial of the right itself to a preliminary investigation. This
fact alone renders preliminary investigation conducted in this case incomplete. And
lastly, it was patent error for the Sandiganbayan to have relied purely on the OMBs
certification of probable cause given the prevailing facts of the case much more so in
the face of the latters flawed report and one side factual findings.
The court cannot accept the Sandiganbayans assertion of having found probable cause
on its own, considering the OMBs defective report and findings, which merely rekied on
the testimonies of the witnesses for the prosecution and disregarded the evidence for
the defense.
Judgment is rendered setting aside the resolution of the Sandiganbayan, ordering the
Sandiganbayan to quash the warrant of arrest and remanding the OMB for completion
of the preliminary investigation.

SILVA VS. PRESIDING JUDGE


[203 SCRA 140; G.R. No. 81756; 21 Oct 1991]
Facts:
Sgt. Villamor, chief of the PC Narcom Detachment in Dumaguete City filed an
"application for search warrant" and "Deposition of witness" against petitioner
Nicomedes Silva and Martin Silva. Judge Nickarter Ontal, then the presiding judge of
RTC of Dumaguete issued Search Warrant No.1 pursuant to the said applications for
violation of RA 6425 Dangerous Drugs ACT of 1972. Such warrant states that there is a
probable cause to believe that Mr. Tama Silva has the possession and control of
marijuana dried leaves, cigarette and joint. The warrant authorizes Sgt. Villamor to
make an immediate search at any time of the room of Mr. Tama Silva at the residence
of his father Comedes Silva and to open aparadors, lockers, cabinets, cartons and
Page 122

containers to look for said illegal drugs. In the course of the search, the officers seized
money belonging to Antonieta Silva in the amount of P1,231.40. Petitioner filed a
motion to quash Search Warrant No.1 on the ground that 1) it was issued on the sole
basis of mimeographed 2) the judge failed to personally examine the complainant and
witness by searching questions and answers.
Issue:
Whether or Not Search Warrant No.1 is invalid. WON the officers abused their authority
in seizing the money of Antonieta Silva.
Held:
Search Warrant No. 1 is invalid due to the failure of the judge to examine the witness in
the form of searching questions and answers. The questions asked were leading as they
are answerable by mere yes or no. Such questions are not sufficiently searching to
establish probable cause. The questions were already mimeographed and all the
witness had to do was fill in their answers on the blanks provided. Judge Ontal is guilty
of grave abuse of discretion when he rejected the motion of Antonieta Silva seeking the
return of her money.
The officers who implemented the search warrant clearly abused their authority when
they seized the money of Antonieta Silva. The warrant did not indicate the seizure of
money but only for marijuana leaves, cigarettes..etc. Search Warrant No. 1 is declared
null and void.
*** Sec 4 Rule 126 Rules of Court
Examination of the complainant, record -the judge before issuing the warrant,
personally examine in the form of searching questions and answers, in writing and
under oath the complainant and any witness he may produce the facts personally
known to them and attach to the record their sworn statements together with their
affidavits.

VEROY VS. LAYAGUE


[210 SCRA 97; G.R. No. 95630; 18 Jun 1992]
Facts:
Page 123

Petitioners are husband and wife who owned and formerly resided at No. 13 Isidro St.,
Skyline Village. Catalunan Grande, Davao City. When petitioner Leopoldo Veroy was
promoted to the position of Assistant Administrator of the Social Security System
sometime in June, 1988, he and his family transferred to 130 K-8th St., East Kamias,
Quezon City, where they are presently residing. The care and upkeep of their residence
in Davao City was left to two (2) houseboys, Jimmy Favia and Eric Burgos, who had their
assigned quarters at a portion of the premises. The Veroys would occasionally send
money to Edna Soguilon for the salary of the said houseboys and other expenses for
the upkeep of their house. While the Veroys had the keys to the interior of the house,
only the key to the kitchen, where the circuit breakers were located, was entrusted to
Edna Soguilon to give her access in case of an emergency. Hence, since 1988, the key
to the master's bedroom as well as the keys to the children's rooms were retained by
herein Petitioners so that neither Edna Soguilon nor the caretakers could enter the
house.
Police Officers had an information that the petitioners residence was being used as a
safehouse of rebel soldiers. They were able to enter the yard with the help of the
caretakers but did not enter the house since the owner was not present and they did
not have a search warrant. Petitioner Ma. Luisa was contacted by telephone in her
Quezon City residence by Capt. Obrero to ask permission to search the house in Davao
City as it was reportedly being used as a hideout and recruitment center of rebel
soldiers. Petitioner Ma. Luisa Veroy responded that she is flying to Davao City to witness
the search but relented if the search would not be conducted in the presence of Major
Ernesto Macasaet, an officer of the PC/INP, Davao City and a long time family friend of
the Veroys.
The following day, Capt. Obrero and Major Macasaet met at the house of herein
petitioners in Skyline Village to conduct the search pursuant to the authority granted by
petitioner Ma. Luisa Veroy. The caretakers facilitated their entry into the yard, and using
the key entrusted to Edna Soguilon, they were able to gain entrance into the kitchen.
However, a locksmith by the name of George Badiang had to be employed to open the
padlock of the door leading to the children's room. Capt. Obrero and Major Macasaet
then entered the children's room and conducted the search. Capt. Obrero recovered a .
45 cal. handgun with a magazine containing seven (7) live bullets in a black clutch bag
inside an unlocked drawer. Three (3) half-full jute sacks containing printed materials of
RAM-SFP were also found in the children's room. A search of the children's recreation
and study area revealed a big travelling bag containing assorted polo shirts, men's
brief, two (2) pieces polo barong and short sleeve striped gray polo. sweat shirt, two (2)
pairs men's socks, a towel made in U.S.A., one blanket, a small black bag, Gandhi
brand, containing a book entitled "Islamic Revolution Future Path of the Nation", a road
map of the Philippines, a telescope, a plastic bag containing assorted medicines and
religious pamphlets was found in the master's bedroom. Sgt. Leo Justalero was
instructed by Capt. Obrero to make an inventory and receipt of the articles seized, in
the house.
The case was referred for preliminary investigation to Quezon City Assistant Prosecutor
Rodolfo Ponferrada who was designated Acting Provincial Prosecutor for Davao City. In a
Page 124

resolution dated August 6, 1990, Fiscal Ponferrada recommended the filing of an


information against herein petitioners for Violation of Presidential Decree No. 1866
(Illegal Possession of Firearms and Ammunitions in Furtherance of Rebellion). No bail
was recommended.
Issue:
Whether or Not Presidential Decree No. 1866, or at least the third paragraph of Section
1 thereof, is unconstitutional for being violative of the due process and equal protection
clauses of the Constitution.
Held:
The issue of constitutionality of Presidential Decree No. 1866 has been laid to rest in
the case of Misolas v. Panga, G.R. No. 83341, January 30, 1990 (181 SCRA 648), where
this Court held that the declaration of unconstitutionality of the third paragraph of
Section 1 of Presidential Decree No. 1866 is wanting in legal basis since it is neither a
bill of attainder nor does it provide a possibility of a double jeopardy.
Petitioners' contention that Republic Act 6968 has repealed Presidential Decree No.
1866 is bereft of merit. It is a cardinal rule of statutory construction that where the
words and phrases of a statute are not obscure or ambiguous. its meaning and the
intention of the legislature must be determined from the language employed, and
where there is no ambiguity in the words, there is no room for construction. Petitioners
contend that Section 1 of Presidential Decree No. 1866 is couched in general or vague
terms. The terms "deal in", "acquire", "dispose" or "possess" are capable of various
interpretations such that there is no definiteness as to whether or not the definition
includes "constructive possession" or how the concept of constructive possession
should be applied. Petitioners were not found in actual possession of the firearm and
ammunitions. They were in Quezon City while the prohibited articles were found in
Davao City. Yet they were being charged under Presidential Decree No. 1866 upon the
sole circumstance that the house wherein the items were found belongs to them.

Petitioners question the admissibility in evidence of the articles seized in violation of


their constitutional right against unreasonable search and seizure. Petitioners aver that
while they concede that Capt. Obrero had permission from Ma. Luisa Veroy to break
open the door of their residence, it was merely for the purpose of ascertaining thereat
the presence of the alleged "rebel" soldiers. The permission did not include any
authority to conduct a room to room search once inside the house. The items taken
were, therefore, products of an illegal search, violative of their constitutional rights As
such, they are inadmissible in evidence against them.
The Constitution guarantees the right of the people to be secure in their persons,
houses, papers and effects against unreasonable searches and seizures (Article III,
Section 2 of the 1987 Constitution). However, the rule that searches and seizures must
be supported by a valid warrant is not an absolute one. Among the recognized
Page 125

exceptions thereto are: (1) a search incidental to an arrest; (2) a search of a moving
vehicle; and (3) seizure of evidence in plain view (People v. Lo Ho Wing, G.R. No. 88017,
January 21, 1991 [193 SCRA 122]).
None of these exceptions pertains to the case at bar. The reason for searching the
house of herein petitioners is that it was reportedly being used as a hideout and
recruitment center for rebel soldiers. While Capt. Obrero was able to enter the
compound, he did not enter the house because he did not have a search warrant and
the owners were not present. This shows that he himself recognized the need for a
search warrant, hence, he did not persist in entering the house but rather contacted
the Veroys to seek permission to enter the same. Permission was indeed granted by Ma.
Luisa Veroy to enter the house but only to ascertain the presence of rebel soldiers.
Under the circumstances it is undeniable that the police officers had ample time to
procure a search warrant but did not.
Undeniably, the offense of illegal possession of firearms is malum prohibitum but it
does not follow that the subject thereof is necessarily illegal per se. Motive is
immaterial in mala prohibita but the subjects of this kind of offense may not be
summarily seized simply because they are prohibited. A search warrant is still
necessary. Hence, the rule having been violated and no exception being applicable, the
articles seized were confiscated illegally and are therefore protected by the
exclusionary principle. They cannot be used as evidence against the petitioners in the
criminal action against them for illegal possession of firearms. (Roan v. Gonzales, 145
SCRA 689-690 [1986]). Besides, assuming that there was indeed a search warrant, still
in mala prohibita, while there is no need of criminal intent, there must be knowledge
that the same existed. Without the knowledge or voluntariness there is no crime.
PREMISES CONSIDERED, the petition as granted and the criminal case against the
petitioners for illegal possession of firearms is DISMISSED.

PEOPLE VS. DEL ROSARIO


[234 SCRA 246; G.R. NO. 109633; 20 JUL 1994]
Facts:
Accused was charged and convicted by the trial court of illegal possession of firearms
and illegal possession and sale of drugs, particularly methamphetamine or shabu. After
the issuance of the search warrant, which authorized the search and seizure of an
undetermined quantity of methamphetamine and its paraphernalias, an entrapment
was planned that led to the arrest of del Rosario and to the seizure of the shabu, its
paraphernalias and of a .22 caliber pistol with 3 live ammunition.
Issue:

Page 126

Whether or Not the seizure of the firearms was proper.


Held:
No. Sec 2 art. III of the constitution specifically provides that a search warrant must
particularly describe the things to be seized. In herein case, the only objects to be
seized that the warrant determined was the methamphetamine and the paraphernalias
therein. The seizure of the firearms was unconstitutional.
Wherefore the decision is reversed and the accused is acquitted.

PEOPLE VS. GESMUNDO


[219 SCRA 743; G.R. NO. 89373; 19 MAR 1993]
Facts:
According to the prosecution, in the morning of Nov. 17, 1986, PO Jose Luciano gave
money and instructed his civilian informer to buy marijuana from the accused at the
Cocoland Hotel. He actually saw the accused selling marijuana to his civilian informer
and that same day Luciano applied for a search warrant.
About 2pm that day, a police raiding team armed with a search warrant went to the
Brgy captain for them to be accompanied in serving the said warrant at the residence
of the accused. The police was allowed to enter the house upon the strength of the
warrant shown to the accused. The accused begged the police not to search and to
leave the house. The police still searched the house and was led to the kitchen. She
pointed a metal basin on top of a table as the hiding place of died marijuana flowering
tops contained in a plastic bag marked ISETANN. The police also recovered from a
native uway cabinet dried marijuana flowering tops wrapped in 3 pieces of komiks
paper.
According to the accused, when the police arrived at her house, she saw Sgt. Yte and
PFC Jose Luciano. She invited Sgt. Yte to enter her house while Luciano was left in the
jeep that was parked near the house. While inside the house Yte showed the accused
something he claimed as a search warrant, when someone coming from the kitchen
uttered eto na They proceeded to the kitchen and saw Luciano holding a plastic bag
with four other companions. They confronted the accused and insisted that the bags
belonged to her. Accused denied the accusation and told them that she doesnt know
Page 127

anything about it. She was made to sign a prepared document. She was brought to the
police station and was detained.
The court renders judgment finding the accused guilty.
Issue:
Whether or Not the evidence was properly obtained by the police.
Held:
In the investigation report prepared by Luciano stated that during the search they
discovered a hole at the backyard of the house of the suspect, there was a big biscuit
can inside the hole and on top of the cover a flower pot was placed wherein the
marijuana was kept. However, there was no mention of any marijuana obtained from a
flower pot in any of their testimonies. There were inconsistencies insofar the
prosecution is concerned, as to what was recovered and where, the trial court
concluded that these inconsistencies are trivial. There must sufficient evidence that the
marijuana was actually surrendered by the accused. As held in PP vs. Remorosa,
Irreconcilable and unexplained contradictions in the testimonies of the prosecution
witnesses cast doubt on the guilt of appellant and his culpability to the crime charged.
The claim that the marijuana was planted was strengthen as the police violated sec 7,
rule 126 rules of the court provides no search of a house, room or any other premise
shall be made except in the presence of the lawful occupant thereof or any member of
his family or in the absence of the latter, in the presence of two (2) witnesses of
sufficient age and discretion residing in the same locality. This requirement is
mandatory to ensure regularity in the execution of the search warrant. Violation of said
rule is in fact punishable under Article 130 of the Revised Penal Code.

The document (PAGPAPATUNAY) was inadmissible to the court as the accused was not
informed of her right not to sign the document neither was she informed that she has
the right to the assistance of a counsel and the fact that it may be used as evidence
against her. It was not proved that the marijuana belonged to her. Not only does the law
require the presence of witnesses when the search is conducted, but it also imposes
upon the person making the search the duty to issue a detailed receipt for the property
seized. He is likewise required to deliver the property seized to the judge who issued
the warrant, together with a true and accurate inventory thereof duly verified under
oath. Again, these duties are mandatory and are required to preclude substitution of
the items seized by interested parties.

Page 128

The guilt of the accused was has not been established. Judgment is reversed.
UMIL VS. RAMOS
[187 SCRA 311; G.R. NO. 81567; 3 OCT 1991]
Facts:
On 1 February 1988, military agents were dispatched to the St. Agnes Hospital,
Roosevelt Avenue, Quezon City, to verify a confidential information which was received
by their office, about a "sparrow man" (NPA member) who had been admitted to the
said hospital with a gunshot wound. That the wounded man in the said hospital was
among the five (5) male "sparrows" who murdered two (2) Capcom mobile patrols the
day before, or on 31 January 1988 at about 12:00 o'clock noon, before a road hump
along Macanining St., Bagong Barrio, Caloocan City. The wounded man's name was
listed by the hospital management as "Ronnie Javellon," twenty-two (22) years old of
Block 10, Lot 4, South City Homes, Bian, Laguna however it was disclosed later that
the true name of the wounded man was Rolando Dural. In view of this verification,
Rolando Dural was transferred to the Regional Medical Servicesof the CAPCOM, for
security reasons. While confined thereat, he was positively identified by the
eyewitnesses as the one who murdered the 2 CAPCOM mobile patrols.
Issue:
Whether or Not Rolando was lawfully arrested.
Held:
Rolando Dural was arrested for being a member of the NPA, an outlawed subversive
organization. Subversion being a continuing offense, the arrest without warrant is
justified as it can be said that he was committing as offense when arrested. The crimes
rebellion, subversion, conspiracy or proposal to commit such crimes, and crimes or
offenses committed in furtherance therefore in connection therewith constitute direct
assaults against the state and are in the nature of continuing crimes.

PEOPLE VS. SUCRO


[195 SCRA 388; G.R. No. 93239; 18 Mar 1991]
Facts:
Pat. Fulgencio went to Arlie Regalados house at C. Quimpo to monitor activities of
Edison SUCRO (accused). Sucro was reported to be selling marijuana at a chapel 2
meters away from Regalados house. Sucro was monitored to have talked and
exchanged things three times. These activities are reported through radio to P/Lt.
Seraspi. A third buyer was transacting with appellant and was reported and later
identified as Ronnie Macabante. From that moment, P/Lt.Seraspi proceeded to the
area. While the police officers were at the Youth Hostel in Maagama St. Fulgencio told
Page 129

Lt. Seraspi to intercept. Macabante was intercepted at Mabini and Maagama crossing
in front of Aklan Medical center. Macabante saw the police and threw a tea bag of
marijuana on the ground. Macabante admitted buying the marijuana from Sucro in
front of the chapel.
The police team intercepted and arrested SUCRO at the corner of C. Quimpo and
Veterans. Recovered were 19 sticks and 4 teabags of marijuana from a cart inside the
chapel and another teabag from Macabante.
Issue:
Whether or Not arrest without warrant is lawful.
Whether or Not evidence from such arrest is admissible.
Held:
Search and seizures supported by a valid warrant of arrest is not an absolute rule. Rule
126, Sec 12 of Rules of Criminal Procedure provides that a person lawfully arrested may
be searched for dangerous weapons or anything, which may be used as proff of the
commission of an offense, without a search warrant.(People v. Castiller) The failure of
the police officers to secure a warrant stems from the fact that their knowledge
required from the surveillance was insufficient to fulfill requirements for its issuance.
However, warantless search and seizures are legal as long as PROBABLE CAUSE
existed. The police officers have personal knowledge of the actual commission of the
crime from the surveillance of the activities of the accused. As police officers were the
ones conducting the surveillance, it is presumed that they are regularly in performance
of their duties.

PEOPLE V. RODRIGUEZA
[205 SCRA 791; G.R. No. 95902; 4 Feb 1992]
Facts:
NARCOM agents staged a buy-bust operation, after gaining information that there was
an ongoing illegal traffic of prohibited drugs in Tagas, Albay. The participating agents
were given money treated with ultraviolet powder. One of the agents went to said
location, asked for a certain Don. Thereafter, the Don, herein accused, met with him
and a certain object wrapped in a plastic later identified as marijuana was given in
exchange for P200. The agent went back to headquarters and made a report, based on
which, a team was subsequently organized and a raid was conducted in the house of
the father of the accused. During the raid, the NARCOM agents were able to confiscate
Page 130

dried marijuana leaves and a plastic syringe among others. There was no authorization
by any search warrant. The accused was found positive of ultraviolet powder. The
lower court, considering the evidences obtained and testimonies from the prosecution,
found him guilty of violating the Dangerous Drugs Act of 1972 and sentenced him to
reclusion perpetua.
Issue:
Whether or Not the lower court was correct in its judgment.

Held:
The NARCOM agents procedure in the entrapment of the accused failed to meet the
qualification that the suspected drug dealer must be caught red-handed in the act of
selling marijuana to a person posing as a buyer, since the operation was conducted
after the actual exchange. Said raid also violated accused right against unreasonable
search and seizure, as the situation did not fall in the circumstances wherein a search
may be validly made even without a search warrant, i.e. when the search is incidental
to a lawful arrest; when it involves prohibited articles in plain view. The NARCOM
agents could not have justified their act by invoking the urgency and necessity of the
situation because the testimonies of the prosecution witnesses reveal that the place
had already been put under surveillance for quite some time. Had it been their
intention to conduct the raid, then they should, because they easily could, have first
secured a search warrant during that time. The Court further notes the confusion and
ambiguity in the identification of the confiscated marijuana leaves and other prohibited
drug paraphernalia presented as evidence against appellant:
CIC Taduran, who acted as the poseur buyer, testified that appellant sold him 100
grams of dried marijuana leaves wrapped in a plastic bag. Surprisingly, and no
plausible explanation has been advanced therefor, what were submitted to and
examined by the PCCL and thereafter utilized as evidence against the appellant were
the following items:
One (1) red and white colored plastic bag containing the following:
Exh. "A"Thirty (30) grams of suspected dried marijuana fruiting tops
contained inside a transparent plastic bag.
Exh. "B" Fifty (50) grams of suspected dried marijuana leaves and seeds
contained inside a white colored plastic labelled "Robertson".
Exh. "C" Four (4) aluminum foils each containing suspected dried
marijuana fruiting tops having a total weight of seven grams then further
wrapped
with a piece of aluminum foil.
Exh. "D" Five (5) small transparent plastic bags each containing
suspected dried marijuana fruiting tops having a total weight of seventeen
grams.
Page 131

Exh. "E" One plastic syringe.


Evidently, these prohibited articles were among those confiscated during the so-called
follow-up raid in the house of Rodriguezas father. The unanswered question then arises
as to the identity of the marijuana leaves that became the basis of appellant's
conviction. In People vs. Rubio, this Court had the occasion to rule that the plastic bag
and the dried marijuana leaves contained therein constitute the corpus delicti of the
crime. As such, the existence thereof must be proved with certainty and
conclusiveness. Failure to do so would be fatal to the cause of the prosecution.
Conviction is reversed and set aside and accused is acquitted.

PEOPLE VS. SY CHUA


[396 SCRA 657; G.R. No.136066-67; 4 Feb 2003]
Facts:
Accused-appellant Binad Sy Chua was charged with violation of Section 16, Article III of
R.A. 6425, as amended by R.A. 7659, and for Illegal Possession of Ammunitions and
Illegal Possession of Drugs in two separate Informations.
SPO2 Nulud and PO2 Nunag received a report from their confidential informant that
accused-appellant was about to deliver drugs that night at the Thunder Inn Hotel in
Balibago, Angeles City. So, the PNP Chief formed a team of operatives. The group
positioned themselves across McArthur Highway near Bali Hai Restaurant, fronting the
hotel. The other group acted as their back up.
Afterwards, their informer pointed to a car driven by accused-appellant which just
arrived and parked near the entrance of the hotel. After accused-appellant alighted
from the car carrying a sealed Zest-O juice box, SPO2 Nulud and PO2 Nunag hurriedly
accosted him and introduced themselves as police officers. As accused-appellant pulled
out his wallet, a small transparent plastic bag with a crystalline substance protruded
from his right back pocket. Forthwith, SPO2 Nulud subjected him to a body search
which yielded twenty (20) pieces of live .22 caliber firearm bullets from his left back
pocket. When SPO2 Nunag peeked into the contents of the Zest-O box, he saw that it
contained a crystalline substance. SPO2 Nulud instantly confiscated the small
transparent plastic bag, the Zest-O juice box, the twenty (20) pieces of .22 caliber
firearm bullets and the car used by accused-appellant. SPO2 Nulud and the other police
operatives who arrived at the scene brought the confiscated items to the office of Col.
Guttierez at the PNP Headquarters in Camp Pepito, Angeles City.
Accused-appellant vehemently denied the accusation against him and narrated a
different version of the incident.
Accused-appellant alleged that he was driving the car of his wife to follow her and his
Page 132

son to Manila. He felt sleepy, so he decided to take the old route along McArthur
Highway. He stopped in front of a small store near Thunder Inn Hotel to buy cigarettes
and candies. While at the store, he noticed a man approaches and examines the inside
of his car. When he called the attention of the onlooker, the man immediately pulled
out a .45 caliber gun and made him face his car with raised hands. The man later on
identified himself as a policeman. During the course of the arrest, the policeman took
out his wallet and instructed him to open his car. He refused, so the policeman took his
car keys and proceeded to search his car. At this time, the police officers companions
arrived at the scene in two cars. PO2 Nulud, who just arrived at the scene, pulled him
away from his car in a nearby bank, while the others searched his car.
Thereafter, he was brought to a police station and was held inside a bathroom for about
fifteen minutes until Col. Guttierez arrived, who ordered his men to call the media. In
the presence of reporters, Col. Guttierez opened the box and accused-appellant was
made to hold the box while pictures were being taken.
The lower court acquitted Sy Chua for the Illegal Possession of Ammunitions, yet
convicted him for Illegal Possession of 1,955.815 grams of shabu. Hence, this appeal to
the Court.
Issue:
Whether or Not the arrest of accused-appellant was lawful; and (2) WON the search of
his person and the subsequent confiscation of shabu allegedly found on him were
conducted in a lawful and valid manner.
Held:
The lower court believed that since the police received information that the accused
will distribute illegal drugs that evening at the Thunder Inn Hotel and its vicinities. The
police officer had to act quickly and there was no more time to secure a search warrant.
The search is valid being akin to a stop and frisk.
The trial court confused the concepts of a stop-and-frisk and of a search incidental to
a lawful arrest. These two types of warrantless searches differ in terms of the requisite
quantum of proof before they may be validly effected and in their allowable scope.
In a search incidental to a lawful arrest, as the precedent arrest determines the validity
of the incidental search, the legality of the arrest is questioned, e.g., whether an arrest
was merely used as a pretext for conducting a search. In this instance, the law requires
that there first be arrest before a search can be madethe process cannot be reversed.
Accordingly, for this exception to apply, two elements must concur: (1) the person to
be arrested must execute an overt act indicating that he has just committed, is actually
committing, or is attempting to commit a crime; and (2) such overt act is done in the
Page 133

presence or within the view of the arresting officer.


We find the two aforementioned elements lacking in the case at bar. Accused-appellant
did not act in a suspicious manner. For all intents and purposes, there was no overt
manifestation that accused-appellant has just committed, is actually committing, or is
attempting to commit a crime. Reliable information alone, absent any overt act
indicative of a felonious enterprise in the presence and within the view of the arresting
officers, is not sufficient to constitute probable cause that would justify an in flagrante
delicto arrest.
With regard to the concept of stop-and frisk: mere suspicion or a hunch will not
validate a stop-and-frisk. A genuine reason must exist, in light of the police officers
experience and surrounding conditions, to warrant the belief that the person detained
has weapons concealed about him. Finally, a stop-and-frisk serves a two-fold interest:
(1) the general interest of effective crime prevention and detection for purposes of
investigating possible criminal behavior even without probable cause; and (2) the
interest of safety and self-preservation which permit the police officer to take steps to
assure himself that the person with whom he deals is not armed with a deadly weapon
that could unexpectedly and fatally be used against the police officer.
A stop-and-frisk was defined as the act of a police officer to stop a citizen on the street,
interrogate him, and pat him for weapon(s) or contraband. It should also be
emphasized that a search and seizure should precede the arrest for this principle to
apply. The foregoing circumstances do not obtain in the case at bar. To reiterate,
accused-appellant was first arrested before the search and seizure of the alleged illegal
items found in his possession. The apprehending police operative failed to make any
initial inquiry into accused-appellants business in the vicinity or the contents of the
Zest-O juice box he was carrying. The apprehending police officers only introduced
themselves when they already had custody of accused-appellant.
In the case at bar, neither the in flagrante delicto nor the stop and frisk principles is
applicable to justify the warrantless arrest and consequent search and seizure made by
the police operatives on accused-appellant.
Wherefore, accused-appellant Binad Sy Chua is hereby Acquitted.

GO VS. COURT OF APPEALS


[206 SCRA 138; G.R. NO. 101837; 11 FEB 1992]
Facts:
Petitioner, while traveling in the wrong direction on a one-way street, almost had a
collision with another vehicle. Petitioner thereafter got out of his car, shot the driver of
Page 134

the other vehicle, and drove off. An eyewitness of the incident was able to take down
petitioners plate number and reported the same to the police, who subsequently
ordered a manhunt for petitioner. 6 days after the shooting, petitioner presented
himself in the police station, accompanied by 2 lawyers, the police detained him.
Subsequently a criminal charge was brought against him. Petitioner posted bail, the
prosecutor filed the case to the lower court, setting and commencing trial without
preliminary investigation. Prosecutor reasons that the petitioner has waived his right to
preliminary investigation as bail has been posted and that such situation, that
petitioner has been arrested without a warrant lawfully, falls under Section 5, Rule 113
and Section 7, Rule 112 of The 1985 Rules of Criminal Procedure which provides for the
rules and procedure pertaining to situations of lawful warrantless arrests. Petitioner in
his petition for certiorari assails such procedure and actions undertaken and files for a
preliminary investigation.
Issue:
Whether or Not warrantless arrest of petitioner was lawful.
Whether or Not petitioner effectively waived his right to preliminary investigation.
Held:
Petitioner and prosecutor err in relying on Umil v. Ramos, wherein the Court upheld the
warrantless arrest as valid effected 1 to 14 days from actual commission of the
offenses, which however constituted continuing crimes, i.e. subversion, membership
in an outlawed organization, etc. There was no lawful warrantless arrest under Section
5, Rule 113. This is because the arresting officers were not actually there during the
incident, thus they had no personal knowledge and their information regarding
petitioner were derived from other sources. Further, Section 7, Rule 112, does not
apply.

Petitioner was not arrested at all, as when he walked in the police station, he neither
expressed surrender nor any statement that he was or was not guilty of any crime.
When a complaint was filed to the prosecutor, preliminary investigation should have
been scheduled to determine probable cause. Prosecutor made a substantive error,
petitioner is entitled to preliminary investigation, necessarily in a criminal charge,
where the same is required appear thereat. Petition granted, prosecutor is ordered to
conduct preliminary investigation, trial for the criminal case is suspended pending
result from preliminary investigation, petitioner is ordered released upon posting a bail
bond.

Page 135

CALLANTA VS. VILLANUEVA


[77 SCRA 377; G.R. NOS. 24646 & L-24674; 20 JUN 1977]
Facts:
Two complaints for grave oral defamation were filed against Faustina Callanta. The City
Judge of Dagupan City, Felipe Villanueva, denied the motions to quash the complaints.
Thus, petitioner Callanta brought the suits for certiorari in the Supreme Court.
Petitioner questions the validity of the issuance of warrant of arrest by respondent,
arguing that the City Fiscal should have conducted the preliminary investigation.
According to petitioners counsel, there was jurisdictional infirmity. After the issuance of
the warrants of arrest and the bail fixed at P600, petitioner posted the bail bond, thus
obtaining her provisional liberty. The City Fiscal in this case did not disagree with the
judges investigation, and agreed with the complaints filed.
Issue:
Whether or Not petitioners contentions are to be given merit.
Held:
Based on many precedent cases of the Supreme Court, where the accused has filed
bail and waived the preliminary investigation proper, he has waived whatever defect, if
any, in the preliminary examination conducted prior to the issuance of the warrant of
arrest. In the case at bar, it is futile for the petitioner to question the validity of the
issuance of the warrant of arrest, because she posted the bail bond. Petitioner also
erred in arguing that only the City Fiscal can conduct a preliminary investigation.
According to the Charter of the City of Dagupan, the City Court of Dagupan City may
also conduct preliminary investigation for any offense, without regard to the limits of
punishment, and may release, or commit and bind over any person charged with such
offense to secure his appearance before the proper court. Petition for certiorari is
denied. Restraining order issued by the Court is lifted and set aside.

POSADAS VS. COURT OF APPEALS


Page 136

[188 SCRA 288; G.R. NO. 89139; 2 AUG 1990]


Facts:
Members of the Integrated National Police (INP) of the Davao Metrodiscom assigned
with the Intelligence Task Force, Pat. Ursicio Ungab and Pat. Umbra Umpar conducted
surveillance along Magallanes Street, Davao City. While in the vicinity of Rizal Memorial
Colleges they spotted petitioner carrying a "buri" bag and they noticed him to be acting
suspiciously. They approached the petitioner and identified themselves as members of
the INP. Petitioner attempted to flee but his attempt to get away was unsuccessful.
They then checked the "buri" bag of the petitioner where they found one (1) caliber .38
Smith & Wesson revolver with Serial No. 770196, two (2) rounds of live ammunition for
a .38 caliber gun, a smoke (tear gas) grenade, and two (2) live ammunitions for a .22
caliber gun. They brought the petitioner to the police station for further investigation. In
the course of the same, the petitioner was asked to show the necessary license or
authority to possess firearms and ammunitions found in his possession but he failed to
do so. He was then taken to the Davao Metrodiscom office and the prohibited articles
recovered from him were indorsed to M/Sgt. Didoy the officer then on duty. He was
prosecuted for illegal possession of firearms and ammunitions in the Regional Trial
Court of Davao City.
Issue:
Whether or Not the warantless search is valid.
Held:
In justifying the warrantless search of the buri bag then carried by the petitioner,
argues that under Section 12, Rule 136 of the Rules of Court a person lawfully arrested
may be searched for dangerous weapons or anything used as proof of a commission of
an offense without a search warrant. It is further alleged that the arrest without a
warrant of the petitioner was lawful under the circumstances.
in the case at bar, there is no question that, indeed, it is reasonable considering that it
was effected on the basis of a probable cause. The probable cause is that when the
petitioner acted suspiciously and attempted to flee with the buri bag there was a
probable cause that he was concealing something illegal in the bag and it was the right
and duty of the police officers to inspect the same.
It is too much indeed to require the police officers to search the bag in the possession
of the petitioner only after they shall have obtained a search warrant for the purpose.
Such an exercise may prove to be useless, futile and much too late.

Page 137

Clearly, the search in the case at bar can be sustained under the exceptions heretofore
discussed, and hence, the constitutional guarantee against unreasonable searches and
seizures has not been violated.

PEOPLE V. MENGOTE
[210 SCRA 174; G.R. NO. 87059; 22 JUN 1992]
Facts:
The Western Police District received a telephone call from an informer that there were
three suspicious looking persons at the corner of Juan Luna and North Bay Boulevard in
Tondo, Manila. A surveillance team of plainclothesmen was forthwith dispatched to the
place. The patrolmen saw two men looking from side to side, one of whom holding his
abdomen. They approached the persons and identified themselves as policemen,
whereupon the two tried to run but unable to escape because the other lawmen
surrounded them. The suspects were then searched. One of them the accusedappellant was found with a .38 caliber with live ammunitions in it, while his companion
had a fan knife. The weapons were taken from them and they were turned over to the
police headquarters for investigation.
An information was filed before the RTC
convicting the accused of illegal possession of firearm arm. A witness testified that the
weapon was among the articles stolen at his shop, which he reported to the police
including the revolver. For his part, Mengote made no effort to prove that he owned the
fire arm or that he was licensed to possess it but instead, he claimed that the weapon
was planted on him at the time of his arrest. He was convicted for violation of P.D.1866
and was sentenced to reclusion perpetua. In his appeal he pleads that the weapon was
not admissible as evidence against him because it had been illegally seized and
therefore the fruit of a poisonous tree.
Issue:
Whether or not the warrantless search and arrest was illegal.
Held:
An evidence obtained as a result of an illegal search and seizure inadmissible in any
proceeding for any purpose as provided by Art. III sec 32 of the Constitution. Rule 113
sec.5 of the Rules of Court, provides arrest without warrant lawful when: (a) the person
to be arrested has committed, is actually committing, or is attempting to commit an
offense, (b) when the offense in fact has just been committed, and he has personal
Page 138

knowledge of the facts indicating the person arrested has committed it and (c) the
person to be arrested has escaped from a penal establishment or a place where he is
serving final judgment or temporarily confined while his case is pending, or has
escaped while being transferred from one confinement to another.
These requirements have not been established in the case at bar. At the time of the
arrest in question, the accused appellant was merely looking from side to side and
holding his abdomen, according to the arresting officers themselves. There was
apparently no offense that has just been committed or was being actually committed or
at least being attempt by Mengote in their presence. Moreover a person may not be
stopped and frisked in a broad daylight or on a busy street on unexplained suspicion.
Judgment is reversed and set aside. Accused-appellant is acquitted.

PEOPLE VS. TANGLIBEN


[184 SCRA 220; G.R. No.L-63630; 6 Apr 1990]
Facts:
Patrolmen Silverio and Romeo Punzalan were conducting surveillance at the San
Fernando Victory Liner Terminal. At around 9:30pm they noticed a person, Medel
Tangliben, carrying a traveling bag who acted suspiciously. They confronted him,
inspected his bag, and there they found marijuana leaves. The accused was then taken
to the Police Headquarters for further investigations. The TC found Tangliben guilty of
violating sec.4 art. 2 of the RA 6425 or the Dangerous Drugs Act of 1972.
Issue:
Whether or Not there was an unlawful search due to lack of search warrant.
Held;
No. Rule 113 sec. 5 provides the a peace officer or a private person may w/o a warrant
arrest a person when in his presence the person to be arrested has committed, is
committing, or is attempting to commit an offense.
In the present case, the accused was found to have been committing possession of
marijuana and can be therefore searched lawfully even without a search warrant.
Another reason is that this case poses urgency on the part of the arresting police
Page 139

officers. It was found out that an informer pointed to the accused telling the policemen
that the accused was carrying marijuana. The police officers had to act quickly and
there was not enough time to secure a search warrant.

PEOPLE VS. MALMSTEDT


[198 SCRA 401; G.R. No. 91107; 19 Jun 1991]
Facts:
In an information filed against the accused- appellant Mikael Malmstead was charged
before the RTC of La Trinidad, Benguet, for violation of Section 4, Art. II of Republic Act
6425, as amended, otherwise known as the Dangerous Drugs Act of 1972, as amended.
Accused Mikael Malmstedt, a Swedish national, entered the Philippines for the third
time in December 1988 as a tourist. He had visited the country sometime in 1982 and
1985.
In the evening of 7 May 1989, accused left for Baguio City. Upon his arrival thereat in
the morning of the following day, he took a bus to Sagada and stayed in that place for
two (2) days. Then in the 7 in the morning of May 11, 1989, the accused went to
Nangonogan bus stop in Sagada.
At about 8: 00 o'clock in the morning of that same day (11 May 1989), Captain Alen
Vasco, the Commanding Officer of the First Regional Command (NARCOM) stationed at
Camp Dangwa, ordered his men to set up a temporary checkpoint at Kilometer 14,
Acop, Tublay, Mountain Province, for the purpose of checking all vehicles coming from
the Cordillera Region. The order to establish a checkpoint in the said area was
prompted by persistent reports that vehicles coming from Sagada were transporting
marijuana and other prohibited drugs. Moreover, information was received by the
Commanding Officer of NARCOM, that same morning that a Caucasian coming from
Sagada had in his possession prohibited drugs. The group composed of seven (7)
NARCOM officers, in coordination with Tublay Police Station, set up a checkpoint at the
designated area at about 10:00 o'clock in the morning and inspected all vehicles
coming from the Cordillera Region.
The two (2) NARCOM officers started their inspection from the front going towards the
rear of the bus. Accused who was the sole foreigner riding the bus was seated at the
rear thereof.
During the inspection, CIC Galutan noticed a bulge on accused's waist. Suspecting the
bulge on accused's waist to be a gun, the officer asked for accused's passport and
other identification papers. When accused failed to comply, the officer required him to
bring out whatever it was that was bulging on his waist. The bulging object turned out
to be a pouch bag and when accused opened the same bag, as ordered, the officer
noticed four (4) suspicious-looking objects wrapped in brown packing tape, prompting
the officer to open one of the wrapped objects. The wrapped objects turned out to
contain hashish, a derivative of marijuana.
Page 140

Thereafter, accused was invited outside the bus for questioning. But before he alighted
from the bus, accused stopped to get two (2) travelling bags from the luggage carrier.
Upon stepping out of the bus, the officers got the bags and opened them. A teddy bear
was found in each bag. Feeling the teddy bears, the officer noticed that there were
bulges inside the same which did not feel like foam stuffing. It was only after the
officers had opened the bags that accused finally presented his passport.
Accused was then brought to the headquarters of the NARCOM at Camp Dangwa, La
Trinidad, Benguet for further investigation. At the investigation room, the officers
opened the teddy bears and they were found to also contain hashish. Representative
samples were taken from the hashish found among the personal effects of accused and
the same were brought to the PC Crime Laboratory for chemical analysis.
In the chemistry report, it was established that the objects examined were hashish. a
prohibited drug which is a derivative of marijuana. Thus, an information was filed
against accused for violation of the Dangerous Drugs Act.
ACCUSEDS DEFENSE
During the arraignment, accused entered a plea of "not guilty." For his defense, he
raised the issue of illegal search of his personal effects. He also claimed that the
hashish was planted by the NARCOM officers in his pouch bag and that the two (2)
travelling bags were not owned by him, but were merely entrusted to him by an
Australian couple whom he met in Sagada. He further claimed that the Australian
couple intended to take the same bus with him but because there were no more seats
available in said bus, they decided to take the next ride and asked accused to take
charge of the bags, and that they would meet each other at the Dangwa Station.
The trial court found the guilt of the accused Mikael Malmstedt established beyond
reasonable doubt.

Seeking the reversal of the decision of the trial court finding him guilty of the crime
charged, accused argues that the search of his personal effects was illegal because it
was made without a search warrant and, therefore, the prohibited drugs which were
discovered during the illegal search are not admissible as evidence against him.
Issue:
Whether or Not the contention of the accused is valid, and therefore the RTC ruling be
reversed.
Held:
The Constitution guarantees the right of the people to be secure in their persons,
houses, papers and effects against unreasonable searches and seizures. However,
where the search is made pursuant to a lawful arrest, there is no need to obtain a
search warrant. A lawful arrest without a warrant may be made by a peace officer or a
private person under the following circumstances.
Sec. 5 Arrest without warrant; when lawful. A peace officer or a private
person may, without a warrant, arrest a person:

Page 141

(a) When, in his presence, the person to be arrested has committed is


actually committing, or is attempting to commit an offense;
(b) When an offense has in fact just been committed, and he has
personal knowledge of facts indicating that the person to be
arrested has committed it; and
(c) When the person to be arrested is a prisoner who has escaped from
a penal establishment or place where he is serving final judgment
or temporarily confined while his case is pending, or has escaped
while being transferred from one confinement to another.
Accused was searched and arrested while transporting prohibited drugs (hashish). A
crime was actually being committed by the accused and he was caught in flagrante
delicto. Thus, the search made upon his personal effects falls squarely under paragraph
(1) of the foregoing provisions of law, which allow a warrantless search incident to a
lawful arrest. While it is true that the NARCOM officers were not armed with a search
warrant when the search was made over the personal effects of accused, however,
under the circumstances of the case, there was sufficient probable cause for said
officers to believe that accused was then and there committing a crime.
Probable cause has been defined as such facts and circumstances which could lead a
reasonable, discreet and prudent man to believe that an offense has been committed,
and that the objects sought in connection with the offense are in the place sought to be
searched. Warrantless search of the personal effects of an accused has been declared
by this Court as valid, because of existence of probable cause, where the smell of
marijuana emanated from a plastic bag owned by the accused, 10 or where the accused
was acting suspiciously, 11 and attempted to flee.
The appealed judgment of conviction by the trial court is hereby affirmed. Costs against
the accused-appellant.

PEOPLE VS. AMMINUDIN


[163 SCRA 402; G.R. L-74869; 6 Jul 1988]
Facts:
Idel Aminnudin, accused-appellant was arrested on June 25, 1984, shortly after
disembarking from the M/V Wilcon 9 at about 8:30 in the evening, in Iloilo City. The PC
officers who were in fact waiting for him because of a tip from one their informers
simply accosted him, inspected his bag and finding what looked liked marijuana leaves
took him to their headquarters for investigation. The two bundles of suspect articles
were confiscated from him and later taken to the NBI laboratory for examination. It was
found to contain three kilos of what were later analyzed as marijuana leaves by an NBI
forensic examiner. An information for violation of the Dangerous Drugs Act was filed
against him. Later, the information was amended to include Farida Ali y Hassen, who
had also been arrested with him that same evening and likewise investigated. Both
were arraigned and pleaded not guilty. Subsequently, the fiscal filed a motion to
dismiss the charge against Ali on the basis of a sworn statement of the arresting
officers absolving her after a 'thorough investigation." The motion was granted, and
Page 142

trial proceeded only against the accused-appellant, who was eventually convicted . In
his defense, Aminnudin disclaimed the marijuana, averring that all he had in his bag
was his clothing consisting of a jacket, two shirts and two pairs of pants. He alleged
that he was arbitrarily arrested and immediately handcuffed. His bag was confiscated
without a search warrant. At the PC headquarters, he was manhandled to force him to
admit he was carrying the marijuana, the investigator hitting him with a piece of wood
in the chest and arms even as he parried the blows while he was still handcuffed. He
insisted he did not even know what marijuana looked like and that his business was
selling watches and sometimes cigarettes. However the RTC rejected his allegations.
Saying that he only has two watches during that time and that he did not sufficiently
proved the injuries allegedly sustained.
Issue:
Whether or not search of defendants bag is legal.
Held:
The search was illegal. Defendant was not caught in flagrante delicto, which could allow
warrantless arrest or search. At the moment of his arrest, he was not committing a
crime. Nor was he about to do so or had just done so. To all appearances, he was like
any of the other passengers innocently disembarking from the vessel. The said
marijuana therefore could not be appreciated as evidence against the defendant, and
furthermore he is acquitted of the crime as charged.

PEOPLE VS. SAYCON


[236 SCRA 325; G.R. NO. 110995; 5 SEPT 1994]
Facts:
On or about 8 July 1992, at about 6:00 in the morning, the Coastguard personnel
received information from NARCOM agent Ruben Laddaran that a suspected "shabu"
courier by the name of Alvaro Saycon was on board the MV Doa Virginia, which was
arriving at that moment in Dumaguete City. Upon receipt of the information, the
Coastguard chief officer CPO Tolin, instructed them to intercept the suspect. A
combined team of NARCOM agents and Philippine Coastguard personnel consisting of
CPO Tolin, a certain Miagme, and Senior Police Officers Ruben Laddaran and Winifredo
Noble of NARCOM posted themselves at the gate of Pier 1. The MV Doa Virginia
docked at 6:00 a.m. that same morning at Pier 1 in Dumaguete City. Alvaro Saycon
alighted from the boat carrying a black bag and went through the checkpoint manned
by the Philippine Coastguard where he was identified by police officer Winifredo Noble
of NARCOM. Saycon was then invited to the Coastguard Headquarters at the Pier area.
He willingly went with them. At the headquarters, the coastguard asked Saycon to open
his bag, and the latter willingly obliged. In it were personal belongings and a maong
Page 143

wallet. Inside that maong wallet, there was a Marlboro pack containing the suspected
"shabu". When police officer Winifredo Noble asked Saycon whether the Marlboro pack
containing the suspected "shabu" was his, Saycon merely bowed his head. Then
Saycon, his bag and the suspected "shabu" were brought to the NARCOM office for
booking. When Alvaro Saycon was arrested, the NARCOM agents did not have a warrant
of arrest. The PNP's Forensic Analyst declared in court that she had conducted an
examination of the specimens and found out that the specimens weighed 4.2 grams in
total, consisted of methamphetamine hydrochloride, more widely known as "shabu."
Issue:
Whether or Not the warrantless search was valid.
Held:
The warrantless search was valid, as the accused was a passenger of a motor vehicle.
There was probable cause to believe that the accused was carrying prohibited drugs.
Three weeks earlier, agents of the Narcotics Command bought methamine
hydrochloride from him. An agent of the Narcotics Command reported that the accused
would be arriving on board the vessel and carrying methamphetamine hydrochloride
with him. Drug couriers do not go about their trade with some external sign indicating
that they are transporting prohibited drugs. This must be taken into account in
determining probable cause.

PEOPLE VS. MUSA


[217 SCRA 597; G.,R. NO. 96177; 27 JAN 1993]
Facts:
A civilian informer gave the information that Mari Musa was engaged in selling
marijuana in Suterville, Zamboanga City. Sgt. Ani was ordered by NARCOM leader
T/Sgt. Belarga, to conduct a surveillance and test buy on Musa. The civilian informer
guided Ani to Musas house and gave the description of Musa. Ani was able to buy one
newspaper-wrapped dried marijuana for P10.00.
The next day, a buy-bust was planned. Ani was to raise his right hand if he successfully
buys marijuana from Musa. As Ani proceeded to the house, the NARCOM team
positioned themselves about 90 to 100 meters away. From his position, Belarga could
see what was going on. Musa came out of the house and asked Ani what he wanted.
Ani said he wanted more marijuana and gave Musa the P20.00 marked money. Musa
went into the house and came back, giving Ani two newspaper wrappers containing
dried marijuana. Ani opened and inspected it. He raised his right hand as a signal to
the other NARCOM agents, and the latter moved in and arrested Musa inside the house.
Page 144

Belarga frisked Musa in the living room but did not find the marked money (gave it to
his wife who slipped away). T/Sgt. Belarga and Sgt. Lego went to the kitchen and found
a cellophane colored white and stripe hanging at the corner of the kitchen. They
asked Musa about its contents but failed to get a response. So they opened it and
found dried marijuana leaves inside. Musa was then placed under arrest.
Issue:
Whether or Not the seizure of the plastic bag and the marijuana inside it is
unreasonable, hence, inadmissible as evidence.
Held:
Yes. It constituted unreasonable search and seizure thus it may not be admitted as
evidence. The warrantless search and seizure, as an incident to a suspects lawful
arrest, may extend beyond the person of the one arrested to include the premises or
surroundings under his immediate control. Objects in the plain view of an officer who
has the right to be in the position to have that view are subject to seizure and may be
presented as evidence. The plain view doctrine is usually applied where a police
officer is not searching for evidence against the accused, but nonetheless inadvertently
comes across an incriminating object. It will not justify the seizure of the object where
the incriminating nature of the object is not apparent from the plain view of the object.
In the case at bar, the plastic bag was not in the plain view of the police. They
arrested the accused in the living room and moved into the kitchen in search for other
evidences where they found the plastic bag. Furthermore, the marijuana inside the
plastic bag was not immediately apparent from the plain view of said object.
Therefore, the plain view does not apply. The plastic bag was seized illegally and
cannot be presented in evidence pursuant to Article III Section 3 (2) of the Constitution.
PITA VS. COURT OF APPEALS
[178 SCRA 362; G.R. NO. 80806; 5 OCT 1989]
Facts:
On December 1 and 3, 1983, pursuing an Anti-Smut Campaign initiated by the Mayor of
the City of Manila, Ramon D. Bagatsing, elements of the Special Anti-Narcotics Group,
Auxilliary Services Bureau, Western Police District, INP of the Metropolitan Police Force
of Manila, seized and confiscated from dealers, distributors, newsstand owners and
peddlers along Manila sidewalks, magazines, publications and other reading materials
believed to be obscene, pornographic and indecent and later burned the seized
materials in public at the University belt along C.M. Recto Avenue, Manila, in the
Page 145

presence of Mayor Bagatsing and several officers and members of various student
organizations.
Among the publications seized, and later burned, was "Pinoy Playboy" magazines
published and co-edited by plaintiff Leo Pita.
Plaintiff filed a case for injunction with prayer for issuance of the writ of preliminary
injunction against Mayor Bagatsing and Narcisco Cabrera, as superintendent of Western
Police District of the City of Manila, seeking to enjoin said defendants and their agents
from confiscating plaintiffs magazines or from preventing the sale or circulation thereof
claiming that the magazine is a decent, artistic and educational magazine which is not
per se obscene, and that the publication is protected by the Constitutional guarantees
of freedom of speech and of the press. Plaintiff also filed an Urgent Motion for issuance
of a temporary restraining order against indiscriminate seizure, confiscation and
burning of plaintiff's "Pinoy Playboy" Magazines, pending hearing on the petition for
preliminary injunction. The Court granted the temporary restraining order. The case was
set for trial upon the lapse of the TRO. RTC ruled that the seizure was valid. This was
affirmed by the CA.
Issue:
Whether or Not the seizure was illegal.
Held:
The Court ruled that the government authorities have not shown the required proof to
justify a ban and to warrant confiscation of the literature. First of all, they were not
possessed of a lawful court order: (1) finding the said materials to be pornography, and
(2) authorizing them to carry out a search and seizure, by way of a search warrant. The
court provides the following guidelines to be observed:
1. The authorities must apply for the issuance of a search warrant from a judge, if
in their opinion an obscenity seizure is in order;
2. The authorities must convince the court that the materials sought to be seized
are obscene and pose a clear and present danger of an evil substantive enough
to warrant State interference and action;
3. The judge must determine whether or not the same are indeed obscene. The
question is to be resolved on a case-to-case basis and on the judges sound
discretion;
4. If in the opinion of the court, probable cause exists, it shall issue the search
warrant prayed for;
5. The proper suit is then brought in the court under Article 201 of the RPC
(Obscene publications).
6. Any conviction is subject to appeal. The appellate court may assess whether or
not the properties seized are indeed obscene.

GUANZON VS. DE VILLA


[181 SCRA 623; G.R. 80508; 30 JAN 1990]
Page 146

Facts:
The 41 petitioners alleged that the "saturation drive" or "aerial target zoning" that were
conducted in their place (Tondo Manila) were unconstitutional. They alleged that there
is no specific target house to be search and that there is no search warrant or warrant
of arrest served. Most of the policemen are in their civilian clothes and without
nameplates or identification cards. The residents were rudely rouse from their sleep by
banging on the walls and windows of their houses. The residents were at the point of
high-powered guns and herded like cows. Men were ordered to strip down to their briefs
for the police to examine their tattoo marks. The residents complained that they're
homes were ransacked, tossing their belongings and destroying their valuables. Some
of their money and valuables had disappeared after the operation. The residents also
reported incidents of maulings, spot-beatings and maltreatment. Those who were
detained also suffered mental and physical torture to extract confessions and tactical
informations. The respondents said that such accusations were all lies. Respondents
contends that the Constitution grants to government the power to seek and cripple
subversive movements for the maintenance of peace in the state. The aerial target
zoning were intended to flush out subversives and criminal elements coddled by the
communities were the said drives were conducted. They said that they have
intelligently and carefully planned months ahead for the actual operation and that local
and foreign media joined the operation to witness and record such event.
Issue:
Whether or Not the saturation drive committed consisted of violation of human rights.
Held:
It is not the police action per se which should be prohibited rather it is the procedure
used or the methods which "offend even hardened sensibilities" .Based on the facts
stated by the parties, it appears to have been no impediment to securing search
warrants or warrants of arrest before any houses were searched or individuals roused
from sleep were arrested. There is no showing that the objectives sought to be attained
by the "aerial zoning" could not be achieved even as th rights of the squatters and low
income families are fully protected. However, the remedy should not be brought by a
tazpaer suit where not one victim complaints and not one violator is properly charged.
In the circumstances of this taxpayers' suit, there is no erring soldier or policeman
whom the court can order prosecuted. In the absence of clear facts no permanent relief
can be given.
In the meantime where there is showing that some abuses were committed, the court
temporary restraint the alleged violations which are shocking to the senses. Petition is
remanded to the RTC of Manila.

Page 147

PEOPLE VS. ARUTA


[288 SCRA 626; G.R. NO. 120515; 13 APR 1998]
Facts:
On Dec. 13, 1988, P/Lt. Abello was tipped off by his informant that a certain Aling
Rosa will be arriving from Baguio City with a large volume of marijuana and assembled
a team. The next day, at the Victory Liner Bus terminal they waited for the bus coming
from Baguio, when the informer pointed out who Aling Rosa was, the team
approached her and introduced themselves as NARCOM agents. When Abello asked
aling Rosa about the contents of her bag, the latter handed it out to the police. They
found dried marijuana leaves packed in a plastic bag marked cash katutak.
Instead of presenting its evidence, the defense filed a demurrer to evidence alleging
the illegality of the search and seizure of the items. In her testimony, the accused
claimed that she had just come from Choice theatre where she watched a movie
Balweg. While about to cross the road an old woman asked her for help in carrying a
shoulder bag, when she was later on arrested by the police. She has no knowledge of
the identity of the old woman and the woman was nowhere to be found. Also, no search
warrant was presented.
The trial court convicted the accused in violation of the dangerous drugs of 1972
Issue:
Whether or Not the police correctly searched and seized the drugs from the accused.
Held:
The following cases are specifically provided or allowed by law:
1.

2.

3.

4.

Warrantless search incidental to a lawful arrest recognized under Section 12,


Rule 126 of the Rules of Court 8 and by prevailing jurisprudence
Seizure of evidence in "plain view," the elements of which are: (a) a prior
valid intrusion based on the valid warrantless arrest in which the police are
legally present in the pursuit of their official duties; (b) the evidence was
inadvertently discovered by the police who had the right to be where they
are; (c) the evidence must be immediately apparent, and (d) "plain view"
justified mere seizure of evidence without further search;
Search of a moving vehicle. Highly regulated by the government, the vehicle's
inherent mobility reduces expectation of privacy especially when its transit in
public thoroughfares furnishes a highly reasonable suspicion amounting to
probable cause that the occupant committed a criminal activity;
Consented warrantless search;
Page 148

5.
6.
7.

Customs search;
Stop and Frisk;
Exigent and Emergency Circumstances.

The essential requisite of probable cause must still be satisfied before a warrantless
search and seizure can be lawfully conducted.
The accused cannot be said to be committing a crime, she was merely crossing the
street and was not acting suspiciously for the Narcom agents to conclude that she was
committing a crime. There was no legal basis to effect a warrantless arrest of the
accuseds bag, there was no probable cause and the accused was not lawfully arrested.
The police had more than 24 hours to procure a search warrant and they did not do so.
The seized marijuana was illegal and inadmissible evidence.

RULE 113, RULES OF COURT


Section 5. Arrest without warrant; when lawful. A peace officer or a private person
may, without a warrant, arrest a person:
(a) When, in his presence, the person to be arrested has committed, is actually
committing, or is attempting to commit an offense;
(b) When an offense has just been committed, and he has probable cause to
believe based on personal knowledge of facts or circumstances that the
person to be arrested has committed it; and
(c) When the person to be arrested is a prisoner who has escaped from a penal
establishment or place where he is serving final judgment or is temporarily
confined while his case is pending, or has escaped while being transferred
from one confinement to another.
In cases falling under paragraph (a) and (b) above, the person arrested without a
warrant shall be forthwith delivered to the nearest police station or jail and shall be
proceeded against in accordance with section 7 of Rule 112.

RULE 126, RULES OF COURT


Section 2. Court where application for search warrant shall be filed. An application
for search warrant shall be filed with the following:
a) Any court within whose territorial jurisdiction a crime was committed.
b) For compelling reasons stated in the application, any court within the judicial
region where the crime was committed if the place of the commission of the crime is
known, or any court within the judicial region where the warrant shall be enforced.
Page 149

However, if the criminal action has already been filed, the application shall only be
made in the court where the criminal action is pending.
Section 7. Right to break door or window to effect search. The officer, if refused
admittance to the place of directed search after giving notice of his purpose and
authority, may break open any outer or inner door or window of a house or any part of
a house or anything therein to execute the warrant or liberate himself or any person
lawfully aiding him when unlawfully detained therein.
Section 12. Delivery of property and inventory thereof to court; return and proceedings
thereon.
(a) The officer must forthwith deliver the property seized to the judge who issued
the warrant, together with a true inventory thereof duly verified under oath.
(b) Ten (10) days after issuance of the search warrant, the issuing judge shall
ascertain if the return has been made, and if none, shall summon the person to
whom the warrant was issued and require him to explain why no return was
made. If the return has been made, the judge shall ascertain whether section 11
of this Rule has been complained with and shall require that the property seized
be delivered to him. The judge shall see to it that subsection (a) hereof has been
complied with.
(c) The return on the search warrant shall be filed and kept by the custodian of the
log book on search warrants who shall enter therein the date of the return, the
result, and other actions of the judge.
A violation of this section shall constitute contempt of court.

Page 150

LIBERTY OF ABODE AND OF TRAVEL

Art 3, Sec. 6.
The liberty of abode and of changing the same within the limits
prescribed by law shall not be impaired except upon lawful order of the court. Neither
shall the right to travel be impaired except in the interest of national security, public
safety, or public health, as may be provided by law.

CAUNCA VS. SALAZAR


[82 PHIL 851; NO.L-2690; 1 JAN 1949]
Facts:
This is an action for habeas corpus brought by Bartolome Caunca in behalf of his cousin
Estelita Flores who was employed by the Far Eastern Employment Bureau, owned by
Julia Salazar, respondent herein. An advanced payment has already been given to
Estelita by the employment agency, for her to work as a maid. However, Estelita
wanted to transfer to another residence, which was disallowed by the employment
agency. Further she was detained and her liberty was restrained. The employment
agency wanted that the advance payment, which was applied to her transportation
expense from the province should be paid by Estelita before she could be allowed to
leave.

Issue:
Whether or Not an employment agency has the right to restrain and detain a maid
without returning the advance payment it gave?
Held:
An employment agency, regardless of the amount it may advance to a prospective
employee or maid, has absolutely no power to curtail her freedom of movement. The
fact that no physical force has been exerted to keep her in the house of the respondent
does not make less real the deprivation of her personal freedom of movement, freedom
to transfer from one place to another, freedom to choose ones residence. Freedom
may be lost due to external moral compulsion, to founded or groundless fear, to
erroneous belief in the existence of an imaginary power of an impostor to cause harm if
not blindly obeyed, to any other psychological element that may curtail the mental
Page 151

faculty of choice or the unhampered exercise of the will. If the actual effect of such
psychological spell is to place a person at the mercy of another, the victim is entitled to
the protection of courts of justice as much as the individual who is illegally deprived of
liberty by duress or physical coercion.

MANOTOC VS. COURT OF APPEALS


[142 SCRA 149; G.R. NO. L-62100; 30 MAY 1986]
Facts:
Petitioner was charged with estafa. He posted bail. Petitioner filed before each of the
trial courts a motion entitled, "motion for permission to leave the country," stating as
ground therefor his desire to go to the United States, "relative to his business
transactions and opportunities." The prosecution opposed said motion and after due
hearing, both trial judges denied the same. Petitioner thus filed a petition for certiorari
and mandamus before the then Court of Appeals seeking to annul the orders dated
March 9 and 26, 1982, of Judges Camilon and Pronove, respectively, as well as the
communication-request of the Securities and Exchange Commission, denying his leave
to travel abroad. He likewise prayed for the issuance of the appropriate writ
commanding the Immigration Commissioner and the Chief of the Aviation Security
Command (AVSECOM) to clear him for departure. The Court of Appeals denied the
petition.
Petitioner contends that having been admitted to bail as a matter of right, neither the
courts which granted him bail nor the Securities and Exchange Commission which has
no jurisdiction over his liberty could prevent him from exercising his constitutional right
to travel.
Issue:
Whether or Not his constitutional right to travel has been violated.
Held:
A court has the power to prohibit a person admitted to bail from leaving the Philippines.
This is a necessary consequence of the nature and function of a bail bond. The
condition imposed upon petitioner to make himself available at all times whenever the
court requires his presence operates as a valid restriction on his right to travel. Indeed,
if the accused were allowed to leave the Philippines without sufficient reason, he may
be placed beyond the reach of the courts. Petitioner has not shown the necessity for his
Page 152

travel abroad. There is no indication that the business transactions cannot be


undertaken by any other person in his behalf.

MARCOS VS. MANGLAPUS


[177 SCRA 668; G.R. NO. 88211; 15 SEPT 1989]
Facts:
This case involves a petition of mandamus and prohibition asking the court to order the
respondents Secretary of Foreign Affairs, etc. To issue a travel documents to former
Pres. Marcos and the immediate members of his family and to enjoin the
implementation of the President's decision to bar their return to the Philippines.
Petitioners assert that the right of the Marcoses to return in the Philippines is
guaranteed by the Bill of Rights, specifically Sections 1 and 6. They contended that
Pres. Aquino is without power to impair the liberty of abode of the Marcoses because
only a court may do so within the limits prescribed by law. Nor the President impair
their right to travel because no law has authorized her to do so.
They further assert that under international law, their right to return to the Philippines
is guaranteed particularly by the Universal Declaration of Human Rights and the
International Covenant on Civil and Political Rights, which has been ratified by the
Philippines.
Issue:
Whether or not, in the exercise of the powers granted by the constitution, the President
(Aquino) may prohibit the Marcoses from returning to the Philippines.
Held:
"It must be emphasized that the individual right involved is not the right to travel from
the Philippines to other countries or within the Philippines. These are what the right to
travel would normally connote. Essentially, the right involved in this case at bar is the
right to return to one's country, a distinct right under international law, independent
from although related to the right to travel. Thus, the Universal Declaration of Human
Rights and the International Covenant on Civil and Political Rights treat the right to
freedom of movement and abode within the territory of a state, the right to leave the
country, and the right to enter one's country as separate and distinct rights. What the
Declaration speaks of is the "right to freedom of movement and residence within the
borders of each state". On the other hand, the Covenant guarantees the right to liberty
of movement and freedom to choose his residence and the right to be free to leave any
country, including his own. Such rights may only be restricted by laws protecting the
Page 153

national security, public order, public health or morals or the separate rights of others.
However, right to enter one's country cannot be arbitrarily deprived. It would be
therefore inappropriate to construe the limitations to the right to return to ones country
in the same context as those pertaining to the liberty of abode and the right to travel.
The Bill of rights treats only the liberty of abode and the right to travel, but it is a well
considered view that the right to return may be considered, as a generally accepted
principle of International Law and under our Constitution as part of the law of the land.
The court held that President did not act arbitrarily or with grave abuse of discretion in
determining that the return of the Former Pres. Marcos and his family poses a serious
threat to national interest and welfare. President Aquino has determined that the
destabilization caused by the return of the Marcoses would wipe away the gains
achieved during the past few years after the Marcos regime.
The return of the Marcoses poses a serious threat and therefore prohibiting their return
to the Philippines, the instant petition is hereby DISMISSED.

SILVERIO VS. COURT OF APPEALS


[195 SCRA 760 ; G.R. 94284; 8 APR 1991]
Facts:
Petitioner was charged with violation of Section 2 (4) of the revised securities act.
Respondent filed to cancel the passport of the petitioner and to issue a hold departure
order. The RTC ordered the DFA to cancel petitioners passport, based on the finding
that the petitioner has not been arraigned and there was evidence to show that the
accused has left the country with out the knowledge and the permission of the court.
Issue:
Whether or Not the right to travel may be impaired by order of the court.
Held:
The bail bond posted by petitioner has been cancelled and warrant of arrest has been
issued by reason that he failed to appear at his arraignments. There is a valid
restriction on the right to travel, it is imposed that the accused must make himself
Page 154

available whenever the court requires his presence. A person facing criminal charges
may be restrained by the Court from leaving the country or, if abroad, compelled to
return (Constitutional Law, Cruz, Isagani A., 1987 Edition, p. 138). So it is also that "An
accused released on bail may be re-arrested without the necessity of a warrant if he
attempts to depart from the Philippines without prior permission of the Court where the
case is pending (ibid., Sec. 20 [2nd
par. ]).
Article III, Section 6 of the 1987 Constitution should be interpreted to mean that while
the liberty of travel may be impaired even without Court Order, the appropriate
executive officers or administrative authorities are not armed with arbitrary discretion
to impose limitations. They can impose limits only on the basis of "national security,
public safety, or public health" and "as may be provided by law," a limitive phrase
which did not appear in the 1973 text (The Constitution, Bernas, Joaquin G.,S.J., Vol. I,
First Edition, 1987, p. 263). Apparently, the phraseology in the 1987 Constitution was a
reaction to the ban on international travel imposed under the previous regime when
there was a Travel Processing Center, which issued certificates of eligibility to travel
upon application of an interested party (See Salonga vs. Hermoso & Travel Processing
Center, No. 53622, 25 April 1980, 97 SCRA 121).
Holding an accused in a criminal case within the reach of the Courts by preventing his
departure from the Philippines must be considered as a valid restriction on his right to
travel so that he may be dealt with in accordance with law. The offended party in any
criminal proceeding is the People of the Philippines. It is to their best interest that
criminal prosecutions should run their course and proceed to finality without undue
delay, with an accused holding himself amenable at all times to Court Orders and
processes

DEFENSOR-SANTIAGO VS. VASQUEZ


[217 SCRA 633; G.R. NOS. 99289-90; 27 JAN 1993]
Facts:
An information was filed against petitioner with the Sandiganbayan for violation of the
Anti Graft and Corrupt Practices Act. The order of arrest was issued with bail for release
fixed at Php. 15,000 so she filed a motion for acceptance of cash bail bond. On the
same day the Sandiganbayan issued a resolution authorizing the petitioner to post cash
bond which the later filed in the amount of Php.15, 000. Her arraignment was set, but
petitioner asked for the cancellation of her bail bond and that she be allowed
provisional release on recognizance. The Sandiganbayan deferred it. The
Sandiganbayan issued a hold departure order against petitioner, by reason of the
announcement she made that she would be leaving for the U.S. to accept a fellowship a
Page 155

Harvard. In the instant motion she submitted before the S.C. she argues that her right
to travel is impaired.
Issue:
Whether or Not the petitioners right to travel is impaired.
Held:
The petitioner does not deny and as a matter of fact even made a public statement,
that she he every intension of leaving the country to pursue higher studies abroad. The
court upholds the course of action of the Sandiganbayan in taking judicial notice of
such fact of petitioners pal to go abroad and in thereafter issuing a sua sponte the hold
departure order is but an exercise of respondent courts inherent power to preserve and
to maintain effectiveness of its jurisdiction over the case and the person of the
accused.
Also, the petitioner assumed obligations, when she posted bail bond. She holds herself
amenable at all times to the orders and process of eth court. She may legally be
prohibited from leaving the country during the pendency of the case. (Manotoc v. C.A.)

MARCOS VS. SANDIGANBAYAN


[247 SCRA 127; G.R. NO. 115132-34; 9 AUG 1995]
Facts:
This is a petition for certiorari to set aside as arbitrary and in grave abuse of discretion
resolutions of the Sandiganbayan's First Division denying petitioner's motion for leave
to travel abroad for medical treatment.
The former first lady Imelda Marcos was found guilty by the First Division of the
Sandiganbayan of violating 3 of the Anti Graft and Corrupt Practices Act. After
conviction she filed a "Motion for Leave to Travel Abroad" to seek diagnostic tests and
treatment by practitioners of oriental medicine in China allegedly because of "a serious
and life threatening medical condition" requiring facilities not available in the
Philippines that was denied. Then she again filed an "Urgent Ex-Parte Motion for
Permission to Travel Abroad" to undergo diagnosis and treatment in China. This was
supported by several medical reports that were prepared by her doctor Roberto
Anastacio.

Page 156

Again another Motion to leave was filed by Mrs. Marcos to US and Europe for treatment
of several Heart diseases alleging that the tests were not available here.
The presiding justice, Garchitorena, contacted Dr. Gregorio B. Patacsil, Officer-in-Charge
of the Philippine Heart Center, and later wrote him a letter, asking for "expert opinion
on coronary medicine". The court still found no merit to allow the petitioners motion to
leave and denied all of the motions.
Petitioner filed a motion for reconsideration and a "Motion to Admit Clinical Summary
and to Resolve Motion for Reconsideration." Attached was a recent medical report and
letters of Vice President Joseph E. Estrada offering to be guarantor for the return of
petitioner and those of twenty four members of the House of Representatives
requesting the court to allow petitioner to travel abroad. This was also denied by the
Court also stating their express disapproval of the involvement of the VP and the
Cabinet members so as to influence the resolutions, decisions or orders or any judicial
action of respondent court.
Issue:
Whether or Not the Sandiganbayan erred in disallowing the Motion for Leave to Travel
Abroad because it (1) disregarded the medical findings (2) it motu propio contacted a
third party asking the latter to give an opinion on petitioner's motion and medical
findings (3) said that there was no necessity to get medical treatment abroad.
Held:
No. The contention of the petitioner that was invalid to contact a third party asking the
latter to give an opinion on petitioner's motion and medical findings was erroneous.
Respondent court had to seek expert opinion because petitioner's motion was based on
the advice of her physician. The court could not be expected to just accept the opinion
of petitioner's physician in resolving her request for permission to travel. What would
be objectionable would be if respondent court obtained information without disclosing
its source to the parties and used it in deciding a case against them.
In disregarding the medical reports, the petitioner failed to prove the necessity for a
trip abroad. It should be emphasized that considering the fact that she is facing charges
before the courts in several cases, in two of which she was convicted although the
decision is still pending reconsideration, petitioner did not have an absolute right to
leave the country and the burden was on her to prove that because of danger to health
if not to her life there was necessity to seek medical treatment in foreign countries.
On the third issue, the Court ordered petitioner to undergo several tests which
summarily states that the required medical treatment was available here in the
Philippines and that the expertise and facilities here were more than adequate to cater
Page 157

to her medical treatment. The heart ailments of the petitioner were not as severe as
that was reported by Dr. Anastacio.
Wherefore, the petitioner is Dismissed without prejudice to the filling of another motion
for leave to travel abroad, should petitioner still desire, based on her heart condition. In
such an event the determination of her medical condition should be made by joint
panel of medical specialists recommended by both the accused and the prosecution.

RUBI VS. PROVINCIAL BOARD OF MINDORO


[39 PHIL 660; NO. 14078; 7 MAR 1919]
Facts:
The provincial board of Mindoro adopted resolution No. 25 wherein non-Christian
inhabitants (uncivilized tribes) will be directed to take up their habitation on sites on
unoccupied public lands. It is resolved that under section 2077 of the Administrative
Code, 800 hectares of public land in the sitio of Tigbao on Naujan Lake be selected as a
site for the permanent settlement of Mangyanes in Mindoro. Further, Mangyans may
only solicit homesteads on this reservation providing that said homestead applications
are previously recommended by the provincial governor.
In that case, pursuant to Section 2145 of the Revised Administrative Code, all the
Mangyans in the townships of Naujan and Pola and the Mangyans east of the Baco
River including those in the districts of Dulangan and Rubi's place in Calapan, were
ordered to take up their habitation on the site of Tigbao, Naujan Lake. Also, that any
Mangyan who shall refuse to comply with this order shall upon conviction be
imprisoned not exceed in sixty days, in accordance with section 2759 of the revised
Administrative Code.
Said resolution of the provincial board of Mindoro were claimed as necessary measures
for the protection of the Mangyanes of Mindoro as well as the protection of public
forests in which they roam, and to introduce civilized customs among them.
It appeared that Rubi and those living in his rancheria have not fixed their dwelling
within the reservation of Tigbao and are liable to be punished.
It is alleged that the Manguianes are being illegally deprived of their liberty by the
provincial officials of that province. Rubi and his companions are said to be held on the
reservation established at Tigbao, Mindoro, against their will, and one Dabalos is said to
be held under the custody of the provincial sheriff in the prison at Calapan for having
run away form the reservation.

Page 158

Issue:
Whether or Not Section 2145 of the Administrative Code deprive a person of his liberty
pf abode. Thus, WON Section 2145 of the Administrative Code of 1917 is constitutional.
Held:
The Court held that section 2145 of the Administrative Code does not deprive a person
of his liberty of abode and does not deny to him the equal protection of the laws, and
that confinement in reservations in accordance with said section does not constitute
slavery and involuntary servitude. The Court is further of the opinion that section 2145
of the Administrative Code is a legitimate exertion of the police power. Section 2145 of
the Administrative Code of 1917 is constitutional.
Assigned as reasons for the action: (1) attempts for the advancement of the nonChristian people of the province; and (2) the only successfully method for educating the
Manguianes was to oblige them to live in a permanent settlement. The Solicitor-General
adds the following; (3) The protection of the Manguianes; (4) the protection of the
public forests in which they roam; (5) the necessity of introducing civilized customs
among the Manguianes.
One cannot hold that the liberty of the citizen is unduly interfered without when the
degree of civilization of the Manguianes is considered. They are restrained for their own
good and the general good of the Philippines.
Liberty regulated by law": Implied in the term is restraint by law for the good of the
individual and for the greater good of the peace and order of society and the general
well-being. No man can do exactly as he pleases.
None of the rights of the citizen can be taken away except by due process of law.
Therefore, petitioners are not unlawfully imprisoned or restrained of their liberty.
Habeas corpus can, therefore, not issue.

Page 159

FREEDOM OF RELIGION

Art 3, Sec. 5.
No law shall be made respecting an establishment of religion, or
prohibiting the free exercise thereof. The free exercise and enjoyment of religious
profession and worship, without discrimination or preference, shall forever be allowed.
No religious test shall be required for the exercise of civil or political rights.

AGLIPAY VS. RUIZ


[64 PHIL 201; G.R. NO. 45459; 13 MAR 1937]
Facts:
Petitioner seeks the issuance of a writ of prohibition against respondent Director of
Posts from issuing and selling postage stamps commemorative of the 33 rd International
Eucharistic Congress.
Petitioner contends that such act is a violation of the
Constitutional provision stating that no public funds shall be appropriated or used in the
benefit of any church, system of religion, etc. This provision is a result of the principle
of the separation of church and state, for the purpose of avoiding the occasion wherein
the state will use the church, or vice versa, as a weapon to further their ends and aims.
Respondent contends that such issuance is in accordance to Act No. 4052, providing for
the appropriation funds to respondent for the production and issuance of postage
stamps as would be advantageous to the government.
Issue:
Whether or Not there was a violation of the freedom to religion.
Held:
What is guaranteed by our Constitution is religious freedom and not mere religious
toleration. It is however not an inhibition of profound reverence for religion and is not a
denial of its influence in human affairs. Religion as a profession of faith to an active
power that binds and elevates man to his Creator is recognized. And in so far as it
instills into the minds the purest principles of morality, its influence is deeply felt and
highly appreciated. The phrase in Act No. 4052 advantageous to the government
does not authorize violation of the Constitution. The issuance of the stamps was not
inspired by any feeling to favor a particular church or religious denomination. They
were not sold for the benefit of the Roman Catholic Church. The postage stamps,
instead of showing a Catholic chalice as originally planned, contains a map of the
Philippines and the location of Manila, with the words Seat XXXIII International
Page 160

Eucharistic Congress. The focus of the stamps was not the Eucharistic Congress but
the city of Manila, being the seat of that congress. This was to to advertise the
Philippines and attract more tourists, the officials merely took advantage of an event
considered of international importance. Although such issuance and sale may be
inseparably linked with the Roman Catholic Church, any benefit and propaganda
incidentally resulting from it was no the aim or purpose of the Government.
GARCES VS. ESTENZO
[104 SCRA 510; G.R. L-53487; 25 MAY 1981]
Facts:
Two resolutions of the Barangay Council of Valencia, Ormoc City were passed:
a. Resolution No. 5- Reviving the traditional socio-religious celebration every
fifth of April. This provided for the acquisition of the image of San Vicente
Ferrer and the construction of a waiting shed. Funds for the said projects will
be obtained through the selling of tickets and cash donations.
b. Resolution No. 6- The chairman or hermano mayor of the fiesta would be the
caretaker of the image of San Vicente Ferrer and that the image would remain
in his residence for one year and until the election of his successor. The image
would be made available to the Catholic Church during the celebration of the
saints feast day.
These resolutions have been ratified by 272 voters, and said projects were
implemented. The image was temporarily placed in the altar of the Catholic Church of
the barangay. However, after a mass, Father Sergio Marilao Osmea refused to return
the image to the barangay council, as it was the churchs property since church funds
were used in its acquisition.
Resolution No. 10 was passed for the authorization of hiring a lawyer for the replevin
case against the priest for the recovery of the image. Resolution No. 12 appointed Brgy.
Captain Veloso as a representative to the case. The priest, in his answer assailed the
constitutionality of the said resolutions. The priest with Andres Garces, a member of
the Aglipayan Church, contends that Sec. 8 Article IV 1 and Sec 18(2) Article VIII) 2 of the
constitution was violated.
Issue:
Whether or Not any freedom of religion clause in the Constitution violated.
Held:
Page 161

No. As said by the Court this case is a petty quarrel over the custody of the image. The
image was purchased in connection with the celebration of the barrio fiesta and not for
the purpose of favoring any religion nor interfering with religious matters or beliefs of
the barrio residents. Any activity intended to facilitate the worship of the patron
saint(such as the acquisition) is not illegal. Practically, the image was placed in a
laymans custody so that it could easily be made available to any family desiring to
borrow the image in connection with prayers and novena. It was the councils funds
that were used to buy the image, therefore it is their property. Right of the
determination of custody is their right, and even if they decided to give it to the
Church, there is no violation of the Constitution, since private funds were used. Not
every government activity which involves the expenditure of public funds and which
has some religious tint is violative of the constitutional provisions regarding separation
of church and state, freedom of worship and banning the use of public money or
property.
AMERICAN BIBLE SOCIETY VS. CITY OF MANILA
[101PHIL 386; G.R. NO. 9637; 30 APR 1957]
Facts:
New Yorks Education Law requires local public school authorities to lend textbooks free
of charge to all students in grade 7 to 12, including those in private schools. The Board
of Education contended that said statute was invalid and violative of the State and
Federal Constitutions. An order barring the Commissioner of Education (Allen) from
removing appellants members from office for failure to comply with the requirement
and an order preventing the use of state funds for the purchase of textbooks to be lent
to parochial schools were sought for. The trial court held the statute unconstitutional.
The Appellate Division reversed the decision and dismissed the complaint since the
appellant have no standing. The New York Court of Appeals, ruled that the appellants
have standing but the law is not unconstitutional.
Issue:
Whether or Not the said ordinances are constitutional and valid (contention: it restrains
the free exercise and enjoyment of the religious profession and worship of appellant).
Held:
Section 1, subsection (7) of Article III of the Constitution, provides that:
(7) No law shall be made respecting an establishment of religion, or prohibiting the
free exercise thereof, and the free exercise and enjoyment of religious profession
Page 162

and worship, without discrimination or preference, shall forever be allowed. No


religion test shall be required for the exercise of civil or political rights.
The provision aforequoted is a constitutional guaranty of the free exercise and
enjoyment of religious profession and worship, which carries with it the right to
disseminate religious information.
It may be true that in the case at bar the price asked for the bibles and other religious
pamphlets was in some instances a little bit higher than the actual cost of the same but
this cannot mean that appellant was engaged in the business or occupation of selling
said "merchandise" for profit. For this reason. The Court believe that the provisions of
City of Manila Ordinance No. 2529, as amended, cannot be applied to appellant, for in
doing so it would impair its free exercise and enjoyment of its religious profession and
worship as well as its rights of dissemination of religious beliefs.
With respect to Ordinance No. 3000, as amended, the Court do not find that it imposes
any charge upon the enjoyment of a right granted by the Constitution, nor tax the
exercise of religious practices.
It seems clear, therefore, that Ordinance No. 3000 cannot be considered
unconstitutional, however inapplicable to said business, trade or occupation of the
plaintiff. As to Ordinance No. 2529 of the City of Manila, as amended, is also not
applicable, so defendant is powerless to license or tax the business of plaintiff Society.
WHEREFORE, defendant shall return to plaintiff the sum of P5,891.45 unduly collected
from it.

GERMAN VS. BARANGAN


[135 SCRA 514; G.R. NO. 68828; 27 MAR 1985]
Facts:
Petitioners converged at J.P. Laurel Street to hear Mass at the St. Jude Chapel, which
adjoined Malacaang. Respondent barred them for security reasons. Petitioners filed a
petition for mandamus.
Issue:
Whether or Not there was a violation of the constitutional freedom.
Held:
Page 163

Petitioners' intention was not really to perform an act of religious worship but to
conduct an anti- government demonstration since they wore yellow T-shirts, raised their
clenched fists and shouted anti- government slogans. While every citizen has the right
to religious freedom, the exercise must be done in good faith. Besides, the restriction
was reasonable as it was designed to protect the lives of the President and his family,
government officials and diplomatic and foreign guests transacting business with
Malacanang. The restriction was also intended to secure the executive offices within the
Malacanang grounds from possible external attacks and disturbances. (Minority
opinion) The sole justification for a prior restraint or limitation on the exercise of the
freedom of religion is the existence of a grave and imminent, of a serious evil to public
safety, public morals, public health or any other legitimate public interest that the State
has a right to prevent. The burden to show the existence of grave and imminent danger
lies on the officials who would restrain petitioners. Respondents were in full control and
had the capability to stop any untoward move. There was no clear and present danger
of any serious evil to public safety or the security of Malacanang.

EBRALINAG VS. DIVISION SUPERINTENDENT OF CEBU


[219 SCRA 256 ; G.R. NO. 95770; 1 MAR 1993]
Facts:
Two special civil actions for certiorari, Mandamus and Prohibition were filed and
consolidated for raising same issue. Petitioners allege that the public respondents
acted without or in excess of their jurisdiction and with grave abuse of discretion.
Respondents ordered expulsion of 68 HS and GS students of Bantayan, Pinamungajan,
Caracar, Taburan and Asturias in Cebu. Public school authorities expelled these
students for refusing to salute the flag, sing the national anthem and recite the
Panatang Makabayan required by RA1265. They are Jehovahs Witnesses believing
that by doing these is religious worship/devotion akin to idolatry against their
teachings. They contend that to compel transcends constitutional limits and invades
protection against official control and religious freedom. The respondents relied on the
precedence of Gerona et al v. Secretary of Education. Gerona doctrine provides that
we are a system of separation of the church and state and the flag is devoid of religious
significance and it doesnt involve any religious ceremony. The freedom of religious
belief guaranteed by the Constitution does not mean exception from non-discriminatory
laws like the saluting of flag and singing national anthem. This exemption disrupts
school discipline and demoralizes the teachings of civic consciousness and duties of
citizenship.
Issue:
Whether or Not religious freedom has been violated.

Page 164

Held:
Religious freedom is a fundamental right of highest priority. The 2 fold aspect of right
to religious worship is: 1.) Freedom to believe which is an absolute act within the realm
of thought. 2.) Freedom to act on ones belief regulated and translated to external acts.
The only limitation to religious freedom is the existence of grave and present danger to
public safety, morals, health and interests where State has right to prevent. The
expulsion of the petitioners from the school is not justified.
The 30 yr old previous GERONA decision of expelling and dismissing students and
teachers who refuse to obey RA1265 is violates exercise of freedom of speech and
religious profession and worship.
Jehovahs Witnesses may be exempted from
observing the flag ceremony but this right does not give them the right to disrupt such
ceremonies. In the case at bar, the Students expelled were only standing quietly
during ceremonies. By observing the ceremonies quietly, it doesnt present any danger
so evil and imminent to justify their expulsion. What the petitioners request is
exemption from flag ceremonies and not exclusion from public schools. The expulsion
of the students by reason of their religious beliefs is also a violation of a citizens right
to free education. The non-observance of the flag ceremony does not totally constitute
ignorance of patriotism and civic consciousness. Love for country and admiration for
national heroes, civic consciousness and form of government are part of the school
curricula. Therefore, expulsion due to religious beliefs is unjustified.
Petition for Certiorari and Prohibition is GRANTED. Expulsion is ANNULLED.

FONACIER VS. COURT OF APPEALS


[96 PHIL 417; G.R. L-5917; 28 JAN 1955]
Facts:
Case was filed by Iglesia Filipina Independiente (IFI), represented by its supreme bishop
Gerardo Bayaca, against Bishop Fonacier seeking to render an accounting of his
administration of all the temporal properties and to recover the same on the ground
that he ceased to be the supreme bishop of IFI. Isabelo De los Reyes Jr. had been
elected as the Supreme Bishop.
Petitioner claims that he was not properly removed as Supreme Bishop and his legal
successor was Juan Jamias. He claims that the there was an accounting of his
administration and was turned over to bishop Jamias. Also, that Isabelo De los Reyes
and Bayaca have abandoned their faith and formally joined the Prostestant Episcopal
Church of America.

Page 165

CFI rendered judgment declaring Isabelo De Los Reyes, Jr. as the sole and legitimate
Supreme Bishop of IFI and ordered Fonacier to render an accounting of his admistration
CA affirmed the decision of the CFI
Issue:
Whether or not the petitioner should still be regarded as the legitimate supreme bishop
of IFI.
Held:
Supreme Court affirmed CAs decision. The legitimate Supreme Bishop of IFI is Isabelo
De los Reyes, Jr. The Supreme Court affirms the validity of the election of Bishop Delos
Reyes as the Supreme Bishop based on their internal laws
To finally dispose of the property issue, the Court, citing Watson v. Jones,368 declared
that the rule in property controversies within religious congregations strictly
independent of any other superior ecclesiastical association (such as the Philippine
Independent Church) is that the rules for resolving such controversies should be those
of any voluntary association. If the congregation adopts the majority rule then the
majority should prevail; if it adopts adherence to duly constituted authorities within the
congregation, then that should be followed.

PAMIL VS. TELECOM


[86 SCRA 413; G.R. 34854; 20 NOV 1978]
Facts:
Fr. Margarito Gonzaga was elected as Municipal Mayor in Alburquerque, Bohol.
Petitioner, also an aspirant for said office, then filed a suit for quo warranto for
Gonzagas disqualification based on the Administrative Code provision: In no case shall
there be elected or appointed to a municipal office ecclesiastics, soldiers in active
service, persons receiving salaries or compensation from provincial or national funds, or
contractors for public works of the municipality." The respondent Judge, in sustaiing Fr.
Gonzagas right to the office, ruled that the provision had already been impliedly
repealed by the Election Code of 1971. Petitioner on the other hand argues that there
was no implied repeal.
Issue:

Page 166

Whether or Not Fr. Gonzaga is eligible for the position of municipal mayor, according to
law.

Whether or Not the prohibition regarding elected or appointed ecclesiastics is


constitutional.
Held:
The court was divided. Five voted that the prohibition was not unconstitutional. Seven
others voted that the provision was impliedly repealed. However, the minority vote
overruled the seven. According to the dissenting seven, there are three reasons for the
said provision to be inoperative. First, the 1935 Constitution stated, No religious test
shall be required for the exercise of civil or political rights. Second, said section 2175 is
superseded by the Constitution. Third, section 2175 has been repealed by Sec. 23 of
the Election Code (1971): Appointive public office holders and active members of the
Armed Forces are no longer disqualified from running for an elective office.
Ecclesiastics were no longer included in the enumeration of persons ineligible under the
said Election Code. On the other hand, the controlling five argued: Section 2175 of the
Administrative Code deals with a matter different from that of section 23 of the Election
Code. Also, section 2175 of the Administrative Code did not violate the right to freedom
of religion because it did not give any requirement for a religious test.
The view of the dissenting seven failed to obtain a vote of eight members, so it was not
controlling. The provision of the Administrative Code remained operative.

ESTRADA VS. ESCRITOR


[492 SCRA 1 ; AM NO P-02-1651; 22 JUN 2006]
Facts:
Escritor is a court interpreter since 1999 in the RTC of Las Pinas City. She has been
living with Quilapio, a man who is not her husband, for more than twenty five years and
had a son with him as well. Respondents husband died a year before she entered into
the judiciary while Quilapio is still legally married to another woman.
Complainant Estrada requested the Judge of said RTC to investigate respondent.
According to complainant, respondent should not be allowed to remain employed
therein for it will appear as if the court allows such act.

Page 167

Respondent claims that their conjugal arrangement is permitted by her religionthe


Jehovahs Witnesses and the Watch Tower and the Bible Trace Society. They allegedly
have a Declaration of Pledging Faithfulness under the approval of their congregation.
Such a declaration is effective when legal impediments render it impossible for a couple
to legalize their union.
Issue:
Whether or Not the State could penalize respondent for such conjugal arrangement.

Held:
No. The State could not penalize respondent for she is exercising her right to freedom
of religion. The free exercise of religion is specifically articulated as one of the
fundamental rights in our Constitution. As Jefferson put it, it is the most inalienable and
sacred of human rights. The States interest in enforcing its prohibition cannot be
merely abstract or symbolic in order to be sufficiently compelling to outweigh a free
exercise claim. In the case at bar, the State has not evinced any concrete interest in
enforcing the concubinage or bigamy charges against respondent or her partner. Thus
the States interest only amounts to the symbolic preservation of an unenforced
prohibition.
Furthermore, a distinction between public and secular morality and religious morality
should be kept in mind. The jurisdiction of the Court extends only to public and secular
morality.
The Court further states that our Constitution adheres the benevolent neutrality
approach that gives room for accommodation of religious exercises as required by the
Free Exercise Clause. This benevolent neutrality could allow for accommodation of
morality based on religion, provided it does not offend compelling state interests.
Assuming arguendo that the OSG has proved a compelling state interest, it has to
further demonstrate that the state has used the least intrusive means possible so that
the free exercise is not infringed any more than necessary to achieve the legitimate
goal of the state. Thus the conjugal arrangement cannot be penalized for it constitutes
an exemption to the law based on her right to freedom of religion.

ISLAMIC DA'WAH COUNCIL OF THE PHILIPPINES VS. EXECUTIVE SECRETARY


[405 SCRA 497;GR 153888; 9 JUL 2003]
Facts:
Page 168

Petitioner Islamic Da'wah Council of the Philippines, Inc. (IDCP) is a corporation that
operates under Department of Social Welfare and Development, a non-governmental
organization that extends voluntary services to the Filipino people, especially to Muslim
communities. It claims to be a federation of national Islamic organizations and an active
member of international organizations such as the Regional Islamic Da'wah Council of
Southeast Asia and the Pacific (RISEAP) and The World Assembly of Muslim Youth. The
RISEAP accredited petitioner to issue halal certifications in the Philippines. Thus, among
the functions petitioner carries out is to conduct seminars, orient manufacturers on
halal food and issue halal certifications to qualified products and manufacturers.
Petitioner alleges that, the actual need to certify food products as halal and also due to
halal food producers' request, petitioner formulated in 1995 internal rules and
procedures based on the Qur'an and the Sunnah for the analysis of food, inspection
thereof and issuance of halal certifications. In that same year, petitioner began to issue,
for a fee, certifications to qualified products and food manufacturers. Petitioner even
adopted for use on its halal certificates a distinct sign or logo registered in the
Philippine Patent Office.
On 2001, respondent Office of the Executive Secretary issued EO 465 creating the
Philippine Halal Certification Scheme and designating respondent OMA to oversee its
implementation. Under the EO, respondent OMA has the exclusive authority to issue
halal certificates and perform other related regulatory activities.
Issue:
Whether or Not EO violates the constitutional provision on the separation of Church and
State.
Held:
It is unconstitutional for the government to formulate policies and guidelines on the
halal certification scheme because said scheme is a function only religious
organizations, entity or scholars can lawfully and validly perform for the Muslims.
According to petitioner, a food product becomes halal only after the performance of
Islamic religious ritual and prayer. Thus, only practicing Muslims are qualified to
slaughter animals for food. A government agency like herein respondent OMA cannot
therefore perform a religious function like certifying qualified food products as halal.
Without doubt, classifying a food product as halal is a religious function because the
standards used are drawn from the Qur'an and Islamic beliefs. By giving OMA the
exclusive power to classify food products as halal, EO 46 encroached on the religious
freedom of Muslim organizations like herein petitioner to interpret for Filipino Muslims
what food products are fit for Muslim consumption. Also, by arrogating to itself the task
of issuing halal certifications, the State has in effect forced Muslims to accept its own
interpretation of the Qur'an and Sunnah on halal food.
Page 169

In the case at bar, we find no compelling justification for the government to deprive
Muslim organizations, like herein petitioner, of their religious right to classify a product
as halal, even on the premise that the health of Muslim Filipinos can be effectively
protected by assigning to OMA the exclusive power to issue halal certifications. The
protection and promotion of the muslim Filipinos' right to health are already provided
for in existing laws and ministered to by government agencies charged with ensuring
that food products released in the market are fit for human consumption, properly
labeled and safe. Unlike EO 46, these laws do not encroach on the religious freedom of
Muslims.

Page 170

FREEDOM OF EXPRESSION

Art 3, Sec. 4.
No law shall be passed abridging the freedom of speech, of
expression, or of the press, or the right of the people peaceably to assemble and
petition the government for redress of grievances.
Art 3, Sec. 7.
The right of the people to information on matters of public concern
shall be recognized. Access to official records, and to documents, and papers
pertaining to official acts, transactions, or decisions, as well as to government research
data used as basis for policy development, shall be afforded the citizen, subject to such
limitations as may be provided by law.
Art 3, Sec. 8.
The right of the people, including those employed in the public and
private sectors, to form unions, associations, or societies for purposes not contrary to
law shall not be abridged.
Art 3, Sec. 18. (1) No person shall be detained solely by reason of his political beliefs
and aspirations.

NEAR VS. MINNESOTA


[283 US 697]
Facts:
A complaint alleged that the defendants, on September 24, 1927, and on eight
subsequent dates in October and November, 1927, published and circulated editions of
The Saturday Press(published in Minneapolis) which were 'largely devoted to
malicious, scandalous and defamatory articles'(based on Session Laws of Minnesota).
The articles charged, in substance, provides that a Jewish gangster was in control of
gambling, bootlegging, and racketeering in Minneapolis, and that law enforcing officers
and agencies were not energetically performing their duties. Most of the charges were
directed against the chief of police; he was charged with gross neglect of duty, illicit
relations with gangsters, and with participation in graft. The county attorney was
charged with knowing the existing conditions and with failure to take adequate
measures to remedy them. The mayor was accused of inefficiency and dereliction. One
member of the grand jury was stated to be in sympathy with the gangsters. A special
grand jury and a special prosecutor were demanded to deal with the situation in
general, and, in particular, to investigate an attempt to assassinate one Guilford, one of
the original defendants, who, it appears from the articles, was shot by gangsters after
the first issue of the periodical had been published. Now defendants challenged the
Minnesota statute which provides for the abatement, as a public nuisance, of a
Page 171

malicious, scandalous and defamatory news paper, magazine or other periodical. The
District Court ruled against defendants. Hence the appeal.
Issue:
Whether or Not the proceeding authorized by the statute herein constitutes an
infringement of the freedom of the press.
Held:
Yes. The insistence that the statute is designed to prevent the circulation of scandal
which tends to disturb the public peace and to provoke assaults and the commission of
crime is unavailing.
The reason for the enactment, as the state court has said, is that prosecutions to
enforce penal statutes for libel do not result in 'efficient repression or suppression of
the evils of scandal.' In the present instance, the proof was that nine editions of the
newspaper or periodical in question were published on successive dates, and that they
were chiefly devoted to charges against public officers and in relation to the prevalence
and protection of crime. In such a case, these officers are not left to their ordinary
remedy in a suit for libel, or the authorities to a prosecution for criminal libel. The
statute not only operates to suppress the offending newspaper or periodical, but to put
the publisher under an effective censorship.
Every freeman has an undoubted right to lay what sentiments he pleases before the
public; to forbid this, is to destroy the freedom of the press; but if he publishes what is
improper, mischievous or illegal, he must take the consequence of his own temerity.
The liberty of the press was to be unrestrained, but he who used it was to be
responsible in case of its abuse.' Public officers, whose character and conduct remain
open to debate and free discussion in the press, find their remedies for false
accusations in actions under libel laws providing for redress and punishment, and not in
proceedings to restrain the publication of newspapers and periodicals.
Characterizing the publication as a business, and the business as a nuisance, does not
permit an invasion of the constitutional immunity against restraint. Nor can it be said
that the constitutional freedom from previous restraint is lost because charges are
made of derelictions which constitute crimes.
The preliminary freedom, by virtue of the very reason for its existence, does not
depend, as this court has said, on proof of truth.

Page 172

GROSJEAN VS. AMERICAN PRESS CO.


[297 US 233]
Facts:
The nine publishers(corporations) who brought the suit publish thirteen newspapers
and these thirteen publications are the only ones within the state of Louisiana having
each a circulation of more than 20,000 copies per week. The suit assailed Act No. 23 1 of
the Louisiana Legislature, as their freedom of the press was abridged in contravention
to the due process clause.
Issue:
Whether or Not Act 23 unconstitutional.
Held:
Yes. Freedom of speech and of the press are rights of the same fundamental character,
safeguarded by the due process of law clause. The word 'liberty' contained in that
amendment embraces not only the right of a person to be free from physical restraint,
but the right to be free in the enjoyment of all his faculties as well.
The Act operates as a restraint in a double sense. First, its effect is to curtail the
amount of revenue realized from advertising; and, second, its direct tendency is to
restrict circulation. This is plain enough when we consider that, if it were increased to a
high degree, as it could be if valid it well might result in destroying both advertising
and circulation.
Judge Cooley has laid down the test to be applied: The evils to be prevented were not
the censorship of the press merely, but any action of the government by means of
which it might prevent such free and general discussion of public matters as seems
absolutely essential to prepare the people for an intelligent exercise of their rights as
citizens.
The tax here involved is bad not because it takes money from the pockets of the
appellees. It is bad because, it is seen to be a deliberate and calculated device in the
guise of a tax to limit the circulation of information to which the public is entitled in
virtue of the constitutional guaranties. A free press stands as one of the great
interpreters between the government and the people.
The form in which the tax is imposed is in itself suspicious. It is not measured or limited
by the volume of advertisements. It is measured alone by the extent of the circulation
of the publication in which the advertisements are carried, with the plain purpose of
penalizing the publishers and curtailing the circulation of a selected group of
newspapers.

NEW YORK TIMES VS. UNITED STATES


Page 173

[403 US 713]
Facts:
The court granted certiorari in the cases in which the United States seeks to enjoin the
New York Times and the Washington Post from publishing the contents of a classified
study entitled "History of U.S. Decision-Making Process on Viet Nam Policy." Said articles
reveal the workings of government that led to the Vietnam war. The Government
argues that "the authority of the Executive Department to protect the nation against
publication of information whose disclosure would endanger the national security stems
from two interrelated sources: the constitutional power of the President over the
conduct of foreign affairs and his authority as Commander-in-Chief. In such case the
Executive Branch seeks judicial aid in preventing publication. The court ruled in favor of
the newspaper companies hence the appeal.
Issue:
Whether or not the freedom of the press was abridged.

Held:
Yes. To find that the President has "inherent power" to halt the publication of news by
resort to the courts would wipe out the First Amendment (Bill of Rights) and destroy the
fundamental liberty and security of the very people the Government hopes to make
"secure."
No branch of government could abridge the people's rights granted by the Constitution
including the freedom of the press. The language of the First Amendment support the
view that the press must be left free to publish news, whatever the source, without
censorship, injunctions, or prior restraints. The press was protected so that it could bare
the secrets of government and inform the people. Only a free and unrestrained press
can effectively expose deception in government. And paramount among the
responsibilities of a free press is the duty to prevent any part of the government from
deceiving the people and sending them off to distant lands to die of foreign fevers and
foreign shot and shell.

GONZALES VS. COMELEC


[27 SCRA 835; G.R. L-27833; 18 APR 1969]
Facts:
Page 174

RA 4880 which took effect on June 17, 1967, prohibiting the too early nomination of
candidates and limiting the period of election campaign or partisan political activity
was challenged on constitutional grounds. More precisely, the basic liberties of free
speech and free press, freedom of assembly and freedom of association are invoked to
nullify the act. Petitioner Cabigao was, at the time of the filing the petition, an
incumbent councilor in the 4th District of Manila and the Nacionalista Party official
candidate for Vice-Mayor of Manila to which he was subsequently elected on November
11, 1967; petitioner Gonzales, on the other hand, is a private individual, a registered
voter in the City of Manila and a political leader of his co-petitioner. There was the
further allegation that the nomination of a candidate and the fixing of period of election
campaign are matters of political expediency and convenience which only political
parties can regulate or curtail by and among themselves through self-restraint or
mutual understanding or agreement and that the regulation and limitation of these
political matters invoking the police power, in the absence of clear and present danger
to the state, would render the constitutional rights of petitioners meaningless and
without effect. Senator Lorenzo M. Taada was asked to appear as amicus curiae, and
elucidated that Act No. 4880 could indeed be looked upon as a limitation on the
preferred rights of speech and press, of assembly and of association. He did justify its
enactment however under the clear and present danger doctrine, there being the
substantive evil of elections, whether for national or local officials, being debased and
degraded by unrestricted campaigning, excess of partisanship and undue concentration
in politics with the loss not only of efficiency in government but of lives as well. The
Philippine Bar Association, the Civil Liberties Union, the U.P. Law Center and the U.P.
Women Lawyers' Circle were requested to give their opinions. Respondents contend
that the act was based on the police power of the state.

Issue:
Whether or Not RA 4880 unconstitutional.
Held:
Yes. As held in Cabansag v. Fernandez there are two tests that may supply an
acceptable criterion for permissible restriction on freedom of speech. These are the
clear and present danger rule and the 'dangerous tendency' rule. The first, means
that the evil consequence of the comment or utterance must be extremely serious and
the degree of imminence extremely high before the utterance can be punished. The
danger to be guarded against is the 'substantive evil' sought to be prevented. It has the
advantage of establishing according to the above decision a definite rule in
constitutional law. It provides the criterion as to what words may be publicly
established. The "dangerous tendency rule" is such that If the words uttered create a
dangerous tendency which the state has a right to prevent, then such words are
punishable. It is not necessary that some definite or immediate acts of force, violence,
or unlawfulness be advocated. It is sufficient that such acts be advocated in general
terms. Nor is it necessary that the language used be reasonably calculated to incite
Page 175

persons to acts of force, violence, or unlawfulness. It is sufficient if the natural tendency


and probable effect of the utterance be to bring about the substantive evil which the
legislative body seeks to prevent.
The challenged statute could have been more narrowly drawn and the practices
prohibited more precisely delineated to satisfy the constitutional requirements as to a
valid limitation under the clear and present danger doctrine. As the author Taada
clearly explained, such provisions were deemed by the legislative body to be part and
parcel of the necessary and appropriate response not merely to a clear and present
danger but to the actual existence of a grave and substantive evil of excessive
partisanship, dishonesty and corruption as well as violence that of late has invariably
marred election campaigns and partisan political activities in this country.
The very idea of a government, republican in form, implies a right on the part of its
citizens to meet peaceably for consultation in respect to public affairs and to petition
for redress of grievances. As in the case of freedom of expression, this right is not to be
limited, much less denied, except on a showing of a clear and present danger of a
substantive evil that Congress has a right to prevent.
The prohibition of any speeches, announcements or commentaries, or the holding of
interviews for or against the election of any party or candidate for public office and the
prohibition of the publication or distribution of campaign literature or materials, against
the solicitation of votes whether directly or indirectly, or the undertaking of any
campaign literature or propaganda for or against any candidate or party is repugnant to
a constitutional command.

IGLESIA NI CRISTO VS. COURT OF APPEALS


[259 SCRA 529; G.R. NO. 119673; 26 JUL 1996]
Facts:
Petitioner has a television program entitled "Ang Iglesia ni Cristo" aired on Channel 2
every Saturday and on Channel 13 every Sunday. The program presents and
propagates petitioner's religious beliefs, doctrines and practices often times in
comparative studies with other religions. Petitioner submitted to the respondent Board
of Review for Moving Pictures and Television the VTR tapes of its TV program Series
Nos. 116, 119, 121 and 128. The Board classified the series as "X" or not for public
viewing on the ground that they "offend and constitute an attack against other religions
which is expressly prohibited by law." On November 28, 1992, it appealed to the Office
of the President the classification of its TV Series No. 128 which allowed it through a
letter of former Executive Secretary Edelmiro A. Amante, Sr., addressed for Henrietta S.
Page 176

Mendez reversing the decision of the respondent Board. According to the letter the
episode in is protected by the constitutional guarantee of free speech and expression
and no indication that the episode poses any clear and present danger. Petitioner also
filed Civil Case. Petitioner alleged that the respondent Board acted without jurisdiction
or with grave abuse of discretion in requiring petitioner to submit the VTR tapes of its
TV program and in x-rating them. It cited its TV Program Series Nos. 115, 119, 121 and
128. In their Answer, respondent Board invoked its power under PD No. 1986 1 in
relation to Article 201 of the Revised Penal Code. The Iglesia ni Cristo insists on the
literal translation of the bible and says that our (Catholic) veneration of the Virgin Mary
is not to be condoned because nowhere it is found in the bible. The board contended
that it outrages Catholic and Protestant's beliefs. RTC ruled in favor of petitioners. CA
however reversed it hence this petition.
Issue:
Whether or Not the "ang iglesia ni cristo" program is not constitutionally protected as a
form of religious exercise and expression.
Held:
Yes. Any act that restrains speech is accompanied with presumption of invalidity. It is
the burden of the respondent Board to overthrow this presumption. If it fails to
discharge this burden, its act of censorship will be struck down. This is true in this case.
So-called "attacks" are mere criticisms of some of the deeply held dogmas and tenets
of other religions. RTCs ruling clearly suppresses petitioner's freedom of speech and
interferes with its right to free exercise of religion. attack is different from offend
any race or religion. The respondent Board may disagree with the criticisms of other
religions by petitioner but that gives it no excuse to interdict such criticisms, however,
unclean they may be. Under our constitutional scheme, it is not the task of the State to
favor any religion by protecting it against an attack by another religion. Religious
dogmas and beliefs are often at war and to preserve peace among their followers,
especially the fanatics, the establishment clause of freedom of religion prohibits the
State from leaning towards any religion. Respondent board cannot censor the speech of
petitioner Iglesia ni Cristo simply because it attacks other religions, even if said religion
happens to be the most numerous church in our country. The basis of freedom of
religion is freedom of thought and it is best served by encouraging the marketplace of
dueling ideas. It is only where it is unavoidably necessary to prevent an immediate and
grave danger to the security and welfare of the community that infringement of
religious freedom may be justified, and only to the smallest extent necessary to avoid
the danger. There is no showing whatsoever of the type of harm the tapes will bring
about especially the gravity and imminence of the threatened harm. Prior restraint on
speech, including religious speech, cannot be justified by hypothetical fears but only by
the showing of a substantive and imminent evil. It is inappropriate to apply the clear
and present danger test to the case at bar because the issue involves the content of
speech and not the time, place or manner of speech. Allegedly, unless the speech is
first allowed, its impact cannot be measured, and the causal connection between the
speech and the evil apprehended cannot be established. The determination of the
question as to whether or not such vilification, exaggeration or fabrication falls within or
Page 177

lies outside the boundaries of protected speech or expression is a judicial function


which cannot be arrogated by an administrative body such as a Board of Censors." A
system of prior restraint may only be validly administered by judges and not left to
administrative agencies.

ADIONG VS. COMELEC


[207 SCRA 712; G.R. NO. 103956; 31 MAR 1992]
Facts:
COMELEC promulgated Resolution No. 2347 which provides that decals and stickers
may be posted only in any of the authorized posting areas, prohibiting posting in
"mobile" places, public or private. Petitioner Blo Umpar Adiong, a senatorial candidate
in the May 11, 1992 elections now assails the Resolution. In addition, the petitioner
believes that with the ban on radio, television and print political advertisements, he,
being a neophyte in the field of politics stands to suffer grave and irreparable injury
with this prohibition.
Issue:
Whether or Not the COMELECs prohibition unconstitutional.
Held:
The prohibition unduly infringes on the citizen's fundamental right of free speech. The
preferred freedom of expression calls all the more for the utmost respect when what
may be curtailed is the dissemination of information to make more meaningful the
equally vital right of suffrage. The so-called balancing of interests individual freedom
on one hand and substantial public interests on the other is made even more difficult
in election campaign cases because the Constitution also gives specific authority to the
Commission on Elections to supervise the conduct of free, honest, and orderly
elections. When faced with border line situations where freedom to speak by a
candidate or party and freedom to know on the part of the electorate are invoked
against actions intended for maintaining clean and free elections, the police, local
officials and COMELEC, should lean in favor of freedom. The regulation of election
campaign activity may not pass the test of validity if it is too general in its terms or not
limited in time and scope in its application, if it restricts one's expression of belief in a
candidate or one's opinion of his or her qualifications, if it cuts off the flow of media
reporting, and if the regulatory measure bears no clear and reasonable nexus with the
constitutionally sanctioned objective.
The posting of decals and stickers in mobile places like cars and other moving vehicles
does not endanger any substantial government interest. There is no clear public
Page 178

interest threatened by such activity so as to justify the curtailment of the cherished


citizen's right of free speech and expression. Under the clear and present danger rule
not only must the danger be patently clear and pressingly present but the evil sought
to be avoided must be so substantive as to justify a clamp over one's mouth or a
writing instrument to be stilled. The regulation strikes at the freedom of an individual to
express his preference and, by displaying it on his car, to convince others to agree with
him. A sticker may be furnished by a candidate but once the car owner agrees to have
it placed on his private vehicle, the expression becomes a statement by the owner,
primarily his own and not of anybody else. The restriction as to where the decals and
stickers should be posted is so broad that it encompasses even the citizen's private
property, which in this case is a privately-owned vehicle. In consequence of this
prohibition, another cardinal rule prescribed by the Constitution would be violated.
Section 1, Article III of the Bill of Rights provides that no person shall be deprived of his
property without due process of law.
The prohibition on posting of decals and stickers on "mobile" places whether public or
private except in the authorized areas designated by the COMELEC becomes
censorship.

NATIONAL PRESS CLUB VS. COMELEC


[201 SCRA 1; G.R. NO. 1026653; 5 MAR 1992]
Facts:
Petitioners in these cases consist of representatives of the mass media which are
prevented from selling or donating space and time for political advertisements; two (2)
individuals who are candidates for office (one for national and the other for provincial
office) in the coming May 1992 elections; and taxpayers and voters who claim that their
right to be informed of election Issue and of credentials of the candidates is being
curtailed. It is principally argued by petitioners that Section 11 (b) of Republic Act No.
66461 invades and violates the constitutional guarantees comprising freedom of
expression. Petitioners maintain that the prohibition imposed by Section 11 (b) amounts
to censorship, because it selects and singles out for suppression and repression with
criminal sanctions, only publications of a particular content, namely, media-based
election or political propaganda during the election period of 1992. It is asserted that
the prohibition is in derogation of media's role, function and duty to provide adequate
channels of public information and public opinion relevant to election Issue. Further,
petitioners contend that Section 11 (b) abridges the freedom of speech of candidates,
and that the suppression of media-based campaign or political propaganda except
those appearing in the Comelec space of the newspapers and on Comelec time of radio
and television broadcasts, would bring about a substantial reduction in the quantity or
volume of information concerning candidates and Issue in the election thereby
curtailing and limiting the right of voters to information and opinion.
Issue:
Page 179

Whether or Not Section 11 (b) of Republic Act No. 6646 constitutional.


Held:
Yes. It seems a modest proposition that the provision of the Bill of Rights which
enshrines freedom of speech, freedom of expression and freedom of the press has to
be taken in conjunction with Article IX (C) (4) which may be seen to be a special
provision applicable during a specific limited period i.e., "during the election period."
In our own society, equality of opportunity to proffer oneself for public office, without
regard to the level of financial resources that one may have at one's disposal, is clearly
an important value. One of the basic state policies given constitutional rank by Article
II, Section 26 of the Constitution is the egalitarian demand that "the State shall
guarantee equal access to opportunities for public service and prohibit political
dynasties as may be defined by law." The essential question is whether or not the
assailed legislative or administrative provisions constitute a permissible exercise of the
power of supervision or regulation of the operations of communication and information
enterprises during an election period, or whether such act has gone beyond permissible
supervision or regulation of media operations so as to constitute unconstitutional
repression of freedom of speech and freedom of the press. The Court considers that
Section 11 (b) has not gone outside the permissible bounds of supervision or regulation
of media operations during election periods.
Section 11 (b) is limited in the duration of its applicability and enforceability. By virtue
of the operation of Article IX (C) (4) of the Constitution, Section 11 (b) is limited in its
applicability in time to election periods. Section 11 (b) does not purport in any way to
restrict the reporting by newspapers or radio or television stations of news or newsworthy events relating to candidates, their qualifications, political parties and programs
of government. Moreover, Section 11 (b) does not reach commentaries and expressions
of belief or opinion by reporters or broadcasters or editors or commentators or
columnists in respect of candidates, their qualifications, and programs and so forth, so
long at least as such comments, opinions and beliefs are not in fact advertisements for
particular candidates covertly paid for. In sum, Section 11 (b) is not to be read as
reaching any report or commentary other coverage that, in responsible media, is not
paid for by candidates for political office. Section 11 (b) as designed to cover only paid
political advertisements of particular candidates.
The limiting impact of Section 11 (b) upon the right to free speech of the candidates
themselves is not unduly repressive or unreasonable.

US VS. BUSTOS
[37 PHIL. 731; G.R. L-12592; 8 MAR 1918]
Facts:
Page 180

In the latter part of 1915, numerous citizens of the Province of Pampanga assembled,
and prepared and signed a petition to the Executive Secretary(privileged
communication) through the law office of Crossfield and O'Brien, and five individuals
signed affidavits, charging Roman Punsalan, justice of the peace of Macabebe and
Masantol, Pampanga, with malfeasance in office and asking for his removal. The
specific charges against the justice of the peace include the solicitation of money from
persons who have pending cases before the judge. Now, Punsalan alleged that accused
published a writing which was false, scandalous, malicious, defamatory, and libelous
against him.
Issue:
Whether or Not accused is entitled to constitutional protection by virtue of his right to
free speech and free press.
Held:
Yes. The guaranties of a free speech and a free press include the right to criticize
judicial conduct. The administration of the law is a matter of vital public concern.
Whether the law is wisely or badly enforced is, therefore, a fit subject for proper
comment. If the people cannot criticize a justice of the peace or a judge the same as
any other public officer, public opinion will be effectively suppressed. It is a duty which
every one owes to society or to the State to assist in the investigation of any alleged
misconduct. It is further the duty of all who know of any official dereliction on the part
of a magistrate or the wrongful act of any public officer to bring the facts to the notice
of those whose duty it is to inquire into and punish them.
The right to assemble and petition is the necessary consequence of republican
institutions and the complement of the part of free speech. Assembly means a right on
the part of citizens to meet peaceably for consultation in respect to public affairs.
Petition means that any person or group of persons can apply, without fear of penalty,
to the appropriate branch or office of the government for a redress of grievances. The
persons assembling and petitioning must, of course, assume responsibility for the
charges made. All persons have an interest in the pure and efficient administration of
justice and of public affairs.
Public policy, the welfare of society, and the orderly administration of government have
demanded protection for public opinion. The inevitable and incontestable result has
been the development and adoption of the doctrine of privilege. All persons have an
interest in the pure and efficient administration of justice and of public affairs. The duty
under which a party is privileged is sufficient if it is social or moral in its nature and this
person in good faith believes he is acting in pursuance thereof although in fact he is
mistaken. Although the charges are probably not true as to the justice of the peace,
they were believed to be true by the petitioners. Good faith surrounded their action.
Probable cause for them to think that malfeasance or misfeasance in office existed is
Page 181

apparent. The ends and the motives of these citizens to secure the removal from
office of a person thought to be venal were justifiable. In no way did they abuse the
privilege.
In the usual case malice can be presumed from defamatory words. Privilege destroys
that presumption. A privileged communication should not be subjected to microscopic
examination to discover grounds of malice or falsity.

PITA VS. COURT OF APPEALS


[178 SCRA 362; G.R. NO.80806; 5 OCT 1989]
Facts:
On December 1 and 3, 1983, pursuing an Anti-Smut Campaign initiated by the Mayor of
the City of Manila, Ramon D. Bagatsing, elements of the Special Anti-Narcotics Group,
Auxilliary Services Bureau, Western Police District, INP of the Metropolitan Police Force
of Manila, seized and confiscated from dealers, distributors, newsstand owners and
peddlers along Manila sidewalks, magazines, publications and other reading materials
believed to be obscene, pornographic and indecent and later burned the seized
materials in public at the University belt along C.M. Recto Avenue, Manila, in the
presence of Mayor Bagatsing and several officers and members of various student
organizations.
Among the publications seized, and later burned, was "Pinoy Playboy" magazines
published and co-edited by plaintiff Leo Pita.

Plaintiff filed a case for injunction with prayer for issuance of the writ of preliminary
injunction against Mayor Bagatsing and Narcisco Cabrera, as superintendent of Western
Police District of the City of Manila, seeking to enjoin said defendants and their agents
from confiscating plaintiffs magazines or from preventing the sale or circulation thereof
claiming that the magazine is a decent, artistic and educational magazine which is not
per se obscene, and that the publication is protected by the Constitutional guarantees
of freedom of speech and of the press. Plaintiff also filed an Urgent Motion for issuance
of a temporary restraining order against indiscriminate seizure, confiscation and
burning of plaintiff's "Pinoy Playboy" Magazines, pending hearing on the petition for
preliminary injunction. The Court granted the temporary restraining order. The case was
set for trial upon the lapse of the TRO. RTC ruled that the seizure was valid. This was
affirmed by the CA.
Issue:
Whether or Not the seizure violative of the freedom of expression of the petitioner.

Page 182

Held:
Freedom of the press is not without restraint as the state has the right to protect
society from pornographic literature that is offensive to public morals, as indeed we
have laws punishing the author, publishers and sellers of obscene publications.
However, It is easier said than done to say, that if the pictures here in question were
used not exactly for art's sake but rather for commercial purposes, the pictures are not
entitled to any constitutional protection. Using the Kottinger rule: the test of obscenity
is "whether the tendency of the matter charged as obscene, is to deprave or corrupt
those whose minds are open to such immoral influences and into whose hands a
publication or other article charged as being obscene may fall." Another is whether it
shocks the ordinary and common sense of men as an indecency. Ultimately "whether a
picture is obscene or indecent must depend upon the circumstances of the case and
that the question is to be decided by the "judgment of the aggregate sense of the
community reached by it." The government authorities in the instant case have not
shown the required proof to justify a ban and to warrant confiscation of the literature
First of all, they were not possessed of a lawful court order: (1) finding the said
materials to be pornography, and (2) authorizing them to carry out a search and
seizure, by way of a search warrant. The court provides that the authorities must apply
for the issuance of a search warrant from a judge, if in their opinion an obscenity
seizure is in order and that;
1. The authorities must convince the court that the materials sought to be seized
are obscene and pose a clear and present danger of an evil substantive enough
to warrant State interference and action;
2. The judge must determine whether or not the same are indeed obscene. The
question is to be resolved on a case-to-case basis and on the judges sound
discretion;

AYER PRODUCTIONS VS. CAPULONG


[160 SCRA 861; G.R. NO. L-82380; 29 APR 1988]
Facts:
Petitioner McElroy an Australian film maker, and his movie production company, Ayer
Productions, envisioned, sometime in 1987, for commercial viewing and for Philippine
and international release, the historic peaceful struggle of the Filipinos at EDSA. The
proposed motion picture entitled "The Four Day Revolution" was endorsed by the
MTRCB as and other government agencies consulted. Ramos also signified his approval
of the intended film production.
It is designed to be viewed in a six-hour mini-series television play, presented in a
"docu-drama" style, creating four fictional characters interwoven with real events, and
utilizing actual documentary footage as background. David Williamson is Australia's
Page 183

leading playwright and Professor McCoy (University of New South Wales) is an American
historian have developed a script.
Enrile declared that he will not approve the use, appropriation, reproduction and/or
exhibition of his name, or picture, or that of any member of his family in any cinema or
television production, film or other medium for advertising or commercial exploitation.
petitioners acceded to this demand and the name of Enrile was deleted from the movie
script, and petitioners proceeded to film the projected motion picture. However, a
complaint was filed by Enrile invoking his right to privacy. RTC ordered for the
desistance of the movie production and making of any reference to plaintiff or his
family and from creating any fictitious character in lieu of plaintiff which nevertheless is
based on, or bears substantial or marked resemblance to Enrile. Hence the appeal.
Issue:
Whether or Not freedom of expression was violated.

Held:
Yes. Freedom of speech and of expression includes the freedom to film and produce
motion pictures and exhibit such motion pictures in theaters or to diffuse them through
television. Furthermore the circumstance that the production of motion picture films is
a commercial activity expected to yield monetary profit, is not a disqualification for
availing of freedom of speech and of expression.
The projected motion picture was as yet uncompleted and hence not exhibited to any
audience. Neither private respondent nor the respondent trial Judge knew what the
completed film would precisely look like. There was, in other words, no "clear and
present danger" of any violation of any right to privacy. Subject matter is one of public
interest and concern. The subject thus relates to a highly critical stage in the history of
the country.
At all relevant times, during which the momentous events, clearly of public concern,
that petitioners propose to film were taking place, Enrile was a "public figure:" Such
public figures were held to have lost, to some extent at least, their right to privacy.
The line of equilibrium in the specific context of the instant case between the
constitutional freedom of speech and of expression and the right of privacy, may be
marked out in terms of a requirement that the proposed motion picture must be fairly
truthful and historical in its presentation of events.
Page 184

LOPEZ VS. SANDIGANBAYAN


[34 SCRA 116; L-26549; 31 JUL 1970]
Facts:
In the early part of January, 1956, there appeared on the front page of The Manila
Chronicle, of which petitioner Lopez was the publisher, as well as on other dailies, a
news story of a sanitary inspector assigned to the Babuyan Islands, Fidel Cruz, sending
a distress signal to a passing United States Airforce plane which in turn relayed the
message to Manila. An American Army plane dropping on the beach of an island an
emergency-sustenance kit containing, among other things, a two-way radio set. He
utilized it to inform authorities in Manila that the people in the place were living in
terror, due to a series of killings committed since Christmas of 1955. Losing no time,
the Philippines defense establishment rushed to the island a platoon of scout rangers.
Upon arriving Major Encarnacion and his men found, instead of the alleged killers, a
man named Fidel Cruz who merely wanted transportation home to Manila. In view of
this finding, Major Encarnacion branded as a "hoax," the report of respondent.
This Week Magazine of the Manila Chronicle, then edited by Gatbonton, devoted a
pictorial article to it in its issue of January 15, 1956. Mention was made that while Fidel
Cruz story turned out to be false it brought attention to the government that people in
that most people in the area are sick sick, only two individuals able to read and write,
food and clothing being scarce.
The magazine carried photographs of the person purporting to be Fidel Cruz.
Unfortunately, the pictures that were published were that of private respondent Fidel G.
Cruz, a businessman contractor from Santa Maria, Bulacan. It turned out that the
photographs of respondent Cruz and that of Fidel Cruz, sanitary inspector, were on file
in the library of the Manila Chronicle but when the news quiz format was prepared, the
two photographs were in advertently switched. However a correction was published
immediately.
Respondent sued petitioners in the Court of First Instance of Manila for the recovery of
damages alleging the defamatory character of the above publication of his picture.
Defense interposed that they are beating the deadline. The court ruled in his favor.
Hence the appeal.
Issue:
Whether or Not petitioners abused the freedom of the press.

Page 185

Held:
No. The SC, quoting Quisumbing v. Lopez, found for plaintiff, but with reduced
damages, since the error in this case could have been checked considering that this
was a weekly magazine and not a daily. The ruling: "there is no evidence in the record
to prove that the publication of the news item under consideration was prompted by
personal ill will or spite, or that there was intention to do harm,' and that on the other
hand there was 'an honest and high sense of duty to serve the best interests of the
public, without self-seeking motive and with malice towards none.' Every citizen of
course has the right to enjoy a good name and reputation, but we do not consider that
the respondents, under the circumstances of this case, had violated said right or
abused the freedom of the press. The newspapers should be given such leeway and
tolerance as to enable them to courageously and effectively perform their important
role in our democracy. In the preparation of stories, press reporters and editors usually
have to race with their deadlines; and consistently with good faith and reasonable care,
they should not be held to account, to a point of suppression, for honest mistakes or
imperfection in the choice of words.
No inroads on press freedom should be allowed in the guise of punitive action visited
on what otherwise could be characterized as libel whether in the form of printed words
or a defamatory imputation resulting from the publication of respondent's picture with
the offensive caption as in the case here complained of. This is merely to underscore
the primacy that freedom of the press enjoys.

PRIMICIAS VS. FUGOSO


[80 PHIL 71; L-1800; 27 JAN 1948]
Facts:
An action was instituted by the petitioner for the refusal of the respondent to issue a
permit to them to hold a public meeting in Plaza Miranda for redress of grievances to
the government. The reason alleged by the respondent in his defense for refusing the
permit is, "that there is a reasonable ground to believe, basing upon previous
utterances and upon the fact that passions, specially on the part of the losing groups,
remains bitter and high, that similar speeches will be delivered tending to undermine
the faith and confidence of the people in their government, and in the duly constituted
authorities, which might threaten breaches of the peace and a disruption of public
order." Giving emphasis as well to the delegated police power to local government.
Stating as well Revised Ordinances of 1927 prohibiting as an offense against public
peace, and penalizes as a misdemeanor, "any act, in any public place, meeting, or
procession, tending to disturb the peace or excite a riot; or collect with other persons in
a body or crowd for any unlawful purpose; or disturb or disquiet any congregation
engaged in any lawful assembly." Included herein is Sec. 1119, Free use of Public
Place.1
Page 186

Issue:
Whether or Not the freedom of speech was violated.
Held:
Yes. Dealing with the ordinance, specifically, Sec. 1119, said section provides for two
constructions: (1) the Mayor of the City of Manila is vested with unregulated discretion
to grant or refuse, to grant permit for the holding of a lawful assembly or meeting,
parade, or procession in the streets and other public places of the City of Manila; (2)
The right of the Mayor is subject to reasonable discretion to determine or specify the
streets or public places to be used with the view to prevent confusion by overlapping,
to secure convenient use of the streets and public places by others, and to provide
adequate and proper policing to minimize the risk of disorder. The court favored the
second construction. First construction tantamount to authorizing the Mayor to prohibit
the use of the streets. Under our democratic system of government no such unlimited
power may be validly granted to any officer of the government, except perhaps in
cases of national emergency.
The Mayors first defense is untenable. Fear of serious injury cannot alone justify
suppression of free speech and assembly. It is the function of speech to free men from
the bondage of irrational fears. To justify suppression of free speech there must be
reasonable ground to fear that serious evil will result if free speech is practiced. There
must be reasonable ground to believe that the danger apprehended is imminent. There
must be reasonable ground to believe that the evil to be prevented is a serious one .
The fact that speech is likely to result in some violence or in destruction of property is
not enough to justify its suppression. There must be the probability of serious injury to
the state.

ZALDIVAR VS. SANDIGANBAYAN


[170 SCRA 1; G.R. NO. 79690-707; 1 FEB 1989]
Facts:
The case stemmed from the resolution of the Supreme Court stopping the respondent
from investigating graft cases involving Antique Gov. Enrique Zaldivar. The Court ruled
that since the adoption of the 1987 Constitution, respondents powers as Tanodbayan
have been superseded by the creation of the Office of the Ombudsman, he however
becomes the Special Prosecutor of the State, and can only conduct an investigation and
file cases only when so authorized by the Ombudsman. A motion for reconsideration
was filed by the respondent wherein he included statements which were unrelated in
the Issue raised in the Court. This include: (a)That he had been approached twice by a
leading member of the court and he was asked to 'go slow on Zaldivar and 'not to be
Page 187

too hard on him; (b) That he "was approached and asked to refrain from investigating
the COA report on illegal disbursements in the Supreme Court because 'it will embarass
the Court;" and (c) that in several instances, the undersigned respondent was called
over the phone by a leading member of the Court and was asked to dismiss the cases
against two Members of the Court." Statements of the respondent saying that the SCs
order '"heightens the people's apprehension over the justice system in this country,
especially because the people have been thinking that only the small fly can get it
while big fishes go scot-free was publicized in leading newspapers.
Now, the Court Resolved to require respondent to explain in writing why he should not
be punished for contempt of court for making such public statements reported in the
media. Respondent then sought to get some members of the Court to inhibit
themselves in the resolution of the Zaldivar case for alleged bias and prejudice against
him. A little later, he in effect asked the whole Court to inhibit itself from passing upon
the Issue involved in proceeding and to pass on responsibility for this matter to the
Integrated Bar of the Philippines, upon the ground that respondent cannot expect due
process from this Court, that the Court has become incapable of judging him impartially
and fairly. The Court found respondent guilty of contempt of court and indefinitely
suspended from the practice of law. Now, he assails said conviction, invoking his
freedom of speech. Counsel for respondent urges that it is error "for this Court to apply
the "visible tendency" rule rather than the "clear and present danger" rule in
disciplinary and contempt charges."
Issue:
Whether or Not there was a violation of the freedom of speech/expression.
Held:
There was no violation. The Court did not purport to announce a new doctrine of
"visible tendency," it was simply paraphrasing Section 3 (d) of Rule 71 of the Revised
Rules of Court which penalizes a variety of contumacious conduct including: "any
improper conduct tending, directly or indirectly, to impede, obstruct or degrade the
administration of justice."
Under either the "clear and present danger" test or the "balancing-of-interest test," the
Court held that the statements made by respondent Gonzalez are of such a nature and
were made in such a manner and under such circumstances, as to transcend the
permissible limits of free speech. What is here at stake is the authority of the Supreme
Court to confront and prevent a "substantive evil" consisting not only of the obstruction
of a free and fair hearing of a particular case but also the avoidance of the broader evil
of the degradation of the judicial system of a country and the destruction of the
standards of professional conduct required from members of the bar and officers of the
courts, which has some implications to the society.
REYES VS. BAGATSING
Page 188

[125 SCRA 553; L-65366; 9 NOV 1983]


Facts:
Petitioner sought a permit from the City of Manila to hold a peaceful march and rally on
October 26, 1983 from 2:00 to 5:00 in the afternoon, starting from the Luneta to the
gates of the United States Embassy. Once there, and in an open space of public
property, a short program would be held. The march would be attended by the local
and foreign participants of such conference. That would be followed by the handing
over of a petition based on the resolution adopted at the closing session of the AntiBases Coalition. There was likewise an assurance in the petition that in the exercise of
the constitutional rights to free speech and assembly, all the necessary steps would be
taken by it "to ensure a peaceful march and rally. However the request was denied.
Reference was made to persistent intelligence reports affirming the plans of
subversive/criminal elements to infiltrate or disrupt any assembly or congregations
where a large number of people is expected to attend. Respondent suggested that a
permit may be issued if it is to be held at the Rizal Coliseum or any other enclosed area
where the safety of the participants themselves and the general public may be
ensured. An oral argument was heard and the mandatory injunction was granted on the
ground that there was no showing of the existence of a clear and present danger of a
substantive evil that could justify the denial of a permit. However Justice Aquino
dissented that the rally is violative of Ordinance No. 7295 of the City of Manila
prohibiting the holding of rallies within a radius of five hundred (500) feet from any
foreign mission or chancery and for other purposes. Hence the Court resolves.
Issue:
Whether or Not the freedom of expression and the right to peaceably assemble
violated.
Held:
Yes. The invocation of the right to freedom of peaceable assembly carries with it the
implication that the right to free speech has likewise been disregarded. It is settled law
that as to public places, especially so as to parks and streets, there is freedom of
access. Nor is their use dependent on who is the applicant for the permit, whether an
individual or a group. There can be no legal objection, absent the existence of a clear
and present danger of a substantive evil, on the choice of Luneta as the place where
the peace rally would start. Time immemorial Luneta has been used for purposes of
assembly, communicating thoughts between citizens, and discussing public questions.
Such use of the public places has from ancient times, been a part of the privileges,
immunities, rights, and liberties of citizens.
With regard to the ordinance, there was no showing that there was violation and even if
it could be shown that such a condition is satisfied it does not follow that respondent
Page 189

could legally act the way he did. The validity of his denial of the permit sought could
still be challenged.
A summary of the application for permit for rally: The applicants for a permit to hold an
assembly should inform the licensing authority of the date, the public place where and
the time when it will take place. If it were a private place, only the consent of the owner
or the one entitled to its legal possession is required. Such application should be filed
well ahead in time to enable the public official concerned to appraise whether there
may be valid objections to the grant of the permit or to its grant but at another public
place. It is an indispensable condition to such refusal or modification that the clear and
present danger test be the standard for the decision reached. Notice is given to
applicants for the denial.
BAYAN VS. EXECUTIVE SECRETARY ERMITA
[488 SCRA 226; G.R. NO. 169838; 25 APR 2006]
Facts:
Rallies of September 20, October 4, 5 and 6, 2005 is at issue. BAYANs rally was
violently dispersed. 26 petitioners were injured, arrested and detained when a peaceful
mass action they was preempted and violently dispersed by the police. KMU asserts
that the right to peaceful assembly, are affected by Batas Pambansa No. 880 and the
policy of Calibrated Preemptive Response (CPR) being followed to implement it. KMU,
et al., claim that on October 4, 2005, a rally KMU co-sponsored was to be conducted at
the Mendiola bridge but police blocked them along C.M. Recto and Lepanto Streets and
forcibly dispersed them, causing injuries to several of their members. They further
allege that on October 6, 2005, a multi-sectoral rally which KMU also co-sponsored was
scheduled to proceed along Espaa Avenue in front of the UST and going towards
Mendiola bridge. Police officers blocked them along Morayta Street and prevented
them from proceeding further. They were then forcibly dispersed, causing injuries on
one of them. Three other rallyists were arrested.
All petitioners assail Batas Pambansa No. 880 The Public Assembly Act of 1985, some of
them in toto and others only Sections 4, 5, 6, 12, 13(a), and 14(a), as well as the policy
of CPR. They seek to stop violent dispersals of rallies under the no permit, no rally
policy and the CPR policy announced on Sept. 21, 2005.
Petitioners Bayan, et al., contend that BP 880 is clearly a violation of the Constitution
and the International Covenant on Civil and Political Rights and other human rights
treaties of which the Philippines is a signatory.
They argue that B.P. No. 880 requires a permit before one can stage a public assembly
regardless of the presence or absence of a clear and present danger. It also curtails the
choice of venue and is thus repugnant to the freedom of expression clause as the time
Page 190

and place of a public assembly form part of the message for which the expression is
sought.
Petitioners Jess del Prado, et al., in turn, argue that B.P. No. 880 is unconstitutional as it
is a curtailment of the right to peacefully assemble and petition for redress of
grievances because it puts a condition for the valid exercise of that right. It also
characterizes public assemblies without a permit as illegal and penalizes them and
allows their dispersal. Thus, its provisions are not mere regulations but are actually
prohibitions. Regarding the CPR policy, it is void for being an ultra vires act that alters
the standard of maximum tolerance set forth in B.P. No. 880, aside from being void for
being vague and for lack of publication.
KMU, et al., argue that the Constitution sets no limits on the right to assembly and
therefore B.P. No. 880 cannot put the prior requirement of securing a permit. And even
assuming that the legislature can set limits to this right, the limits provided are
unreasonable: First, allowing the Mayor to deny the permit on clear and convincing
evidence of a clear and present danger is too comprehensive. Second, the five-day
requirement to apply for a permit is too long as certain events require instant public
assembly, otherwise interest on the issue would possibly wane.As to the CPR policy,
they argue that it is preemptive, that the government takes action even before the
rallyists can perform their act, and that no law, ordinance or executive order supports
the policy. Furthermore, it contravenes the maximum tolerance policy of B.P. No. 880
and violates the Constitution as it causes a chilling effect on the exercise by the people
of the right to peaceably assemble.
Respondents argued that petitioners have no standing. BP 880 entails traffic re-routing
to prevent grave public inconvenience and serious or undue interference in the free
flow of commerce and trade. It is content-neutral regulation of the time, place and
manner of holding public assemblies. According to Atienza RA. 7160 gives the Mayor
power to deny a permit independently of B.P. No. 880. and that the permit is for the use
of a public place and not for the exercise of rights; and that B.P. No. 880 is not a
content-based regulation because it covers all rallies.
Issue:
Whether or Not BP 880 and the CPR Policy unconstitutional.
Held:
No question as to standing. Their right as citizens to engage in peaceful assembly and
exercise the right of petition, as guaranteed by the Constitution, is directly affected by
B.P. No. 880. B.P. 880 is not an absolute ban of public assemblies but a restriction that
simply regulates the time, place and manner of the assemblies. It refers to all kinds of
public assemblies that would use public places. The reference to lawful cause does
not make it content-based because assemblies really have to be for lawful causes,
Page 191

otherwise they would not be peaceable and entitled to protection. Maximum


tolerance1 is for the protection and benefit of all rallyists and is independent of the
content of the expressions in the rally. There is, likewise, no prior restraint, since the
content of the speech is not relevant to the regulation.
The so-called calibrated preemptive response policy has no place in our legal
firmament and must be struck down as a darkness that shrouds freedom. It merely
confuses our people and is used by some police agents to justify abuses. Insofar as it
would purport to differ from or be in lieu of maximum tolerance, this was declared null
and void.
The Secretary of the Interior and Local Governments, are DIRECTED to take all
necessary steps for the immediate compliance with Section 15 of Batas Pambansa No.
880 through the establishment or designation of at least one suitable freedom park or
plaza in every city and municipality of the country. After thirty (30) days from the
finality of this Decision, subject to the giving of advance notices, no prior permit shall
be required to exercise the right to peaceably assemble and petition in the public parks
or plazas of a city or municipality that has not yet complied with Section 15 of the law.

FERNANDO VS. ESTORNINOS


[G.R. NO 159751; 6 DEC 2006]
Facts:
Acting on reports of sale and distribution of pornographic materials, officers of the PNP
Criminal Investigation and Detection Group in the National Capital Region conducted
police surveillance on the store Gaudencio E. Fernando Music Fair (Music Fair) in
Quiapo. A Search Warrant for violation of Article 201 of RPC against petitioner and a
certain Warren Tingchuy and the seizure of the following items:
a.

Copies of New Rave Magazines with nude obscene pictures;

b.

Copies of IOU Penthouse Magazine with nude obscene pictures;

c.

Copies of Hustler International Magazine with nude obscene pictures; and

d.

Copies of VHS tapes containing pornographic shows.

The police searched the premises and confiscated twenty-five VHS tapes(among of
which is Kahit sa Pangarap Lang with Myra Manibog as actress who is naked) and ten
different magazines(Dalaga, Penthouse, Swank, Erotic, Rave, Playhouse, Gallery, QUI),
which they deemed pornographic. Petitioners were charged and convicted. CA affirmed
the decision hence this appeal.

Page 192

Issue:
Whether or Not the CA erred in affirming RTCs decision.
Held:
No. As obscenity is an unprotected speech which the State has the right to regulate,
the State in pursuing its mandate to protect the public from obscene, immoral and
indecent materials must justify the regulation or limitation. (Kottinger Rule Applied).

MALABANAN VS. RAMENTO


[129 SCRA 359; G.R. NO.62270; 21 MAY 1984]
Facts:
Petitioners were officers of the Supreme Student Council of respondent University. They
sought and were granted by the school authorities a permit to hold a meeting from
8:00 A.M. to 12:00 P.M, on August 27, 1982. Pursuant to such permit, along with other
students, they held a general assembly at the Veterinary Medicine and Animal Science
basketball court (VMAS), the place indicated in such permit, not in the basketball court
as therein stated but at the second floor lobby. At such gathering they manifested in
vehement and vigorous language their opposition to the proposed merger of the
Institute of Animal Science with the Institute of Agriculture. The same day, they
marched toward the Life Science Building and continued their rally. It was outside the
area covered by their permit. Even they rallied beyond the period allowed. They were
asked to explain on the same day why they should not be held liable for holding an
illegal assembly. Then on September 9, 1982, they were informed that they were under
preventive suspension for their failure to explain the holding of an illegal assembly. The
validity thereof was challenged by petitioners both before the Court of First Instance of
Rizal against private respondents and before the Ministry of Education, Culture, and
Sports. Respondent Ramento found petitioners guilty of the charge of illegal assembly
which was characterized by the violation of the permit granted resulting in the
disturbance of classes and oral defamation. The penalty was suspension for one
academic year. Hence this petition.
Issue:
Whether on the facts as disclosed resulting in the disciplinary action and the penalty
imposed, there was an infringement of the right to peaceable assembly and its cognate
right of free speech.
Held:
Page 193

Yes. Student leaders are likely to be assertive and dogmatic. They would be ineffective
if during a rally they speak in the guarded and judicious language of the academe. But
with the activity taking place in the school premises and during the daytime, no clear
and present danger of public disorder is discernible. This is without prejudice to the
taking of disciplinary action for conduct, "materially disrupts classwork or involves
substantial disorder or invasion of the rights of others."
The rights to peaceable assembly and free speech are guaranteed students of
educational institutions. Necessarily, their exercise to discuss matters affecting their
welfare or involving public interest is not to be subjected to previous restraint or
subsequent punishment unless there be a showing of a clear and present danger to a
substantive evil that the state, has a right to present. As a corollary, the utmost leeway
and scope is accorded the content of the placards displayed or utterances made. The
peaceable character of an assembly could be lost, however, by an advocacy of disorder
under the name of dissent, whatever grievances that may be aired being susceptible to
correction through the ways of the law. If the assembly is to be held in school premises,
permit must be sought from its school authorities, who are devoid of the power to deny
such request arbitrarily or unreasonably. In granting such permit, there may be
conditions as to the time and place of the assembly to avoid disruption of classes or
stoppage of work of the non-academic personnel. Even if, however, there be violations
of its terms, the penalty incurred should not be disproportionate to the offense.

NON VS. DAMES


[185 SCRA 523; G.R. NO. 89317; 20 MAY 1990]
Facts:
Petitioners, students in private respondent Mabini Colleges, Inc. in Daet, Camarines
Norte, were not allowed to re-enroll by the school for the academic year 1988-1989 for
leading or participating in student mass actions against the school in the preceding
semester. The subject of the protests is not, however, made clear in the pleadings.
Petitioners filed a petition in the court seeking their readmission or re-enrollment to the
school, but the trial court dismissed the petition. They now petition the court to reverse
its ruling in Alcuaz vs. PSBA1, which was also applied in the case. The court said that
petitioners waived their privilege to be admitted for re-enrollment with respondent
college when they adopted, signed, and used its enrollment form for the first semester
of school year 1988-89, which states that: The Mabini College reserves the right to
deny admission of students whose scholarship and attendance are unsatisfactory and
to require withdrawal of students whose conduct discredits the institution and/or whose
activities unduly disrupts or interfere with the efficient operation of the college.
Students, therefore, are required to behave in accord with the Mabini College code of
conduct and discipline.
Page 194

Issue:
Whether or Not the students right to freedom of speech and assembly infringed.
Held:
Yes. The protection to the cognate rights of speech and assembly guaranteed by the
Constitution is similarly available to students is well-settled in our jurisdiction. However
there are limitations. The permissible limitation on Student Exercise of Constitutional
Rights within the school presupposes that conduct by the student, in class or out of it,
which for any reason whether it stems from time, place, or type of behavior should not
materially disrupt classwork or must not involve substantial disorder or invasion of the
rights of others.

IN RE: TULFO
[A.M. NO. 90-4-1545-0; 17 APR 1990]
Facts:
In Oct. 13, 1989, Tulfo wrote an article in his column in PDI 'On Target' stating that the
Supreme Court rendered an idiotic decision in legalizing checkpoints, and again on Oct.
16, 1989, where he called the Supreme Court stupid and "sangkatutak na mga bobo
justices of the Philippine Supreme Court". Tulfo was required to show cause why he
should not be punished for contempt. Tulfo said that he was just reacting emotionally
because he had been a victim of harassment in the checkpoints, and "idiotic" meant
illogical and unwise, and "bobo" was just quoted from other attorneys, and since the
case had been decided and terminated, there was not contempts. Lastly, the article
does not pose any clear and present danger to the Supreme court.
Issue:
Whether or Not Tulfo is in contempt.

Held:
Yes. At the time Tulfo wrote the article, the checkpoints case had not yet been decided
upon, and the Supreme Court was still acting on an MR filed from the CA. The power to
punish is inherent as it is essential for self-preservation. Contempt of court is defiance
Page 195

of the authority, justice and dignity of the courts. It brings disrepute to the court. There
are two kinds of publications which can be punished for contempt:
a. those whose object is to affect the decision in a pending case.
b. those whose object is to bring courts to discredit. Tulfo's article constituted
both.
It should have been okay to criticize if respectful language was used, but if its object is
only to degrade and ridicule, then it is clearly an obstruction of justice. Nothing
constructive can be gained from them. Being emotional is no excuse for being insulting.
Quoting is not an excuse also, because at the end of his article, Tulfo said, "So you bobo
justices, watch out!" Also, he said he was not sorry for having written the articles.
Tulfo is found in contempt of court and is gravely censured.

PBM EMPLOYEES VS. PBM


[51 SCRA 189; G.R. NO. L-31195; 5 JUN 1993]
Facts:
The petitioner Philippine Blooming Mills Employees Organization (PBMEO) is a
legitimate labor union composed of the employees of the respondent Philippine
Blooming Mills Co., Inc., and petitioners. Benjamin Pagcu and Rodulfo Munsod are
officers and members of the petitioner Union. Petitioners claim that on March 1, 1969,
they decided to stage a mass demonstration at Malacaang on March 4, 1969, in
protest against alleged abuses of the Pasig police. PBMEO thru Pagcu confirmed the
planned demonstration and stated that the demonstration or rally cannot be cancelled
because it has already been agreed upon in the meeting. Pagcu explained further that
the demonstration has nothing to do with the Company because the union has no
quarrel or dispute with Management. The Management, thru Atty. C.S. de Leon,
Company personnel manager, informed PBMEO that the demonstration is an inalienable
right of the union guaranteed by the Constitution but emphasized that any
demonstration for that matter should not unduly prejudice the normal operation of the
Company. Workers who without previous leave of absence approved by the Company,
particularly , the officers present who are the organizers of the demonstration, who
shall fail to report for work the following morning shall be dismissed, because such
failure is a violation of the existing CBA and, therefore, would be amounting to an illegal
strike. Because the petitioners and their members numbering about 400 proceeded
with the demonstration despite the pleas of the respondent Company that the first shift
workers should not be required to participate in the demonstration and that the workers
in the second and third shifts should be utilized for the demonstration from 6 A.M. to 2
P.M. on March 4, 1969, filed a charge against petitioners and other employees who
composed the first shift, for a violation of Republic Act No. 875(Industrial Peace Act),
and of the CBA providing for 'No Strike and No Lockout.' Petitioners were held guilty in
by CIR for bargaining in bad faith, hence this appeal.
Page 196

Issue:
Whether or Not the petitioners right to freedom of speech and to peaceable assemble
violated.
Held:
Yes. A constitutional or valid infringement of human rights requires a more stringent
criterion, namely existence of a grave and immediate danger of a substantive evil
which the State has the right to prevent. This is not present in the case. It was to the
interest herein private respondent firm to rally to the defense of, and take up the
cudgels for, its employees, so that they can report to work free from harassment,
vexation or peril and as consequence perform more efficiently their respective tasks
enhance its productivity as well as profits. Herein respondent employer did not even
offer to intercede for its employees with the local police. In seeking sanctuary behind
their freedom of expression well as their right of assembly and of petition against
alleged persecution of local officialdom, the employees and laborers of herein private
respondent firm were fighting for their very survival, utilizing only the weapons afforded
them by the Constitution the untrammelled enjoyment of their basic human rights.
The pretension of their employer that it would suffer loss or damage by reason of the
absence of its employees from 6 o'clock in the morning to 2 o'clock in the afternoon, is
a plea for the preservation merely of their property rights. The employees' pathetic
situation was a stark reality abused, harassment and persecuted as they believed
they were by the peace officers of the municipality. As above intimated, the condition in
which the employees found themselves vis-a-vis the local police of Pasig, was a matter
that vitally affected their right to individual existence as well as that of their families.
Material loss can be repaired or adequately compensated. The debasement of the
human being broken in morale and brutalized in spirit-can never be fully evaluated in
monetary terms. As heretofore stated, the primacy of human rights freedom of
expression, of peaceful assembly and of petition for redress of grievances over
property rights has been sustained. To regard the demonstration against police officers,
not against the employer, as evidence of bad faith in collective bargaining and hence a
violation of the collective bargaining agreement and a cause for the dismissal from
employment of the demonstrating employees, stretches unduly the compass of the
collective bargaining agreement, is "a potent means of inhibiting speech" and therefore
inflicts a moral as well as mortal wound on the constitutional guarantees of free
expression, of peaceful assembly and of petition. Circulation is one of the aspects of
freedom of expression. If demonstrators are reduced by one-third, then by that much
the circulation of the Issue raised by the demonstration is diminished. The more the
participants, the more persons can be apprised of the purpose of the rally. Moreover,
the absence of one-third of their members will be regarded as a substantial indication
of disunity in their ranks which will enervate their position and abet continued alleged
police persecution.

Page 197

THE IMPAIRMENT CLAUSE

Art 3, Sec. 10. No law impairing the obligation of contracts shall be passed.

RUTTER VS. ESTEBAN


[93 PHIL 68; NO.L-3708; 18 MAY 1953]
Facts:
On August 20,1941 Rutter sold to Esteban two parcels of land situated in the Manila for
P9,600 of which P4,800 were paid outright, and the balance was made payable as
follows: P2,400 on or before August 7, 1942, and P2,400 on or before August 27, 1943,
with interest at the rate of 7 percent per annum. To secure the payment of said balance
of P4,800, a first mortgage has been constituted in favor of the plaintiff. Esteban failed
to pay the two installments as agreed upon, as well as the interest that had accrued
and so Rutter instituted an action to recover the balance due, the interest due and the
attorney's fees. The complaint also contains a prayer for sale of the properties
mortgaged in accordance with law. Esteban claims that this is a prewar obligation
contracted and that he is a war sufferer, having filed his claim with the Philippine War
Damage Commission for the losses he had suffered as a consequence of the last war;
and that under section 2 of RA 342(moratorium law), payment of his obligation cannot
be enforced until after the lapse of eight years. The complaint was dismissed. A motion
for recon was made which assails the constitutionality of RA 342.
Issue:
Whether or Not RA 342 unconstitutional on non-impairment clause grounds.
Held:
Yes. The moratorium is postponement of fulfillment of obligations decreed by the state
through the medium of the courts or the legislature. Its essence is the application of
police power. The economic interests of the State may justify the exercise of its
continuing and dominant protective power notwithstanding interference with contracts.
The question is not whether the legislative action affects contracts incidentally, or
directly or indirectly, but whether the legislation is addressed to a legitimate end and
the measures taken are reasonable and appropriate to that end.
However based on the Presidents general SONA and consistent with what the Court
believes to be as the only course dictated by justice, fairness and righteousness,
Page 198

declared that the continued operation and enforcement of RA 342 at the present time is
unreasonable and oppressive, and should not be prolonged should be declared null and
void and without effect. This holds true as regards Executive Orders Nos. 25 and 32,
with greater force and reason considering that said Orders contain no limitation
whatsoever in point of time as regards the suspension of the enforcement and
effectivity of monetary obligations.

ORTIGAS VS. FEATI BANK


[94 SCRA 533; NO.L-24670; 14 DEC 1979]
Facts:
Plaintiff is engaged in real estate business, developing and selling lots to the public,
particularly the Highway Hills Subdivision along EDSA. On March 4, 1952, plaintiff, as
vendor, and Augusto Padilla and Natividad Angeles, as vendees, entered into separate
agreements of sale on installments over two parcels of land of the Subdivision. On July
19, 1962, the said vendees transferred their rights and interests over the aforesaid lots
in favor of one Emma Chavez. Upon completion of payment of the purchase price, the
plaintiff executed the corresponding deeds of sale in favor of Emma Chavez. Both the
agreements (of sale on installment) and the deeds of sale contained the stipulations or
restrictions that:
1. The parcel of land shall be used exclusively for residential purposes, and she
shall not be entitled to take or remove soil, stones or gravel from it or any other
lots belonging to the Seller.
2. All buildings and other improvements (except the fence) which may be
constructed at any time in said lot must be, (a) of strong materials and properly
painted, (b) provided with modern sanitary installations connected either to the
public sewer or to an approved septic tank, and (c) shall not be at a distance of
less than two (2) meters from its boundary lines.
Eventually said lots were bought by defendant. Lot 5 directly from Chavez and Lot 6
from Republic Flour Mills by deed of exchange, with same restrictions. Plaintiff claims
that restriction is for the beautification of the subdivision. Defendant claimed of the
commercialization of western part of EDSA. Defendant began constructing a
commercial bank building. Plaintiff demand to stop it, which forced him to file a case,
which was later dismissed, upholding police power. Motion for recon was denied, hence
the appeal.
Issue:
Page 199

Whether or Not non-impairment clause violated.


Held:
No. Resolution is a valid exercise of police power. EDSA, a main traffic artery which runs
through several cities and municipalities in the Metro Manila area, supports an endless
stream of traffic and the resulting activity, noise and pollution are hardly conducive to
the health, safety or welfare of the residents in its route. Health, safety, peace, good
order and general welfare of the people in the locality are justifications for this. It
should be stressed, that while non-impairment of contracts is constitutionally
guaranteed, the rule is not absolute, since it has to be reconciled with the legitimate
exercise of police power.

LOZANO VS. MARTINEZ


[146 SCRA 323; NO.L-63419; 18 DEC 1986]
Facts:
A motion to quash the charge against the petitioners for violation of the BP 22 was
made, contending that no offense was committed, as the statute is unconstitutional.
Such motion was denied by the RTC. The petitioners thus elevate the case to the
Supreme Court for relief. The Solicitor General, commented that it was premature for
the accused to elevate to the Supreme Court the orders denying their motions to
quash. However, the Supreme Court finds it justifiable to intervene for the review of
lower court's denial of a motion to quash.
Issue:
Whether or Not BP 22 impairs freedom of contract. Whether or not BP 22 transgresses
the constitutional inhibition against imprisonment for debt.
Held:
The freedom of contract which is constitutionally protected is freedom to enter into
"lawful" contracts. Contracts which contravene public policy are not lawful. Checks can
not be categorized as mere contracts. It is a commercial instrument which, in this
modem day and age, has become a convenient substitute for money; it forms part of
the banking system and therefore not entirely free from the regulatory power of the
state.

Page 200

The offense punished by BP 22 is the act of making and issuing a worthless check or a
check that is dishonored upon its presentation for payment. It is not the non-payment
of an obligation which the law punishes. The law is not intended or designed to coerce
a debtor to pay his debt. The thrust of the law is to prohibit, under pain of penal
sanctions, the making of worthless checks and putting them in circulation.

Page 201

EX POST FACTO LAWS

Art 3, Sec. 22. No ex post facto law or bill of attainder shall be enacted.

PEOPLE VS. FERRER


[48 SCRA 382; NOS.L-32613-14; 27 DEC 1972]
Facts:
Hon. Judge Simeon Ferrer is the Tarlac trial court judge that declared RA1700 or the
Anti-Subversive Act of 1957 as a bill of attainder. Thus, dismissing the information of
subversion against the following: 1.) Feliciano Co for being an officer/leader of the
Communist Party of the Philippines (CPP) aggravated by circumstances of contempt
and insult to public officers, subversion by a band and aid of armed men to afford
impunity. 2.) Nilo Tayag and 5 others, for being members/leaders of the NPA, inciting,
instigating people to unite and overthrow the Philippine Government. Attended by
Aggravating Circumstances of Aid or Armed Men, Craft, and Fraud. The trial court is of
opinion that 1.) The Congress usurped the powers of the judge 2.) Assumed judicial
magistracy by pronouncing the guilt of the CPP without any forms of safeguard of a
judicial trial. 3.) It created a presumption of organizational guilt by being members of
the CPP regardless of voluntariness.
The Anti Subversive Act of 1957 was approved 20June1957. It is an act to outlaw the
CPP and similar associations penalizing membership therein, and for other purposes. It
defined the Communist Party being although a political party is in fact an organized
conspiracy to overthrow the Government, not only by force and violence but also by
deceit, subversion and other illegal means. It declares that the CPP is a clear and
present danger to the security of the Philippines. Section 4 provided that affiliation
with full knowledge of the illegal acts of the CPP is punishable. Section 5 states that
due investigation by a designated prosecutor by the Secretary of Justice be made prior
to filing of information in court. Section 6 provides for penalty for furnishing false
evidence. Section 7 provides for 2 witnesses in open court for acts penalized by prision
mayor to death. Section 8 allows the renunciation of membership to the CCP through
writing under oath. Section 9 declares the constitutionality of the statute and its valid
exercise under freedom if thought, assembly and association.
Issue:
Whether or not RA1700 is a bill of attainder/ ex post facto law.
Page 202

Whether or Not RA1700 violates freedom of expression.


Held:
The court holds the VALIDITY Of the Anti-Subversion Act of 1957.
A bill of attainder is solely a legislative act. It punishes without the benefit of the trial.
It is the substitution of judicial determination to a legislative determination of guilt. In
order for a statute be measured as a bill of attainder, the following requisites must be
present: 1.) The statute specifies persons, groups.
2.) the statute is applied
retroactively and reach past conduct. (A bill of attainder relatively is also an ex post
facto law.)
In the case at bar, the statute simply declares the CPP as an organized conspiracy for
the overthrow of the Government for purposes of example of SECTION 4 of the Act. The
Act applies not only to the CPP but also to other organizations having the same purpose
and their successors.
The Acts focus is on the conduct not person.
Membership to this organizations, to be UNLAWFUL, it must be shown that membership
was acquired with the intent to further the goals of the organization by overt acts. This
is the element of MEMBERSHIP with KNOWLEDGE that is punishable. This is the
required proof of a members direct participation. Why is membership punished.
Membership renders aid and encouragement to the organization. Membership makes
himself party to its unlawful acts.
Furthermore, the statute is PROSPECTIVE in nature. Section 4 prohibits acts committed
after approval of the act. The members of the subversive organizations before the
passing of this Act is given an opportunity to escape liability by renouncing
membership in accordance with Section 8. The statute applies the principle of mutatis
mutandis or that the necessary changes having been made.
The declaration of that the CPP is an organized conspiracy to overthrow the Philippine
Government should not be the basis of guilt. This declaration is only a basis of Section
4 of the Act. The EXISTENCE OF SUBSTANTIVE EVIL justifies the limitation to the
exercise of Freedom of Expression and Association in this matter. Before the
enactment of the statute and statements in the preamble, careful investigations by the
Congress were done. The court further stresses that whatever interest in freedom of
speech and association is excluded in the prohibition of membership in the CPP are
weak considering NATIONAL SECURITY and PRESERVATION of DEMOCRACY.
The court set basic guidelines to be observed in the prosecution under RA1700. In
addition to proving circumstances/ evidences of subversion, the following elements
must also be established:

Page 203

1. Subversive Organizations besides the CPP, it must be proven that the


organization purpose is to overthrow the present Government of the
Philippines and establish a domination of a FOREIGN POWER. Membership is
willfully and knowingly done by overt acts.
2. In case of CPP, the continued pursuance of its subversive purpose.
Membership is willfully and knowingly done by overt acts.
The court did not make any judgment on the crimes of the accused under the Act. The
Supreme Court set aside the resolution of the TRIAL COURT.

BAYOT VS. SANDIGANBAYAN


[128 SCRA 383; NO.L-61776 TO NO.L-61861; 23 MAR 1984]
Facts:
Bayot is one of the several persons who was accused in more than 100 counts of estafa
thru falsification of Public documents before the Sandiganbayan. The said charges
started from his alleged involvement as a government auditor of the commission on
audit assigned to the Ministry of education and culture, with some other employees
from the said ministry. The bureau of treasury and the teachers camp in Baguio City for
the preparation and encashment of fictitious TCAA checks for the nom-existent
obligations of the teachers camp resulting in damage to the government of several
millions. The 1st 32 cases were filed on july 25, 1987, while Bayot ran for municipal
mayor of Amadeo Cavite and was elected on January 1980. but on May 1980
Sandiganbayan promulgated a decision convicting the accused together with his other
co-accused in all but one of the thirty two cases filed against them.
On Mach 16, 1982 Batas Pambansa Blg 195 was passed amending RA 3019.
Issue:

Page 204

Whether or Not it would be violative of the constitutional guarantee against an ex post


facto law.
Held:
The court finds no merit in the petitioners contention that RA 3019 as amended by
Batas Pambansa Blg 195, which includes the crime of estafa through falsification of
Public Documents as among crimes subjecting the public officer charged therewith
with suspension from public office pending action in court, is a penal provision which
violates the constitutional prohibition against the enactment of ex post facto law. Accdg
to the RPC suspension from employment and public office during trial shall not be
considered as a penalty. It is not a penalty because it is not a result of a judicial
proceeding. In fact, if acquitted the official who is suspended shall be entitled to
reinstatement and the salaries and benefits which he failed to receive during
suspension. And does not violate the constitutional provision against ex post facto law.
The claim of the petitioner that he cannot be suspended because he is currently
occupying a position diffren tfrom that under which he is charged is untenable. The
amendatory provision clearly states that any incumbent public officer against whom
any criminal prosecution under a valid information under RA 3019 for any offense
involving fraud upon the government or public funds or property or whatever stage of
execution and mode of participation shall be suspended from office. The use of the
word office applies to any office which the officer charged may be holding and not
only the particular office under which he was charged.

PEOPLE VS. SANDIGANBAYAN


[211 SCRA 241; G.R. NO. 101724; 3 JUL 1992]
Facts:
Two letter complaints were filed with the Tanodbayan by Teofilo Gelacio on October
28,1986 and December 9, 1986, a political leader of Governor Valentina Plaza, wife of
Congressman Democrito Plaza of Agusan del Sur, shortly after private respondent had
replaced Mrs. Plaza as OIC/provincial Governor of Agusan del Sur on March 1986 The
complaint questioned the issuance to Governor Paredes, when he was still the
provincial attorney in 1976 of a free patent title for a lot in the Rosario public land
subdivision in San Francisco, Agusan del Sur. He misrepresented to a Lands Inspector of
the Bureau of Lands that the lands subject herein are disposable lands, thereby
inducing said inspector to recommend approval of his application for free patent. On
August 10, 1989 an information for violation of RA 3019 Anti-Graft and Corrupt
Practices Act was then filed in the Sandiganbayan after an ex parte preliminary
Page 205

investigation. A motion to quash the information was filed by the private respondent
contending among others that he is charged for an offence which has prescribed. Said
motion was granted. The crime was committed on January 21, 1976, period of
prescription was 10 years, therefore it has prescribed in 1986. Now the motion to quash
was being assailed.
Issue:
Whether or Not the motion to quash validly granted.
Held:
Yes. RA 3019, being a special law the computation of the period for the prescription of
the crime is governed by Sec. 29 of Act No. 3326, which begins to run from the day of
the commission of the crime and not the discovery of it. Additionally, BP 195 which was
approved on March 16, 1982, amending Sec. 11 of RA 3019 by increasing ten to fifteen
years of the period for the prescription or extinguishment of a violation of RA 3019 may
not be given retroactive application to the crime which was committed by Paredes, as it
is prejudicial to the accused. To apply BP 195 to Paredes would make it an ex post facto
law1 for it would alter his situation to his disadvantage by making him criminally liable
for a crime that had already been extinguished under the law existing when it was
committed.

Page 206

NON-IMPRISONMENT FOR DEBT

Art 3, Sec. 20. No person shall be imprisoned for debt or non-payment of a poll tax.

SERAFIN VS. LINDAYAG


[67 SCRA 166; ADM. MATTER. NO. 297-MJ; 30 SEPT 1975]
Facts:
Plaintiff failed to pay a simple indebtedness for P1500 Carmelito Mendoza, then
municipal secretary and his wife Corazon Mendoza and therefore an estafa case was
filed against her. Complainant admitted complaint. Now complainant filed a case
against respondent Judge for not dismissing the case and issuing a warrant of arrest as
it falls on the category of a simple indebtedness, since elements of estafa are not
present. Further she contended that no person should be imprisoned for non-payment
of a loan of a sum of money. Two months after respondent dismissed plaintiffs case.
(Judge here committed gross ignorance of law. Even if complainant desisted case was
pursued.)
Issue:
Whether or Not there was a violation committed by the judge when it ordered the
imprisonment of plaintiff for non-payment of debt?
Held:
Yes. Since plaintiff did not commit any offense as, his debt is considered a simple loan
granted by her friends to her. There is no collateral or security because complainant
was an old friend of the spouses who lent the money and that when they wrote her a
letter of demand she promised to pay them and said that if she failed to keep her
promise, they could get her valuable things at her home. Under the Constitution she is
protected. Judge therefore in admitting such a "criminal complaint" that was plainly civil
in aspects from the very face of the complaint and the "evidence" presented, and
issuing on the same day the warrant of arrest upon his utterly baseless finding "that the
accused is probably guilty of the crime charged," respondent grossly failed to perform
his duties properly.

LOZANO VS. MARTINEZ


Page 207

[146 SCRA 323; NO.L-63419; 18 DEC 1986]


Facts:
A motion to quash the charge against the petitioners for violation of the BP 22 was
made, contending that no offense was committed, as the statute is unconstitutional.
Such motion was denied by the RTC. The petitioners thus elevate the case to the
Supreme Court for relief. The Solicitor General, commented that it was premature for
the accused to elevate to the Supreme Court the orders denying their motions to
quash. However, the Supreme Court finds it justifiable to intervene for the review of
lower court's denial of a motion to quash.
Issue:
Whether or not BP 22 is constitutional as it is a proper exercise of police power of the
State.
Held:
The enactment of BP 22 a valid exercise of the police power and is not repugnant to the
constitutional inhibition against imprisonment for debt.
The offense punished by BP 22 is the act of making and issuing a worthless check or a
check that is dishonored upon its presentation for payment. It is not the non-payment
of an obligation which the law punishes. The law is not intended or designed to coerce
a debtor to pay his debt.
The law punishes the act not as an offense against property, but an offense against
public order. The thrust of the law is to prohibit, under pain of penal sanctions, the
making of worthless checks and putting them in circulation. An act may not be
considered by society as inherently wrong, hence, not malum in se but because of the
harm that it inflicts on the community, it can be outlawed and criminally punished as
malum prohibitum. The state can do this in the exercise of its police power.

Page 208

INVOLUNTARY SERVITUDE

Art 3, Sec. 18. (2)


No involuntary servitude in any form shall exist except as a
punishment for a crime whereof the party shall have been duly convicted.

CAUNCA VS. SALAZAR


[82 PHIL 851; NO.L-2690; 1 JAN 1949]
Facts:
This is an action for habeas corpus brought by Bartolome Caunca in behalf of his cousin
Estelita Flores who was employed by the Far Eastern Employment Bureau, owned by
Julia Salazar, respondent herein. An advanced payment has already been given to
Estelita by the employment agency, for her to work as a maid. However, Estelita
wanted to transfer to another residence, which was disallowed by the employment
agency. Further she was detained and her liberty was restrained. The employment
agency wanted that the advance payment, which was applied to her transportation
expense from the province should be paid by Estelita before she could be allowed to
leave.
Issue:
Whether or Not an employment agency has the right to restrain and detain a maid
without returning the advance payment it gave?
Held:
An employment agency, regardless of the amount it may advance to a prospective
employee or maid, has absolutely no power to curtail her freedom of movement. The
fact that no physical force has been exerted to keep her in the house of the respondent
does not make less real the deprivation of her personal freedom of movement, freedom
to transfer from one place to another, freedom to choose ones residence. Freedom
may be lost due to external moral compulsion, to founded or groundless fear, to
erroneous belief in the existence of an imaginary power of an impostor to cause harm if
not blindly obeyed, to any other psychological element that may curtail the mental
faculty of choice or the unhampered exercise of the will. If the actual effect of such
psychological spell is to place a person at the mercy of another, the victim is entitled to
the protection of courts of justice as much as the individual who is illegally deprived of
liberty by duress or physical coercion.

Page 209

Page 210

THE WRIT OF HABEAS CORPUS

Art 3, Sec. 15. The privilege of the writ of habeas corpus shall not be suspended
except in cases of invasion or rebellion when the public safety requires it.

LANSANG VS. GARCIA


[42 SCRA 448; L-33964; 11 Dec 1971]
Facts:
In the evening of August 21, 1971, at about 9 p.m., while the Liberal Party of the
Philippines was holding a public meeting at Plaza Miranda, Manila, for the presentation
of its candidates in the general elections scheduled for November 8, 1971, two hand
grenades were thrown at the platform where said candidates and other persons were.
Eight persons were killed and many more injured. Proclamation 889 was issued by the
President suspending privilege of writ of habeas corpus stating that there is a
conspiracy of rebellion and insurrection in order to forcibly seize political power.
Petitions for writ of habeas corpus were filed by persons (13) who have been arrested
without a warrant.
It was stated that one of the safeguards of the proclamation was that it is to be applied
to persons caught in flagrante delicto. Incidentally, Proc. 889-A was issued as an
amendment, inserting the word actually staging. Proc. 889-B was also issued lifting
the suspension of privilege in 27 provinces, 3 sub-provinces and 26 cities. Proc. 889-C
was issued restoring the suspension in 13 provinces and cities(mostly in Mindanao).
Proc. 889-D further lifted the suspension in 7 provinces and 4 cities. Only 18 provinces
and sub-provinces and 2 cities whose privilege was suspended. Petitioners maintained
that Proclamation No. 889 did not declare the existence of actual "invasion insurrection
or rebellion or imminent danger thereof, however it became moot and academic since
it was amended. Petitioners further contend that public safety did not require the
issuance of proclamations stating: (a) that there is no rebellion; (b) that, prior to and at
the time of the suspension of the privilege, the Government was functioning normally,
as were the courts; (c) that no untoward incident, confirmatory of an alleged JulyAugust Plan, has actually taken place after August 21, 1971; (d) that the President's
alleged apprehension, because of said plan, is non-existent and unjustified; and (e) that
the Communist forces in the Philippines are too small and weak to jeopardize public
safety to such extent as to require the suspension of the privilege of the writ of habeas
corpus.
A resolution was issued by majority of the Court having tentatively arrived at a
consensus that it may inquire in order to satisfy itself of the existence of the factual
bases for the proclamations. Now the Court resolves after conclusive decision reached
by majority.
Page 211

Issue:
Whether or Not the authority to decide whether the exigency has arisen requiring
suspension (of the privilege of the writ of habeas corpus) belongs to the President and
his decision is final and conclusive upon the courts and upon all other persons.
Whether or Not public safety require the suspension of the privilege of the writ of
habeas corpus decreed in Proclamation No. 889-A.
Held:
The President has authority however it is subject to judicial review. Two conditions must
concur for the valid exercise of the authority to suspend the privilege to the writ (a)
there must be "invasion, insurrection, or rebellion" or "imminent danger thereof," and
(b) "public safety" must require the suspension of the privilege. President has three (3)
courses of action: (a) to call out the armed forces; (b) to suspend the privilege of the
writ of habeas corpus; and (c) to place the Philippines or any part thereof under martial
law. He had, already, called out the armed forces, proved inadequate. Of the two other
alternatives, the suspension of the privilege is the least harsh.
Petitioners contention that CPP-NPA has no ability, is negatived by the killing of 5
mayors, 20 barrio captains and 3 chiefs of police; that there were fourteen (14)
meaningful bombing incidents in the Greater Manila Area in 1970. CPP has managed to
infiltrate or establish and control nine major labor organizations; has exploited the (11)
major student or youth organizations; about thirty (30) mass organizations actively
advancing the CPP.

Page 212

RIGHTS OF THE ACCUSED

Art 3, Sec. 12. (1) Any person under investigation for the commission of an offense
shall have the right to be informed of his right to remain silent and to have competent
and independent counsel preferably of his own choice. If the person cannot afford the
services of counsel, he must be provided with one. These rights cannot be waived
except in writing and in the presence of counsel.
(2)
No torture, force, violence, threat, intimidation, or any other means which vitiate
the free will shall be used against him. Secret detention places, solitary,
incommunicado, or other similar forms of detention are prohibited.
(3)
Any confession or admission obtained in violation of this or Section 17 hereof
shall be inadmissible in evidence against him.
(4)
The law shall provide for penal and civil sanctions for violations of this section as
well as compensation to and rehabilitation of victims of torture or similar practices, and
their families.
Art 3, Sec. 14. (1) No person shall be held to answer for a criminal offense without
due process of law.
(2) In all criminal prosecutions, the accused shall be presumed innocent until the
contrary is proved, and shall enjoy the right to be heard by himself and counsel, to be
informed of the nature and cause of the accusation against him, to have a speedy,
impartial, and public trial, to meet the witnesses face to face, and to have compulsory
process to secure the attendance of witnesses and the production of evidence in his
behalf. However, after arraignment, trial may proceed notwithstanding the absence of
the accused provided that he has been duly notified and his failure to appear is
unjustifiable.
Art 3, Sec. 11. Free access to the courts and quasi-judicial bodies and adequate
legal assistance shall not be denied to any person by reason of poverty.
Art 3, Sec. 16. All persons shall have the right to a speedy disposition of their cases
before all judicial, quasi-judicial, or administrative bodies.
Art 3, Sec. 17. No person shall be compelled to be a witness against himself.
Art 3, Sec. 19. (1) Excessive fines shall not be imposed, nor cruel, degrading or
inhuman punishment inflicted. Neither shall death penalty be imposed, unless, for
compelling reasons involving heinous crimes, the Congress hereafter provides for it.
Any death penalty already imposed shall be reduced to reclusion perpetua.
Page 213

(2)
The employment of physical, psychological, or degrading punishment against
any prisoner or detainee or the use of substandard or inadequate penal facilities under
subhuman conditions shall be dealt with by law.
Art 3, Sec. 21. No person shall be twice put in jeopardy of punishment for the same
offense. If an act is punished by a law and an ordinance, conviction or acquittal under
either shall constitute a bar to another prosecution for the same act.
GAMBOA VS. CRUZ
[162 SCRA 642;L-56291; 27 JUN 1988]
Facts:
Petitioner was arrested for vagrancy without a warrant. During a line-up of 5 detainees
including petitioner, he was identified by a complainant to be a companion in a
robbery, thereafter he was charged. Petitioner filed a Motion to Acquit on the ground
that the conduct of the line-up, without notice and in the absence of his counsel
violated his constitutional rights to counsel and to due process. The court denied said
motion. Hearing was set, hence the petition.
Issue:
Whether or Not petitioners right to counsel and due process violated.
Held:
No. The police line-up was not part of the custodial inquest, hence, petitioner was not
yet entitled, at such stage, to counsel. He had not been held yet to answer for a
criminal offense. The moment there is a move or even an urge of said investigators to
elicit admissions or confessions or even plain information which may appear innocent
or innocuous at the time, from said suspect, he should then and there be assisted by
counsel, unless he waives the right, but the waiver shall be made in writing and in the
presence of counsel.
On the right to due process, petitioner was not, in any way, deprived of this substantive
and constitutional right, as he was duly represented by a counsel. He was accorded all
the opportunities to be heard and to present evidence to substantiate his defense; only
that he chose not to, and instead opted to file a Motion to Acquit after the prosecution
had rested its case. What due process abhors is the absolute lack of opportunity to be
heard.

Page 214

PEOPLE VS. JUDGE AYSON


[175 SCRA 216; G.R. NO. 85215; 7 JUL 1989]
Facts:
Felipe Ramos was a ticket freight clerk of the Philippine Airlines, assigned at its Baguio
City station. It was alleged that he was involved in irregularities in the sales of plane
tickets, the PAL management notified him of an investigation to be conducted. That
investigation was scheduled in accordance with PAL's Code of Conduct and Discipline,
and the Collective Bargaining Agreement signed by it with the Philippine Airlines
Employees' Association (PALEA) to which Ramos pertained. A letter was sent by Ramos
stating his willingness to settle the amount of P76,000. The findings of the Audit team
were given to him, and he refuted that he misused proceeds of tickets also stating that
he was prevented from settling said amounts. He proffered a compromise however this
did not ensue. Two months after a crime of estafa was charged against Ramos. Ramos
pleaded not guilty. Evidence by the prosecution contained Ramos written admission
and statement, to which defendants argued that the confession was taken without the
accused being represented by a lawyer. Respondent Judge did not admit those stating
that accused was not reminded of his constitutional rights to remain silent and to have
counsel. A motion for reconsideration filed by the prosecutors was denied. Hence this
appeal.
Issue:
Whether or Not the respondent Judge correct in making inadmissible as evidence the
admission and statement of accused.
Held:
No. Section 20 of the 1987 constitution provides that the right against self-incrimination
(only to witnesses other than accused, unless what is asked is relating to a different
crime charged- not present in case at bar).
This is accorded to every person who gives evidence, whether voluntarily or under
compulsion of subpoena, in any civil, criminal, or administrative proceeding. The right is
not to "be compelled to be a witness against himself. It prescribes an "option of refusal
to answer incriminating questions and not a prohibition of inquiry." the right can be
claimed only when the specific question, incriminatory in character, is actually put to
the witness. It cannot be claimed at any other time. It does not give a witness the right
to disregard a subpoena, to decline to appear before the court at the time appointed, or
to refuse to testify altogether. It is a right that a witness knows or should know. He must
claim it and could be waived.

Page 215

Rights in custodial interrogation as laid down in miranda v. Arizona: the rights of the
accused include:
1) he shall have the right to remain silent and to counsel, and to be informed of
such right.
2) nor force, violence, threat, intimidation, or any other means which vitiates the
free will shall be used against him.
3) any confession obtained in violation of these rights shall be inadmissible in
evidence.
The individual may knowingly and intelligently waive these rights and agree to answer
or make a statement. But unless and until such rights and waivers are demonstrated by
the prosecution at the trial, no evidence obtained as a result of interrogation can be
used against him.

PEOPLE VS. MAQUEDA


[242 SCRA 565; G.R. NO.112983; 22 MAR 1994]
Facts:
British Horace William Barker (consultant of WB) was slain inside his house in Tuba,
Benguet while his Filipino wife, Teresita Mendoza was badly battered with lead pipes on
the occasion of a robbery. Two household helpers of the victims identified Salvamante
(a former houseboy of the victims) and Maqueda as the robbers. Mike Tabayan and his
friend also saw the two accused a kilometer away from the house of the victims that
same morning, when the two accused asked them for directions.
Maqueda was then arrested in Guinyangan, Quezon. He was taken to Calauag, Quezon
where he signed a Sinumpaang Salaysay wherein he narrated his participation in the
crime. According to SPO3 Molleno, he informed Maqueda of his constitutional rights
before he signed such document. Afterwards he was brought to the Benguet Provincial
Jail. While he was under detention, Maqueda filed a Motion to Grant Bail. He stated
therein that "he is willing and volunteering to be a State witness in the above entitled
case, it appearing that he is the least guilty among the accused in this case."
Maqueda also admitted his involvement in the commission of the robbery to Prosecutor
Zarate and to Salvosa.
Issue:

Page 216

Whether or Not the trial court was correct in holding that the Sinumpaan Salaysay is
admissible as evidence.
Held:
No. The Sinumpaang Salaysay is inadmissible because it was in clear violation of the
constitutional rights of the accused. First, he was not informed of his right to remain
silent and his right to counsel. Second, he cannot be compelled to be a witness against
himself. At the time of the confession, the accused was already facing charges in court.
He no longer had the right to remain silent and to counsel but he had the right to refuse
to be a witness and not to have any prejudice whatsoever result to him by such refusal.
And yet, despite his knowing fully well that a case had already been filed in court, he
still confessed when he did not have to do so.
The contention of the trial court that the accused is not entitled to such rights anymore
because the information has been filed and a warrant of arrest has been issued already,
is untenable. The exercise of the rights to remain silent and to counsel and to be
informed thereof under Section 12(1) of the Bill of Rights are not confined to that period
prior to the filing of a criminal complaint or information but are available at that stage
when a person is "under investigation for the commission of an offense."
Pursuant to Section 12(3) of the Bill of Rights therefore, such extra-judicial admission is
inadmissible as evidence.
As to the admissions made by Maqueda to Prosecutor Zarate and Ray Dean Salvosa,
the trial court admitted their testimony thereon only to prove the tenor of their
conversation but not to prove the truth of the admission because such testimony was
objected to as hearsay. Maqueda voluntarily and freely made them to Prosecutor Zarate
not in the course of an investigation, but in connection with Maqueda's plea to be
utilized as a state witness; and as to the other admission (Salvosa), it was given to a
private person therefore admissible.
Note: a distinction between a confession and admission has been made by the SC:
Admission of a party. The act, declaration or omission of party as to a relevant fact
may be given in evidence against him.
Confession. The declaration of an accused acknowledging his guilt of the offense
charged, or of any offense necessarily included therein, may be given in evidence
against him.

PEOPLE VS. BANDULA


[232 SCRA 566; G.R. NO. 89223; 27 MAY 1994]
Page 217

Facts:
Six armed men barged into the compound of Polo Coconut Plantation in Tanjay, Negros
Oriental. The armed men were identified by Security Guard, including accused. Salva
and Pastrano, security guards were hogtied and accused proceeded to the Atty. Garay,
counsel of plantation. They ransacked the place and took with them money and other
valuables. Atty. Garay was killed. Accused-appellant is charged with robbery with
homicide along with 3 others who were acquitted for insufficiency of evidence.
Appellant was convicted.
Now, appellant argues that the extrajudicial confessions he and accused Dionanao
executed suffer from constitutional infirmities, hence, inadmissible in evidence
considering that they were extracted under duress and intimidation, and were merely
countersigned later by the municipal attorney who, by the nature of his position, was
not entirely an independent counsel nor counsel of their choice. Consequently, without
the extrajudicial confessions, the prosecution is left without sufficient evidence to
convict him of the crime charged.
Issue:
Whether or Not extrajudicial confessions of appellant is admissible as evidence against
him.
Held:
No. When accused-appellant Bandula and accused Dionanao were investigated
immediately after their arrest, they had no counsel present. If at all, counsel came in
only a day after the custodial investigation with respect to accused Dionanao, and two
weeks later with respect to appellant Bandula. And, counsel who supposedly assisted
both accused was Atty. Ruben Zerna, the Municipal Attorney of Tanjay. On top of this,
there are telltale signs that violence was used against the accused. Certainly, these are
blatant violations of the Constitution which mandates in
Sec. 12, Art. III. Irregularities present include:
1. The investigators did not inform the accused of their right to remain silent and to
have competent and independent counsel, preferably of their own choice, even
before attempting to elicit statements that would incriminate them.
2. Investigators continuously disregard the repeated requests of the accused for
medical assistance. Reason for Accused Sedigos "black eye" which even
Pat. Baldejera admitted is not established, as well as Bandulas fractured rib.

Page 218

3. Counsel must be independent. He cannot be a special counsel, public or private


prosecutor, counsel of the police, or a municipal attorney whose interest is
admittedly adverse to the accused.

PEOPLE VS. LUCERO


[244 SCRA 425; G.R. NO.97936; 29 MAY 1995]
Facts:
Alejandro Lucero, Bienvenido Echavez, Balbino Echavez, Peter Doe, Richard Doe and
John Doe were charged with the crime of robbery with homicide.
The prosecution:
Accused-appellant (alighted from a gray-reddish car), armed with handgun, blocked the
way of the said complainant who was on board a Mercedes Benz passing along Road
14, Mindanao Avenue, Pag-asa, QC, rob and carry away cash money; one gold necklace
with cross pendant, 7 karat; one gold Rolex watch; one 3 karat gold ring; one 2 karat
gold ring, domino style; one solid gold bracelet; all worth P363,600.00, belonging to DR.
DEMETRIO Z. MADRID. Accused shot LORENZO BERNALES y ALERIA, a driver of the said
offended party, thus inflicting upon him mortal wounds, which resulted to the
instantaneous death of ALERIA.
Only the accused Echavez brothers and Alejandro Lucero were apprehended.
When Lucero told him that he had no lawyer, in due time, Atty. Diosdado Peralta
conferred with Lucero. He apprised Lucero of his constitutional rights. He observed no
reaction from Lucero. Nonetheless, Atty. Peralta gathered the impression that Lucero
understood his advice.
When the investigator started asking the preliminary questions, Atty. Peralta left to
attend the wake of his friend. The next morning, Lucero was accompanied by CIS
agents to Atty. Peralta's house. The extrajudicial statement of Lucero was presented to
Atty. Peralta. It was already signed by Lucero.
The three accused denied complicity in the crime charged.
Appellant Lucero's defense is alibi. He testified that he was at his house in Caloocan
City.

Page 219

He said he was surprised when several unidentified men accosted him while he was
walking towards his house. They chased him, handcuffed and blindfolded him and
pushed him into a jeep. He was blindfolded the whole night and did not know where he
was taken. The men turned out to be police officers.
The next day, he learned he was in Camp Crame. He claimed that he was tortured. He
was not informed of the offense for which he was being investigated. Neither did they
reveal the identity of the complainant.
Lucero denied knowing Dr. Madrid, the Echavez brothers and the other accused in this
case. He said he only met Dr. Madrid at the CIS Office during the police line-up. He was
made to line-up four (4) times before Dr. Madrid finally identified him on the fourth
time.
Lucero also claimed he signed the extrajudicial confession under duress. He denied
engaging the services of Atty, Peralta. He likewise confirmed that Atty. Peralta was not
present during his actual custodial interrogation.
After trial, the court a quo acquitted the Echavez brothers for insufficient evidence. The
trial court, however, convicted accused Lucero GUILTY as principal by direct
participation of Robbery with Homicide and sentenced to suffer an imprisonment term
of RECLUSION PERPETUA.
Issue:
Whether or Not the lower court erred in convicting accused-appellant.
Held:
Appellant's conviction cannot be based on his extrajudicial confession.
Constitution requires that a person under investigation for the commission of a crime
should be provided with counsel. The Court have constitutionalized the right to counsel
because of hostility against the use of duress and other undue influence in extracting
confessions from a suspect. Force and fraud tarnish confessions can render them
inadmissible.
The records show that Atty. Peralta, who was not the counsel of choice of appellant.
Atty. Peralta himself admitted he received no reaction from appellant although his
impression was that appellant understood him. More so, it was during his absence that
appellant gave an uncounselled confession.

Page 220

Constitution requires the right to counsel, it did not mean any kind of counsel but
effective and vigilant counsel. The circumstances clearly demonstrate that appellant
received no effective counseling from Atty. Peralta.
Whereof, Decision convicting appellant Alejandro Lucero y Cortel is hereby reversed.

PEOPLE VS. AGUSTIN


[240 SCRA 541; G.R. NO. 110290; 25 JAN 1995]
Facts:
Dr. Bayquen, a dentist, together with his son, Anthony; Anthony's girlfriend, Anna
Theresa; his daughter, Dominic; and Danny, a family friend, were on their way aboard
their Brasilia to the doctor's residence at Malvar Street, Baguio City. While they were
cruising along Malvar Street and nearing the Baptist church, a man came out from the
right side of a car parked about two meters to the church. The man approached the
Brasilia, aimed his armalite rifle through its window, and fired at the passengers. The
Brasilia swerved and hit a fence. The gunman immediately returned to the parked car
which then sped away. All those in the car were hit and Dr. Bayquen and Anna Theresa
died on the spot. Dominic was bale to get out of the Brasilia to run to the Alabanza
store where she telephoned her mother. Later, she and her mother brought her father
and Anthony to the hospital. Danny went home and was then brought to the Hospital
for treatment.
Accused Quiao, an alleged former military agent who had been picked up by the police
authorities, confessed during the investigation conducted by Baguio City Fiscal Erdolfo
Balajadia in his office that he was the triggerman. He implicated Abenoja, Jr., who
engaged him to kill Dr. Bayquen for a fee, Cartel, who provided the armalite, and a
certain "Jimmy." During the investigation, Wilfredo Quiao was assisted by Atty.
Reynaldo Cajucom. Stenographic notes of the proceedings during the investigation as
transcribed with the sworn statement of Quiao was signed, with the assistance of Atty.
Cajucom, and swore to before City Fiscal Balajadia. The following day, Agustin was
apprehended, and was investigated and was afforded the privileges like that of Quijano.
Agustins defense interpose that he was forced to admit involvement at gunpoint in the
Kennon Road. He further declared that although he was given a lawyer, Cajucom (a law
partner of the private prosecutor), he nevertheless, asked for his uncle Atty. Oliver
Tabin, and that Atty. Cajucom interviewed him from only two minutes in English and
Tagalog but not in Ilocano, the dialect he understands. The promise that he would be
discharged as a witness did not push through since Quijano escaped. However the RTC
convicted him, since conspiracy was established. Hence the appeal.
Issue:

Page 221

Whether or Not accused-appellants extrajudicial statements admissible as evidence.


Held:
No. Extrajudicial statement is not extrajudicial confession. In a confession, there is an
acknowledgment of guilt of the accused, while an admission is a statement direct or
implied of facts pertinent to the issue. The rule on inadmissibility, however expressly
includes admissions, not just confessions.The extrajudicial admission of the appellant,
contained in twenty-two pages appear to be signed by him and Atty. Cajucom but for
reasons not explained in the records, the transcript of the notes which consists of
twelve pages was not signed by the appellant. Since the court cannot even read or
decipher the stenographic notes it cannot be expected that appellant, who is a farmer
and who reached only the fourth grade, to read or decipher its contents. The appellant,
therefore was deprived of his rights under Section 12(1), Article III of the Constitution.
Firstly, he was not fully and properly informed of his rights. The appellant was not
explicitly told of his right to have a competent and independent counsel of his choice,
specifically asked if he had in mind any such counsel and, if so, whether he could afford
to hire his services, and, if he could not, whether he would agree to be assisted by one
to be provided for him. He was not categorically informed that he could waive his rights
to remain silent and to counsel and that this waiver must be in writing and in the
presence of his counsel. He had, in fact, waived his right to remain silent by agreeing to
be investigated. Yet, no written waiver of such right appears in the transcript and no
other independent evidence was offered to prove its existence. In short, after the
appellant said that he wanted to be assisted by counsel, the City fiscal, through
suggestive language, immediately informed him that Atty. Cajucom was ready to assist
him. Moreso said counsel is not independent since he is an associate of the private
prosecutor.

PEOPLE VS. BOLANOS


[211 SCRA 262; G.R. NO. 101808; 3 JUL 1992]
Facts:
Oscar Pagdalian was murdered in Marble Supply, Balagtas Bulacan. According to Pat.
Rolando Alcantara and Francisco Dayao, deceased was with two companions on the
previous night, one of whom the accused who had a drinking spree with the deceased.
When they apprehended the accused they found the firearm of the deceased on the
chair where the accused was allegedly seated. They boarded accused along with
Magtibay, other accused on the police vehicle and brought them to the police station.
While in the vehicle Bolanos admitted that he killed the deceased. RTC convicted him
hence the appeal.
Issue:
Page 222

Whether or Not accused-appellant deprived of his constitutional right to counsel.


Held:
Yes. Being already under custodial investigation while on board the police patrol jeep on
the way to the Police Station where formal investigation may have been conducted,
appellant should have been informed of his Constitutional rights under Article III,
Section 12 of the 1987 Constitution, more particularly par. 1 and par. 3.

PEOPLE VS. MACAM


[238 SCRA 306; G.R. NOS. 91011-12; 24 NOV 1994]
Facts:
Prosecutions version:
On Aug 18,1987, Eduardo Macam, Antonio Cedro, Eugenio Cawilan Jr., Danilo Roque and
Ernesto Roque went to the house of Benito Macam (uncle of Eduardo Macam) located at
43 Ferma Road QC. Upon the arrival of the accused, Benito invited the former to have
lunch. Benito asked his maid Salvacion Enrera to call the companions of Eduardo who
were waiting in a tricycle outside the house. A. Cedro, E. Cawilan and D. Roque entered
the house while E. Roque remained in the tricycle. After all the accused had taken their
lunch, Eduardo Macam grabbed the clutch bag of Benito Macam and pulled out his
uncles gun then declared a hold-up. They tied up the wife (Leticia Macam), children,
maid (Salvacion) and Nilo Alcantara and brought them to the room upstairs. After a
while Leticia was brought to the bathroom and after she screamed she was stabbed
and killed by A. Cedro. Benito, Nilo and Salvacion was also stabbed but survived. The
total value of the items taken was P536, 700.00.
Defenses version:
Danilo Roque stated that he being a tricycle driver drove the 4 accused to Benitos
house for a fee of P50.00. Instead of paying him, he was given a calling card by
Eduardo Macam so that he can be paid the following day. Upon arriving, he went with
the accused inside the house to have lunch. Thereafter he washed the dishes and
swept the floor. When Eugenio Cawilan pulled a gun and announced the hold-up, he
was asked to gather some things and which he abided out of fear. While putting the
said thins inside the car of Benito (victim) he heard the accused saying kailangan
patayin ang mga taong yan dahil kilala ako ng mga yan. Upon hearing such phrase he
escaped and went home using his tricycle. He also testified that his brother Ernesto
Roque has just arrived from the province and in no way can be involved in the case at
bar. On the following day, together with his brother, they went to the factory of the
Page 223

Zesto Juice (owned by the father of Eduardo Macam) for him to get his payment (50.00)
. He and his brother was suddenly apprehended by the security guards and brought to
the police headquarters in Q.C. They were also forced to admit certain things.
After which, he together with all the accused, in handcuffs and bore contusions on their
faces caused by blows inflicted in their faces during investigation, was brought to the
QC General Hospital before each surviving victims and made to line-up for
identification. Eugenio Cawilan was also charged with Anti-fencing Law but was
acquitted in the said case.
Issue:
Whether or Not their right to counsel has been violated. WON the arrest was valid. WON
the evidence from the line-up is admissible.
Held:
It is appropriate to extend the counsel guarantee to critical stages of prosecution even
before trial. A police line-up is considered a critical stage of the proceedings. Any
identification of an uncounseled accused made in a police line-up is inadmissible.
HOWEVER, the prosecution did not present evidence regarding appellants identification
at the line-up. The witnesses identified the accused again in open court. Also, accused
did not object to the in-court identification as being tainted by illegal line-up.
The arrest of the appellants was without a warrant. HOWEVER, they are estopped from
questioning the legality of such arrest because they have not moved to quash the said
information and therefore voluntarily submitted themselves to the jurisdiction of the
trial court by entering a plea of not guilty and participating in trial.
The court believed the version of the prosecution. Ernesto Roque, while remaining
outside the house served as a looked out.
Wherefore, decision of lower court is Affirmed. Danilo Roque and Ernesto Roque is guilty
of the crime of robbery with homicide as co-conspirators of the other accused to suffer
reclusion perpetua.
Things taken: 2 toygun, airgun riffle, CO2 refiller, TV, betamax tapes, betamax
rewinder, Samsonite attache case, typewriter, chessboard, TOYOTA Crown Car Plate No.
CAS-997, assorted jewelry. .22 gun and money.

PEOPLE VS. DY
Page 224

[158 SCRA 111; G.R. 74517; 23 FEB 1988]


Facts:
Pat. Padilla reported along with Benny Dy, with caliber .38 as suspect to the shooting
incident at "Benny's Bar," at Sitio Angol, Manoc-Manoc Malay, Aklan (Boracay) situated
on the Island which caused the death of Christian Langel Philippe, tourist, 24 years old
and a Swiss nationale. He was charged with the Murder With the Use of Unlicensed
firearms. Appellant alleges that he carried the victim to the shore to be brought to the
hospital to save the latter, and who facilitated the surrender to Pat. Padilla a gun which
his helper found the following morning while cleaning the bar. Accused posted bail
which was granted. The accused denied having made any oral confession alleging that
he went to Pat. Padilla not to report the incident but to state that a boy helper in the
bar had found a gun on the sand floor while cleaning and that Pat. Padilla picked up the
gun from the bar at his request. The Accused argues that even if he did make such a
confession, the same would be inadmissible in evidence. He was found guilty in the
RTC. Hence the appeal.
Issue:
Whether or Not the lower court correct in saying that the constitutional procedure on
custodial interrogation is not applicable in the instant case.
Held:
YES. Appellant's assertion that the gun he had surrendered was merely found by a boy
helper while cleaning the bar deserves no credence for, if it were so, it would have
been absurd for him to have placed himself under police custody in the early morning
after the incident. Sworn Complaint for "Murder with Use of Unlicensed Firearm" signed
by the Chief of Police also attests to Appellant's oral confession. That Complaint forms
part of the record of the proceedings before the Municipal Circuit Trial Court of
Buruanga, Aklan, and is prima facie evidence of the facts therein stated. Appellant's
voluntary surrender implies no violation as "no warrant of arrest is issued for the
apprehension of the accused for the reason that he is already under police custody
before the filing of the complaint." What was told by the Accused to Pat, Padilla was a
spontaneous statement not elicited through questioning, but given in ordinary manner.
No written confession was sought to be presented in evidence as a result of formal
custodial investigation.

NAVALLO VS. SANDIGANBAYAN


[234 SCRA 177; G.R. NO. 97214; 18 JUL 1994]
Page 225

Facts:
Accused was the Collecting and Disbursing Officer of the Numancia National Vocational
School, which school is also located at del Carmen, Surigao del Norte. His duties
included the collection of tuition fees, preparation of vouchers for salaries of teachers
and employees, and remittance of collections exceeding P500.00 to the National
Treasury. An information for malversation of public funds was filed. A warrant of arrest
was issued, but accused-petitioner could not be found. on 10 December 1978,
Presidential Decree No. 1606 took effect creating the Sandiganbayan and conferring on
it original and exclusive jurisdiction over crimes committed by public officers embraced
in Title VII of the Revised Penal Code. On 15 November 1984, Navallo was finally
arrested. He was released on provisional liberty upon the approval of his property bail
bond. When arraigned by the RTC on 18 July 1985, he pleaded not guilty. Upon motion
of the prosecution, the RTC transferred the case and transmitted its records to the
Sandiganbayan. Special Prosecutor Luz L. Quiones-Marcos opined that since Navallo
had already been arraigned before the case was transferred to the Sandiganbayan, the
RTC should continue taking cognizance of the case. The matter was referred to the
Office of the Ombudsman which held otherwise. The information was then docketed
with the Sandiganbayan. A new order for Navallo's arrest was issued by the
Sandiganbayan. The warrant was returned with a certification by the RTC Clerk of Court
that the accused had posted a bail bond. Navallo filed a motion to quash, contending
(1) that the Sandiganbayan had no jurisdiction over the offense and the person of the
accused and (2) that since the accused had already been arraigned by the RTC, the
attempt to prosecute him before the Sandiganbayan would constitute double jeopardy.
However this was denied and trial ensued and he was found guilty.
Issue:
Whether or Not the constitutional right against double jeopardy and in custodial
investigations in favor of the accused violated.
Held:
No. Double jeopardy requires the existence of the following requisites:
(1) The previous complaint or information or other formal charge is sufficient in form
and substance to sustain a conviction;
(2) The court has jurisdiction to try the case;
(3) The accused has been arraigned and has pleaded to the charge; and
(4) The accused is convicted or acquitted or the case is dismissed without his
express consent.

Page 226

The RTC was devoid of jurisdiction when it conducted an arraignment of the accused
which by then had already been conferred on the Sandiganbayan. Moreover, neither did
the case there terminate with conviction or acquittal nor was it dismissed.
No. Appellant is not in custodial investigation. A person under a normal audit
examination is not under custodial investigation. An audit examiner himself can hardly
be deemed to be the law enforcement officer contemplated in the above rule. In any
case, the allegation of his having been "pressured" to sign the Examination Report
prepared by Dulguime (examined cash, as ordered by Espino, the provincial auditor)
appears to be belied by his own testimony.

PEOPLE VS. ALICANDO


[251 SCRA 293; G.R. NO. 117487; 2 DEC 1995]
Facts:
Appellant was charged with the crime of rape with homicide of Khazie Mae Penecilla, a
minor, four years of age, choking her with his right hand. The incident happened after
appellant drank liquor. A neighbor, Leopoldo Santiago found the victims body and the
parents and police were informed. Appellant was living in his uncle's house some five
arm's length from Penecilla's house. Appellant was arrested and interrogated by PO3
Danilo Tan. He verbally confessed his guilt without the assistance of counsel. On the
basis of his uncounselled verbal confession and follow up interrogations, the police
came to know and recovered from appellant's house, Khazie Mae's green slippers, a
pair of gold earrings, a buri mat, a stained pillow and a stained T-shirt all of which were
presented as evidence for the prosecution. He was arraigned with the assistance of
Atty. Rogelio Antiquiera of the PAO. Appellant pleaded guilty. The RTC convicted him.
Hence an automatic review for the imposition of death penalty.
Issue:
Whether or Not the death penalty proper.
Held:
No. The records do not reveal that the Information against the appellant was read in the
language or dialect known to him. The Information against the appellant is written in
the English language. It is unknown whether the appellant knows the English language.
Neither is it known what dialect is understood by the appellant. Nor is there any
showing that the Information couched in English was translated to the appellant in his
own dialect before his plea of guilt. The RTC violated section 1(a) of Rule 116, the rule
implementing the constitutional right of the appellant to be informed of the nature and
Page 227

cause of the accusation against him. It also denied appellant his constitutional right to
due process of law. It is urged that we must presume that the arraignment of the
appellant was regularly conducted. When life is at stake, we cannot lean on this
rebuttable presumption. There could be no presumption. The court must be sure.
The trial court violated section 3 of Rule 116 when it accepted the plea of guilt of the
appellant. Said section requires that the court shall conduct a searching inquiry the
voluntariness and full comprehension of the consequences of his plea and require the
prosecution to prove his guilt and the precise degree of culpability. The accused may
also present evidence in his behalf. The trial court simply inquired if appellant had
physical marks of maltreatment. It did not ask the appellant when he was arrested, who
arrested him, how and where he was interrogated, whether he was medically examined
before and after his interrogation, etc. It limited its efforts trying to discover late body
marks of maltreatment as if involuntariness is caused by physical abuse alone.
Further, there are physical evidence to prove Khazie was raped. These consists of a
pillow with bloodstains in its center 14 and the T-shirt 15 of the accused colored white
with bloodstains on its bottom. These physical evidence are evidence of the highest
order. They strongly corroborate the testimony of Luisa Rebada that the victim was
raped.These are inadmissible evidence for they were gathered by PO3 Danilo Tan of the
Iloilo City PNP as a result of custodial interrogation where appellant verbally confessed
to the crime without the benefit of counsel.

PEOPLE VS. DE GUZMAN


[224 SCRA 93; G.R. NOS. 98321-24; 30 JUN 1993]
Facts:
All the accused were charged before the Regional Trial Court of Cebu with three counts
of murder and one count of frustrated murder in four Informations. The victim Jose
Bantug was found with gunshots in the head, body, and skull. The other three
informations charged them with the murder of Francisco Carteciano y Sorilla and
Antonio S. Carteciano, and the frustrated murder of Lorna V. Carteciano. The other 8
accused were acquitted on the ground of reasonable doubt, while Victor Nuez was
found guilty. The facts shown by evidence are: One morning, Major Antonio Carteciano
was driving his private jeep Camp General Arcadio Maxilom in Lahug, Cebu City where
he was stationed as medical officer of the PC/INP Provincial Command. In the front seat
with him is his wife Lorna, and at the backseat are his mother in law, son, brother
Francisco, neighbor Bantug, and Bantugs son. Near the intersection, gunshots were
heard from the left side of the street. Major Carteciano took his .45 cal pistol and fired.
However, gunshots were fired in succession, and Major Carteciano, his brother
Francisco, Jose Bantug, and his wife Lorna were hit. When the jeep stopped, several
gunmen approached them. Nuez demanded Lorna to give Nuez her husbands pistol.
Lorna asked to take her valuables instead. Then, Nuez shot Major Cartecianos head
point blank. Then the gunmen hijacked another jeep and took off. Lorna, her mother
Page 228

Juanita Ricaplaza, and her son Reiser Carteciano positively identified the accused. Lorna
identified Nuez as the one who shot her husband. Nuez claimed that his arrest was
illegal and that he was deprived of his right to counsel when he was subjected to a
paraffin test without the assistance of counsel.
Issue:
Whether or not the accused Nuezs constitutional right was violated

Held:
No. Nuez pleaded not guilty at the arraignment. Therefore, he is estopped from
questioning the validity of his arrest. Furthermore, the illegal arrest of an accused is not
sufficient cause for setting aside a valid judgment rendered upon a sufficient complaint
after trial free from error. The witnesses also positively identified the accused, so he
cannot question the credibility of the witnesses. Regarding his right to counsel, the
Supreme Court held that-- the right to counsel attaches only upon the start of an
investigation, that is, when the investigating officer starts to ask questions to elicit
information and/or confessions or admissions from the accused. At such point or stage,
the person being interrogated must be assisted by counsel to avoid the pernicious
practice of extorting false or coerced admissions or confessions from the lips of the
person undergoing interrogation. In the case at bar, when accused was subjected to a
paraffin test, he was not then under custodial investigation. Accused-appellant also
argued that since his co-accused were acquitted, then their acquittal negates
conspiracy among them, and he should not be convicted with the charges filed.
However, the Court held that conspiracy was still proven by the evidence, and the other
co-accused were acquitted only because there was reasonable doubt. Therefore,
accused-appellant is still convicted of the four charges against him.
We, therefore, find that the conviction of accused-appellant for the crimes charged has
been established beyond reasonable doubt and the penalty imposed is in accordance
with law. However, the civil indemnity imposed by the trial court should be increased to
P50,000 in conformity with our recent rulings on the matter.
WHEREFORE, except for the modification that the civil indemnity to be paid by accusedappellant Victor Nuez, Jr. to the heirs of each victim who died is hereby increased to
P50,000, the appealed decision is hereby affirmed in all other respects, with costs
against accused-appellant

PEOPLE VS. JUDGE DONATO


[198 SCRA 130; G.R. NO.79269; 5 JUN 1991]

Page 229

Facts:
Private respondent and his co-accused were charged of rebellion on October 2, 1986 for
acts committed before and after February 1986. Private respondent filed with a Motion
to Quash alleging that: (a) the facts alleged do not constitute an offense; (b) the Court
has no jurisdiction over the offense charged; (c) the Court has no jurisdiction over the
persons of the defendants; and (d) the criminal action or liability has been
extinguished. This was denied. May 9, 1987 Respondent filed a petition for bail, which
was opposed that the respondent is not entitled to bail anymore since rebellion became
a capital offense under PD 1996, 942 and 1834 amending ART. 135 of RPC. On 5 June
1987 the President issued Executive Order No. 187 repealing, among others, P.D. Nos.
1996, 942 and 1834 and restoring to full force and effect Article 135 of the Revised
Penal Code as it existed before the amendatory decrees. Judge Donato now granted the
bail, which was fixed at P30,000.00 and imposed a condition that he shall report to the
court once every two months within the first ten days of every period thereof. Petitioner
filed a supplemental motion for reconsideration indirectly asking the court to deny bail
to and to allow it to present evidence in support thereof considering the "inevitable
probability that the accused will not comply with this main condition of his bail. It was
contended that:
1. The accused has evaded the authorities for thirteen years and was an escapee
from detention when arrested; (Chairman of CPP-NPA)
2. He was not arrested at his residence as he had no known address;
3. He was using the false name "Manuel Mercado Castro" at the time of his arrest
and presented a Driver's License to substantiate his false identity;
4. The address he gave "Panamitan, Kawit, Cavite," turned out to be also a false
address;
5. He and his companions were on board a private vehicle with a declared owner
whose identity and address were also found to be false;
6. Pursuant to Ministry Order No. 1-A dated 11 January 1982 , a reward of
P250,000.00 was offered and paid for his arrest.
This however was denied. Hence the appeal.
Issue:
Whether or Not the private respondent has the right to bail.
Held:
Yes. Bail in the instant case is a matter of right. It is absolute since the crime is not a
capital offense, therefore prosecution has no right to present evidence. It is only when
Page 230

it is a capital offense that the right becomes discretionary. However it was wrong for the
Judge to change the amount of bail from 30K to 50K without hearing the prosecution.
Republic Act No. 6968 approved on 24 October 1990, providing a penalty of reclusion
perpetua to the crime of rebellion, is not applicable to the accused as it is not favorable
to him.
Accused validly waived his right to bail in another case(petition for habeas corpus).
Agreements were made therein: accused to remain under custody, whereas his codetainees Josefina Cruz and Jose Milo Concepcion will be released immediately, with a
condition that they will submit themselves in the jurisdiction of the court. Said petition
for HC was dismissed. Bail is the security given for the release of a person in custody of
the law. Ergo, there was a waiver. We hereby rule that the right to bail is another of the
constitutional rights which can be waived. It is a right which is personal to the accused
and whose waiver would not be contrary to law, public order, public policy, morals, or
good customs, or prejudicial to a third person with a right recognized by law.

CARPIO VS. MAGLALANG


[196 SCRA 41; G.R. NO. 78162; 19 APR 1991]
Facts:
On January 8, 1987, information for the murder of Mayor Jose Payumo of Dinalupihan
Bataan was filed against Escao and ten other unindentified persons by the provincial
fiscal in the RTC of Bataan at Balanga. Four days later, the Acting Executive Judge of
said court issued an order of arrest against Escao recommending no bail for his
provisional liberty. Pat. Cesar Diego who acted on the warrant returned to the court with
a certification issued by NBI agent Gonzales, stating therein that accused was still
under investigation.
Through counsel Rolando T. Cainoy, Escao filed in court an urgent ex-parte motion for
his commitment at the provincial jail of Bataan on the ground that he wanted to be
where his family and counsel could have easy access to him. He alleged therein that his
detention at the NBI headquarters in Manila was irregular and in defiance of the
warrant of arrest issued by the court. This was granted.
A motion for reconsideration was filed by Director Carpio stating that the NBI needed
physical custody of Escao for the identification of the other accused in the case who
were still the objects of a manhunt by NBI agents; that in view of the finding of NBI
agents that the other accused and suspects in the case were subversive elements or
members of the New People's Army, it was for the best interest of Escao that he be
detained at the NBI lock-up cell where security measures were adequate; and that the
NBI would produce the person of Escao before the court whenever required and every
Page 231

time that there would be a hearing on the case. However another motion was executed
by Escao stating that he now wants to be detained in the NBI, alleging that he did not
authorize his counsel to execute the first motion. Also, Escao's counsel Rolando T.
Cainoy filed an application for bail stating that Escao was arrested by NBI agents on
December 7, 1986 without a warrant having been presented to him and that since then
he had been detained in the lock-up cell of the NBI; that said agents, also without a
warrant, searched his house when he was arrested; that he was subjected to inhuman
torture and forced to admit participation in the killing of Mayor Payumo and to implicate
other persons, and that during the custodial investigation, he was not represented by
counsel. In opposing said application, the public prosecutor averred that the accused
was charged with a capital offense for which no bail may be availed of, that the reasons
advanced in said application would be overcome by strong and sufficient evidence; and
that during the custodial investigation, he was represented by counsel. The court
granted the application for bail fixing the same at P30,000, having found no sufficient
evidence against accused. Director Carpio was ordered to justify his actions and so as
not to be considered in contempt.
Issue:
Whether or Not the order granting right to bail was proper.
Held:
No. The order granting bail had been rendered moot not only by the fact that he had
been released from NBI custody, but also because Escao jumped bail and did not
appear on the date set for his arraignment. Notwithstanding, the Court resolved the
issue of the legality of the order granting bail to Escao. Although the right to bail is
principally for the benefit of the accused, in the judicial determination of the availability
of said right, the prosecution should be afforded procedural due process. Thus, in the
summary proceeding on a motion praying for admission to bail, the prosecution should
be given the opportunity to present evidence and, thereafter, the court should spell out
at least a resume of the evidence on which its order granting or denying bail is based.
Otherwise, the order is defective and voidable. In the case at bar the RTC erred in not
summarizing the factual basis of its order granting bail, the court merely stated the
number of prosecution witnesses but not their respective testimonies, and concluded
that the evidence presented by the prosecution was not "sufficiently strong" to deny
bail to Escao.
The facts, however, that Mayor Payumo was killed on August 20, 1986 when the 1973
Constitution allowing the death penalty was still in force and that the application for
bail was made on March 5, 1987 during the effectivity of the 1987 Constitution which
abolished the death penalty, should not have gotten in the way of resolving the
application for bail in accordance with the Constitution and procedural rules. Section
13, Article III of the Constitution explicitly provides that "(a)ll persons, except those
charged with offenses punishable by reclusion perpetua when evidence of guilt is
strong, shall before conviction, be bailable by sufficient sureties, or be released on
recognizance as may be provided by law." As the phrase "capital offenses" has been
Page 232

replaced by the phrase "offenses punishable by reclusion perpetua, 25 crimes


punishable by reclusion perpetua instead of those punishable by the death penalty,
when evidence of guilt is strong, are the exceptions to the rule that the right to bail
should be made available to all accused. As the court itself acknowledged in its order of
April 2, 1987 that "capital punishment" in Section 4, Rule 114 has been amended to
reclusion perpetua, the court should have proceeded accordingly: i.e., resolved the
application for bail pursuant to Section 13, Article III of the Constitution. It did not have
to invoke the abolition of the death penalty and the lack of legislative enactment
restoring it in justifying the grant of bail. All it had to do was to determine whether
evidence of guilt is strong in the light of the provision of Section 13, Article III.
The RTC has the discretion in the consideration of the strength of the evidence at hand.
However, in the exercise of said discretion, the court is controlled by the following: first,
the applicable provisions of the Constitution and the statutes; second, by the rules
which this Court may promulgate; and third, by those principles of equity and justice
that are deemed to be part of the laws of the land. 27 The lower court not only failed to
properly apply the pertinent provisions of the Constitution and the Rules but it also
disregarded equity and justice by its failure to take into account the factual milieu
surrounding the detention of Escao

PEOPLE VS. FORTES


[223 SCRA 619; G.R. NO. 90643; 25 JUN 1993]
Facts:
Agripino Gine of Barangay Naburacan, Municipality of Matnog, Province of Sorsogon,
accompanied his 13-year old daughter, Merelyn, to the police station of the said
municipality to report a rape committed against the latter by the accused. Following
this, the accused was apprehended and charged. A bond of P25000 was granted for
accuseds provisional release. The MCTC found him guilty. An appeal to RTC was filed,
the request for the fixing of bond was denied. Now accused assails denial of bail on the
ground that the same amounted to an undue denial of his constitutional right to bail.
Issue:
Whether or Not the accuseds right to bail violated.
Held:
No. It is clear from Section 13, Article III of the 1987 Constitution and Section 3, Rule
114 of the Revised Rules of Court, as amended, that before conviction bail is either a
matter of right or of discretion. It is a matter of right when the offense charged is
Page 233

punishable by any penalty lower than reclusion perpetua. To that extent the right is
absolute. If the offense charged is punishable by reclusion perpetua bail becomes a
matter of discretion. It shall be denied if the evidence of guilt is strong. The court's
discretion is limited to determining whether or not evidence of guilt is strong. But once
it is determined that the evidence of guilt is not strong, bail also becomes a matter of
right. If an accused who is charged with a crime punishable by reclusion perpetua is
convicted by the trial court and sentenced to suffer such a penalty, bail is neither a
matter of right on the part of the accused nor of discretion on the part of the court.

COMMENDADOR VS. DE VILLA


[200 SCRA 80; G.R. NO. 93177; 2 AUG 1991]
Facts:
The petitioners in G.R. Nos. 93177 and 96948 who are officers of the AFP were directed
to appear in person before the Pre-Trial Investigating Officers for the alleged
participation the failed coup on December 1 to 9, 1989. Petitioners now claim that there
was no pre-trial investigation of the charges as mandated by Article of War 71. A
motion for dismissal was denied. Now, their motion for reconsideration. Alleging denial
of due process.
In G.R. No. 95020, Ltc Jacinto Ligot applied for bail on June 5, 1990, but the application
was denied by GCM No.14. He filed with the RTC a petition for certiorari and mandamus
with prayer for provisional liberty and a writ of preliminary injunction. Judge of GCM
then granted the provisional liberty. However he was not released immediately. The RTC
now declared that even military men facing court martial proceedings can avail the
right to bail.
The private respondents in G.R. No. 97454 filed with SC a petition for habeas corpus on
the ground that they were being detained in Camp Crame without charges. The petition
was referred to RTC. Finding after hearing that no formal charges had been filed against
the petitioners after more than a year after their arrest, the trial court ordered their
release.
Issue:
Whether or Not there was a denial of due process.
Whether or not there was a violation of the accused right to bail.
Held:
Page 234

NO denial of due process. Petitioners were given several opportunities to present their
side at the pre-trial investigation, first at the scheduled hearing of February 12, 1990,
and then again after the denial of their motion of February 21, 1990, when they were
given until March 7, 1990, to submit their counter-affidavits. On that date, they filed
instead a verbal motion for reconsideration which they were again asked to submit in
writing. They had been expressly warned in the subpoena that "failure to submit
counter-affidavits on the date specified shall be deemed a waiver of their right to
submit controverting evidence." Petitioners have a right to pre-emptory challenge.
(Right to challenge validity of members of G/SCM)
It is argued that since the private respondents are officers of the Armed Forces accused
of violations of the Articles of War, the respondent courts have no authority to order
their release and otherwise interfere with the court-martial proceedings. This is without
merit. * The Regional Trial Court has concurrent jurisdiction with the Court of Appeals
and the Supreme Court over petitions for certiorari, prohibition or mandamus against
inferior courts and other bodies and on petitions for habeas corpus and quo warranto.
The right to bail invoked by the private respondents has traditionally not been
recognized and is not available in the military, as an exception to the general rule
embodied in the Bill of Rights. The right to a speedy trial is given more emphasis in the
military where the right to bail does not exist.
On the contention that they had not been charged after more than one year from their
arrest, there was substantial compliance with the requirements of due process and the
right to a speedy trial. The AFP Special Investigating Committee was able to complete
the pre-charge investigation only after one year because hundreds of officers and
thousands of enlisted men were involved in the failed coup.
Accordingly, in G.R. No. 93177, the petition is dismissed for lack of merit. In G.R. No.
96948, the petition is granted, and the respondents are directed to allow the petitioners
to exercise the right of peremptory challenge under article 18 of the articles of war. In
G.R. Nos. 95020 and 97454, the petitions are also granted, and the orders of the
respondent courts for the release of the private respondents are hereby reversed and
set aside. No costs.

MANOTOC VS. COURT OF APPEALS


[142 SCRA 149; G.R. NO. L-62100; 30 MAY 1986]
Facts:
Petitioner was charged with estafa. He posted bail. Petitioner filed before each of the
trial courts a motion entitled, "motion for permission to leave the country," stating as
Page 235

ground therefor his desire to go to the United States, "relative to his business
transactions and opportunities." The prosecution opposed said motion and after due
hearing, both trial judges denied the same. Petitioner thus filed a petition for certiorari
and mandamus before the then Court of Appeals seeking to annul the orders dated
March 9 and 26, 1982, of Judges Camilon and Pronove, respectively, as well as the
communication-request of the Securities and Exchange Commission, denying his leave
to travel abroad. He likewise prayed for the issuance of the appropriate writ
commanding the Immigration Commissioner and the Chief of the Aviation Security
Command (AVSECOM) to clear him for departure. The Court of Appeals denied the
petition.
Petitioner contends that having been admitted to bail as a matter of right, neither the
courts which granted him bail nor the Securities and Exchange Commission which has
no jurisdiction over his liberty could prevent him from exercising his constitutional right
to travel.
Issue:
Whether or Not the right to bail a matter of right.
Held:
The right to bail is a matter of right if the charge is not a capital offense or punishable
by reclusion perpetua to death.
A court has the power to prohibit a person admitted to bail from leaving the Philippines.
This is a necessary consequence of the nature and function of a bail bond.
The condition imposed upon petitioner to make himself available at all times whenever
the court requires his presence operates as a valid restriction on his right to travel.
Indeed, if the accused were allowed to leave the Philippines without sufficient reason,
he may be placed beyond the reach of the courts.
Petitioner has not shown the necessity for his travel abroad. There is no indication that
the business transactions cannot be undertaken by any other person in his behalf.

CALLANTA VS. VILLANUEVA


[77 SCRA 377; G.R. NOS. 24646 & L-24674; 20 JUN 1977]
Facts:
Two complaints for grave oral defamation were filed against Faustina Callanta. The City
Judge of Dagupan City, Felipe Villanueva, denied the motions to quash the complaints.
Thus, petitioner Callanta brought the suits for certiorari in the Supreme Court.
Petitioner questions the validity of the issuance of warrant of arrest by respondent,
arguing that the City Fiscal should have conducted the preliminary investigation.
Page 236

According to petitioners counsel, there was jurisdictional infirmity. After the issuance of
the warrants of arrest and the bail fixed at P600, petitioner posted the bail bond, thus
obtaining her provisional liberty. The City Fiscal in this case did not disagree with the
judges investigation, and agreed with the complaints filed.
Issue:
Whether or Not petitioners contentions are to be given merit.
Held:
Based on many precedent cases of the Supreme Court, where the accused has filed
bail and waived the preliminary investigation proper, he has waived whatever defect, if
any, in the preliminary examination conducted prior to the issuance of the warrant of
arrest. In the case at bar, it is futile for the petitioner to question the validity of the
issuance of the warrant of arrest, because she posted the bail bond. Petitioner also
erred in arguing that only the City Fiscal can conduct a preliminary investigation.
According to the Charter of the City of Dagupan, the City Court of Dagupan City may
also conduct preliminary investigation for any offense, without regard to the limits of
punishment, and may release, or commit and bind over any person charged with such
offense to secure his appearance before the proper court. Petition for certiorari is
denied. Restraining order issued by the Court is lifted and set aside.

TATAD VS. SANDIGANBAYAN


[159 SCRA 70; G.R. NOS. L-72335-39; 21 MAR 1988]
Facts:
The complainant, Antonio de los Reyes, originally filed what he termed "a report" with
the Legal Panel of the Presidential Security Command (PSC) on October 1974,
containing charges of alleged violations of Rep. Act No. 3019 against then Secretary of
Public Information Francisco S. Tatad. The "report" was made to "sleep" in the office of
the PSC until the end of 1979 when it became widely known that Secretary (then
Minister) Tatad had a falling out with President Marcos and had resigned from the
Cabinet. On December 12, 1979, the 1974 complaint was resurrected in the form of a
formal complaint filed with the Tanodbayan. The Tanodbayan acted on the complaint on
April 1, 1980 which was around two months after petitioner Tatad's resignation was
accepted by Pres. Marcos by referring the complaint to the CIS, Presidential Security
Command, for investigation and report. On June 16, 1980, the CIS report was submitted
to the Tanodbayan, recommending the filing of charges for graft and corrupt practices
against former Minister Tatad and Antonio L. Cantero. By October 25, 1982, all affidavits
and counter-affidavits were in the case was already for disposition by the Tanodbayan.
However, it was only on June 5, 1985 that a resolution was approved by the
Page 237

Tanodbayan. Five criminal informations were filed with the Sandiganbayan on June 12,
1985, all against petitioner Tatad alone. (1) Section 3, paragraph (e) of RA. 3019 for
giving D' Group, a private corporation controlled by his brother-in-law, unwarranted
benefits, advantage or preference in the discharge of his official functions; (2) Violation
of Section 3, paragraph (b) for receiving a check of P125,000.00 from Roberto Vallar,
President/General Manager of Amity Trading Corporation as consideration for the
release of a check of P588,000.00 to said corporation for printing services rendered for
the Constitutional Convention Referendum in 1973; (3) Violation of Section 7 on three
(3) counts for his failure to file his Statement of Assets and Liabilities for the calendar
years 1973, 1976 and 1978. A motion to quash the information was made alleging that
the prosecution deprived accused of due process of law and of the right to a speedy
disposition of the cases filed against him. It was denied hence the appeal.
Issue:
Whether or not petitioner was deprived of his rights as an accused.

Held:
YES. Due process (Procedural) and right to speedy disposition of trial were violated.
Firstly, the complaint came to life, as it were, only after petitioner Tatad had a falling
out with President Marcos. Secondly, departing from established procedures prescribed
by law for preliminary investigation, which require the submission of affidavits and
counter-affidavits by the complainant and the respondent and their witnesses, the
Tanodbayan referred the complaint to the Presidential Security Command for finding
investigation and report. The law (P.D. No. 911) prescribes a ten-day period for the
prosecutor to resolve a case under preliminary investigation by him from its
termination. While we agree with the respondent court that this period fixed by law is
merely "directory," yet, on the other hand, it can not be disregarded or ignored
completely, with absolute impunity. A delay of close to three (3) years can not be
deemed reasonable or justifiable in the light of the circumstance obtaining in the case
at bar.

GALMAN VS. SANDIGANBAYAN


[144 SCRA 43; G.R. NO.72670; 12 SEP 1986]
Facts:
Assassination of former Senator Benigno "Ninoy" Aquino, Jr. He was killed from his
plane that had just landed at the Manila International Airport. His brain was smashed by
Page 238

a bullet fired point-blank into the back of his head by an assassin. The military
investigators reported within a span of three hours that the man who shot Aquino
(whose identity was then supposed to be unknown and was revealed only days later as
Rolando Galman) was a communist-hired gunman, and that the military escorts gunned
him down in turn.
President was constrained to create a Fact Finding Board to investigate due to large
masses of people who joined in the ten-day period of national mourning yearning for
the truth, justice and freedom.
The fact is that both majority and minority reports were one in rejecting the military
version stating that "the evidence shows to the contrary that Rolando Galman had no
subversive affiliations. Only the soldiers in the staircase with Sen. Aquino could have
shot him; that Ninoy's assassination was the product of a military conspiracy, not a
communist plot. Only difference between the two reports is that the majority report
found all the twenty-six private respondents above-named in the title of the case
involved in the military conspiracy; " while the chairman's minority report would
exclude nineteen of them.
Then Pres. Marcos stated that evidence shows that Galman was the killer.
Petitioners pray for issuance of a TRO enjoining respondent court from rendering a
decision in the two criminal cases before it, the Court resolved by nine-to-two votes 11
to issue the restraining order prayed for. The Court also granted petitioners a five-day
period to file a reply to respondents' separate comments and respondent Tanodbayan a
three-day period to submit a copy of his 84-page memorandum for the prosecution.
But ten days later, the Court by the same nine-to-two-vote ratio in reverse, resolved to
dismiss the petition and to lift the TRO issued ten days earlier enjoining the
Sandiganbayan from rendering its decision. The same Court majority denied
petitioners' motion for a new 5-day period counted from receipt of respondent
Tanodbayan's memorandum for the prosecution (which apparently was not served on
them).
Thus, petitioners filed a motion for reconsideration, alleging that the dismissal did not
indicate the legal ground for such action and urging that the case be set for a full
hearing on the merits that the people are entitled to due process.
However, respondent Sandiganbayan issued its decision acquitting all the accused of
the crime charged, declaring them innocent and totally absolving them of any civil
liability. Respondents submitted that with the Sandiganbayan's verdict of acquittal, the
instant case had become moot and academic. Thereafter, same Court majority denied
petitioners' motion for reconsideration for lack of merit.

Page 239

Hence, petitioners filed their motion to admit their second motion for reconsideration
alleging that respondents committed serious irregularities constituting mistrial and
resulting in miscarriage of justice and gross violation of the constitutional rights of the
petitioners and the sovereign people of the Philippines to due process of law.
Issue:
Whether or not petitioner was deprived of his rights as an accused.
Whether or not there was a violation of the double jeopardy clause.
Held:
Petitioners' second motion for reconsideration is granted and ordering a re-trial of the
said cases which should be conducted with deliberate dispatch and with careful regard
for the requirements of due process.
Deputy Tanodbayan Manuel Herrera (made his expose 15 months later when former
Pres. was no longer around) affirmed the allegations in the second motion for
reconsideration that he revealed that the Sandiganbayan Justices and Tanodbayan
prosecutors were ordered by Marcos to whitewash the Aquino-Galman murder case.
Malacaang wanted dismissal to the extent that a prepared resolution was sent to the
Investigating Panel. Malacaang Conference planned a scenario of trial where the
former President ordered then that the resolution be revised by categorizing the
participation of each respondent; decided that the presiding justice, Justice Pamaran,
(First Division) would personally handle the trial. A conference was held in an inner
room of the Palace. Only the First Lady and Presidential Legal Assistant Justice Lazaro
were with the President. The conferees were told to take the back door in going to the
room where the meeting was held, presumably to escape notice by the visitors in the
reception hall waiting to see the President. During the conference, and after an
agreement was reached, Pres. Marcos told them 'Okay, mag moro-moro na lamang
kayo;' and that on their way out of the room Pres. Marcos expressed his thanks to the
group and uttered 'I know how to reciprocate'.
The Court then said that the then President (code-named Olympus) had stage-managed
in and from Malacaang Palace "a scripted and predetermined manner of handling and
disposing of the Aquino-Galman murder case;" and that "the prosecution in the AquinoGalman case and the Justices who tried and decided the same acted under the
compulsion of some pressure which proved to be beyond their capacity to resist. Also
predetermined the final outcome of the case" of total absolution of the twenty-six
respondents-accused of all criminal and civil liability. Pres. Marcos came up with a
public statement aired over television that Senator Aquino was killed not by his military
escorts, but by a communist hired gun. It was, therefore, not a source of wonder that
President Marcos would want the case disposed of in a manner consistent with his
announced theory thereof which, at the same time, would clear his name and his
Page 240

administration of any suspected guilty participation in the assassination. such a


procedure would be a better arrangement because, if the accused are charged in court
and subsequently acquitted, they may claim the benefit of the doctrine of double
jeopardy and thereby avoid another prosecution if some other witnesses shall appear
when President Marcos is no longer in office.
More so was there suppression of vital evidence and harassment of witnesses. The
disappearance of witnesses two weeks after Ninoy's assassination. According to J.
Herrera, "nobody was looking for these persons because they said Marcos was in
power. The assignment of the case to Presiding Justice Pamaran; no evidence at all that
the assignment was indeed by virtue of a regular raffle, except the uncorroborated
testimony of Justice Pamaran himself. The custody of the accused and their
confinement in a military camp, instead of in a civilian jail. The monitoring of
proceedings and developments from Malacaang and by Malacaang personnel. The
partiality of Sandiganbayan betrayed by its decision: That President Marcos had wanted
all of the twenty-six accused to be acquitted may not be denied. In rendering its
decision, the Sandiganbayan overdid itself in favoring the presidential directive. Its bias
and partiality in favor of the accused was clearly obvious. The evidence presented by
the prosecution was totally ignored and disregarded.
The record shows that the then President misused the overwhelming resources of the
government and his authoritarian powers to corrupt and make a mockery of the judicial
process in the Aquino-Galman murder cases. "This is the evil of one-man rule at its very
worst." Our Penal Code penalizes "any executive officer who shall address any order or
suggestion to any judicial authority with respect to any case or business coming within
the exclusive jurisdiction of the courts of justice."
Impartial court is the very essence of due process of law. This criminal collusion as to
the handling and treatment of the cases by public respondents at the secret
Malacaang conference (and revealed only after fifteen months by Justice Manuel
Herrera) completely disqualified respondent Sandiganbayan and voided ab initio its
verdict. The courts would have no reason to exist if they were allowed to be used as
mere tools of injustice, deception and duplicity to subvert and suppress the truth. More
so, in the case at bar where the people and the world are entitled to know the truth,
and the integrity of our judicial system is at stake.
There was no double jeopardy. Courts' Resolution of acquittal was a void judgment for
having been issued without jurisdiction. No double jeopardy attaches, therefore. A void
judgment is, in legal effect, no judgment at all. By it no rights are divested. It neither
binds nor bars anyone. All acts and all claims flowing out of it are void.
Motion to Disqualify/Inhibit should have been resolved ahead. In this case, petitioners'
motion for reconsideration of the abrupt dismissal of their petition and lifting of the TRO
enjoining the Sandiganbayan from rendering its decision had been taken cognizance of
by the Court which had required the respondents', including the Sandiganbayan's,
comments. Although no restraining order was issued anew, respondent Sandiganbayan
should not have precipitately issued its decision of total absolution of all the accused
Page 241

pending the final action of this Court. All of the acts of the respondent judge manifest
grave abuse of discretion on his part amounting to lack of jurisdiction which
substantively prejudiced the petitioner.
With the declaration of nullity of the proceedings, the cases must now be tried before
an impartial court with an unbiased prosecutor. Respondents accused must now face
trial for the crimes charged against them before an impartial court with an unbiased
prosecutor with all due process.
The function of the appointing authority with the mandate of the people, under our
system of government, is to fill the public posts. Justices and judges must ever realize
that they have no constituency, serve no majority nor minority but serve only the
public interest as they see it in accordance with their oath of office, guided only the
Constitution and their own conscience and honor.

PEOPLE VS. DRAMAYO


[42 SCRA 60; G.R. L-21325; 29 OCT 1971]
Facts:
Dramayo brought up the idea of killing Estelito Nogaliza so that he could not testify in
the robbery case where he is an accused. The idea was for Dramayo and Ecubin to
ambush Estelito, who was returning from Sapao. The others were to station themselves
nearby. Only Dramayo and Ecubin were convicted in the RTC for murder. Hence the
appeal
Issue:
Whether or not the accuseds criminal liability proved beyond reasonable doubt.
Held:
Yes. It is to be admitted that the starting point is the Presumption of innocence. So it
must be, according to the Constitution. That is a right safeguarded both appellants.
Accusation is not, according to the fundamental law, synonymous with guilt. It is
incumbent on the prosecution demonstrate that culpability lies. Appellants were not
even called upon then to offer evidence on their behalf. Their freedom is forfeit only if
the requisite quantum of proof necessary for conviction be in existence. Their guilt be
shown beyond reasonable doubt. What is required then is moral certainty. "By
reasonable doubt is meant that which of possibility may arise, but it is doubt
engendered by an investigation of the whole proof and an inability, after such
investigation, to let the mind rest easy upon the certainty of guilt. Absolute certain of
guilt is not demanded by the law to convict of any carnal charge but moral certainty is
Page 242

required, and this certainty is required as to every proposition of proof regular to


constitute the offense."
The judgment of conviction should not have occasioned any surprise on the part of the
two appellants, as from the evidence deserving of the fullest credence, their guilt had
been more than amply demonstrated. The presumption of innocence could not come to
their rescue as it was more than sufficiently overcome by the proof that was offered by
the prosecution. The principal contention raised is thus clearly untenable. It must be
stated likewise that while squarely advanced for the first time, there had been cases
where this Court, notwithstanding a majority of the defendants being acquitted, the
element of conspiracy likewise being allegedly present, did hold the party or parties,
responsible for the offense guilty of the crime charged, a moral certainty having arisen
as to their capability.

DUMLAO VS. COMELEC


[95 SCRA 392; L-52245; 22 JAN 1980]
Facts:
Petitioner Dumlao questions the constitutionality of Sec. 4 of Batas Pambansa Blg 52 as
discriminatory and contrary to equal protection and due process guarantees of the
Constitution. Sec. 4 provides that any retired elective provicial or municipal official who
has received payments of retirement benefits and shall have been 65 years of age at
the commencement of the term of office to which he seeks to be elected, shall not be
qualified to run for the same elective local office from which he has retired. According
to Dumlao, the provision amounts to class legislation. Petitioners Igot and Salapantan Jr.
also assail the validity of Sec. 4 of Batas Pambansa Blg 52, which states that any
person who has committed any act of disloyalty to the State, including those
amounting to subversion, insurrection, rebellion, or other similar crimes, shall not be
qualified for any of the offices covered by the act, or to participate in any partisan
activity therein: provided that a judgment of conviction of those crimes shall be
conclusive evidence of such fact and the filing of charges for the commission of such
crimes before a civil court or military tribunal after preliminary investigation shall be
prima facie evidence of such fact.
Issue:
Whether or not the aforementioned statutory provisions violate the Constitution and
thus, should be declared null and void
Held:

Page 243

In regards to the unconstitutionality of the provisions, Sec. 4 of BP Blg 52 remains


constitutional and valid. The constitutional guarantee of equal protection of the laws is
subject to rational classification. One class can be treated differently from another
class. In this case, employees 65 years of age are classified differently from younger
employees. The purpose of the provision is to satisfy the need for new blood in the
workplace. In regards to the second paragraph of Sec. 4, it should be declared null and
void for being violative of the constitutional presumption of innocence guaranteed to an
accused. Explicit is the constitutional provision that, in all criminal prosecutions, the
accused shall be presumed innocent until the contrary is proved, and shall enjoy the
right to be heard by himself and counsel (Article IV, section 19, 1973 Constitution). An
accusation, according to the fundamental law, is not synonymous with guilt. The
challenged proviso contravenes the constitutional presumption of innocence, as a
candidate is disqualified from running for public office on the ground alone that charges
have been filed against him before a civil or military tribunal. It condemns before one is
fully heard. In ultimate effect, except as to the degree of proof, no distinction is made
between a person convicted of acts of dislotalty and one against whom charges have
been filed for such acts, as both of them would be ineligible to run for public office. A
person disqualified to run for public office on the ground that charges have been filed
against him is virtually placed in the same category as a person already convicted of a
crime with the penalty of arresto, which carries with it the accessory penalty of
suspension of the right to hold office during the term of the sentence (Art. 44, Revised
Penal Code).
And although the filing of charges is considered as but prima facie evidence, and
therefore, may be rebutted, yet. there is "clear and present danger" that because of the
proximity of the elections, time constraints will prevent one charged with acts of
disloyalty from offering contrary proof to overcome the prima facie evidence against
him.
Additionally, it is best that evidence pro and con of acts of disloyalty be aired before the
Courts rather than before an administrative body such as the COMELEC. A highly
possible conflict of findings between two government bodies, to the extreme detriment
of a person charged, will thereby be avoided. Furthermore, a legislative/administrative
determination of guilt should not be allowed to be substituted for a judicial
determination.
Being infected with constitutional infirmity, a partial declaration of nullity of only that
objectionable portion is mandated. It is separable from the first portion of the second
paragraph of section 4 of Batas Pambansa Big. 52 which can stand by itself.
Wherefore, the first paragraph of section 4 of Batas pambansa Bilang 52 is hereby
declared valid and that portion of the second paragraph of section 4 of Batas Pambansa
Bilang 52 is hereby declared null and void, for being violative of the constitutional
presumption of innocence guaranteed to an accused.

Page 244

PEOPLE VS. ALCANTARA


[240 SCRA 122; G.R. NO. 91283; 17 JAN 1995]
Facts:
On July 19, 1988, Venancio Patricio, accompanied by Larry Salvador, drove a tenwheeler truck a Coca-Cola plant in Antipolo to load cases of softdrinks. They were about
to leave the plant at 10:00pm when several men approached them to hitch for a ride.
Ascertaining that Salvador knew appellant, Venancio accommodated appellant's
request. Appellant had four companions. At Ortigas Ave., one of them poked a gun at
Venancio and grabbed the steering wheel. At the North Diversion Road, Venancio and
Salvador(helper) were brought down from the vehicle and tied to the fence of the
expressway, thereafter they were stabbed and left bleeding to death. Venancio survived
but Salvador did not.Appellant was arrested in the vicinity of Otis Street in Pandacan,
Manila. A few days later, he was turned over to the Constabulary Highway Patrol Group.
Sgt. Alberto Awanan brought the appellant to the MCU hospital and was presented to
Venancio for identification. Appellant was brought to the Headquarters at Camp Crame
where he confessed.
Appelants Defense: Denial and alibi. He said that he was just applying to be a driver
and stayed there even if he was told that no work was available, to confirm with the
truck drivers. While he was applying for CONCEPCION TRUCKING located across Otis
street from the Coca-cola plant. He was arrested. He denied any knowledge of the "hit"
on the Coca-cola delivery truck. He remained in the custody of the police for two days
and two nights. On the third day of his detention, he was turned over to the
Constabulary Highway Patrol Group. Appellant was the brought to the MCU hospital. He
was made to confront Venancio whom he saw for the first time. CHPG Sgt. Awanan
asked Venancio twice if appellant was among those who hijacked the truck he was
driving. On both times, Venancio did not respond. Undaunted, Sgt. Awanan, called to a
photographer present, forced appellant to stand about a foot from Venancio, and told
the latter to just point at the suspect. "Basta ituro mo lang," Sgt. Awanan directed.
Venancio obeyed, and pictures of him pointing to the suspect were taken. From the
hospital, appellant was brought to the Constabulary Highway Patrol Group headquarters
at Camp Crame. Without being apprised of his rights nor provided with counsel, he was
interrogated and urged to confess his guilt. He balked. At ten o'clock that night, hours
after questioning began, appellant's interrogators started boxing him and kicking him.
He was also hit on the back with a chair, and electrocuted. Still, he refused to admit to
the crime. In the midst of his ordeal, appellant heard someone say, "Tubigan na iyan."
He was then blindfolded and brought to another room where he was made to lie down.
Water was slowly and continuously poured on his face, over his mouth. Appellant could
no longer bear the pain caused by the water treatment. Finally, he confessed to being
one of the hijackers. He was led to another room, where he was handcuffed and left
Page 245

until the following day. Later, he was made to sign prepared statements containing his
full confession.
Alcantara was arraigned under an information charging him and four others (at large)
with the crime of robbery with Homicide and Frustrated Homicide.
The trial court convicted the accused despite the following inconsistency between
Venancios affidavit and testimony:
Affidavit

Testimony

1. mentioned 5 assailants

- only Alcantara was identified

2. stabbing was preceded by a

- only 3 assailants had a

3. conference by all assailants

- conference

4. claimed to have allowed assailants

- failed to identify Alcantara

5. to hitch a ride because Alcantara

- at the hospital and in open

6. was familiar to them

- court (pointed to another person)

Issue:
Whether or not the rights of the accused was violated.

Held:
YES. The peoples evidence failed to meet the quantum required to overcome the
presumption. The second identification which correctly pointed to accused by Venancio
should not be credited. There is no reason for him to err as they know each other for 3
years. It was also incorrect to give too much weight to Police Sgt. Awanans testimony
as to the previous identification at the hospital. The testimony of Sgt. Awanan was
not corroborated by Venancio.
The identification procedure was irregular. Due process demands that the identification
procedure of criminal suspects must be free from impermissible suggestions as the
influence of improper suggestion probably accounts for more miscarriages of justice
than any other single factor. Conviction must be based on the strength of the
prosecution and not the weakness of the defense. There was blatant violation of the
constitutional rights of appellant as an accused. Appellant belongs to the economically
deprived in our society. He is nearly illiterate(third grade education). Our Constitution
and our laws strictly ordain their protection following the Magsaysay desideratum that
those who have less in life should have more in law.
Page 246

CORPUZ VS. REPUBLIC


[194 SCRA 73; G.R. NO. 74259; 14 FEB 1991]
Facts:
Generoso Corpuz is the Supervising Accounting Clerk in the Office of the Provincial
Treasurer of Nueva Viscaya. He was designated Acting Supervising Cashier in the said
office. In this capacity, he received collections, disbursed funds and made bank
deposits and withdrawals pertaining to government accounts. On April 13, 1981 his
designation as Acting Supervising Cashier was terminated and a transfer of
accountabilities was effected between him and his successor. The Certificate of
turnover revealed a shortage of P72,823.00. He was able to pay only P10,159.50. After
a final demand letter for the total of P50,596.07 which was not met, a case of
malversation was filed against him. Corpuz did not deny such facts but he insists that
the shortage was malversed by other persons. He alleged that Paymaster Diosdado
Pineda through 1 of 4 separate checks (PNB) issued and encashed such checks while he
was of leave. Also, Acting Deputy Provincial Treasurer Bernardo Aluning made to post
the amount on his cashbook although he had not received the said amount. He was
convicted in Sandiganbayan.
Issue:
Whether or Not Corpuz is guilty of malversation.
Held:
It is a subtle way of camouflaging the embezzlement of the money equivalent when 1
of the 4 checks issued and encashed in the same day was entered in the accuseds
cash book 3 months after such encashments. Also, Corpuz claim that he was absent
when Paymaster Diosdado Pineda through 1 of 4 separate checks (PNB) issued and
encashed such checks, was not proven.
Post-Audit is not a preliminary requirement to filing a malversation case. The failure of
the public officer to have duly forthcoming any public funds with which he is
chargeable, upon demand by an authorized officer shall be a prima facie evidence that
he has put such missing funds to personal use.
The equipoise rule(balancing test) which is the presumption of innocence is applicable
only where the evidence of the parties is evenly balance, in which case the scale of
justice should be tilt in favor of the accused. There is no such balance in the case at
bar. The evidence of the prosecution is overwhelming and has not been overcome by
Page 247

the petitioner with his claims. The presumed innocence must yield to the positive
finding that he is guilty of malversation.
Wherefore his petition is denied. He is guilty as principal of Malversation of Public
Funds.

PEOPLE VS. HOLGADO


[85 PHIL 752; G.R.L-2809; 22 MAR 1950]
Facts:
Appellant Frisco Holgado was charged in the court of First Instance of Romblon with
slight illegal detention because according to the information, being a private person, he
did "feloniously and without justifiable motive, kidnap and detain one Artemia Fabreag
in the house of Antero Holgado for about eight hours thereby depriving said Artemia
Fabreag of her personal liberty. He pleaded guilty (without a counsel) and said that he
was just instructed by Mr. Ocampo, which no evidence was presented to indict the
latter.
Issue:
Whether or Not there was any irregularity in the proceedings in the trial court.
Held:
Yes. Rule 112, section 3 of ROC that : If the defendant appears without attorney, he
must be informed by the court that it is his right to have attorney being arraigned., and
must be asked if he desires the aid of attorney, the Court must assign attorney de oficio
to defend him. A reasonable time must be allowed for procuring attorney. This was
violated. Moreso the guarantees of our Constitution that "no person shall be held to
answer for a criminal offense without due process of law", and that all accused "shall
enjoy the right to be heard by himself and counsel." In criminal cases there can be no
fair hearing unless the accused be given the opportunity to be heard by counsel.
The trial court failed to inquire as to the true import of the qualified plea of accused.
The record does not show whether the supposed instructions of Mr. Ocampo was real
and whether it had reference to the commission of the offense or to the making of the
plea guilty. No investigation was opened by the court on this matter in the presence of
the accused and there is now no way of determining whether the supposed instruction
is a good defense or may vitiate the voluntariness of the confession. Apparently the
court became satisfied with the fiscal's information that he had investigated Mr.
Ocampo and found that the same had nothing to do with this case. Such attitude of the
Page 248

court was wrong for the simple reason that a mere statement of the fiscal was not
sufficient to overcome a qualified plea of the accused. But above all, the court should
have seen to it that the accused be assisted by counsel especially because of the
qualified plea given by him and the seriousness of the offense found to be capital by
the court.

PEOPLE VS. MAGSI


[124 SCRA 64; G.R. NO.L-32888; 12 AUG 1983]
Facts:
Soon after appellant was apprehended on August 20, 1970, his arraignment was
scheduled before the Criminal Circuit Court of San Fernando, La Union. The case was
actually set and rescheduled for six (6) times, first of which was on August 1, 1970. On
that date, despite appointment by the court of Atty. Mario Rivera as de officio counsel
for the accused, hearing was re-set to September 8, 1970 on motion of Atty. Rivera,
who was prompted to ask for it because of accused desire to be represented by a de
parte counsel. Prior to the next hearing, Atty. Rivera moved to withdraw as de officio
counsel and it was favorably acted on by the court on September 7, 1970. At the
second hearing on September 8, 1970, for failure of the de officio and de parte
counsels to appear, despite a second call of the case, the hearing was re-set for the
next day and the court appointed Atty. Dominador Cariaso de officio counsel for the
accused. On the third hearing date, neither the de parte nor the de officio counsel was
in Court, so Atty. Rivera was reappointed that day as de officio counsel for arraignment
purposes only. The accused del Rosario entered a plea of guilty but qualified it with the
allegation that he committed the crime out of fear of his co-accused Eloy Magsi and the
other coaccused. Appellant was found guilty of murder and made to suffer the death
penalty.
Issue:
Whether or not there was a violation of the rights of the accused.
Held:
YES. The desire to speed up the disposition of cases should not be effected at the
sacrifice of the basic rights of the accused. Citing People vs. Domingo (55 SCRA 243244): the trial courts should exercise solicitous care before sentencing the accused on a
plea of guilty especially in capital offenses by first insuring that the accused fully
understands the gravity of the offense, the severity of the consequences attached
thereto as well as the meaning and significance of his plea of guilty; and that the
prudent and proper thing to do in capital cases is to take testimony, to assure the court
that the accused has not misunderstood the nature and effect of his plea of guilty. Mere
pro-forma appointment of de officio counsel, who fails to genuinely protect the interests
Page 249

of the accused, resetting of hearing by the court for alleged reception of evidence when
in fact none was conducted, perfunctory queries addressed to the accused whether he
understands the charges and the gravity of the penalty, are not sufficient compliance.

SORIANO VS. SANDIGANBAYAN


[131 SCRA 184; G.R. NO.L-65952; 31 JUL 1984]
Facts:
Tan was accused of qualified theft. The petitioner, who was an Asst. Fiscal, was
assigned to investigate. In the course of the investigation, petitioner demanded
Php.4000 from Tan as price for dismissing the case. Tan reported it to the NBI which set
up an entrapment. Tan was given a Php.2000, marked bill, and he had supplied the
other half. The entrapment succeeded and an information was filed with the
Sandiganbayan. After trial, the Sandiganbayan rendered a decision finding the
petitioner guilty as a principal in violating the Anti Graft and Corrupt Practices Act
(R.A.3019). A motion for reconsideration was denied by the Sandiganbayan, hence this
instant petition.
Issue:
Whether or Not the investigation conducted by the petitioner can be regarded as
contract or transaction within the purview of .RA.3019.
Held:
R.A. 3019 Sec.3. Corrupt practices of public officers - In addition to acts or omissions of
public officers already penalized by existing laws, the following shall constitute corrupt
practices of any public officer and are hereby declared to be unlawful: xxx b. Directly or
indirectly requesting or receiving any gift, present, share percentage or benefit, for
himself or for other person, in connection with any contract or transaction between the
Govt. and any other party wherein the public officer in his official capacity has to
intervene under the law.
The petitioner stated that the facts make out a case of direct bribery under Art.210 of
the RPC and not a violation of R.A. 3019 sec.3 (b). The offense of direct bribery is not
the offense charged and is not included in the offense charged which is violation of
R.A.3019 sec.3 (b).

Page 250

The respondent claimed that, transaction as used hereof, is not limited to commercial
or business transaction, but includes all kinds of transaction whether commercial, civil,
or administrative in nature.
The court agrees with the petitioner. It is obvious that the investigation conducted by
the petitioner was neither a contract nor transaction. A transaction like a contract is
one which involves some consideration as in credit transactions. And this element is
absent in the investigation conducted by the petitioner.
Judgment modified. Petitioner is guilty of direct bribery under Art.210 of the RPC.

BORJA VS. MENDOZA


[77 SCRA 422; G.R. NO.L-45667; 20 JUN 1977]
Facts:
Borja was accused of slight physical injuries in the City of Cebu. However, he was not
arraigned. That not withstanding, respondent Judge Senining proceeded with the trial in
absentia and rendered a decision finding petitioner guilty of the crime charged. The
case was appealed to the Court o First Instance in Cebu presided by respondent Judge
Mendoza. It was alleged that the failure to arraign him is a violation of his constitutional
rights. It was also alleged that without any notice to petitioner and without requiring
him to submit his memorandum, a decision on the appealed case was rendered The
Solicitor General commented that the decision should be annulled because there was
no arraignment.
Issue:
Whether or Not petitioners constitutional right was violated when he was not
arraigned.
Held:
Yes. Procedural due process requires that the accused be arraigned so that he may be
informed as to why he was indicted and what penal offense he has to face, to be
convicted only on a showing that his guilt is shown beyond reasonable doubt with full
opportunity to disprove the evidence against him. It is also not just due process that
Page 251

requires an arraignment. It is required in the Rules that an accused, for the first time, is
granted the opportunity to know the precise charge that confronts him. It is imperative
that he is thus made fully aware of possible loss of freedom, even of his life, depending
on the nature of the crime imputed to him. At the very least then, he must be fully
informed of why the prosecuting arm of the state is mobilized against him. Being
arraigned is thus a vital aspect of the constitutional rights guaranteed him. Also,
respondent Judge Senining convicted petitioner notwithstanding the absence of an
arraignment. With the violation of the constitutional right to be heard by himself and
counsel being thus manifest, it is correct that the Solicitor General agreed with
petitioner that the sentence imposed on him should be set aside for being null. The
absence of an arraignment can be invoked at anytime in view of the requirements of
due process to ensure a fair and impartial trial.
Wherefore, the petition for certiorari is granted. The decision of respondent Judge
Romulo R. Senining dated December 28, 1973, finding the accused guilty of the crime
of slight physical injuries, is nullified and set aside. Likewise, the decision of respondent
Judge Rafael T. Mendoza dated November 16, 1976, affirming the aforesaid decision of
Judge Senining, is nullified and set aside. The case is remanded to the City Court of
Cebu for the prosecution of the offense of slight physical injuries, with due respect and
observance of the provisions of the Rules of Court, starting with the arraignment of
petitioner.

CONDE VS. RIVERA


[45 PHIL 650; G.R. NO. 21741; 25 JAN 1924]
Facts:
Aurelia Conde, formerly a municipal midwife in Lucena, Tayabas, has been forced to
respond to no less the five information for various crimes and misdemeanors, has
appeared with her witnesses and counsel at hearings no less than on eight different
occasions only to see the cause postponed, has twice been required to come to the
Supreme Court for protection, and now, after the passage of more than one year from
the time when the first information was filed, seems as far away from a definite
resolution of her troubles as she was when originally charged.
Issue:
Whether or Not petitioner has been denied her right to a speedy and impartial trial.
Held:
Page 252

Philippine organic and statutory law expressly guarantee that in all criminal
prosecutions the accused shall enjoy the right to have a speedy trial. Aurelia Conde,
like all other accused persons, has a right to a speedy trial in order that if innocent she
may go free, and she has been deprived of that right in defiance of law. We lay down
the legal proposition that, where a prosecuting officer, without good cause, secures
postponements of the trial of a defendant against his protest beyond a reasonable
period of time, as in this instance for more than a year, the accused is entitled to relief
by a proceeding in mandamus to compel a dismissal of the information, or if he be
restrained of his liberty, by habeas corpus to obtain his freedom.

PEOPLE VS. TAMPAL


[244 SCRA 202; G.R. NO. 102485; 22 MAY 1995]
Facts:
Luis Tampal, Domingo Padumon, Arsenio Padumon, Samuel Padumon, Pablito Suco,
Dario Suco and Galvino Cadling were charged of robbery with homicide and multiple
serious physical injuries in the Regional Trial Court of Zamboanga with Hon. Wilfredo
Ochotorena as presiding judge. However, only private respondents, Luis Tampal,
Domingo Padumon, Arsenio Padumon, and Samuel Padumon were arrested, while the
others remained at large.
The case was set for hearing on July 26, 1991, but Assistant Provincial Prosecutor
Wilfredo Guantero moved for postponement due to his failure to contact the material
witnesses. The case was reset without any objection from the defense counsel. The
case was called on September 20, 1991 but the prosecutor was not present. The
respondent judge considered the absence of the prosecutor as unjustified, and
dismissed the criminal case for failure to prosecute. The prosecution filed a motion for
reconsidereation, claiming that his absence was because such date was a Muslim
holiday and the office of the Provincial prosecutor was closed on that day. The motion
was denied by respondent judge.
Issue:
Whether or Not the postponement is a violation of the right of the accused to a speedy
disposition of their cases.
Whether or Not the dismissal serves as a bar to reinstatement of the case.
Held:
In determining the right of an accused to speedy disposition of their case, courts should
do more than a mathematical computation of the number of postponements of the
scheduled hearings of the case. What are violative of the right of the accused to speedy
trial are unjustified postponements which prolong trial for an unreasonable length of
Page 253

time. In the facts above, there was no showing that there was an unjust delay caused
by the prosecution, hence, the respondent judge should have given the prosecution a
fair opportunity to prosecute its case.
The private respondents cannot invoke their right against double jeopardy. In several
cases it was held that dismissal on the grounds of failure to prosecute is equivalent to
an acquittal that would bar another prosecution for the same offense, but in this case,
this does not apply, considering that the rights of the accused to a speedy trial was not
violated by the State. Therefore, the order of dismissal is annulled and the case is
remanded to the court of origin for further proceedings.

REPUBLIC ACT NO. 8493 THE SPEEDY TRIAL ACT


The arraignment of an accused shall be held within 30 days from filing of the
information, or from the date the accused has appeared before the justice, judge or
court in which the charge is pending, whichever date last occurs. Thereafter, where a
plea of not guilty is entered, the accused shall have at least 15 days to prepare for trial.
Trial shall commence within 30 days from arraignment as fixed by the court. In no case
shall the entire trial period exceed 180 days from the 1 st day of trial, except as
otherwise authorized by the Chief Justice of the Supreme Court.

RE: REQUEST FOR LIVE TV OF TRIAL OF JOSEPH ESTRADA


[360 SCRA 248; A.M. NO 01-4-03-SC; 29 JUN 2001]
Facts:
The Kapisanan ng mga Brodkaster ng Pilipinas (KBP) sent a letter requesting the Court
to allow live media coverage of the anticipated trial of the plunder and other criminal
cases filed against former President Joseph E. Estrada before the Sandiganbayan in
order "to assure the public of full transparency in the proceedings of an unprecedented
case in our history." The request was seconded by Mr. Cesar N. Sarino and, still later, by
Senator Renato Cayetano and Attorney Ricardo Romulo.
Issue:
Whether or Not live media coverage of the trial of the plunder and other criminal cases
filed against former President Joseph E. Estrada should be permitted by the court.
Held:
The propriety of granting or denying the instant petition involve the weighing out of the
constitutional guarantees of freedom of the press and the right to public information,
on the other hand, along with the constitutional power of a court to control its
Page 254

proceedings in ensuring a fair and impartial trial. When these rights race against one
another, jurisprudence tells us that the right of the accused must be preferred to win.
Due process guarantees the accused a presumption of innocence until the contrary is
proved in a trial that is not lifted about its individual settings nor made an object of
publics attention and where the conclusions reached are induced not by any outside
force or influence but only be evidence and argument given in open court, where fitting
dignity and calm ambiance is demanded.
An accused has a right to a public trial but it is a right that belongs to him, more than
anyone else, where his life or liberty can be held critically in balance. A public trial aims
to ensure that he is fairly dealt with and would not be unjustly condemned and that his
rights are not compromised in secret conclaves of long ago. A public trial is not
synonymous with publicized trial, it only implies that the court doors must be open to
those who wish to come, sit in the available seats, conduct themselves with decorum
and observe the trial process.
The courts recognize the constitutionally embodied freedom of the press and the right
to public information. It also approves of media's exalted power to provide the most
accurate and comprehensive means of conveying the proceedings to the public.
Nevertheless, within the courthouse, the overriding consideration is still the paramount
right of the accused to due process which must never be allowed to suffer diminution in
its constitutional proportions.

PEOPLE VS. SALAS


[143 SCRA 163; G.R. NO. L-66469; 29 JUL 1986]
Facts:
At about 6:00 o'clock in the morning of March 6, 1992, a 60 year old woman, identified
as Virginia Talens was found lying dead in a canal at Bo. San Nicolas, Mexico,
Pampanga; she was last seen alive at about 3:00 o'clock early morning of March 6,
1992 by Orlando Pangan and Richard Pangan who were with her going home coming
from the wake of one Leonardo Flores; both Orlando and Richard Pangan testified that
accused was with them in going home at about 3:00 o'clock in the morning of March 6,
1992; Orlando and Richard Pangan reached first their house and left the two on the way
and that was the last time Virginia was seen alive; just a few minutes after reaching his
house and while inside his house, Orlando Pangan heard a shout; another woman, one
Serafia Gutierrez, testified that she likewise was awakened by a shout at about 3:00 in
the morning; Dr. Aguda who autopsied the victim found hematoma on the head and
chest, an abrasion on the left chin and stabwound on the neck which stabwound, the
doctor claims, was the cause of death of the victim; Police Investigator Gonzales who
immediately responded upon report, recovered at the scene a pin, the victim's
wristwatch, earring, a ring and P135.00 money; he likewise found on March 9, 1992
when he continued his investigation bloodstain on the front door of the house of the
Page 255

accused which bloodstain when submitted for examination was found to be of human
blood; one Resultay was with Virginia Talens at about 5:00 afternoon of March 5, 1992
in going to the wake, who claims that Virginia had money on a purse as while they were
on the way Virginia bet on a jueteng she saw Virginia got money from her purse a
P500.00 bill but as she had no change she instead took P8.00 from her other pocket;
one Ramil Talens, a son of the victim corroborated the claim of Resultay that Virginia
had with her at that time money worth P2,000.00 as in the morning of March 5, 1992
he gave her mother for safekeeping the sum of P1,500.00 which he claims his mother
placed in her purse and claims further that at the wake, he asked and was given P50.00
by his mother as he also participated in the gambling thereat, however, the purse of
Virginia containing about P2,000.00 was no longer to be found when she was found
dead; Orlando Pangan saw the accused gambled in the wake; Virginia likewise gambled
at the wake; accused had been working for three days before March 6 at Sta. Ana,
Pampanga and up to March 5, 1992, but the following day, he did not anymore report
for work at Sta. Ana, Pampanga, was no longer to be found and was last seen at about
3:00 morning together with Virginia Talens on their way home coming from the wake;
the parents of [the] accused were informed by Investigator Gonzales that their son was
the suspect and adviced them to surrender him, but since March 6, 1992 when accused
left Mexico, Pampanga, he returned only on September 19, 1992 at Arayat, Pampanga,
not at Mexico, Pampanga where he was ultimately apprehended by the Mexico Police
on September 22, 1992 after chancing on a radio message by the police of Arayat to
their Provincial commander that a vehicular incident occurred at Arayat, Pampanga
where one Elmer Salas was the victim and was hospitalized at the district hospital at
Arayat, Pampanga where he used the name of Rommel Salas and not Elmer Salas. The
trial court rendered convicting Salas for Robbery with Homicide
Issue:
Whether or Not there is evidence sufficient to sustain a conviction of the appellant of
the crime of Robbery with Homicide.
Whether or Not the appellants crime homicide or robbery with homicide.
Held:
There was no eyewitness or direct evidence; either to the robbery or to the homicide
and none of the things allegedly stolen were ever recovered. However, direct evidence
is not the only matrix from which the trial court may draw its findings and conclusion of
culpability. Resort to circumstantial evidence is essential when to insist on direct
testimony would result in setting felons free.
For circumstantial evidence to be sufficient to support a conviction, all the
circumstances must be consistent with each other, consistent with the theory that the
accused is guilty of the offense charged, and at the same time inconsistent with the
hypothesis that he is innocent and with every other possible, rational hypothesis
excepting that of guilt. All the circumstances established must constitute an unbroken
chain which leads to one and fair and reasonable conclusion pointing solely to the
accused, to the exclusion of all other persons, as the author of the crime. The facts and
circumstances consistent with the guilt of the accused and inconsistent with his
innocence can constitute evidence which, in weight and probative value, may be
deemed to surpass even direct evidence in its effect on the court.
Page 256

The fatal stabbing of Virginia Talens occurred at around 3:00 a.m. of March 6, 1992.
Appellant hastily abandoned his house in Barrio San Nicolas, Mexico, Pampanga, his
residence since childhood, on that very date. Appellant was nowhere when his coworker and barrio mate, Eduardo Bagtas, came to appellant's house to fetch him for
work at around 6:30 to 7:00 a.m. of March 6, 1992. Appellant also abandoned his job as
a painter in Sta. Ana, Pampanga, on March 6, 1992, the date of the crime, leaving
behind an unfinished painting project. He was not seen again from said date. Police
investigators found human bloodstains on the front door of appellant's house, on his
clothing, and on his yellow slippers after the victim was killed. Despite efforts of the
police to find appellant as the principal suspect, a fact known to appellant's family and
neighbors, appellant did not present himself to the authorities. Appellant was
apprehended only a full six months after the date of the crime, following his
confinement in a hospital in Arayat, Pampanga because he was sideswiped by a Victory
Liner bus in Arayat. When hospitalized, appellant used the alias Rommel Salas, instead
of his true name Elmer Salas. These circumstances denote flight, which when
unexplained, has always been considered by the courts as indicative of guilt.
Both appellant and victim gambled at the wake they attended. The victim was, in fact,
enjoying a winning streak when her son, Ramil Talens, came to fetch her but which he
failed to do because his mother was winning, and she refused to leave. The purse of
Talens containing cash was gone when her corpse was found in the canal with a stab
wound and bruises. What was left was a safety pin which victim used to fasten the
missing purse to her clothes.
Denial is an inherently weak defense which must be buttressed by strong evidence of
non-culpability to merit credibility. Denial is negative and self-serving and cannot be
given greater evidentiary weight over the testimonies of credible witnesses who
positively testified that appellant was at the locus criminis and was the last person seen
with the victim alive.
The absence of evidence showing any improper motive on the part of the principal
witness for the prosecution to falsely testify against the appellant strongly tends to
buttress the conclusion that no such improper motive exists and that the testimony of
said witnesses deserve full faith and credit.
The essence of voluntary surrender is spontaneity and the intent of the accused to give
himself up and submit himself unconditionally to the authorities either because he
acknowledges his guilt or he wants to save the State the trouble of having to effect his
arrest. Spontaneity and intent to give one's self up are absent where the accused went
into hiding for six months after the incident and had to resort to an alias when he was
involved in an accident being investigated by the police authorities.
Robbery with Homicide is a special complex crime against property. Homicide is
incidental to the robbery which is the main purpose of the criminal. In charging Robbery
with Homicide, the onus probandi is to establish: "(a) the taking of personal property
with the use of violence or intimidation against a person; (b) the property belongs to
another; (c) the taking is characterized with animus lucrandi; and (d) on the occasion of
the robbery or by reason thereof, the crime of homicide, which is used in the generic
sense, was committed." Although there was no witness as to the actual robbing of the
Page 257

victim, there is testimony that the victim had more or less P2,000.00; and wore gold
earrings valued at P750.00. These were never recovered.
While there is indeed no direct proof that Virginia Talens was robbed at the time she
was killed, we may conclude from four circumstances that the robbery occasioned her
killing: (1) Both appellant and victim gambled at the wake. (2) The appellant knew that
victim was winning. (3) The victim was last seen alive with appellant. (4) The victim's
purse containing her money and earrings were missing from her body when found.
The decision of the regional trial court is affirmed. Costs against appellant. So ordered.

PEOPLE VS. MAGPALAO


[197 SCRA 79; G.R. NO. 92415; 14 MAY 1991]
Facts:
Eleven (11) people rode in a Ford Fiera going to Baguio. Namely they are: Felizardo
Galvez, Jimmy Jetwani, Simeon Calama, Rene Salonga, Eduardo Lopez, Adolfo
Quiambao, Aliman Bara-akal, Anwar Hadji Edris, Gumanak Ompa and defendantappelants in this case, Omar Magpalao and Rex Magumnang.
After an hour of driving, the car stopped so that one of the passengers could urinate.
While the car was stopped the Bara-akal, Edris, Ompa, Magpalao and Magumnang
pointed guns and knives at the other passengers and divested them of their properties.
On of the robbers then ordered Galvez to drive the car towards the precipice (bangin).
When the car was near the precipice, Galvez then stepped to the brakes. The other
passengers jumped out of the car and went to different directions to escape. Galvez
however, was left in side the car and was stabbed by one of the robbers. The robbers
then escaped. Quiambao, who owned the car helped Galvez to get to a hospital. Galvez
died in the hospital. The robbers were then apprehended with the exception of Edris
who remain at large. Mangumnang however escaped while being in detention and
Bara-akal died inside the jail. Since Mangumnang was not arrested, the trial in absentia
continued as to him. Ompa, Magpalao, and Magumnang were all held guilty as principal
by direct participation of the crime of Robbery with Homicide.
Issue:
Whether or Not the lower court erred in failing to apply the Constitutional mandate on
the presumption of innocence and proof beyond reasonable doubt when it allowed the
trial in absentia to push through on the part of defendant-appellant Magumnang.
Held:
Page 258

The Court affirmed the decision of the lower court. The reason is that the lower court
has jurisdiction over Magumnang the moment the latter was in custody. Jurisdiction
once acquired is not lost upon the instance of parties but until the case is terminated.
Since all the requisites of trial in absentia are complete, the court has jurisdiction over
Magumnang.
In addition, Magumnang was presumed innocent during his trial in absentia. The
prosecution had strong evidence against him as proof beyond reasonable doubt that he
is a principal by direct participation in the crime of Robbery with Homicide. Thus, the
Constitutional mandate was not violated.

PEOPLE VS. ACABAL


[226 SCRA 694 ; G.R. NO. 103604, 23 SEP 1993]
Facts:
The accusatory portion in the information for murder. Facts are as follows:
"That sometime in the evening of the 28th of January, 1980, at Nagbinlod, Municipality
of Sta. Catalina, Province of Negros Oriental, Philippines, and within the jurisdiction of
this Honorable Court, the accused, including several 'John Does', conspiring and
confederating with one another, with intent to kill, and with treachery and evident
premeditation and being then armed with bolos and 'pinuti', did then and there willfully,
unlawfully and feloniously attack, assault and use personal violence on the person of
one Rizalina Apatan Silvano while the latter was about to leave her house and inflicting
upon her injuries, to wit: 'right leg amputated below the knee; left leg hacked behind
the knee; abdomen hacked with viscerae evacerated,' and did then and there set the
house on fire while the aforementioned Rizalina Apatan Silvano was inside said house
trying to escape therefrom, and allowing her to be burned inside said house which was
burned to the ground, thereby causing upon said Rizalina Apatan Silvano her death and
burning her beyond recognition.
But on 16 May 1987, a fire gutted the building where Branch 37 was located and the
records of these two cases were burned. The records were subsequently reconstituted
upon petition of the prosecuting fiscal. The testimonies of the witnesses were retaken,
however, before it could commence, accused Engracio Valeriano jumped bail and the
warrant for his arrest issued on 16 November 1987 was returned unserved because he
could not be found. An alias warrant for his arrest was issued on 26 June 1989, but
he remains at large up to the present.

Page 259

After the completion of the re-taking of the testimonies of the witnesses in Branch 37,
Criminal Cases Nos. 4584 and 4585 were re-raffled to Branch 33 of the trial court, then
presided over by Judge Pacifico S. Bulado.
The decision of the trial court, per Judge Pacifico S. Bulado, dated 31 October 1991 but
promulgated on 20 December 1991, contained no specific dispositive portion. Its
rulings are found in the last two paragraphs which read as follows:
"The elements of murder in this case, Criminal Case No. 4585 for the killing of Rizalina
Apatan-Silvano having been proved by the prosecution beyond doubt, the accused
JUANITO RISMUNDO, MACARIO ACABAL and ABUNDIO NAHID, considering the attendant
qualifying aggravating circumstances of nighttime, use of fire by burning the house of
victim Rizalina Apatan-Silvano in order to forcibly drive her out of her house and hack
her to death, the abuse of superior strength, the penalty impossable [sic] here will be in
its maximum degree, that is reclusion perpetua taking into account Article 248 of the
Revised Penal Code, the penalty now for murder is Reclusion Temporal to Reclusion
Perpetua, and for all the accused to indemnify the heirs of the victim the sum of Thirty
Thousand (P30,000.00) Pesos since this case occurred [sic] in 1980. For the wounding
of the victim Wilson A. Silvano, this Court believes that simple frustrated homicide only
is committed by the accused Engracio Valeriano only.
But since the person who actually inflicted the injuries of victim Wilson Silvano, accused
Engracio Valeriano only is nowhere to be found, hence, not brought to the bar of justice,
he being a fugitive or at large, no penalty could be imposed on him since he is beyond
the jurisdiction of this court to reach. All the other two (2) accused, JUANITO RISMUNDO
and ABUNDIO NAHID are hereby ordered and declared absolved from any criminal
responsibility from frustrated homicide.
The bail bond put up by the three accused, namely: Juanito Rismundo, Macario Acabal
and Abundio Nahid are hereby ordered cancelled and let a warrant of arrest be issued
for their immediate confinement."
Issue:
Whether or not the judgment complied with the Rules of Court.
Whether or not the cancellation of the bail bonds of the accused is valid.
Whether or not the accused may be tried in absentia.
Whether or not the accused is guilty of the crime of frustrated murder.
Held:
Page 260

We find that the decision substantially complies with the Rules of Court on judgments
as it did sentence the accused-appellants to reclusion perpetua. A judgment of
conviction shall state (a) the legal qualification of the offense constituted by the acts
committed by the accused, and the aggravating or mitigating circumstances attending
the commission, if there are any; (b) the participation of the accused in the commission
of the offense, whether as principal, accomplice or accessory after the fact; (c) the
penalty imposed upon the accused; and (d) the civil liability or damages caused by the
wrongful act to be recovered from the accused by the offended party, if there is any,
unless the enforcement of the civil liability by a separate action has been reserved or
waived.
It is obvious that they clearly understood that they were found guilty beyond
reasonable doubt of the crime of murder and were sentenced to suffer the penalty of
reclusion perpetua in Criminal Case No. 4585. Were it otherwise, they would not have
declared in open court their intention to appeal immediately after the promulgation of
the decision and would not have subsequently filed their written notice of appeal.
Accused-appellants contend that the trial court did not impose any sentence and so
cannot cancel anymore their bail bonds and direct their arrest and immediate
commitment because it already lost jurisdiction over their persons when they perfected
their appeal.
The decision did impose the penalty of reclusion perpetua. Since the order cancelling
their bail bonds and directing their arrest is contained in the decision itself, it is
apparent that their abovementioned contention is highly illogical. At the time the order
in question was made, the trial court still had jurisdiction over the persons of the
accused-appellants.
The trial court further erred in holding that no penalty could be imposed on accused
Engracio Valeriano in Criminal Case No. 4584 because he "is nowhere to be found,
hence, not brought to the bar of justice, he being a fugitive or at large." The court
ignored the fact that Engracio jumped bail after he had been arraigned, just before the
retaking of evidence commenced. Paragraph (2), Section 14, Article III of the
Constitution permits trial in absentia after the accused has been arraigned provided he
has been duly notified of the trial and his failure to appear thereat is unjustified. One
who jumps bail can never offer a justifiable reason for his non-appearance during the
trial.
Accordingly, after the trial in absentia, the court can render judgment in the case and
promulgation may be made by simply recording the judgment in the criminal docket
with a copy thereof served upon his counsel, provided that the notice requiring him to
be present at the promulgation is served through his bondsmen or warden and counsel.
In conclusion, because of reasonable doubt as to their guilt, the accused-appellants
must be acquitted. Every accused is presumed innocent until the contrary is proved;
Page 261

that presumption is solemnly guaranteed by the Bill of Rights. The contrary requires
proof beyond reasonable doubt, or that degree of proof which produces conviction in an
unprejudiced mind. Short of this, it is not only the right of the accused to be freed; it is
even the constitutional duty of the court to acquit him.

US VS. TAN TENG


[23 PHIL 145; G.R. NO. 7081; 7 SEP 1912]
Facts:
The defendant herein raped Oliva Pacomio, a seven-year-old girl. Tan Teng was
gambling near the house of the victim and it was alleged that he entered her home and
threw the victim on the floor and place his private parts over hers. Several days later,
Pacomio was suffering from a disease called gonorrhea. Pacomio told her sister about
what had happened and reported it to the police.
Tan Teng was called to appear in a police line-up and the victim identified him. He was
then stripped of his clothing and was examined by a policeman. He was found to have
the same symptoms of gonorrhea. The policeman took a portion of the substance
emitting from the body of the defendant and turned it over to the Bureau of Science.
The results showed that the defendant was suffering from gonorrhea.
The lower court held that the results show that the disease that the victim had acquired
came from the defendant herein. Such disease was transferred by the unlawful act of
carnal knowledge by the latter. The defendant alleged that the said evidence should be
inadmissible because it was taken in violation of his right against self-incrimination.
Issue:
Whether or Not the physical examination conducted was a violation of the defendants
rights against self-incrimination.
Held:
The court held that the taking of a substance from his body was not a violation of the
said right. He was neither compelled to make any admissions or to answer any
questions. The substance was taken from his body without his objection and was
examined by competent medical authority.
The prohibition of self-incrimination in the Bill of Rights is a prohibition of the use of
physical or moral compulsion to extort communications from him, and not an exclusion
Page 262

of his body as evidence, when it may be material. It would be the same as if the
offender apprehended was a thief and the object stolen by him may be used as
evidence against him.

VILLAFLOR VS. SUMMERS


[41 PHIL 62; G.R. NO. 16444; 8 SEP 1920]
Facts:
Petitioner Villaflor was charged with the crime of adultery. The trial judge ordered the
petitioner to subject herself into physical examination to test whether or not she was
pregnant to prove the determine the crime of adultery being charged to her. Herein
petitioner refused to such physical examination interposing the defense that such
examination was a violation of her constitutional rights against self-incrimination.
Issue:
Whether or Not the physical examination was a violation of the petitioners
constitutional rights against self-incrimination.
Held:
No. It is not a violation of her constitutional rights. The rule that the constitutional
guaranty, that no person shall be compelled in any criminal case to be a witness
against himself, is limited to a prohibition against compulsory testimonial selfincrimination. The corollary to the proposition is that, an ocular inspection of the body
of the accused is permissible.

BELTRAN VS. SAMSON


[53 PHIL 570; G.R. NO. 32025; 23 SEPT 1929]
Facts:
Beltran, as a defendant for the crime of Falsification, refused to write a sample of his
handwriting as ordered by the respondent Judge. The petitioner in this case contended
that such order would be a violation of his constitutional right against self-incrimination
because such examination would give the prosecution evidence against him, which the
latter should have gotten in the first place. He also argued that such an act will make
him furnish evidence against himself.
Issue:

Page 263

Whether or not the writing from the fiscal's dictation by the petitioner for the purpose
of comparing the latter's handwriting and determining whether he wrote certain
documents supposed to be falsified, constitutes evidence against himself within the
scope and meaning of the constitutional provision under examination.
Held:
The court ordered the respondents and those under their orders desist and abstain
absolutely and forever from compelling the petitioner to take down dictation in his
handwriting for the purpose of submitting the latter for comparison. Writing is
something more than moving the body, or the hands, or the fingers; writing is not a
purely mechanical act, because it requires the application of intelligence and attention;
and in the case at bar writing means that the petitioner herein is to furnish a means to
determine whether or not he is the falsifier, as the petition of the respondent fiscal
clearly states. Except that it is more serious, we believe the present case is similar to
that of producing documents or chattels in one's possession. We say that, for the
purposes of the constitutional privilege, there is a similarity between one who is
compelled to produce a document, and one who is compelled to furnish a specimen of
his handwriting, for in both cases, the witness is required to furnish evidence against
himself. It cannot be contended in the present case that if permission to obtain a
specimen of the petitioner's handwriting is not granted, the crime would go
unpunished. Considering the circumstance that the petitioner is a municipal treasurer,
it should not be a difficult matter for the fiscal to obtained genuine specimens of his
handwriting. But even supposing it is impossible to obtain specimen or specimens
without resorting to the means complained herein, that is no reason for trampling upon
a personal right guaranteed by the constitution. It might be true that in some cases
criminals may succeed in evading the hand of justice, but such cases are accidental
and do not constitute the raison d' etre of the privilege. This constitutional privilege
exists for the protection of innocent persons.

PASCUAL VS. BME


[28 SCRA 345; G.R. NO. 25018; 26 MAY 1969]
Facts:
Petitioner Arsenio Pascual, Jr. filed an action for prohibition against the Board of Medical
Examiners. It was alleged therein that at the initial hearing of an administrative case for
alleged immorality, counsel for complainants announced that he would present as his
first witness the petitioner. Thereupon, petitioner, through counsel, made of record his
objection, relying on the constitutional right to be exempt from being a witness against
himself. Petitioner then alleged that to compel him to take the witness stand, the Board
of Examiners was guilty, at the very least, of grave abuse of discretion for failure to
respect the constitutional right against self-incrimination.

Page 264

The answer of respondent Board, while admitting the facts stressed that it could call
petitioner to the witness stand and interrogate him, the right against self-incrimination
being available only when a question calling for an incriminating answer is asked of a
witness. They likewise alleged that the right against self-incrimination cannot be
availed of in an administrative hearing.
Petitioner was sustained by the lower court in his plea that he could not be compelled
to be the first witness of the complainants, he being the party proceeded against in an
administrative charge for malpractice. Hence, this appeal by respondent Board.
Issue:
Whether or Not compelling petitioner to be the first witness of the complainants
violates the Self-Incrimination Clause.
Held:
The Supreme Court held that in an administrative hearing against a medical
practitioner for alleged malpractice, respondent Board of Medical Examiners cannot,
consistently with the self-incrimination clause, compel the person proceeded against to
take the witness stand without his consent. The Court found for the petitioner in
accordance with the well-settled principle that "the accused in a criminal case may
refuse, not only to answer incriminatory questions, but, also, to take the witness stand."
If petitioner would be compelled to testify against himself, he could suffer not the
forfeiture of property but the revocation of his license as a medical practitioner. The
constitutional guarantee protects as well the right to silence: "The accused has a
perfect right to remain silent and his silence cannot be used as a presumption of his
guilt." It is the right of a defendant "to forego testimony, to remain silent, unless he
chooses to take the witness stand with undiluted, unfettered exercise of his own free
genuine will."
The reason for this constitutional guarantee, along with other rights granted an
accused, stands for a belief that while crime should not go unpunished and that the
truth must be revealed, such desirable objectives should not be accomplished
according to means or methods offensive to the high sense of respect accorded the
human personality. More and more in line with the democratic creed, the deference
accorded an individual even those suspected of the most heinous crimes is given due
weight. The constitutional foundation underlying the privilege is the respect a
government ... must accord to the dignity and integrity of its citizens.

PEOPLE VS. BALISACAN


[17 SCRA 1119; G.R. NO. L-26376; 31 AUG 1966]
Page 265

Facts:
Aurelio Balisacan was charged with homicide in the CFI of Ilocos Norte. Upon being
arraigned, he entered into a plea of guilty. In doing so, he was assisted y counsel. At his
counsel de officio, he was allowed to present evidence and consequently testified that
he stabbed the deceased in self-defense. In addition, he stated that he surrendered
himself voluntarily to the police authorities. On the basis of the testimony of the
accused, he was acquitted. Thus, the prosecution appealed.
Issue:
Whether or Not the appeal placed the accused in double jeopardy.
Held:
The Supreme Court held that it is settled that the existence of plea is an essential
requisite to double jeopardy. The accused had first entered a plea of guilty but however
testified that he acted in complete self-defense. Said testimony had the effect of
vacating his plea of guilty and the court a quo should have required him to plead a
new charge, or at least direct that a new plea of not guilty be entered for him. This was
not done. Therefore, there has been no standing of plea during the judgment of
acquittal, so there can be no double jeopardy with respect to the appeal herein.

PEOPLE VS. OBSANIA


[23 SCRA 1249; G.R. L-24447; 29 JUN 1968]

Facts:
The accused was charged with Robbery with Rape before the Municipal Court of
Balungao, Pangasinan. He pleaded not guilty. His counsel moved for the dismissal of the
charge for failure to allege vivid designs in the info. Said motion was granted. From this
order of dismissal the
prosecution appealed.
Issue:
Whether or Not the present appeal places the accused in Double Jeopardy.
Held:
Page 266

In order that the accused may invoke double jeopardy, the following requisites must
have obtained in the original prosecution, a) valid complaint, b) competent court, c) the
defendant had pleaded to the charge, d) defendant was acquitted or convicted or the
case against him was dismissed or otherwise terminated without his express consent.
In the case at bar, the converted dismissal was ordered by the Trial Judge upon the
defendant's motion to dismiss. The doctrine of double jeopardy as enunciated in P.vs.
Salico
applies to wit when the case is dismissed with the express consent of the
defendant, the dismissal will not be a bar to another prosecution for the same offense
because his action in
having the case is dismissed constitutes a waiver of his
constitutional right/privilege for the reason that he thereby prevents the Court from
proceeding to the trial on the merits and rendering a judgment of conviction against
him.
In essence, where a criminal case is dismissed provisionally not only with the express
consent of the accused but even upon the urging of his counsel there can be no double
jeopardy under Sect. 9 Rule 113, if the indictment against him is revived by the fiscal.

PAULIN VS. GIMENEZ


[217 SCRA 386; G.R. NO. 103323; 21 JAN 1993]
Facts:
Respondent and Brgy Capt. Mabuyo, while in a jeep, were smothered with dust when
they were overtaken by the vehicle owned by Petitioner Spouses. Irked by such,
Mabuyo followed the vehicle until the latter entered the gate of an establishment. He
inquired the nearby security guard for the identity of the owner of the vehicle. Later
that day, while engaged in his duties, petitioners allegedly pointed their guns at him.
Thus, he immediately ordered his subordinate to call the police and block road to
prevent the petitioners escape. Upon the arrival of the police, petitioners put their
guns down and were immediately apprehended.
A complaint grave threats was filed against the petitioners (Criminal Case No. 5204).
It was dismissed by the court acting on the motion of the petitioners. Mabuyo filed a
MOR thus the dismissal was reversed. Thereafter, petitioners filed for certiorari,
prohibition, damages, with relief of preliminary injunction and the issuance of a TRO
(CEB-9207). Petition is dismissed for lack of merit and for being a prohibited pleading
and ordered to proceed with the trial of the case. Hence, this instant petition.
Issue:

Page 267

Whether or Not the dismissal of 5204 was a judgment of acquittal.


Whether or Not the judge ignored petitioners right against double jeopardy by
dismissing CEB-9207.
Held:
For double jeopardy to attach, the dismissal of the case must be without the express
consent of the accused. Where the dismissal was ordered upon motion or with the
express assent of the accused, he has deemed to have waived his protection against
double jeopardy. In the case at bar, the dismissal was granted upon motion of the
petitioners. Double jeopardy thus did not attach.
Furthermore, such dismissal is not considered as an acquittal. The latter is always
based on merit that shows that the defendant is beyond reasonable doubt not guilty.
While the former, in the case at bar, terminated the proceedings because no finding
was made as to the guilt or innocence of the petitioners.
The lower court did not violate the rule when it set aside the order of dismissal for the
reception of further evidence by the prosecution because it merely corrected its error
when it prematurely terminated and dismissed the case without giving the prosecution
the right to complete the presentation of its evidence. The rule on summary procedure
was correctly applied.

PEOPLE VS. COURT OF SILAY


[74 SCRA 248; G.R. NO. L-43790; 9 DEC 1976]
Facts:
That sometime on January 4,1974, accused Pacifico Sensio, Romeo Millan and Wilfredo
Jochico who were then scalers at the Hawaiian-Philippine Company, weighed cane cars
No.1743,1686 and 1022 loaded with sugar canes which were placed in tarjetas (weight
report cards), Apparently, it was proven and shown that there was padding of the
weight of the sugar canes and that the information on the tarjetas were to be false
making it appear to be heavier than its actual weight. The three accused then were
charged with Falsification by private individuals and use of falsified document. After
the prosecution had presented, the respondent moved to dismiss the charge against
them on the ground that the evidences presented were not sufficient to establish their
guilt beyond reasonable doubt. Acting on the motion, respondent court issued its order
dismissing the case on the ground that the acts committed by the accused do not
constituted the crime of falsification as strictly enumerated in the revised penal code
defining the crime of falsification which was charged earlier and that their case be
Page 268

dismissed. People asserts that the plea of double jeopardy is not tenable even if the
case at bar was dismissed because according to them, it was done with the consent of
the accused therefore waiving there defense of double jeopardy. The accused on the
other hand, reiterated the fact that the dismissal was due to lack of merits of the
prosecution which would have the same effect as an acquittal which will bar the
prosecution from prosecuting the accused for it will be unjust and unconstitutional for
the accused due to double jeopardy rule thus the appeal of the plaintiff.
Issue:
Whether or Not the grant of petition by the court would place the accused Sensio,
Millan and Jochico in double jeopardy
Held:
Yes the revival of the case will put the accused in double jeopardy for the very reason
that the case has been dismissed earlier due to lack of merits. It is true that the
criminal case of falsification was dismissed on a motion of the accused however this
was a motion filed after the prosecution had rested its case, calling for the evidence
beyond reasonable ground which the prosecution had not been able to do which would
be tantamount to acquittal therefore will bar the prosecution of another case. As it was
stated on the requirements of a valid defense of double jeopardy it says: That there
should be a valid complaint, second would be that such complaint be filed before a
competent court and to which the accused has pleaded and that defendant was
previously acquitted, convicted or dismissed or otherwise terminated without express
consent of the accused in which were all present in the case at bar. There was indeed a
valid, legitimate complaint and concern against the accused Sensio, Millan and Jochico
which was filed at a competent court with jurisdiction on the said case. It was also
mentioned that the accused pleaded not guilty and during the time of trial, it was
proven that the case used against the accused were not sufficient to prove them guilty
beyond reasonable doubt therefore dismissing the case which translates to acquittal. It
explained further that there are two instances when we can conclude that there is
jeopardy when first is that the ground for the dismissal of the case was due to
insufficiency of evidence and second, when the proceedings have been reasonably
prolonged as to violate the right of the accused to a speedy trial. In the 2 requisites
given, it was the first on that is very much applicable to our case at bar where there
was dismissal of the case due to insufficiency of evidence which will bar the approval of
the petition in the case at bar for it will constitute double jeopardy on the part of the
accused which the law despises.

PEOPLE VS. RELOVA


[149 SCRA 292; G.R. NO.L-45129; 6 MAR 1987]

Page 269

FACTS: In this petition for certiorari and mandamus, People of the Philippines seeks to
set aside the orders of Respondent Judge Hon. Relova quashing an information for theft
filed against Mr. Opulencia on the ground of double jeopardy and denying the
petitioners motion for reconsideration.. On Feb.1 1975, Batangas police together with
personnel of Batangas Electric Light System, equipped with a search warrant issued by
a city judge of Batangas to search and examine the premises of the Opulencia Carpena
Ice Plant owned by one Manuel Opulencia. They discovered electric wiring devices
have been installed without authority from the city government and architecturally
concealed inside the walls of the building. Said devices are designed purposely to
lower or decrease the readings of electric current consumption in the plants electric
meter. The case was dismissed on the ground of prescription for the complaint was filed
nine months prior to discovery when it should be 2months prior to discovery that the
act being a light felony and prescribed the right to file in court. On Nov 24, 1975,
another case was filed against Mr. Opulencia by the Assistant City Fiscal of Batangas for
a violation of a Batangas Ordinance regarding unauthorized electrical installations with
resulting damage and prejudice to City of Batangas in the amount of P41,062.16.
Before arraignment, Opulencia filed a motion to quash on the ground of double
jeopardy. The Assistant fiscals claim is that it is not double jeopardy because the first
offense charged against the accused was unauthorized installation of electrical devices
without the approval and necessary authority from the City Government which was
punishable by an ordinance, where in the case was dismissed, as opposed to the
second offense which is theft of electricity which is punishable by the Revised Penal
Code making it a different crime charged against the 1 st complaint against
Mr.Opulencia.
Issue:
Whether or Not the accused Mr. Opulencia can invoke double jeopardy as defense to
the second offense charged against him by the assistant fiscal of Batangas on the
ground of theft of electricity punishable by a statute against the Revised Penal Code.
Held:
Yes, Mr. Opulencia can invoke double jeopardy as defense for the second offense
because as tediously explained in the case of Yap vs Lutero, the bill of rights give two
instances or kinds of double jeopardy. The first would be that No person shall be twice
put in jeopardy of punishment for the same offense and the second sentence states
that If an act is punishable by a law or an ordinance, the conviction or acquittal shall
bar to another prosecution for the same act. In the case at bar, it was very evident
that the charges filed against Mr. Opulencia will fall on the 2 nd kind or definition of
double jeopardy wherein it contemplates double jeopardy of punishment for the same
act. It further explains that even if the offenses charged are not the same, owing that
the first charge constitutes a violation of an ordinance and the second charge was a
violation against the revised penal code, the fact that the two charges sprung from one
and the same act of conviction or acquittal under either the law or the ordinance shall
bar a prosecution under the other thus making it against the logic of double jeopardy.
The fact that Mr. Opulencia was acquitted on the first offense should bar the 2 nd
Page 270

complaint against him coming from the same identity as that of the 1 st offense charged
against Mr.Opulencia.

ESMENA VS. POGOY


[102 SCRA 861; G.R. NO. L-54110; 20 FEB 1981]
Facts:
Petitioners Esmea and Alba were charged with grave coercion in the Court of Cebu
City for allegedly forcing Fr. Thomas Tibudan to withdraw a sum of money worth P5000
from the bank to be given to them because the priest lost in a game of chance. During
arraignment, petitioners pleaded Not Guilty. No trial came in after the arraignment
due to the priests request to move it on another date. Sometime later Judge Pogoy
issued an order setting the trial Aug.16,1979 but the fiscal informed the court that it
received a telegram stating that the complainant was sick. The accused invoked their
right to speedy trial. Respondent judge dismissed the case because the trial was
already dragging the accused and that the priests telegram did not have a medical
certificate attached to it in order for the court to recognize the complainants reason to
be valid in order to reschedule again another hearing. After 27 days the fiscal filed a
motion to revive the case and attached the medical certificate of the priest proving the
fact that the priest was indeed sick of influenza. On Oct.24,1979, accused Esmea and
Alba filed a motion to dismiss the case on the ground of double jeopardy.
Issue:
Whether or Not the revival of grave coercion case, which was dismissed earlier due to
complainants failure to appear at the trial, would place the accused in double jeopardy
Held:
Yes, revival of the case will put the accused in double jeopardy for the very reason that
the case has been dismissed already without the consent of the accused which would
have an effect of an acquittal on the case filed. The dismissal was due to complainants
incapability to present its evidence due to non appearance of the witnesses and
complainant himself which would bar further prosecution of the defendant for the same
offense. For double jeopardy to exist these three requisites should be present, that
one, there is a valid complaint or information filed second, that it is done before a court
of competent jurisdiction and third, that the accused has been arraigned and has
pleaded to the complaint or information. In the case at bar, all three conditions were
present, as the case filed was grave coercion, filed in a court of competent jurisdiction
as to where the coercion took place and last the accused were arraigned and has
pleaded to the complaint or the information. When these three conditions are present
then the acquittal, conviction of the accused, and the dismissal or termination of the
Page 271

case without his express consent constitutes res judicata and is a bar to another
prosecution for the offense charged. In the case, it was evidently shown that the
accused invoked their right to a speedy trial and asked for the trial of the case and not
its termination which would mean that respondents had no expressed consent to the
dismissal of the case which would make the case filed res judicata and has been
dismissed by the competent court in order to protect the respondents as well for their
right to speedy trial which will be equivalent to acquittal of the respondents which
would be a bar to further prosecution.

PEOPLE VS. DE LA TORRE


[380 SCRA 586; G.R. NOS. 137953-58; 11 MAR 2002]
Facts:
Wilfredo dela Torre, appellee, has three children from a common-law relationship, the
eldest of which is Mary Rose. When Mary Rose was 7 yearsold, her mother left them
together with her youngest brother so she and her other brother were left to the care of
her father.
Mary Rose was the brightest in her class despite their poverty. However, in January
1997, a sudden change in Mary Roses behavior behavior was noticed. She was twelve
years old at that time. She appeared sleepy, snobbish and she also urinated on her
panty. When confronted by her head teacher, Mary Rose admitted that she was abused
repeatedly by her father. Her father, however, denied vehemently the charges being
imputed to him by her daughter.
The RTC convicted appellee of two counts of acts of lasciviousness and four counts of
murder. However, the RTC refused to impose the supreme penalty of death on
appellee. It maintained that there were circumstances that mitigated the gravity of the
offenses such as the absence of any actual physical violence or intimidation on the
commission of the acts, that after the mother of Mary Rose left the conjugal home, for
more than five years, Wilfredo, Mary Rose and her brother were living together as a
family and Mary Rose was never molested by her father.
The prosecution seeks to modify the RTC Decision by imposing the supreme penalty of
death of the accused. It argues that it has proven that the victim is the daughter of the
accused, and that she was below eighteen years old when the rapes took place. As a
consequence, the trial court should have been imposed the penalty of death pursuant
to Section 11 of R.A. 7659. .
Issue:

Page 272

Whether or Not the Court erred in penalizing the appellee with reclusion perpetua in
each of the four indictments of rape, instead of imposing the supreme penalty of death
as mandated by R.A. 7659.
Held:
Under Section 1, Rule 122 of the 2000 Rules of Criminal Procedure, any party may
appeal from a judgment or final order unless the accused will be put in double jeopardy.
In People vs. Leones, it declared that:
while it is true that this Court is the Court of last resort, there are
allegations of error committed by a lower court which we ought not to look
into to uphold the right of the accused. Such is the case in an appeal by
the prosecution seeking to increase the penalty imposed upon the accused
for this runs afoul of the right of the accused against double jeopardy
When the accused after conviction by the trial court did not appeal his
decision, an appeal by the government seeking to increase the penalty
imposed by the trial court places the accused in double jeopardy and
should therefore be dismissed.
The ban on double jeopardy primarily prevents the State from using its criminal
processes as an instrument of harassment to wear out the accused by a multitude of
cases with accumulated trials. It also serves as a deterrent from successively retrying
the defendant in the hope of securing a conviction. And finally, it prevents the State,
following conviction, from retrying the defendant again in the hope of securing a
greater penalty.
Being violative of the right against double jeopardy, the appeal of the prosecution
cannot prosper.

Page 273

CITIZENSHIP

Page 274

CITIZENSHIP

Art. 4
Sec. 1.

The following are citizens of the Philippines:

(1)

Those who are citizens of the Philippines at the time of the adoption of this
Constitution;

(2)

Those whose fathers or mothers are citizens of the Philippines;

(3)

Those born before January 17, 1973, of Filipino mothers, who elect Philippine
citizenship upon reaching the age of majority; and

(4)

Those who are naturalized in accordance with law.

Sec. 2.
Natural-born citizens are those who are citizens of the Philippines from
birth without having to perform any act to acquire or perfect their Philippine
citizenship. Those who elect Philippine citizenship in accordance with paragraph (3),
Section 1 hereof shall be deemed natural-born citizens.
Sec. 3.
law.

Philippine citizenship may be lost or reacquired in the manner provided by

Sec. 4.
Citizens of the Philippines who marry aliens shall retain their citizenship,
unless by their act or omission they are deemed, under the law, to have renounced it.
Sec. 5.
Dual allegiance of citizens is inimical to the national interest and shall be
dealt with by law.

FRIVALDO VS. COMELEC


[174 SCRA 245; G.R. NO. 87193; 23 JUN 1989]
Facts:
Petitioner Juan G. Frivaldo was proclaimed governor-elect of the province of Sorsogon
on January 22, 1988, and assumed office in due time. On October 27, 1988, the League
of Municipalities, Sorsogon Chapter, represented by its President, Estuye, who was also
suing in his personal capacity, filed with the COMELEC a petition for the annulment of
Frivaldo; election and proclamation on the ground that he was not a Filipino citizen,
having been naturalized in the United States on January 20, 1983. In his answer dated
May 22, 1988, Frivaldo admitted that he was naturalized in the United States as alleged
but pleaded the special and affirmative defenses that he had sought American
citizenship only to protect himself against President Marcos. His naturalization, he said,
Page 275

was "merely forced upon himself as a means of survival against the unrelenting
persecution by the Martial Law Dictator's agents abroad." He added that he had
returned to the Philippines after the EDSA revolution to help in the restoration of
democracy. In their Comment, the private respondents reiterated their assertion that
Frivaldo was a naturalized American citizen and had not reacquired Philippine
citizenship on the day of the election on January 18, 1988. He was therefore not
qualified to run for and be elected governor. They also argued that their petition in the
Commission on Elections was not really for quo warranto under Section 253 of the
Omnibus Election Code. The ultimate purpose was to prevent Frivaldo from continuing
as governor, his candidacy and election being null and void ab initio because of his
alienage. Speaking for the public respondent, the Solicitor General supported the
contention that Frivaldo was not a citizen of the Philippines and had not repatriated
himself after his naturalization as an American citizen. As an alien, he was disqualified
from public office in the Philippines. His election did not cure this defect because the
electorate of Sorsogon could not amend the Constitution, the Local Government Code,
and the Omnibus Election Code. He also joined in the private respondent's argument
that Section 253 of the Omnibus Election Code was not applicable because what the
League and Estuye were seeking was not only the annulment of the proclamation and
election of Frivaldo. He agreed that they were also asking for the termination of
Frivaldo's incumbency as governor of Sorsogon on the ground that he was not a Filipino.
Issue:
Whether or Not petitioner Juan G. Frivaldo was a citizen of the Philippines at the time of
his election on January 18, 1988, as provincial governor of Sorsogon.
Held:
The reason for this inquiry is the provision in Article XI, Section 9, of the Constitution
that all public officials and employees owe the State and the Constitution "allegiance at
all times" and the specific requirement in Section 42 of the Local Government Code that
a candidate for local elective office must be inter alia a citizen of the Philippines and a
qualified voter of the constituency where he is running. Section 117 of the Omnibus
Election Code provides that a qualified voter must be, among other qualifications, a
citizen of the Philippines, this being an indispensable requirement for suffrage under
Article V, Section 1, of the Constitution.
In the certificate of candidacy he filed on November 19, 1987, Frivaldo described
himself as a "natural-born" citizen of the Philippines, omitting mention of any
subsequent loss of such status. The evidence shows, however, that he was naturalized
as a citizen of the United States in 1983 per the following certification from the United
States District Court, Northern District of California, as duly authenticated by Vice
Consul Amado P. Cortez of the Philippine Consulate General in San Francisco, California,
U.S.A.
The Court sees no reason not to believe that the petitioner was one of the enemies of
the Marcos dictatorship. Even so, it cannot agree that as a consequence thereof he was
coerced into embracing American citizenship. His feeble suggestion that his

Page 276

naturalization was not the result of his own free and voluntary choice is totally
unacceptable and must be rejected outright.
This Court will not permit the anomaly of a person sitting as provincial governor in this
country while owing exclusive allegiance to another country. The fact that he was
elected by the people of Sorsogon does not excuse this patent violation of the salutary
rule limiting public office and employment only to the citizens of this country. The
qualifications prescribed for elective office cannot be erased by the electorate alone.
The will of the people as expressed through the ballot cannot cure the vice of
ineligibility, especially if they mistakenly believed, as in this case, that the candidate
was qualified. Obviously, this rule requires strict application when the deficiency is lack
of citizenship. If a person seeks to serve in the Republic of the Philippines, he must owe
his total loyalty to this country only, abjuring and renouncing all fealty and fidelity to
any other state.
It is true as the petitioner points out that the status of the natural-born citizen is
favored by the Constitution and our laws, which is all the more reason why it should be
treasured like a pearl of great price. But once it is surrendered and renounced, the gift
is gone and cannot be lightly restored. This country of ours, for all its difficulties and
limitations, is like a jealous and possessive mother. Once rejected, it is not quick to
welcome back with eager arms its prodigal if repentant children. The returning
renegade must show, by an express and unequivocal act, the renewal of his loyalty and
love.
Petition Dismissed. Petitioner JUAN G. FRIVALDO is hereby declared not a citizen of the
Philippines and therefore disqualified from serving as Governor of the Province of
Sorsogon. Accordingly, he is ordered to vacate his office and surrender the same to the
duly elected Vice-Governor of the said province once this decision becomes final and
executory.

MERCADO VS. MANZANO


[307 SCRA 630; G.R. NO. 135083; 26 MAY 1999]
Facts:
Petitioner Ernesto Mercado and Private respondent Eduardo Manzano are candidates for
the position of Vice-Mayor of Makati City in the May, 1998 elections. Private respondent
was the winner of the said election but the proclamation was suspended due to the
petition of Ernesto Mamaril regarding the citizenship of private respondent. Mamaril
alleged that the private respondent is not a citizen of the Philippines but of the United
States. COMELEC granted the petition and disqualified the private respondent for being
a dual citizen, pursuant to the Local Government code that provides that persons who
possess dual citizenship are disqualified from running any public position. Private
respondent filed a motion for reconsideration which remained pending until after
election. Petitioner sought to intervene in the case for disqualification. COMELEC
reversed the decision and declared private respondent qualified to run for the position.
Pursuant to the ruling of the COMELEC, the board of canvassers proclaimed private
Page 277

respondent as vice mayor. This petition sought the reversal of the resolution of the
COMELEC and to declare the private respondent disqualified to hold the office of the
vice mayor of Makati.
Issue:
Whether or Not private respondent is qualified to hold office as Vice-Mayor.
Held:
Dual citizenship is different from dual allegiance. The former arises when, as a result of
the concurrent application of the different laws of two or more states, a person is
simultaneously considered a national by the said states. For instance, such a situation
may arise when a person whose parents are citizens of a state which adheres to the
principle of jus sanguinis is born in a state which follows the doctrine of jus soli. Private
respondent is considered as a dual citizen because he is born of Filipino parents but was
born in San Francisco, USA. Such a person, ipso facto and without any voluntary act on
his part, is concurrently considered a citizen of both states. Considering the citizenship
clause (Art. IV) of our Constitution, it is possible for the following classes of citizens of
the Philippines to posses dual citizenship: (1) Those born of Filipino fathers and/or
mothers in foreign countries which follow the principle of jus soli; (2) Those born in the
Philippines of Filipino mothers and alien fathers if by the laws of their fathers country
such children are citizens of that country; (3) Those who marry aliens if by the laws of
the latters country the former are considered citizens, unless by their act or omission
they are deemed to have renounced Philippine citizenship. Dual allegiance, on the
other hand, refers to the situation in which a person simultaneously owes, by some
positive act, loyalty to two or more states. While dual citizenship is involuntary, dual
allegiance is the result of an individuals volition.
By filing a certificate of candidacy when he ran for his present post, private respondent
elected Philippine citizenship and in effect renounced his American citizenship. The
filing of such certificate of candidacy sufficed to renounce his American citizenship,
effectively removing any disqualification he might have as a dual citizen.
By declaring in his certificate of candidacy that he is a Filipino citizen; that he is not a
permanent resident or immigrant of another country; that he will defend and support
the Constitution of the Philippines and bear true faith and allegiance thereto and that
he does so without mental reservation, private respondent has, as far as the laws of
this country are concerned, effectively repudiated his American citizenship and
anything which he may have said before as a dual citizen. On the other hand, private
respondents oath of allegiance to the Philippine, when considered with the fact that he
has spent his youth and adulthood, received his education, practiced his profession as
an artist, and taken part in past elections in this country, leaves no doubt of his election
of Philippine citizenship.

TECSON VS. COMELEC


[424 SCRA 277; G.R. No. 161434; 3 Mar 2004]
Facts:
Page 278

Victorino X. Fornier, petitioner initiated a petition before the COMELEC to disqualify FPJ
and to deny due course or to cancel his certificate of candidacy upon the thesis that FPJ
made a material misrepresentation in his certificate of candidacy by claiming to be a
natural-born Filipino citizen when in truth, according to Fornier, his parents were
foreigners; his mother, Bessie Kelley Poe, was an American, and his father, Allan Poe,
was a Spanish national, being the son of Lorenzo Pou, a Spanish subject. Granting,
petitioner asseverated, that Allan F. Poe was a Filipino citizen, he could not have
transmitted his Filipino citizenship to FPJ, the latter being an illegitimate child of an
alien mother. Petitioner based the allegation of the illegitimate birth of respondent on
two assertions - first, Allan F. Poe contracted a prior marriage to a certain Paulita Gomez
before his marriage to Bessie Kelley and, second, even if no such prior marriage had
existed, Allan F. Poe, married Bessie Kelly only a year after the birth of respondent.

Issue:
Whether or Not FPJ is a natural born Filipino citizen.
Held:
It is necessary to take on the matter of whether or not respondent FPJ is a natural-born
citizen, which, in turn, depended on whether or not the father of respondent, Allan F.
Poe, would have himself been a Filipino citizen and, in the affirmative, whether or not
the alleged illegitimacy of respondent prevents him from taking after the Filipino
citizenship of his putative father. Any conclusion on the Filipino citizenship of Lorenzo
Pou could only be drawn from the presumption that having died in 1954 at 84 years old,
Lorenzo would have been born sometime in the year 1870, when the Philippines was
under Spanish rule, and that San Carlos, Pangasinan, his place of residence upon his
death in 1954, in the absence of any other evidence, could have well been his place of
residence before death, such that Lorenzo Pou would have benefited from the "en
masse Filipinization" that the Philippine Bill had effected in 1902. That citizenship (of
Lorenzo Pou), if acquired, would thereby extend to his son, Allan F. Poe, father of
respondent FPJ. The 1935 Constitution, during which regime respondent FPJ has seen
first light, confers citizenship to all persons whose fathers are Filipino citizens
regardless of whether such children are legitimate or illegitimate.
But while the totality of the evidence may not establish conclusively that respondent
FPJ is a natural-born citizen of the Philippines, the evidence on hand still would
preponderate in his favor enough to hold that he cannot be held guilty of having made
a material misrepresentation in his certificate of candidacy in violation of Section 78, in
relation to Section 74, of the Omnibus Election Code.

BENGZON VS. HRET


Page 279

[357 SCRA 545; G. R. No. 142840; 7 May 2001]


Facts:
Respondent Teodoro Cruz was a natural-born citizen of the Philippines. He was born in
San Clemente, Tarlac, on April 27, 1960, of Filipino parents. The fundamental law then
applicable was the 1935 Constitution. On November 5, 1985, however, respondent Cruz
enlisted in the United States Marine Corps and without the consent of the Republic of
the Philippines, took an oath of allegiance to the United States. As a Consequence, he
lost his Filipino citizenship for under Commonwealth Act No. 63, section 1(4), a Filipino
citizen may lose his citizenship by, among other, "rendering service to or accepting
commission in the armed forces of a foreign country. He was naturalized in US in 1990.
On March 17, 1994, respondent Cruz reacquired his Philippine citizenship through
repatriation under Republic Act No. 2630. He ran for and was elected as the
Representative of the Second District of Pangasinan in the May 11, 1998 elections. He
won over petitioner Antonio Bengson III, who was then running for reelection.
Issue:
Whether or Not respondent Cruz is a natural born citizen of the Philippines in view of
the constitutional requirement that "no person shall be a Member of the House of
Representative unless he is a natural-born citizen.
Held:
Respondent is a natural born citizen of the Philippines. As distinguished from the
lengthy process of naturalization, repatriation simply consists of the taking of an oath of
allegiance to the Republic of the Philippine and registering said oath in the Local Civil
Registry of the place where the person concerned resides or last resided. This means
that a naturalized Filipino who lost his citizenship will be restored to his prior status as a
naturalized Filipino citizen. On the other hand, if he was originally a natural-born citizen
before he lost his Philippine citizenship, he will be restored to his former status as a
natural-born Filipino.

Page 280

You might also like